med surg final

Ace your homework & exams now with Quizwiz!

A 68-year-old patient is being admitted with a possible stroke. Which information from the assessment indicates that the nurse should consult with the health care provider before giving the prescribed aspirin? a. The patient has dysphasia. b. The patient has atrial fibrillation. c. The patient reports that symptoms began with a severe headache. d. The patient has a history of brief episodes of right-sided hemiplegia.

ANS: C A sudden onset headache is typical of a subarachnoid hemorrhage, and aspirin is contraindicated. Atrial fibrillation, dysphasia, and transient ischemic attack (TIA) are not contraindications to aspirin use, so the nurse can administer the aspirin. DIF: Cognitive Level: Apply (application) REF: 1392-1393 TOP: Nursing Process: Assessment MSC: NCLEX: Physiological Integrity

The nurse is caring for a patient who has been experiencing stroke symptoms for 60 minutes. Which action can the nurse delegate to a licensed practical/vocational nurse (LPN/LVN)? a. Assess the patient's gag and cough reflexes. b. Determine when the stroke symptoms began. c. Administer the prescribed short-acting insulin. d. Infuse the prescribed IV metoprolol (Lopressor).

ANS: C Administration of subcutaneous medications is included in LPN/LVN education and scope of practice. The other actions require more education and scope of practice and should be done by the registered nurse (RN). DIF: Cognitive Level: Apply (application) REF: 1405 OBJ: Special Questions: Delegation TOP: Nursing Process: Planning MSC: NCLEX: Safe and Effective Care Environment

A 72-year-old patient who has a history of a transient ischemic attack (TIA) has an order for aspirin 160 mg daily. When the nurse is administering medications, the patient says, "I don't need the aspirin today. I don't have a fever." Which action should the nurse take? a. Document that the aspirin was refused by the patient. b. Tell the patient that the aspirin is used to prevent a fever. c. Explain that the aspirin is ordered to decrease stroke risk. d. Call the health care provider to clarify the medication order.

ANS: C Aspirin is ordered to prevent stroke in patients who have experienced TIAs. Documentation of the patient's refusal to take the medication is an inadequate response by the nurse. There is no need to clarify the order with the health care provider. The aspirin is not ordered to prevent aches and pains. DIF: Cognitive Level: Apply (application) REF: 1396 TOP: Nursing Process: Implementation MSC: NCLEX: Physiological Integrity

When caring for a patient with a new right-sided homonymous hemianopsia resulting from a stroke, which intervention should the nurse include in the plan of care? a. Apply an eye patch to the right eye. b. Approach the patient from the right side. c. Place objects needed on the patient's left side. d. Teach the patient that the left visual deficit will resolve.

ANS: C During the acute period, the nurse should place objects on the patient's unaffected side. Because there is a visual defect in the right half of each eye, an eye patch is not appropriate. The patient should be approached from the left side. The visual deficit may not resolve, although the patient can learn to compensate for the defect. DIF: Cognitive Level: Apply (application) REF: 1407 TOP: Nursing Process: Planning MSC: NCLEX: Physiological Integrity

3. A patient is scheduled for pulmonary function testing. Which action should the nurse take to prepare the patient for this procedure? a. Give the rescue medication immediately before testing. b. Administer oral corticosteroids 2 hours before the procedure. c. Withhold bronchodilators for 6 to 12 hours before the examination. d. Ensure that the patient has been NPO for several hours before the test.

ANS: C Bronchodilators are held before pulmonary function testing (PFT) so that a baseline assessment of airway function can be determined. Testing is repeated after bronchodilator use to determine whether the decrease in lung function is reversible. There is no need for the patient to be NPO. Oral corticosteroids should be held before PFTs. Rescue medications (which are bronchodilators) would not be given until after the baseline pulmonary function was assessed. DIF: Cognitive Level: Apply (application) REF: 566 TOP: Nursing Process: Implementation MSC: NCLEX: Physiological Integrity

A 38-year old woman receiving chemotherapy for breast cancer develops a Candida albicans oral infection. The nurse will anticipate the need for a. hydrogen peroxide rinses. b. the use of antiviral agents. c. administration of nystatin (Mycostatin) tablets. d. referral to a dentist for professional tooth cleaning.

ANS: C Candida albicans is treated with an antifungal such as nystatin. Oral saltwater rinses may be used but will not cure the infection. Antiviral agents are used for viral infections such as herpes simplex. Referral to a dentist is indicated for gingivitis but not for Candida infection

24. A 22-year-old man tells the nurse at the health clinic that he has recently had some problems with erectile dysfunction. Which question should the nurse ask first to assess for possible etiologic factors?

"Do you use any recreational drugs or drink alcohol?"

A nurse is teaching a patient with contact dermatitis of the arms and legs about ways to decrease pruritus. Which information should the nurse include in the teaching plan (select all that apply)?

-Cool, wet cloths or dressings can be used to reduce itching. -Take cool or tepid baths several times daily to decrease itching. -Use of an over-the-counter (OTC) antihistamine can reduce scratching. Cool or tepid baths, cool dressings, and OTC antihistamines all help reduce pruritus and scratching. Adding oil to bath water is not recommended because of the increased risk for falls. The patient should use the towel to pat (not rub) the skin dry.

COMPLETION 1. When assessing a patient with possible peripheral artery disease (PAD), the nurse obtains a brachial blood pressure (BP) of 147/82 mm Hg and an ankle pressure of 112/74 mm Hg. The nurse calculates the patient's ankle-brachial index (ABI) as ________ (round up to the nearest hundredth).

0.76 The ABI is calculated by dividing the ankle systolic BP by the brachial systolic BP. DIF: Cognitive Level: Apply (application) REF: 805 TOP: Nursing Process: Implementation MSC: NCLEX: Physiological

A new mother expresses concern about her baby developing allergies and asks what the health care provider meant by "passive immunity." Which example should the nurse use to explain this type of immunity? a. Early immunization b. Bone marrow donation c. Breastfeeding her infant d. Exposure to communicable diseases

ANS: C Colostrum provides passive immunity through antibodies from the mother. These antibodies protect the infant for a few months. However, memory cells are not retained, so the protection is not permanent. Active immunity is acquired by being immunized with vaccinations or having an infection. It requires that the infant has an immune response after exposure to an antigen. Cell-mediated immunity is acquired through T lymphocytes and is a form of active immunity.

32. The following male patients recently arrived in the emergency department. Which one should the nurse assess first?

19-year-old who is complaining of severe scrotal pain

To evaluate an obese patient for adverse effects of lorcaserin (Belviq), which action will the nurse take? a. Take the apical pulse rate. b. Check sclera for jaundice. c. Ask about bowel movements. d. Assess for agitation or restlessness.

ANS: C Constipation is a common side effect of lorcaserin. The other assessments would be appropriate for other weight-loss medications.

There is one opening in the schedule at the dermatology clinic, and 4 patients are seeking appointments today. Which patient will the nurse schedule for the available opening?

38-year old with a 7-mm nevus on the face that has recently become darker The description of the lesion is consistent with possible malignant melanoma. This patient should be assessed as soon as possible by the health care provider. Itching is common after using topical fluorouracil and redness is an expected finding a few days after a chemical peel. Skin tags are common, benign lesions after midlife.

The nurse working in the vision and hearing clinic receives telephone calls from several patients who want appointments in the clinic as soon as possible. Which patient should be seen first?

71-year-old who has noticed increasing loss of peripheral vision

A 34-year-old female patient with a new ileostomy asks how much drainage to expect. The nurse explains that after the bowel adjusts to the ileostomy, the usual drainage will be about _____ cups. a. 2 b. 3 c. 4 d. 5

A

A patient complains of gas pains and abdominal distention two days after a small bowel resection. Which nursing action is best to take? a. Encourage the patient to ambulate. b. Instill a mineral oil retention enema. c. Administer the ordered IV morphine sulfate. d. Offer the ordered promethazine (Phenergan) suppository.

A

A patient with a right retinal detachment had a pneumatic retinopexy procedure. Which information will be included in the discharge teaching plan? a. The purpose of maintaining the head in a prescribed position b. The use of eye patches to reduce movement of the operative eye c. The need to wear dark glasses to protect the eyes from bright light d. The procedure for dressing changes when the eye dressing is saturated

A

During the preoperative assessment of the patient scheduled for a right cataract extraction and intraocular lens implantation, it is most important for the nurse to assess a. the visual acuity of the patient's left eye. b. how long the patient has had the cataract. c. for a white pupil in the patient's right eye. d. for a history of reactions to general anesthetics.

A

The nurse admitting a patient with acute diverticulitis explains that the initial plan of care is to a. administer IV fluids. b. give stool softeners and enemas. c. order a diet high in fiber and fluids. d. prepare the patient for colonoscopy.

A

The nurse is assessing a patient who has recently been treated with amoxicillin for acute otitis media of the right ear. Which finding is a priority to report to the health care provider? a. The patient has a temperature of 100.6° F. b. The patient complains of "popping" in the ear. c. The patient frequently asks the nurse to repeat information. d. The patient states that the right ear has a feeling of fullness.

A

The priority nursing diagnosis for a patient experiencing an acute attack with Meniere's disease is a. risk for falls related to dizziness. b. impaired verbal communication related to tinnitus. c. self-care deficit (bathing and dressing) related to vertigo. d. imbalanced nutrition: less than body requirements related to nausea.

A

Which information will the nurse include for a patient contemplating a cochlear implant? a. Cochlear implants require training in order to receive the full benefit. b. Cochlear implants are not useful for patients with congenital deafness. c. Cochlear implants are most helpful as an early intervention for presbycusis. d. Cochlear implants improve hearing in patients with conductive hearing loss.

A

A patient has acute bronchitis with a nonproductive cough and wheezes. Which topic should the nurse plan to include in the teaching plan? a. Purpose of antibiotic therapy b. Ways to limit oral fluid intake c. Appropriate use of cough suppressants d. Safety concerns with home oxygen therapy

ANS: C Cough suppressants are frequently prescribed for acute bronchitis. Because most acute bronchitis is viral in origin, antibiotics are not prescribed unless there are systemic symptoms. Fluid intake is encouraged. Home oxygen is not prescribed for acute bronchitis, although it may be used for chronic bronchitis

A patient who has undergone a left tympanoplasty should be instructed to a. remain on bed rest. b. keep the head elevated. c. avoid blowing the nose. d. irrigate the left ear canal.

ANS: C Coughing or blowing the nose increases pressure in the eustachian tube and middle ear cavity and disrupts postoperative healing. There is no postoperative need for prolonged bed rest, elevation of the head, or continuous antibiotic irrigation

A patient is diagnosed with both human immunodeficiency virus (HIV) and active tuberculosis (TB) disease. Which information obtained by the nurse is most important to communicate to the health care provider? a. The Mantoux test had an induration of 7 mm. b. The chest-x-ray showed infiltrates in the lower lobes. c. The patient is being treated with antiretrovirals for HIV infection. d. The patient has a cough that is productive of blood-tinged mucus.

ANS: C Drug interactions can occur between the antiretrovirals used to treat HIV infection and the medications used to treat TB. The other data are expected in a patient with HIV and TB.

13. Which diet choice by the patient with an acute exacerbation of inflammatory bowel disease (IBD) indicates a need for more teaching? a. Scrambled eggs b. White toast and jam c. Oatmeal with cream d. Pancakes with syrup

ANS: C During acute exacerbations of IBD, the patient should avoid high-fiber foods such as whole grains. High-fat foods also may cause diarrhea in some patients. The other choices are low residue and would be appropriate for this patient.

24. The nurse is caring for a patient immediately after repair of an abdominal aortic aneurysm. On assessment, the patient has absent popliteal, posterior tibial, and dorsalis pedis pulses. The legs are cool and mottled. Which action should the nurse take first? a. Notify the surgeon and anesthesiologist. b. Wrap both the legs in a warming blanket. c. Document the findings and recheck in 15 minutes. d. Compare findings to the preoperative assessment of the pulses.

A Lower extremity pulses may be absent for a short time after surgery because of vasospasm and hypothermia. Decreased or absent pulses together with a cool and mottled extremity may indicate embolization or graft occlusion. These findings should be reported to the surgeon immediately because this is an emergency situation. Because pulses are marked before surgery, the nurse would know whether pulses were present before surgery before notifying the health care providers about the absent pulses. Because the patient's symptoms may indicate graft occlusion or multiple emboli and a possible need to return to surgery, it is not appropriate to wait 15 minutes before taking action. A warming blanket will not improve the circulation to the patient's legs. DIF: Cognitive Level: Analyze (analysis) REF: 814 OBJ: Special Questions: Prioritization TOP: Nursing Process: Implementation MSC: NCLEX: Physiological

4. A patient in the outpatient clinic has a new diagnosis of peripheral artery disease (PAD). Which group of drugs will the nurse plan to include when teaching about PAD management? a. Statins c. Thrombolytics b. Antibiotics d. Anticoagulants

A Research indicates that statin use by patients with PAD improves multiple outcomes. There is no research that supports the use of the other drug categories in PAD. DIF: Cognitive Level: Apply (application) REF: 805 TOP: Nursing Process: Planning MSC: NCLEX: Physiological

After administering a dose of promethazine (Phenergan) to a patient with nausea and vomiting, the nurse explains that which of the following may be experienced as a common temporary adverse effect of the medication? A) Drowsiness B) Reduced hearing C) Sensation of falling D) Photosensitivity

A) Drowsiness (Although being given to this patient as an antiemetic, promethazine also has sedative and amnesic properties. For this reason, the patient is likely to experience drowsiness as an adverse effect of the medication.)

The nurse notes new onset confusion in an older patient who is normally alert and oriented. In which order should the nurse take the following actions? (Put a comma and a space between each answer choice [A, B, C, D].) a. Obtain the oxygen saturation. b. Check the patient's pulse rate. c. Document the change in status. d. Notify the health care provider

ANS: A, B, D, C Assessment for physiologic causes of new onset confusion such as pneumonia, infection, or perfusion problems should be the first action by the nurse. Airway and oxygenation should be assessed first, then circulation. After assessing the patient, the nurse should notify the health care provider. Finally, documentation of the assessments and care should be done

In which order will the nurse take the following actions when caring for a patient who develops watery diarrhea and a fever after prolonged omeprazole (Prilosec) therapy? (Put a comma and a space between each answer choice [A, B, C, D].) a. Contact the health care provider. b. Assess blood pressure and heart rate. c. Give the PRN acetaminophen (Tylenol). d. Place the patient on contact precautions.

ANS: D, B, A, C Proton pump inhibitors including omeprazole (Prilosec) may increase the risk of Clostridium difficile-associated colitis. Because the patient's history and symptoms are consistent with C. difficile infection, the initial action should be initiation of infection control measures to protect other patients. Assessment of blood pressure and pulse is needed to determine whether the patient has symptoms of hypovolemia and/or shock. The health care provider should be notified so that actions such as obtaining stool specimens and antibiotic therapy can be started. Tylenol may be administered, but is the lowest priority of the actions.

A patient is admitted to the hospital with acute rejection of a kidney transplant. Which intervention will the nurse prepare for this patient? a. Administration of immunosuppressant medications b. Insertion of an arteriovenous graft for hemodialysis c. Placement of the patient on the transplant waiting list d. A blood draw for human leukocyte antigen (HLA) matching

ANS: A Acute rejection is treated with the administration of additional immunosuppressant drugs such as corticosteroids. Because acute rejection is potentially reversible, there is no indication that the patient will require another transplant or hemodialysis. There is no indication for repeat HLA testing.

25. A patient in the clinic with cystic fibrosis (CF) reports increased sweating and weakness during the summer months. Which action by the nurse would be most appropriate? a. Have the patient add dietary salt to meals. b. Teach the patient about the signs of hypoglycemia. c. Suggest decreasing intake of dietary fat and calories. d. Instruct the patient about pancreatic enzyme replacements.

ANS: A Added dietary salt is indicated whenever sweating is excessive, such as during hot weather, when fever is present, or from intense physical activity. The management of pancreatic insufficiency includes pancreatic enzyme replacement of lipase, protease, and amylase (e.g., Pancreaze, Creon, Ultresa, Zenpep) administered before each meal and snack. This patient is at risk for hyponatremia based on reported symptoms. Adequate intake of fat, calories, protein, and vitamins is important. Fat-soluble vitamins (vitamins A, D, E, and K) must be supplemented because they are malabsorbed. Use of caloric supplements improves nutritional status. Hyperglycemia due to pancreatic insufficiency is more likely to occur than hypoglycemia.DIF: Cognitive Level: Apply (application) REF: 605 TOP: Nursing Process: Implementation MSC: NCLEX: Physiological Integrity

A patient who has vague symptoms of fatigue, headaches, and a positive test for human immunodeficiency virus (HIV) antibodies using an enzyme immunoassay (EIA) test. What instructions should the nurse give to this patient? a. "The EIA test will need to be repeated to verify the results." b. "A viral culture will be done to determine the progression of the disease." c. "It will probably be 10 or more years before you develop acquired immunodeficiency syndrome (AIDS)." d. "The Western blot test will be done to determine whether acquired immunodeficiency syndrome (AIDS) has developed."

ANS: A After an initial positive EIA test, the EIA is repeated before more specific testing such as the Western blot is done. Viral cultures are not usually part of HIV testing. It is not appropriate for the nurse to predict the time frame for AIDS development. The Western blot tests for HIV antibodies, not for AIDS.

25. A 34-year-old female patient with a new ileostomy asks how much drainage to expect. The nurse explains that after the bowel adjusts to the ileostomy, the usual drainage will be about _____ cups. a. 2 b. 3 c. 4 d. 5

ANS: A After the proximal small bowel adapts to reabsorb more fluid, the average amount of ileostomy drainage is about 500 mL daily. One cup is about 240 mL.

The priority nursing diagnosis for a patient experiencing an acute attack with Meniere's disease is a. risk for falls related to dizziness. b. impaired verbal communication related to tinnitus. c. self-care deficit (bathing and dressing) related to vertigo. d. imbalanced nutrition: less than body requirements related to nausea.

ANS: A All the nursing diagnoses are appropriate, but because sudden attacks of vertigo can lead to "drop attacks," the major focus of nursing care is to prevent injuries associated with dizziness

2. Which integumentary assessment data from an older patient admitted with bacterial pneumonia is of most concern for the nurse? a. Reports a history of allergic rashes b. Scattered macular brown areas on extremities c. Skin brown and wrinkled, skin tenting on forearm d. Longitudinal nail bed ridges noted; sparse scalp hair

ANS: A Because the patient will be receiving antibiotics to treat the pneumonia, the nurse should be most concerned about her history of allergic rashes. The nurse needs to do further assessment of possible causes of the allergic rashes and whether she has ever had allergic reactions to any drugs, especially antibiotics. The assessment data in the other response would be normal for an older patient. DIF: Cognitive Level: Apply (application) REF: 398 TOP: Nursing Process: Assessment

1. Which finding for a 19-year-old female who is a vegan may indicate the need for cobalamin supplementation? a. Paresthesias b. Ecchymoses c. Dry, scaly skin d. Gingival swelling

ANS: A Cobalamin (vitamin B12) cannot be obtained from foods of plant origin, so the patient will be most at risk for signs of cobalamin deficiency, such as paresthesias, peripheral neuropathy, and anemia. The other symptoms listed are associated with other nutritional deficiencies but would not be associated with a vegan diet.

15. The nurse is caring for a patient with cor pulmonale. The nurse should monitor the patient for which expected finding? a. Peripheral edema b. Elevated temperature c. Clubbing of the fingers d. Complaints of chest pain

ANS: A Cor pulmonale causes clinical manifestations of right ventricular failure, such as peripheral edema. The other clinical manifestations may occur in the patient with other complications of chronic obstructive pulmonary disease (COPD) but are not indicators of cor pulmonale. DIF: Cognitive Level: Apply (application) REF: 586 TOP: Nursing Process: Evaluation MSC: NCLEX: Physiological Integrity

The nurse will anticipate preparing a 71-year-old female patient who is vomiting "coffee-ground" emesis for a. endoscopy. b. angiography. c. barium studies. d. gastric analysis.

ANS: A Endoscopy is the primary tool for visualization and diagnosis of upper gastrointestinal (GI) bleeding. Angiography is used only when endoscopy cannot be done because it is more invasive and has more possible complications. Barium studies are helpful in determining the presence of gastric lesions, but not whether the lesions are actively bleeding. Gastric analysis testing may help with determining the cause of gastric irritation, but it is not used for acute GI bleeding

A patient with a right retinal detachment had a pneumatic retinopexy procedure. Which information will be included in the discharge teaching plan? a. The purpose of maintaining the head in a prescribed position b. The use of eye patches to reduce movement of the operative eye c. The need to wear dark glasses to protect the eyes from bright light d. The procedure for dressing changes when the eye dressing is saturated

ANS: A Following pneumatic retinopexy, the patient will need to position the head so the air bubble remains in contact with the retinal tear. The dark lenses and bilateral eye patches are not required after this procedure. Saturation of any eye dressings would not be expected following this procedure

43. Which finding in a patient hospitalized with bronchiectasis is most important to report to the health care provider? a. Cough productive of bloody, purulent mucus b. Scattered rhonchi and wheezes heard bilaterally c. Respiratory rate 28 breaths/minute while ambulating in hallway d. Complaint of sharp chest pain with deep breathing

ANS: A Hemoptysis may indicate life-threatening hemorrhage and should be reported immediately to the health care provider. The other findings are frequently noted in patients with bronchiectasis and may need further assessment but are not indicators of life-threatening complications. DIF: Cognitive Level: Apply (application) REF: 607 OBJ: Special Questions: Prioritization TOP: Nursing Process: Assessment MSC: NCLEX: Safe and Effective Care Environment

The nurse assesses the chest of a patient with pneumococcal pneumonia. Which finding would the nurse expect? a. Increased tactile fremitus b. Dry, nonproductive cough c. Hyperresonance to percussion d. A grating sound on auscultation

ANS: A Increased tactile fremitus over the area of pulmonary consolidation is expected with bacterial pneumonias. Dullness to percussion would be expected. Pneumococcal pneumonia typically presents with a loose, productive cough. Adventitious breath sounds such as crackles and wheezes are typical. A grating sound is more representative of a pleural friction rub rather than pneumonia

The nurse will be teaching self-management to patients after gastric bypass surgery. Which information will the nurse plan to include? a. Drink fluids between meals but not with meals. b. Choose high-fat foods for at least 30% of intake. c. Developing flabby skin can be prevented by exercise. d. Choose foods high in fiber to promote bowel function.

ANS: A Intake of fluids with meals tends to cause dumping syndrome and diarrhea. Food choices should be low in fat and fiber. Exercise does not prevent the development of flabby skin.

15. Which action for a patient receiving tube feedings through a percutaneous endoscopic gastrostomy (PEG) may be delegated to a licensed practical/vocational nurse (LPN/LVN)? a. Providing skin care to the area around the tube site b. Teaching the patient how to administer tube feedings c. Determining the need for adding water to the feedings d. Assessing the patient's nutritional status at least weekly

ANS: A LPN/LVN education and scope of practice include actions such as dressing changes and wound care. Patient teaching and complex assessments (such as patient nutrition and hydration status) require registered nurse (RN)-level education and scope of practice.

45. A 51-year-old woman with Crohn's disease who is taking infliximab (Remicade) calls the nurse in the outpatient clinic about new symptoms. Which symptom is most important to communicate to the health care provider? a. Fever b. Nausea c. Joint pain d. Headache

ANS: A Since infliximab suppresses the immune response, rapid treatment of infection is essential. The other patient complaints are common side effects of the medication, but they do not indicate any potentially life-threatening complications.

To evaluate the effectiveness of antiretroviral therapy (ART), which laboratory test result will the nurse review? a. Viral load testing b. Enzyme immunoassay c. Rapid HIV antibody testing d. Immunofluorescence assay

ANS: A The effectiveness of ART is measured by the decrease in the amount of virus detectable in the blood. The other tests are used to detect HIV antibodies, which remain positive even with effective ART.

A 40-year-old obese woman reports that she wants to lose weight. Which question should the nurse ask first? a. "What factors led to your obesity?" b. "Which types of food do you like best?" c. "How long have you been overweight?" d. "What kind of activities do you enjoy?"

ANS: A The nurse should obtain information about the patient's perceptions of the reasons for the obesity to develop a plan individualized to the patient. The other information also will be obtained from the patient, but the patient is more likely to make changes when the patient's beliefs are considered in planning.

A 72-year-old patient with a history of benign prostatic hyperplasia (BPH) is admitted with acute urinary retention and elevated blood urea nitrogen (BUN) and creatinine levels. Which prescribed therapy should the nurse implement first? a. Insert urethral catheter. b. Obtain renal ultrasound. c. Draw a complete blood count. d. Infuse normal saline at 50 mL/hour.

ANS: A The patient's elevation in BUN is most likely associated with hydronephrosis caused by the acute urinary retention, so the insertion of a retention catheter is the first action to prevent ongoing postrenal failure for this patient. The other actions also are appropriate, but should be implemented after the retention catheter

Ten days after receiving a bone marrow transplant, a patient develops a skin rash. What would the nurse suspect is the cause of this patient's skin rash? a. The donor T cells are attacking the patient's skin cells. b. The patient's antibodies are rejecting the donor bone marrow. c. The patient is experiencing a delayed hypersensitivity reaction. d. The patient will need treatment to prevent hyperacute rejection.

ANS: A The patient's history and symptoms indicate that the patient is experiencing graft-versus-host disease, in which the donated T cells attack the patient's tissues. The history and symptoms are not consistent with rejection or delayed hypersensitivity

When caring for a patient with a left arm arteriovenous fistula, which action will the nurse include in the plan of care to maintain the patency of the fistula? a. Auscultate for a bruit at the fistula site. b. Assess the quality of the left radial pulse. c. Compare blood pressures in the left and right arms. d. Irrigate the fistula site with saline every 8 to 12 hours.

ANS: A The presence of a thrill and bruit indicates adequate blood flow through the fistula. Pulse rate and quality are not good indicators of fistula patency. Blood pressures should never be obtained on the arm with a fistula. Irrigation of the fistula might damage the fistula, and typically only dialysis staff would access the fistula

An older adult patient who is having an annual check-up tells the nurse, "I feel fine, and I don't want to pay for all these unnecessary cancer screening tests!" Which information should the nurse plan to teach this patient? a. Consequences of aging on cell-mediated immunity b. Decrease in antibody production associated with aging c. Impact of poor nutrition on immune function in older people d. Incidence of cancer-stimulating infections in older individuals

ANS: A The primary impact of aging on immune function is on T cells, which are important for immune surveillance and tumor immunity. Antibody function is not affected as much by aging. Poor nutrition can also contribute to decreased immunity, but there is no evidence that it is a contributing factor for this patient. Although some types of cancer are associated with specific infections, this patient does not have an active infection.

A patient who is scheduled for a therapeutic abortion tells the nurse, "Having an abortion is not right." Which functional health pattern should the nurse further assess? a. Value-belief b. Cognitive-perceptual c. Sexuality-reproductive d. Coping-stress tolerance

ANS: A The value-belief pattern includes information about conflicts between a patient's values and proposed medical care. In the cognitive-perceptual pattern, the nurse will ask questions about pain and sensory intactness. The sexuality-reproductive pattern includes data about the impact of the surgery on the patient's sexuality. The coping-stress tolerance pattern assessment will elicit information about how the patient feels about the surgery.

Following assessment of a patient with pneumonia, the nurse identifies a nursing diagnosis of ineffective airway clearance. Which assessment data best supports this diagnosis? a. Weak, nonproductive cough effort b. Large amounts of greenish sputum c. Respiratory rate of 28 breaths/minute d. Resting pulse oximetry (SpO2) of 85%

ANS: A The weak, nonproductive cough indicates that the patient is unable to clear the airway effectively. The other data would be used to support diagnoses such as impaired gas exchange and ineffective breathing pattern

The nurse is caring for a patient who has a right-sided chest tube after a right lower lobectomy. Which nursing action can the nurse delegate to the unlicensed assistive personnel (UAP)? a. Document the amount of drainage every eight hours. b. Obtain samples of drainage for culture from the system. c. Assess patient pain level associated with the chest tube. d. Check the water-seal chamber for the correct fluid level.

ANS: A UAP education includes documentation of intake and output. The other actions are within the scope of practice and education of licensed nursing personnel

8. A young adult patient who denies any history of smoking is seen in the clinic with a new diagnosis of chronic obstructive pulmonary disease (COPD). It is most appropriate for the nurse to teach the patient about a. α1-antitrypsin testing. b. use of the nicotine patch. c. continuous pulse oximetry. d. effects of leukotriene modifiers.

ANS: A When COPD occurs in young patients, especially without a smoking history, a genetic deficiency in α1-antitrypsin should be suspected. Because the patient does not smoke, a nicotine patch would not be ordered. There is no indication that the patient requires continuous pulse oximetry. Leukotriene modifiers would be used in patients with asthma, not with COPD. DIF: Cognitive Level: Apply (application) REF: 582-583 TOP: Nursing Process: Planning MSC: NCLEX: Physiological Integrity

Which of the nurse's assigned patients should be referred to the dietitian for a complete nutritional assessment (select all that apply)? a. A 23-year-old who has a history of fluctuating weight gains and losses b. A 35-year-old who complains of intermittent nausea for the past 2 days c. A 64-year-old who is admitted for débridement of an infected surgical wound d. A 52-year-old admitted with chest pain and possible myocardial infarction (MI) e. A 48-year-old with rheumatoid arthritis who takes prednisone (Deltasone) daily

ANS: A, C, E Weight fluctuations, use of corticosteroids, and draining or infected wounds all suggest that the patient may be at risk for malnutrition. Patients with chest pain or MI are not usually poorly nourished. Although vomiting that lasts 5 days places a patient at risk, nausea that has persisted for 2 days does not always indicate poor nutritional status or risk for health problems caused by poor nutrition.

3. The nurse assesses a circular, flat, reddened lesion about 5 cm in diameter on a middle-aged patients ankle. How should the nurse determine if the lesion is related to intradermal bleeding? a. Elevate the patients leg. b. Press firmly on the lesion. c. Check the temperature of the skin around the lesion. d. Palpate the dorsalis pedis and posterior tibial pulses.

ANS: B If the lesion is caused by intradermal or subcutaneous bleeding or a nonvascular cause, the discoloration will remain when direct pressure is applied to the lesion. If the lesion is caused by blood vessel dilation, blanching will occur with direct pressure. The other assessments will assess circulation to the leg, but will not be helpful in determining the etiology of the lesion. DIF: Cognitive Level: Apply (application) REF: 401 TOP: Nursing Process: Assessment

A 38-year-old patient who had a kidney transplant 8 years ago is receiving the immunosuppressants tacrolimus (Prograf), cyclosporine (Sandimmune), and prednisone (Deltasone). Which assessment data will be of most concern to the nurse? a. The blood glucose is 144 mg/dL. b. There is a nontender axillary lump. c. The patient's skin is thin and fragile. d. The patient's blood pressure is 150/92.

ANS: B A nontender lump suggests a malignancy such as a lymphoma, which could occur as a result of chronic immunosuppressive therapy. The elevated glucose, skin change, and hypertension are possible side effects of the prednisone and should be addressed, but they are not as great a concern as the possibility of a malignancy

After the nurse has completed teaching a patient with newly diagnosed eosinophilic esophagitis about the management of the disease, which patient action indicates that the teaching has been effective? a. Patient orders nonfat milk for each meal. b. Patient uses the prescribed corticosteroid inhaler. c. Patient schedules an appointment for allergy testing. d. Patient takes ibuprofen (Advil) to control throat pain.

ANS: C Eosinophilic esophagitis is frequently associated with environmental allergens, so allergy testing is used to determine possible triggers. Corticosteroid therapy may be prescribed, but the medication will be swallowed, not inhaled. Milk is a frequent trigger for attacks. NSAIDs are not used for eosinophilic esophagitis

After a patient experienced a brief episode of tinnitus, diplopia, and dysarthria with no residual effects, the nurse anticipates teaching the patient about a. cerebral aneurysm clipping. b. heparin intravenous infusion. c. oral low-dose aspirin therapy. d. tissue plasminogen activator (tPA).

ANS: C The patient's symptoms are consistent with transient ischemic attack (TIA), and drugs that inhibit platelet aggregation are prescribed after a TIA to prevent stroke. Continuous heparin infusion is not routinely used after TIA or with acute ischemic stroke. The patient's symptoms are not consistent with a cerebral aneurysm. tPA is used only for acute ischemic stroke, not for TIA. DIF: Cognitive Level: Apply (application) REF: 1391 | 1396 TOP: Nursing Process: Planning MSC: NCLEX: Physiological Integrity

During the change of shift report a nurse is told that a patient has an occluded left posterior cerebral artery. The nurse will anticipate that the patient may have a. dysphasia. b. confusion. c. visual deficits. d. poor judgment.

ANS: C Visual disturbances are expected with posterior cerebral artery occlusion. Aphasia occurs with middle cerebral artery involvement. Cognitive deficits and changes in judgment are more typical of anterior cerebral artery occlusion. DIF: Cognitive Level: Apply (application) REF: 1393 TOP: Nursing Process: Assessment MSC: NCLEX: Physiological Integrity

4. A 26-year-old woman is being evaluated for vomiting and abdominal pain. Which question from the nurse will be most useful in determining the cause of the patient's symptoms? a. "What type of foods do you eat?" b. "Is it possible that you are pregnant?" c. "Can you tell me more about the pain?" d. "What is your usual elimination pattern?"

ANS: C A complete description of the pain provides clues about the cause of the problem. Although the nurse should ask whether the patient is pregnant to determine whether the patient might have an ectopic pregnancy and before any radiology studies are done, this information is not the most useful in determining the cause of the pain. The usual diet and elimination patterns are less helpful in determining the reason for the patient's symptoms.

12. The nurse interviews a patient with a new diagnosis of chronic obstructive pulmonary disease (COPD). Which information is most helpful in confirming a diagnosis of chronic bronchitis? a. The patient tells the nurse about a family history of bronchitis. b. The patient's history indicates a 30 pack-year cigarette history. c. The patient complains about a productive cough every winter for 3 months. d. The patient denies having any respiratory problems until the last 12 months.

ANS: C A diagnosis of chronic bronchitis is based on a history of having a productive cough for 3 months for at least 2 consecutive years. There is no family tendency for chronic bronchitis. Although smoking is the major risk factor for chronic bronchitis, a smoking history does not confirm the diagnosis. DIF: Cognitive Level: Apply (application) REF: 579 TOP: Nursing Process: Assessment MSC: NCLEX: Physiological Integrity

Which finding in the mouth of a patient who uses smokeless tobacco is suggestive of oral cancer? a. Bleeding during tooth brushing b. Painful blisters at the lip border c. Red, velvety patches on the buccal mucosa d. White, curdlike plaques on the posterior tongue

ANS: C A red, velvety patch suggests erythroplasia, which has a high incidence (greater than 50%) of progression to squamous cell carcinoma. The other lesions are suggestive of acute processes (e.g., gingivitis, oral candidiasis, herpes simplex).

8. A patient in the dermatology clinic is scheduled for removal of a 15-mm multicolored and irregular mole from the upper back. The nurse should prepare the patient for which type of biopsy? a. Shave biopsy b. Punch biopsy c. Incisional biopsy d. Excisional biopsy

ANS: C An incisional biopsy would remove the entire mole and the tissue borders. The appearance of the mole indicates that it may be malignant. A shave biopsy would not remove the entire mole. The mole is too large to be removed with punch biopsy. Excisional biopsies are done for smaller lesions and where a good cosmetic effect is desired, such as on the face. DIF: Cognitive Level: Apply (application) REF: 405 TOP: Nursing Process: Planning

The health care provider asks the nurse whether a patient's angioedema has responded to prescribed therapies. Which assessment should the nurse perform? a. Ask the patient about any clear nasal discharge. b. Obtain the patient's blood pressure and heart rate. c. Check for swelling of the patient's lips and tongue. d. Assess the patient's extremities for wheal and flare lesions.

ANS: C Angioedema is characterized by swelling of the eyelids, lips, and tongue. Wheal and flare lesions, clear nasal drainage, and hypotension and tachycardia are characteristic of other allergic reactions.

Which assessment is most important for the nurse to make regarding a patient with myasthenia gravis? a. Pupil size b. Grip strength c. Respiratory effort d. Level of consciousness

ANS: C Because respiratory insufficiency may be life threatening, it will be most important to monitor respiratory function. The other data also will be assessed but are not as critical. DIF: Cognitive Level: Apply (application) REF: 1438-1439 OBJ: Special Questions: Prioritization TOP: Nursing Process: Assessment MSC: NCLEX: Physiological Integrity

7. A malnourished patient is receiving a parenteral nutrition (PN) infusion containing amino acids and dextrose from a bag that was hung 24 hours ago. The nurse observes that about 50 mL remain in the PN container. Which action is best for the nurse to take? a. Ask the health care provider to clarify the written PN order. b. Add a new container of PN using the current tubing and filter. c. Hang a new container of PN and change the IV tubing and filter. d. Infuse the remaining 50 mL and then hang a new container of PN.

ANS: B All PN solutions are changed at 24 hours. PN solutions containing dextrose and amino acids require a change in tubing and filter every 72 hours rather than daily. Infusion of the additional 50 mL will increase patient risk for infection. Changing the IV tubing and filter more frequently than required will unnecessarily increase costs. The nurse (not the health care provider) is responsible for knowing the indicated times for tubing and filter changes.

Which information about a patient who had a stapedotomy yesterday is most important for the nurse to communicate to the health care provider? a. The patient complains of "fullness" in the ear. b. The patient's oral temperature is 100.8° F (38.1° C). c. The patient says "My hearing is worse now than it was right after surgery." d. There is a small amount of dried bloody drainage on the patient's dressing.

ANS: B An elevated temperature may indicate a postoperative infection. Although the nurse would report all the data, a temporary decrease in hearing, bloody drainage on the dressing, and a feeling of congestion (because of the accumulation of blood and drainage in the ear) are common after this surgery

Which finding for a patient who has been taking orlistat (Xenical) is most important to report to the health care provider? a. The patient frequently has liquid stools. b. The patient is pale and has many bruises. c. The patient complains of bloating after meals. d. The patient is experiencing a weight loss plateau.

ANS: B Because orlistat blocks the absorption of fat-soluble vitamins, the patient may not be receiving an adequate amount of vitamin K, resulting in a decrease in clotting factors. Abdominal bloating and liquid stools are common side effects of orlistat and indicate that the nurse should remind the patient that fat in the diet may increase these side effects. Weight loss plateaus are normal during weight reduction.

A 49-year-old man has been admitted with hypotension and dehydration after 3 days of nausea and vomiting. Which order from the health care provider will the nurse implement first? a. Insert a nasogastric (NG) tube. b. Infuse normal saline at 250 mL/hr. c. Administer IV ondansetron (Zofran). d. Provide oral care with moistened swabs.

ANS: B Because the patient has severe dehydration, rehydration with IV fluids is the priority. The other orders should be accomplished as quickly as possible after the IV fluids are initiated

41. Which information obtained by the nurse interviewing a 30-year-old male patient is most important to communicate to the health care provider? a. The patient has a history of constipation. b. The patient has noticed blood in the stools. c. The patient had an appendectomy at age 27. d. The patient smokes a pack/day of cigarettes.

ANS: B Blood in the stools is a possible clinical manifestation of colorectal cancer and requires further assessment by the health care provider. The other patient information will also be communicated to the health care provider, but does not indicate an urgent need for further testing or intervention.

Which information will the nurse monitor in order to determine the effectiveness of prescribed calcium carbonate (Caltrate) for a patient with chronic kidney disease (CKD)? a. Blood pressure b. Phosphate level c. Neurologic status d. Creatinine clearance

ANS: B Calcium carbonate is prescribed to bind phosphorus and prevent mineral and bone disease in patients with CKD. The other data will not be helpful in evaluating the effectiveness of calcium carbonate

Which item should the nurse offer to the patient who is to restart oral intake after being NPO due to nausea and vomiting? a. Glass of orange juice b. Dish of lemon gelatin c. Cup of coffee with cream d. Bowl of hot chicken broth

ANS: B Clear cool liquids are usually the first foods started after a patient has been nauseated. Acidic foods such as orange juice, very hot foods, and coffee are poorly tolerated when patients have been nauseated

Which statement by a patient with bacterial conjunctivitis indicates a need for further teaching? a. "I will wash my hands often during the day." b. "I will remove my contact lenses at bedtime." c. "I will not share towels with my friends or family." d. "I will monitor my family for eye redness or drainage."

ANS: B Contact lenses should not be used when patients have conjunctivitis because they can further irritate the conjunctiva. Hand washing is the major means to prevent the spread of conjunctivitis. Infection may be spread by sharing towels or other contact. It is common for bacterial conjunctivitis to spread through a family or other group in close contact

Which information will the nurse prioritize in planning preoperative teaching for a patient undergoing a Roux-en-Y gastric bypass? a. Educating the patient about the nasogastric (NG) tube b. Instructing the patient on coughing and breathing techniques c. Discussing necessary postoperative modifications in lifestyle d. Demonstrating passive range-of-motion exercises for the legs

ANS: B Coughing and deep breathing can prevent major postoperative complications such as carbon monoxide retention and hypoxemia. Information about passive range of motion, the NG tube, and postoperative modifications in lifestyle will also be discussed, but avoidance of respiratory complications is the priority goal after surgery.

Which information in a patient's history indicates to the nurse that the patient is not an appropriate candidate for kidney transplantation? a. The patient has type 1 diabetes. b. The patient has metastatic lung cancer. c. The patient has a history of chronic hepatitis C infection. d. The patient is infected with the human immunodeficiency virus.

ANS: B Disseminated malignancies are a contraindication to transplantation. The conditions of the other patients are not contraindications for kidney transplant

Which information will the nurse include for a patient with newly diagnosed gastroesophageal reflux disease (GERD)? a. "Peppermint tea may reduce your symptoms." b. "Keep the head of your bed elevated on blocks." c. "You should avoid eating between meals to reduce acid secretion." d. "Vigorous physical activities may increase the incidence of reflux."

ANS: B Elevating the head of the bed will reduce the incidence of reflux while the patient is sleeping. Peppermint will decrease lower esophageal sphincter (LES) pressure and increase the chance for reflux. Small, frequent meals are recommended to avoid abdominal distention. There is no need to make changes in physical activities because of GERD

Which nursing action should be included in the postoperative plan of care for a patient after a laparoscopic esophagectomy? a. Notify the doctor about bloody nasogastric (NG) drainage. b. Elevate the head of the bed to at least 30 degrees. c. Reposition the NG tube if drainage stops. d. Start oral fluids when the patient has active bowel sounds.

ANS: B Elevation of the head of the bed decreases the risk for reflux and aspiration of gastric secretions. The NG tube should not be repositioned without consulting with the health care provider. Bloody NG drainage is expected for the first 8 to 12 hours. A swallowing study is needed before oral fluids are started

16. A 24-year-old woman with Crohn's disease develops a fever and symptoms of a urinary tract infection (UTI) with tan, fecal-smelling urine. What information will the nurse add to a general teaching plan about UTIs in order to individualize the teaching for this patient? a. Bacteria in the perianal area can enter the urethra. b. Fistulas can form between the bowel and bladder. c. Drink adequate fluids to maintain normal hydration. d. Empty the bladder before and after sexual intercourse.

ANS: B Fistulas between the bowel and bladder occur in Crohn's disease and can lead to UTI. Teaching for UTI prevention in general includes good hygiene, adequate fluid intake, and voiding before and after intercourse.

37. Which assessment finding in a patient who has received omalizumab (Xolair) is most important to report immediately to the health care provider? a. Pain at injection site b. Flushing and dizziness c. Peak flow reading 75% of normal d. Respiratory rate 22 breaths/minute

ANS: B Flushing and dizziness may indicate that the patient is experiencing an anaphylactic reaction, and immediate intervention is needed. The other information should also be reported, but do not indicate possibly life-threatening complications of omalizumab therapy. DIF: Cognitive Level: Apply (application) REF: 572 OBJ: Special Questions: Prioritization TOP: Nursing Process: Assessment MSC: NCLEX: Physiological Integrity

Monitored anesthesia care (MAC) is going to be used for a closed, manual reduction of a dislocated shoulder. What action does the nurse anticipate? a. Securing an airtight fit for the inhalation mask b. Starting a 20-gauge IV in the patient's unaffected arm c. Obtaining a nonocclusive dressing to place over the administration site d. Teaching the patient about epidural patient-controlled anesthesia (PCA) use

ANS: B For MAC, IV sedatives, such as the benzodiazepines, are administered. Therefore the patient needs IV access. Inhaled, epidural, and topical agents are not included in MAC. An occlusive dressing would be placed over a topical agent such as eutectic mixture of local anesthetics (EMLA) cream

An experienced nurse instructs a new nurse about how to care for a patient with dyspnea caused by a pulmonary fungal infection. Which action by the new nurse indicates a need for further teaching? a. Listening to the patient's lung sounds several times during the shift b. Placing the patient on droplet precautions and in a private hospital room c. Increasing the oxygen flow rate to keep the oxygen saturation above 90% d. Monitoring patient serology results to identify the specific infecting organism

ANS: B Fungal infections are not transmitted from person to person. Therefore no isolation procedures are necessary. The other actions by the new nurse are appropriate

A patient has just been admitted with probable bacterial pneumonia and sepsis. Which order should the nurse implement first? a. Chest x-ray via stretcher b. Blood cultures from two sites c. Ciprofloxacin (Cipro) 400 mg IV d. Acetaminophen (Tylenol) rectal suppository

ANS: B Initiating antibiotic therapy rapidly is essential, but it is important that the cultures be obtained before antibiotic administration. The chest x-ray and acetaminophen administration can be done last

When teaching a patient about the treatment of acoustic neuroma, the nurse will include information about a. a low sodium diet. b. ways to avoid falls. c. how to apply sunscreen. d. the chemotherapy side effects.

ANS: B Intermittent vertigo occurs with acoustic neuroma, so the nurse should include information about how to prevent falls. Diet is not a risk factor for acoustic neuroma and no dietary changes are needed. Sunscreen would be used to prevent skin cancers on the external ear. Acoustic neuromas are benign and do not require chemotherapy

The home health registered nurse (RN) is planning care for a patient with a seizure disorder related to a recent head injury. Which nursing action can be delegated to a licensed practical/vocational nurse (LPN/LVN)? a. Make referrals to appropriate community agencies. b. Place medications in the home medication organizer. c. Teach the patient and family how to manage seizures. d. Assess for use of medications that may precipitate seizures.

ANS: B LPN/LVN education includes administration of medications. The other activities require RN education and scope of practice. DIF: Cognitive Level: Apply (application) REF: 1426 OBJ: Special Questions: Delegation TOP: Nursing Process: Planning MSC: NCLEX: Safe and Effective Care Environment

54. Which information will the nurse teach a 23-year-old patient with lactose intolerance? a. Ice cream is relatively low in lactose. b. Live-culture yogurt is usually tolerated. c. Heating milk will break down the lactose. d. Nonfat milk is a better choice than whole milk.

ANS: B Lactose-intolerant individuals can usually eat yogurt without experiencing discomfort. Ice cream, nonfat milk, and milk that has been heated are all high in lactose.

The nurse in the dialysis clinic is reviewing the home medications of a patient with chronic kidney disease (CKD). Which medication reported by the patient indicates that patient teaching is required? a. Multivitamin with iron b. Magnesium hydroxide c. Acetaminophen (Tylenol) d. Calcium phosphate (PhosLo)

ANS: B Magnesium is excreted by the kidneys, and patients with CKD should not use over-the-counter products containing magnesium. The other medications are appropriate for a patient with CKD.

Which intervention will the nurse include in the plan of care for a patient with primary restless legs syndrome (RLS) who is having difficulty sleeping? a. Teach about the use of antihistamines to improve sleep. b. Suggest that the patient exercise regularly during the day. c. Make a referral to a massage therapist for deep massage of the legs. d. Assure the patient that the problem is transient and likely to resolve.

ANS: B Nondrug interventions such as getting regular exercise are initially suggested to improve sleep quality in patients with RLS. Antihistamines may aggravate RLS. Massage does not alleviate RLS symptoms and RLS is likely to progress in most patients. DIF: Cognitive Level: Apply (application) REF: 1427 TOP: Nursing Process: Planning MSC: NCLEX: Physiological Integrity

52. After change-of-shift report, which patient should the nurse assess first? a. 40-year-old male with celiac disease who has frequent frothy diarrhea b. 30-year-old female with a femoral hernia who has abdominal pain and vomiting c. 30-year-old male with ulcerative colitis who has severe perianal skin breakdown d. 40-year-old female with a colostomy bag that is pulling away from the adhesive wafer

ANS: B Pain and vomiting with a femoral hernia suggest possible strangulation, which will necessitate emergency surgery. The other patients have less urgent problems.

An 82-year-old patient who is being admitted to the hospital repeatedly asks the nurse to "speak up so that I can hear you." Which action should the nurse take? a. Overenunciate while speaking. b. Speak normally but more slowly. c. Increase the volume when speaking. d. Use more facial expressions when talking.

ANS: B Patient understanding of the nurse's speech will be enhanced by speaking at a normal tone, but more slowly. Increasing the volume, overenunciating, and exaggerating facial expressions will not improve the patient's ability to comprehend the nurse

The nurse provides discharge instructions to a patient who was hospitalized for pneumonia. Which statement, if made by the patient, indicates a good understanding of the instructions? a. "I will call the doctor if I still feel tired after a week." b. "I will continue to do the deep breathing and coughing exercises at home." c. "I will schedule two appointments for the pneumonia and influenza vaccines." d. "I'll cancel my chest x-ray appointment if I'm feeling better in a couple weeks."

ANS: B Patients should continue to cough and deep breathe after discharge. Fatigue is expected for several weeks. The Pneumovax and influenza vaccines can be given at the same time in different arms. Explain that a follow-up chest x-ray needs to be done in 6 to 8 weeks to evaluate resolution of pneumonia

When a patient with acute kidney injury (AKI) has an arterial blood pH of 7.30, the nurse will expect an assessment finding of a. persistent skin tenting b. rapid, deep respirations. c. bounding peripheral pulses. d. hot, flushed face and neck.

ANS: B Patients with metabolic acidosis caused by AKI may have Kussmaul respirations as the lungs try to regulate carbon dioxide. Bounding pulses and vasodilation are not associated with metabolic acidosis. Because the patient is likely to have fluid retention, poor skin turgor would not be a finding in AKI

The nurse at the eye clinic made a follow-up telephone call to a patient who underwent cataract extraction and intraocular lens implantation the previous day. Which information is the priority to communicate to the health care provider? a. The patient has questions about the ordered eye drops. b. The patient has eye pain rated at a 5 (on a 0 to 10 scale). c. The patient has poor depth perception when wearing an eye patch. d. The patient complains that the vision has not improved very much.

ANS: B Postoperative cataract surgery patients usually experience little or no pain, so pain at a level 5 on a 10-point pain scale may indicate complications such as hemorrhage, infection, or increased intraocular pressure. The other information given by the patient indicates a need for patient teaching but does not indicate that complications of the surgery may be occurring

Which action will the nurse plan to take for a 40-year-old patient with multiple sclerosis (MS) who has urinary retention caused by a flaccid bladder? a. Decrease the patient's evening fluid intake. b. Teach the patient how to use the Credé method. c. Suggest the use of adult incontinence briefs for nighttime only. d. Assist the patient to the commode every 2 hours during the day.

ANS: B The Credé method can be used to improve bladder emptying. Decreasing fluid intake will not improve bladder emptying and may increase risk for urinary tract infection (UTI) and dehydration. The use of incontinence briefs and frequent toileting will not improve bladder emptying. DIF: Cognitive Level: Apply (application) REF: eNCP 59-3 TOP: Nursing Process: Planning MSC: NCLEX: Physiological Integrity

18. A patient is receiving 35% oxygen via a Venturi mask. To ensure the correct amount of oxygen delivery, which action by the nurse is most important? a. Teach the patient to keep mask on at all times. b. Keep the air entrainment ports clean and unobstructed. c. Give a high enough flow rate to keep the bag from collapsing. d. Drain moisture condensation from the oxygen tubing every hour.

ANS: B The air entrainment ports regulate the oxygen percentage delivered to the patient, so they must be unobstructed. A high oxygen flow rate is needed when giving oxygen by partial rebreather or non-rebreather masks. Draining oxygen tubing is necessary when caring for a patient receiving mechanical ventilation. The mask is uncomfortable and can be removed when the patient eats. DIF: Cognitive Level: Apply (application) REF: 591 TOP: Nursing Process: Implementation MSC: NCLEX: Physiological Integrity

16. The nurse is admitting a patient diagnosed with an acute exacerbation of chronic obstructive pulmonary disease (COPD).What is the best way for the nurse to determine the appropriate oxygen flow rate? a. Minimize oxygen use to avoid oxygen dependency. b. Maintain the pulse oximetry level at 90% or greater. c. Administer oxygen according to the patient's level of dyspnea. d. Avoid administration of oxygen at a rate of more than 2 L/minute.

ANS: B The best way to determine the appropriate oxygen flow rate is by monitoring the patient's oxygenation either by arterial blood gases (ABGs) or pulse oximetry. An oxygen saturation of 90% indicates adequate blood oxygen level without the danger of suppressing the respiratory drive. For patients with an exacerbation of COPD, an oxygen flow rate of 2 L/min may not be adequate. Because oxygen use improves survival rate in patients with COPD, there is no concern about oxygen dependency. The patient's perceived dyspnea level may be affected by other factors (such as anxiety) besides blood oxygen level. DIF: Cognitive Level: Apply (application) REF: 589 TOP: Nursing Process: Implementation MSC: NCLEX: Physiological Integrity

A 48-year-old patient with stage 2 chronic kidney disease (CKD) is scheduled for an intravenous pyelogram (IVP). Which order for the patient will the nurse question? a. NPO for 6 hours before procedure b. Ibuprofen (Advil) 400 mg PO PRN for pain c. Dulcolax suppository 4 hours before procedure d. Normal saline 500 mL IV infused before procedure

ANS: B The contrast dye used in IVPs is potentially nephrotoxic, and concurrent use of other nephrotoxic medications such as the nonsteroidal anti-inflammatory drugs (NSAIDs) should be avoided. The suppository and NPO status are necessary to ensure adequate visualization during the IVP. IV fluids are used to ensure adequate hydration, which helps reduce the risk for contrast-induced renal failure

6. A 20-year-old man with extensive facial injuries from a motor vehicle crash is receiving tube feedings through a percutaneous endoscopic gastrostomy (PEG). Which action will the nurse include in the plan of care? a. Keep the patient positioned on the left side. b. Check the gastric residual volume every 4 to 6 hours. c. Avoid giving bolus tube feedings through the PEG tube. d. Obtain a daily abdominal x-ray to verify tube placement.

ANS: B The gastric residual volume is assessed every 4 to 6 hours to decrease the risk for aspiration. The patient does not need to be positioned on the left side. Bolus feedings can be administered through a PEG tube. An x-ray is obtained immediately after placement of the PEG tube to check position, but daily x-rays are not needed.

The nurse monitors a patient after chest tube placement for a hemopneumothorax. The nurse is most concerned if which assessment finding is observed? a. A large air leak in the water-seal chamber b. 400 mL of blood in the collection chamber c. Complaint of pain with each deep inspiration d. Subcutaneous emphysema at the insertion site

ANS: B The large amount of blood may indicate that the patient is in danger of developing hypovolemic shock. An air leak would be expected immediately after chest tube placement for a pneumothorax. Initially, brisk bubbling of air occurs in this chamber when a pneumothorax is evacuated. The pain should be treated but is not as urgent a concern as the possibility of continued hemorrhage. Subcutaneous emphysema should be monitored but is not unusual in a patient with pneumothorax. A small amount of subcutaneous air is harmless and will be reabsorbed

The nurse evaluates that wearing bifocals improved the patient's myopia and presbyopia by assessing for a. strength of the eye muscles. b. both near and distant vision. c. cloudiness in the eye lenses. d. intraocular pressure changes.

ANS: B The lenses are prescribed to correct the patient's near and distant vision. The nurse may also assess for cloudiness of the lenses, increased intraocular pressure, and eye movement, but these data do not evaluate whether the patient's bifocals are effective

Which prescribed medication should the nurse give first to a patient who has just been admitted to a hospital with acute angle-closure glaucoma? a. Morphine sulfate 4 mg IV b. Mannitol (Osmitrol) 100 mg IV c. Betaxolol (Betoptic) 1 drop in each eye d. Acetazolamide (Diamox) 250 mg orally

ANS: B The most immediate concern for the patient is to lower intraocular pressure, which will occur most rapidly with IV administration of a hyperosmolar diuretic such as mannitol. The other medications are also appropriate for a patient with glaucoma but would not be the first medication administered

A 74-year-old who is progressing to stage 5 chronic kidney disease asks the nurse, "Do you think I should go on dialysis? Which initial response by the nurse is best? a. "It depends on which type of dialysis you are considering." b. "Tell me more about what you are thinking regarding dialysis." c. "You are the only one who can make the decision about dialysis." d. "Many people your age use dialysis and have a good quality of life."

ANS: B The nurse should initially clarify the patient's concerns and questions about dialysis. The patient is the one responsible for the decision and many people using dialysis do have good quality of life, but these responses block further assessment of the patient's concerns. Referring to which type of dialysis the patient might use only indirectly responds to the patient's question

A patient with newly diagnosed lung cancer tells the nurse, "I don't think I'm going to live to see my next birthday." Which response by the nurse is best? a. "Would you like to talk to the hospital chaplain about your feelings?" b. "Can you tell me what it is that makes you think you will die so soon?" c. "Are you afraid that the treatment for your cancer will not be effective?" d. "Do you think that taking an antidepressant medication would be helpful?"

ANS: B The nurse's initial response should be to collect more assessment data about the patient's statement. The answer beginning "Can you tell me what it is" is the most open-ended question and will offer the best opportunity for obtaining more data. The answer beginning, "Are you afraid" implies that the patient thinks that the cancer will be immediately fatal, although the patient's statement may not be related to the cancer diagnosis. The remaining two answers offer interventions that may be helpful to the patient, but more assessment is needed to determine whether these interventions are appropriate

Five minutes after receiving the ordered preoperative midazolam (Versed) by IV injection, the patient asks to get up to go to the bathroom to urinate. Which action by the nurse is most appropriate? a. Assist the patient to the bathroom and stay with the patient to prevent falls. b. Offer a urinal or bedpan and position the patient in bed to promote voiding. c. Allow the patient up to the bathroom because medication onset is 10 minutes. d. Ask the patient to wait because catheterization is performed just before the surgery.

ANS: B The patient will be at risk for a fall after receiving the sedative, so the best nursing action is to have the patient use a bedpan or urinal. Having the patient get up either with assistance or independently increases the risk for a fall. The patient will be uncomfortable and risk involuntary incontinence if the bladder is full during transport to the operating room.

Which intervention will be included in the plan of care for a male patient with acute kidney injury (AKI) who has a temporary vascular access catheter in the left femoral vein? a. Start continuous pulse oximetry. b. Restrict physical activity to bed rest. c. Restrict the patient's oral protein intake. d. Discontinue the urethral retention catheter.

ANS: B The patient with a femoral vein catheter must be on bed rest to prevent trauma to the vein. Protein intake is likely to be increased when the patient is receiving dialysis. The retention catheter is likely to remain in place because accurate measurement of output will be needed. There is no indication that the patient needs continuous pulse oximetry

4. A 48-year-old woman has a body mass index (BMI) of 31 kg/m2, a normal C-reactive protein level, and low serum transferrin and albumin levels. The nurse will plan patient teaching to increase the patient's intake of foods that are high in a. iron. b. protein. c. calories. d. carbohydrate.

ANS: B The patient's C-reactive protein and transferrin levels indicate low protein stores. The BMI is in the obese range, so increasing caloric intake is not indicated. The data do not indicate a need for increased carbohydrate or iron intake.

A hospitalized patient complains of a bilateral headache, 4/10 on the pain scale, that radiates from the base of the skull. Which prescribed PRN medications should the nurse administer initially? a. Lorazepam (Ativan) b. Acetaminophen (Tylenol) c. Morphine sulfate (Roxanol) d. Butalbital and aspirin (Fiorinal)

ANS: B The patient's symptoms are consistent with a tension headache, and initial therapy usually involves a nonopioid analgesic such as acetaminophen, which is sometimes combined with a sedative or muscle relaxant. Lorazepam may be used in conjunction with acetaminophen but would not be appropriate as the initial monotherapy. Morphine sulfate and butalbital and aspirin would be more appropriate for a headache that did not respond to a nonopioid analgesic. DIF: Cognitive Level: Apply (application) REF: 1414 OBJ: Special Questions: Prioritization TOP: Nursing Process: Implementation MSC: NCLEX: Physiological Integrity

A 44-year-old man admitted with a peptic ulcer has a nasogastric (NG) tube in place. When the patient develops sudden, severe upper abdominal pain, diaphoresis, and a firm abdomen, which action should the nurse take? a. Irrigate the NG tube. b. Check the vital signs. c. Give the ordered antacid. d. Elevate the foot of the bed.

ANS: B The patient's symptoms suggest acute perforation, and the nurse should assess for signs of hypovolemic shock. Irrigation of the NG tube, administration of antacids, or both would be contraindicated because any material in the stomach will increase the spillage into the peritoneal cavity. Elevating the foot of the bed may increase abdominal pressure and discomfort, as well as making it more difficult for the patient to breathe

29. A 62-year-old patient has had a hemorrhoidectomy at an outpatient surgical center. Which instructions will the nurse include in discharge teaching? a. Maintain a low-residue diet until the surgical area is healed. b. Use ice packs on the perianal area to relieve pain and swelling. c. Take prescribed pain medications before a bowel movement is expected. d. Delay having a bowel movement for several days until healing has occurred.

ANS: C Bowel movements may be very painful, and patients may avoid defecation unless pain medication is taken before the bowel movement. A high-residue diet will increase stool bulk and prevent constipation. Delay of bowel movements is likely to lead to constipation. Warm sitz baths rather than ice packs are used to relieve pain and keep the surgical area clean.

A 38-year-old female is admitted for an elective surgical procedure. Which information obtained by the nurse during the preoperative assessment is most important to report to the anesthesiologist before surgery? a. The patient's lack of knowledge about postoperative pain control measures b. The patient's statement that her last menstrual period was 8 weeks previously c. The patient's history of a postoperative infection following a prior cholecystectomy d. The patient's concern that she will be unable to care for her children postoperatively

ANS: B This statement suggests that the patient may be pregnant, and pregnancy testing is needed before administration of anesthetic agents. Although the other data may also be communicated with the surgeon and anesthesiologist, they will affect postoperative care and do not indicate a need for further assessment before surgery.

29. Which instruction should the nurse include in an exercise teaching plan for a patient with chronic obstructive pulmonary disease (COPD)? a. "Stop exercising if you start to feel short of breath." b. "Use the bronchodilator before you start to exercise." c. "Breathe in and out through the mouth while you exercise." d. "Upper body exercise should be avoided to prevent dyspnea."

ANS: B Use of a bronchodilator before exercise improves airflow for some patients and is recommended. Shortness of breath is normal with exercise and not a reason to stop. Patients should be taught to breathe in through the nose and out through the mouth (using a pursed lip technique). Upper-body exercise can improve the mechanics of breathing in patients with COPD. DIF: Cognitive Level: Apply (application) REF: 572 TOP: Nursing Process: Implementation MSC: NCLEX: Physiological Integrity

A patient with a pleural effusion is scheduled for a thoracentesis. Which action should the nurse take to prepare the patient for the procedure? a. Start a peripheral IV line to administer the necessary sedative drugs. b. Position the patient sitting upright on the edge of the bed and leaning forward. c. Obtain a large collection device to hold 2 to 3 liters of pleural fluid at one time. d. Remove the water pitcher and remind the patient not to eat or drink anything for 6 hours.

ANS: B When the patient is sitting up, fluid accumulates in the pleural space at the lung bases and can more easily be located and removed. The patient does not usually require sedation for the procedure, and there are no restrictions on oral intake because the patient is not sedated or unconscious. Usually only 1000 to 1200 mL of pleural fluid is removed at one time. Rapid removal of a large volume can result in hypotension, hypoxemia, or pulmonary edema

A 46-year-old female with gastroesophageal reflux disease (GERD) is experiencing increasing discomfort. Which patient statement indicates that additional teaching about GERD is needed? a. "I take antacids between meals and at bedtime each night." b. "I sleep with the head of the bed elevated on 4-inch blocks." c. "I eat small meals during the day and have a bedtime snack." d. "I quit smoking several years ago, but I still chew a lot of gum."

ANS: C GERD is exacerbated by eating late at night, and the nurse should plan to teach the patient to avoid eating at bedtime. The other patient actions are appropriate to control symptoms of GERD

A 55-year-old patient with end-stage kidney disease (ESKD) is scheduled to receive a prescribed dose of epoetin alfa (Procrit). Which information should the nurse report to the health care provider before giving the medication? a. Creatinine 1.6 mg/dL b. Oxygen saturation 89% c. Hemoglobin level 13 g/dL d. Blood pressure 98/56 mm Hg

ANS: C High hemoglobin levels are associated with a higher rate of thromboembolic events and increased risk of death from serious cardiovascular events (heart attack, heart failure, stroke) when erythropoietin (EPO) is administered to a target hemoglobin of >12 g/dL. Hemoglobin levels higher than 12 g/dL indicate a need for a decrease in epoetin alfa dose. The other information also will be reported to the health care provider but will not affect whether the medication is administered

An 80-year-old who is hospitalized with peptic ulcer disease develops new-onset auditory hallucinations. Which prescribed medication will the nurse discuss with the health care provider before administration? a. Sucralfate (Carafate) b. Omeprazole (Prilosec) c. Metoclopramide (Reglan) d. Aluminum hydroxide (Amphojel)

ANS: C Metoclopramide can cause central nervous system (CNS) side effects ranging from anxiety to hallucinations. Hallucinations are not a side effect of proton-pump inhibitors, mucosal protectants, or antacids

8. A patient's capillary blood glucose level is 120 mg/dL 6 hours after the nurse initiated a parenteral nutrition (PN) infusion. The most appropriate action by the nurse is to a. obtain a venous blood glucose specimen. b. slow the infusion rate of the PN infusion. c. recheck the capillary blood glucose in 4 to 6 hours. d. notify the health care provider of the glucose level.

ANS: C Mild hyperglycemia is expected during the first few days after PN is started and requires ongoing monitoring. Because the glucose elevation is small and expected, notification of the health care provider is not necessary. There is no need to obtain a venous specimen for comparison. Slowing the rate of the infusion is beyond the nurse's scope of practice and will decrease the patient's nutritional intake.

A patient who is taking rifampin (Rifadin) for tuberculosis calls the clinic and reports having orange discolored urine and tears. Which is the best response by the nurse? a. Ask if the patient is experiencing shortness of breath, hives, or itching. b. Ask the patient about any visual abnormalities such as red-green color discrimination. c. Explain that orange discolored urine and tears are normal while taking this medication. d. Advise the patient to stop the drug and report the symptoms to the health care provider.

ANS: C Orange-colored body secretions are a side effect of rifampin. The patient does not have to stop taking the medication. The findings are not indicative of an allergic reaction. Alterations in red-green color discrimination commonly occurs when taking ethambutol (Myambutol), which is a different TB medication

Which action by a 70-year-old patient who is using peritoneal dialysis (PD) indicates that the nurse should provide more teaching about PD? a. The patient leaves the catheter exit site without a dressing. b. The patient plans 30 to 60 minutes for a dialysate exchange. c. The patient cleans the catheter while taking a bath each day. d. The patient slows the inflow rate when experiencing abdominal pain.

ANS: C Patients are encouraged to take showers rather than baths to avoid infections at the catheter insertion side. The other patient actions indicate good understanding of peritoneal dialysis

Which topic is most important for the nurse to discuss preoperatively with a patient who is scheduled for abdominal surgery for an open cholecystectomy? a. Care for the surgical incision b. Medications used during surgery c. Deep breathing and coughing techniques d. Oral antibiotic therapy after discharge home

ANS: C Preoperative teaching, demonstration, and redemonstration of deep breathing and coughing are needed on patients having abdominal surgery to prevent postoperative atelectasis. Incisional care and the importance of completing antibiotics are better discussed after surgery, when the patient will be more likely to retain this information. The patient does not usually need information about medications that are used intraoperatively.

The nurse designs a program to decrease the incidence of human immunodeficiency virus (HIV) infection in the adolescent and young adult populations. Which information should the nurse assign as the highest priority? a. Methods to prevent perinatal HIV transmission b. Ways to sterilize needles used by injectable drug users c. Prevention of HIV transmission between sexual partners d. Means to prevent transmission through blood transfusions

ANS: C Sexual transmission is the most common way that HIV is transmitted. The nurse should also provide teaching about perinatal transmission, needle sterilization, and blood transfusion, but the rate of HIV infection associated with these situations is lower.

A patient undergoing an emergency appendectomy has been using St. John's wort to prevent depression. Which complication would the nurse expect in the postanesthesia care unit? a. Increased pain b. Hypertensive episodes c. Longer time to recover from anesthesia d. Increased risk for postoperative bleeding

ANS: C St. John's wort may prolong the effects of anesthetic agents and increase the time to waken completely after surgery. It is not associated with increased bleeding risk, hypertension, or increased pain.

Which action will the nurse include in the plan of care for a patient with benign paroxysmal positional vertigo (BPPV)? a. Teach the patient about use of medications to reduce symptoms. b. Place the patient in a dark, quiet room to avoid stimulating BPPV attacks. c. Teach the patient that canalith repositioning may be used to reduce dizziness. d. Speak slowly and in a low-pitch to ensure that the patient is able to hear instructions.

ANS: C The Epley maneuver is used to reposition "ear rocks" in BPPV. Medications and placement in a dark room may be used to treat Ménière's disease, but are not necessary for BPPV. There is no hearing loss with BPPV

The nurse is caring for a patient with idiopathic pulmonary arterial hypertension (IPAH) who is receiving epoprostenol (Flolan). Which assessment information requires the most immediate action by the nurse? a. The oxygen saturation is 94%. b. The blood pressure is 98/56 mm Hg. c. The patient's central IV line is disconnected. d. The international normalized ratio (INR) is prolonged.

ANS: C The half-life of this drug is 6 minutes, so the nurse will need to restart the infusion as soon as possible to prevent rapid clinical deterioration. The other data also indicate a need for ongoing monitoring or intervention, but the priority action is to reconnect the infusion

Which action by the nurse will be most effective in decreasing the spread of pertussis in a community setting? a. Providing supportive care to patients diagnosed with pertussis b. Teaching family members about the need for careful hand washing c. Teaching patients about the need for adult pertussis immunizations d. Encouraging patients to complete the prescribed course of antibiotics

ANS: C The increased rate of pertussis in adults is thought to be due to decreasing immunity after childhood immunization. Immunization is the most effective method of protecting communities from infectious diseases. Hand washing should be taught, but pertussis is spread by droplets and contact with secretions. Supportive care does not shorten the course of the disease or the risk for transmission. Taking antibiotics as prescribed does assist with decreased transmission, but patients are likely to have already transmitted the disease by the time the diagnosis is made

A patient with acute kidney injury (AKI) has longer QRS intervals on the electrocardiogram (ECG) than were noted on the previous shift. Which action should the nurse take first? a. Notify the patient's health care provider. b. Document the QRS interval measurement. c. Check the medical record for most recent potassium level. d. Check the chart for the patient's current creatinine level.

ANS: C The increasing QRS interval is suggestive of hyperkalemia, so the nurse should check the most recent potassium and then notify the patient's health care provider. The BUN and creatinine will be elevated in a patient with AKI, but they would not directly affect the electrocardiogram (ECG). Documentation of the QRS interval is also appropriate, but interventions to decrease the potassium level are needed to prevent life-threatening dysrhythmias

After successfully losing 1 lb weekly for several months, a patient at the clinic has not lost any weight for the last month. The nurse should first a. review the diet and exercise guidelines with the patient. b. instruct the patient to weigh and record weights weekly. c. ask the patient whether there have been any changes in exercise or diet patterns. d. discuss the possibility that the patient has reached a temporary weight loss plateau.

ANS: C The initial nursing action should be assessment of any reason for the change in weight loss. The other actions may be needed, but further assessment is required before any interventions are planned or implemented.

Employee health test results reveal a tuberculosis (TB) skin test of 16-mm induration and a negative chest x-ray for a staff nurse working on the pulmonary unit. The nurse has no symptoms of TB. Which information should the occupational health nurse plan to teach the staff nurse? a. Standard four-drug therapy for TB b. Need for annual repeat TB skin testing c. Use and side effects of isoniazid (INH) d. Bacille Calmette-Guérin (BCG) vaccine

ANS: C The nurse is considered to have a latent TB infection and should be treated with INH daily for 6 to 9 months. The four-drug therapy would be appropriate if the nurse had active TB. TB skin testing is not done for individuals who have already had a positive skin test. BCG vaccine is not used in the United States for TB and would not be helpful for this individual, who already has a TB infection

Which of these patients being seen at the human immunodeficiency virus (HIV) clinic should the nurse assess first? a. Patient whose latest CD4+ count is 250/µL b. Patient whose rapid HIV-antibody test is positive c. Patient who has had 10 liquid stools in the last 24 hours d. Patient who has nausea from prescribed antiretroviral drugs

ANS: C The nurse should assess the patient for dehydration and hypovolemia. The other patients also will require assessment and possible interventions, but do not require immediate action to prevent complications such as hypovolemia and shock

49. A 72-year-old male patient with dehydration caused by an exacerbation of ulcerative colitis is receiving 5% dextrose in normal saline at 125 mL/hour. Which assessment finding by the nurse is most important to report to the health care provider? a. Patient has not voided for the last 4 hours. b. Skin is dry with poor turgor on all extremities. c. Crackles are heard halfway up the posterior chest. d. Patient has had 5 loose stools over the last 6 hours.

ANS: C The presence of crackles in an older patient receiving IV fluids at a high rate suggests volume overload and a need to reduce the rate of the IV infusion. The other data will also be reported, but are consistent with the patient's age and diagnosis and do not require a change in the prescribed treatment.

Which action most effectively demonstrates that a new staff member understands the role of scrub nurse? a. Documents all patient care accurately b. Labels all specimens to send to the lab c. Keeps both hands above the operating table level d. Takes the patient to the postanesthesia recovery area

ANS: C The scrub nurse role includes maintaining asepsis in the operating field. The other actions would be performed by the circulating nurse.

A 61-year-old man is being admitted for bariatric surgery. Which nursing action can the nurse delegate to unlicensed assistive personnel (UAP)? a. Demonstrate use of the incentive spirometer. b. Plan methods for bathing and turning the patient. c. Assist with IV insertion by holding adipose tissue out of the way. d. Develop strategies to provide privacy and decrease embarrassment.

ANS: C UAP can assist with IV placement by assisting with patient positioning or holding skinfolds aside. Planning for care and patient teaching require registered nurse (RN)-level education and scope of practice.

42. Which care activity for a patient with a paralytic ileus is appropriate for the registered nurse (RN) to delegate to unlicensed assistive personnel (UAP)? a. Auscultation for bowel sounds b. Nasogastric (NG) tube irrigation c. Applying petroleum jelly to the lips d. Assessment of the nares for irritation

ANS: C UAP education and scope of practice include patient hygiene such as oral care. The other actions require education and scope of practice appropriate to the RN.

A patient who received a corneal transplant 2 weeks ago calls the ophthalmology clinic to report that his vision has not improved with the transplant. Which action should the nurse take? a. Suggest the patient arrange a ride to the clinic immediately. b. Ask about the presence of "floaters" in the patient's visual field. c. Remind the patient it may take months to restore vision after transplant. d. Teach the patient to continue using prescribed pupil-dilating medications.

ANS: C Vision may not be restored for up to a year after corneal transplant. Because the patient is not experiencing complications of the surgery, an emergency clinic visit is not needed. Because "floaters" are not associated with complications of corneal transplant, the nurse will not need to ask the patient about their presence. Corticosteroid drops, not mydriatic drops, are used after corneal transplant surgery

A patient with diabetes who has bacterial pneumonia is being treated with IV gentamicin (Garamycin) 60 mg IV BID. The nurse will monitor for adverse effects of the medication by evaluating the patient's a. blood glucose. b. urine osmolality. c. serum creatinine. d. serum potassium.

ANS: C When a patient at risk for chronic kidney disease (CKD) receives a potentially nephrotoxic medication, it is important to monitor renal function with BUN and creatinine levels. The other laboratory values would not be useful in assessing for the adverse effects of the gentamicin

When caring for a preoperative patient on the day of surgery, which actions included in the plan of care can the nurse delegate to unlicensed assistive personnel (UAP)? (Select all that apply.) a. Teach incentive spirometer use. b. Explain preoperative routine care. c. Obtain and document baseline vital signs. d. Remove nail polish and apply pulse oximeter. e. Transport the patient by stretcher to the operating room.

ANS: C, D, E Obtaining vital signs, removing nail polish, pulse oximeter placement, and transport of the patient are routine skills that are appropriate to delegate. Teaching patients about the preoperative routine and incentive spirometer use require critical thinking and should be done by the registered nurse.

A patient admitted with possible stroke has been aphasic for 3 hours and his current blood pressure (BP) is 174/94 mm Hg. Which order by the health care provider should the nurse question? a. Keep head of bed elevated at least 30 degrees. b. Infuse normal saline intravenously at 75 mL/hr. c. Administer tissue plasminogen activator (tPA) per protocol. d. Administer a labetalol (Normodyne) drip to keep BP less than 140/90 mm Hg.

ANS: D Because elevated BP may be a protective response to maintain cerebral perfusion, antihypertensive therapy is recommended only if mean arterial pressure (MAP) is >130 mm Hg or systolic pressure is >220 mm Hg. Fluid intake should be 1500 to 2000 mL daily to maintain cerebral blood flow. The head of the bed should be elevated to at least 30 degrees, unless the patient has symptoms of poor tissue perfusion. tPA may be administered if the patient meets the other criteria for tPA use. DIF: Cognitive Level: Apply (application) REF: 1397 TOP: Nursing Process: Implementation MSC: NCLEX: Physiological Integrity

A 58-year-old patient with a left-brain stroke suddenly bursts into tears when family members visit. The nurse should a. use a calm voice to ask the patient to stop the crying behavior. b. explain to the family that depression is normal following a stroke. c. have the family members leave the patient alone for a few minutes. d. teach the family that emotional outbursts are common after strokes.

ANS: D Patients who have left-sided brain stroke are prone to emotional outbursts that are not necessarily related to the emotional state of the patient. Depression after a stroke is common, but the suddenness of the patient's outburst suggests that depression is not the major cause of the behavior. The family should stay with the patient. The crying is not within the patient's control and asking the patient to stop will lead to embarrassment. DIF: Cognitive Level: Apply (application) REF: 1409 TOP: Nursing Process: Implementation MSC: NCLEX: Psychosocial Integrity

An alcoholic and homeless patient is diagnosed with active tuberculosis (TB). Which intervention by the nurse will be most effective in ensuring adherence with the treatment regimen? a. Arrange for a friend to administer the medication on schedule. b. Give the patient written instructions about how to take the medications. c. Teach the patient about the high risk for infecting others unless treatment is followed. d. Arrange for a daily noon meal at a community center where the drug will be administered.

ANS: D Directly observed therapy is the most effective means for ensuring compliance with the treatment regimen, and arranging a daily meal will help ensure that the patient is available to receive the medication. The other nursing interventions may be appropriate for some patients but are not likely to be as helpful for this patient

34. Which question from the nurse would help determine if a patient's abdominal pain might indicate irritable bowel syndrome? a. "Have you been passing a lot of gas?" b. "What foods affect your bowel patterns?" c. "Do you have any abdominal distention?" d. "How long have you had abdominal pain?"

ANS: D One criterion for the diagnosis of irritable bowel syndrome (IBS) is the presence of abdominal discomfort or pain for at least 3 months. Abdominal distention, flatulence, and food intolerance are also associated with IBS, but are not diagnostic criteria.

11. Which abnormality on the skin of an older patient is the priority to discuss immediately with the health care provider? a. Several dry, scaly patches on the face b. Numerous varicosities noted on both legs c. Dilation of small blood vessels on the face d. Petechiae present on the chest and abdomen

ANS: D Petechiae are caused by pinpoint hemorrhages and are associated with a variety of serious disorders such as meningitis and coagulopathies. The nurse should contact the patients health care provider about this finding for further diagnostic follow-up. The other skin changes are associated with aging. Although the other changes will also require ongoing monitoring or intervention by the nurse, they do not indicate a need for urgent action. DIF: Cognitive Level: Apply (application) REF: 397 OBJ: Special Questions: Prioritization TOP: Nursing Process: Assessment

A patient received inhalation anesthesia during surgery. Postoperatively the nurse should monitor the patient for which complication? a. Tachypnea b. Myoclonus c. Hypertension d. Laryngospasm

ANS: D Possible complications of inhalation anesthetics include coughing, laryngospasm, and increased secretions. Hypertension and tachypnea are not associated with general anesthetics. Myoclonus may occur with nonbarbiturate hypnotics but not with the inhalation agents

7. A 27-year-old female patient is admitted to the hospital for evaluation of right lower quadrant abdominal pain with nausea and vomiting. Which action should the nurse take? a. Encourage the patient to sip clear liquids. b. Assess the abdomen for rebound tenderness. c. Assist the patient to cough and deep breathe. d. Apply an ice pack to the right lower quadrant.

ANS: D The patient's clinical manifestations are consistent with appendicitis, and application of an ice pack will decrease inflammation at the area. Checking for rebound tenderness frequently is unnecessary and uncomfortable for the patient. The patient should be NPO in case immediate surgery is needed. The patient will need to know how to cough and deep breathe postoperatively, but coughing will increase pain at this time.

10. A 60-year-old man who is hospitalized with an abdominal wound infection has only been eating about 50% of meals and states, "Nothing on the menu sounds good." Which action by the nurse will be most effective in improving the patient's oral intake? a. Order six small meals daily. b. Make a referral to the dietitian. c. Teach the patient about high-calorie foods. d. Have family members bring in favorite foods.

ANS: D The patient's statement that the hospital foods are unappealing indicates that favorite home-cooked foods might improve intake. The other interventions may also help improve the patient's intake, but the most effective action will be to offer the patient more appealing foods.

Which prescribed intervention will the nurse implement first for a patient in the emergency department who is experiencing continuous tonic-clonic seizures? a. Give phenytoin (Dilantin) 100 mg IV. b. Monitor level of consciousness (LOC). c. Obtain computed tomography (CT) scan. d. Administer lorazepam (Ativan) 4 mg IV.

ANS: D To prevent ongoing seizures, the nurse should administer rapidly acting antiseizure medications such as the benzodiazepines. A CT scan is appropriate, but prevention of any seizure activity during the CT scan is necessary. Phenytoin will also be administered, but it is not rapidly acting. Patients who are experiencing tonic-clonic seizures are nonresponsive, although the nurse should assess LOC after the seizure. DIF: Cognitive Level: Apply (application) REF: 1424 OBJ: Special Questions: Prioritization TOP: Nursing Process: Implementation MSC: NCLEX: Physiological Integrity

4. Which information will the nurse include in the asthma teaching plan for a patient being discharged? a. Use the inhaled corticosteroid when shortness of breath occurs. b. Inhale slowly and deeply when using the dry powder inhaler (DPI). c. Hold your breath for 5 seconds after using the bronchodilator inhaler. d. Tremors are an expected side effect of rapidly acting bronchodilators.

ANS: D Tremors are a common side effect of short-acting β2-adrenergic (SABA) medications and not a reason to avoid using the SABA inhaler. Inhaled corticosteroids do not act rapidly to reduce dyspnea. Rapid inhalation is needed when using a DPI. The patient should hold the breath for 10 seconds after using inhalers. DIF: Cognitive Level: Apply (application) REF: 572 TOP: Nursing Process: Implementation MSC: NCLEX: Physiological Integrity

A 50-year-old man vomiting blood-streaked fluid is admitted to the hospital with acute gastritis. To determine possible risk factors for gastritis, the nurse will ask the patient about a. the amount of saturated fat in the diet. b. any family history of gastric or colon cancer. c. a history of a large recent weight gain or loss. d. use of nonsteroidal antiinflammatory drugs (NSAIDs).

ANS: D Use of an NSAID is associated with damage to the gastric mucosa, which can result in acute gastritis. Family history, recent weight gain or loss, and fatty foods are not risk factors for acute gastritis

25. According to the Center for Disease Control (CDC) guidelines, which personal protective equipment will the nurse put on when assessing a patient who is on contact precautions for diarrhea caused by Clostridium difficile (select all that apply)? o Mask o Gown o Gloves o Shoe covers o Eye protection

ANS: Gown, Gloves

31. A patient with renal calculi is hospitalized with gross hematuria and severe colicky left flank pain. Which nursing action will be of highest priority at this time?

Administer prescribed analgesics.

A patient is admitted for hypovolemia associated with multiple draining wounds. Which assessment would be the most accurate way for the nurse to evaluate fluid balance? a. Skin turgor b. Daily weight c. Presence of edema d. Hourly urine output

Answer B

When caring for a patient with renal failure on a low phosphate diet, the nurse will inform unlicensed assistive personnel (UAP) to remove which food from the patient's food tray? a. Grape juice b. Milk carton c. Mixed green salad d. Fried chicken breast

Answer B Foods high in phosphate include milk and other dairy products, so these are restricted on low-phosphate diets. Green, leafy vegetables; high-fat foods; and fruits/juices are not high in phosphate and are not restricted.

The nurse notes the presence of white lesions that resemble milk curds in the back of a patients throat. Which question by the nurse is appropriate at this time?

Are you taking any medications at present? The appearance of the lesions is consistent with an oral candidiasis (thrush) infection, which can occur in patients who are taking medications such as immunosuppressants or antibiotics. Candidiasis is not associated with poor oral hygiene or lower respiratory infections. The lesions do not look like an oral herpes infection.

Which action should the nurse take when providing patient teaching to a 76-year-old with mild presbycusis?

Ask for permission to turn off the television before teaching

34. Following an open loop resection and fulguration of the bladder, a patient is unable to void. Which nursing action should be implemented first?

Assist the patient to take a 15-minute sitz bath.

16. A 78-year-old who has been admitted to the hospital with dehydration is confused and incontinent of urine. Which nursing action will be best to include in the plan of care?

Assist the patient to the bathroom every 2 hours during the day.

A 71-year-old male patient tells the nurse that growing old causes constipation so he has been using a suppository for constipation every morning. Which action should the nurse take first? a. Encourage the patient to increase oral fluid intake. b. Assess the patient about risk factors for constipation. c. Suggest that the patient increase intake of high-fiber foods. d. Teach the patient that a daily bowel movement is unnecessary.

B

A 74-year-old patient preparing to undergo a colon resection for cancer of the colon asks about the elevated carcinoembryonic antigen (CEA) test result. The nurse explains that the test is used to a. identify any metastasis of the cancer. b. monitor the tumor status after surgery. c. confirm the diagnosis of a specific type of cancer. d. determine the need for postoperative chemotherapy.

B

A new 19-year-old male patient has familial adenomatous polyposis (FAP). Which action will the nurse in the gastrointestinal clinic include in the plan of care? a. Obtain blood samples for DNA analysis. b. Schedule the patient for yearly colonoscopy. c. Provide preoperative teaching about total colectomy. d. Discuss lifestyle modifications to decrease cancer risk.

B

A nurse should include which instructions when teaching a patient with repeated hordeolum how to prevent further infection? a. Apply cold compresses at the first sign of recurrence. b. Discard all open or used cosmetics applied near the eyes. c. Wash the scalp and eyebrows with an antiseborrheic shampoo. d. Be examined for recurrent sexually transmitted infections (STIs).

B

A patient being admitted with an acute exacerbation of ulcerative colitis reports crampy abdominal pain and passing 15 or more bloody stools a day. The nurse will plan to a. administer IV metoclopramide (Reglan). b. discontinue the patient's oral food intake. c. administer cobalamin (vitamin B12) injections. d. teach the patient about total colectomy surgery.

B

A patient with glaucoma who has been using timolol (Timoptic) drops for several days tells the nurse that the eye drops cause eye burning and visual blurriness for a short time after administration. The best response to the patient's statement is a. "Those symptoms may indicate a need for an increased dosage of the eye drops." b. "The drops are uncomfortable, but it is important to use them to retain your vision." c. "These are normal side effects of the drug, which should be less noticeable with time." d. "Notify your health care provider so that different eye drops can be prescribed for you."

B

The nurse preparing for the annual physical exam of a 50-year-old man will plan to teach the patient about a. endoscopy. b. colonoscopy. c. computerized tomography screening. d. carcinoembryonic antigen (CEA) testing.

B

The nurse will determine that teaching a 67-year-old man to irrigate his new colostomy has been effective if the patient a. inserts the irrigation tubing 4 to 6 inches into the stoma. b. hangs the irrigating container 18 inches above the stoma. c. stops the irrigation and removes the irrigating cone if cramping occurs. d. fills the irrigating container with 1000 to 2000 mL of lukewarm tap water.

B

The occupational health nurse is caring for an employee who is complaining of bilateral eye pain after a cleaning solution splashed into the employee's eyes. Which action will the nurse take first? a. Apply ice packs to both eyes. b. Flush the eyes with sterile saline. c. Apply antiseptic ophthalmic ointment to the eyes. d. Cover the eyes with dry sterile patches and shields.

B

To determine whether treatment is effective for a patient with primary open-angle glaucoma (POAG), the nurse can evaluate the patient for improvement by a. questioning the patient about blurred vision. b. noting any changes in the patient's visual field. c. asking the patient to rate the pain using a 0 to 10 scale. d. assessing the patient's depth perception when climbing stairs.

B

Which information obtained by the nurse interviewing a 30-year-old male patient is most important to communicate to the health care provider? a. The patient has a history of constipation. b. The patient has noticed blood in the stools. c. The patient had an appendectomy at age 27. d. The patient smokes a pack/day of cigarettes.

B

Which nursing activity is appropriate for the registered nurse (RN) working in the eye clinic to delegate to experienced unlicensed assistive personnel (UAP)? a. Instilling antiviral drops for a patient with a corneal ulcer b. Application of a warm compress to a patient's hordeolum c. Instruction about hand washing for a patient with herpes keratitis d. Looking for eye irritation in a patient with possible conjunctivitis

B

Which patient statement indicates that the nurse's teaching about sulfasalazine (Azulfidine) for ulcerative colitis has been effective? a. "The medication will be tapered if I need surgery." b. "I will need to use a sunscreen when I am outdoors." c. "I will need to avoid contact with people who are sick." d. "The medication will prevent infections that cause the diarrhea."

B

Which prescribed intervention for a 61-year-old female patient with chronic short bowel syndrome will the nurse question? a. Ferrous sulfate (Feosol) 325 mg daily b. Senna (Senokot) 1 tablet every day c. Psyllium (Metamucil) 2.1 grams 3 times daily d. Diphenoxylate with atropine (Lomotil) prn loose stools

B

Which prescribed medication should the nurse give first to a patient who has just been admitted to a hospital with acute angle-closure glaucoma? a. Morphine sulfate 4 mg IV b. Mannitol (Osmitrol) 100 mg IV c. Betaxolol (Betoptic) 1 drop in each eye d. Acetazolamide (Diamox) 250 mg orally

B

Which statement by a patient with bacterial conjunctivitis indicates a need for further teaching? a. "I will wash my hands often during the day." b. "I will remove my contact lenses at bedtime." c. "I will not share towels with my friends or family." d. "I will monitor my family for eye redness or drainage."

B

Which assessment finding alerts the nurse to provide patient teaching about cataract development?

Blurred vision and light sensitivity

27. Which assessment data reported by a 28-year-old male patient is consistent with a lower urinary tract infection (UTI)?

Burning on urination

A 19-year-old female is brought to the emergency department with a knife handle protruding from the abdomen. During the initial assessment of the patient, the nurse should a. remove the knife and assess the wound. b. determine the presence of Rovsing sign. c. check for circulation and tissue perfusion. d. insert a urinary catheter and assess for hematuria.

C

A 26-year-old woman is being evaluated for vomiting and abdominal pain. Which question from the nurse will be most useful in determining the cause of the patient's symptoms? a. "What type of foods do you eat?" b. "Is it possible that you are pregnant?" c. "Can you tell me more about the pain?" d. "What is your usual elimination pattern?"

C

The nurse at the outpatient surgery unit obtains the following information about a patient who is scheduled for cataract extraction and implantation of an intraocular lens. Which information is most important to report to the health care provider at this time? a. The patient has had blurred vision for 3 years. b. The patient has not eaten anything for 8 hours. c. The patient takes 2 antihypertensive medications. d. The patient gets nauseated with general anesthesia.

C

The nurse is providing preoperative teaching for a 61-year-old man scheduled for an abdominal-perineal resection. Which information will the nurse include? a. Another surgery in 8 to 12 weeks will be used to create an ileal-anal reservoir. b. The patient will begin sitting in a chair at the bedside on the first postoperative day. c. The patient will drink polyethylene glycol lavage solution (GoLYTELY) preoperatively. d. IV antibiotics will be started at least 24 hours before surgery to reduce the bowel bacteria.

C

The nurse learns that a newly admitted patient has functional blindness and that the spouse has cared for the patient for many years. During the initial assessment of the patient, it is most important for the nurse to a. obtain more information about the cause of the patient's vision loss. b. obtain information from the spouse about the patient's special needs. c. make eye contact with the patient and ask about any need for assistance. d. perform an evaluation of the patient's visual acuity using a Snellen chart.

C

To decrease the risk for future hearing loss, which action should the nurse who is working with college students at the on-campus health clinic implement? a. Arrange to include otoscopic examinations for all patients. b. Administer influenza immunizations to all students at the clinic. c. Discuss the importance of limiting exposure to amplified music. d. Perform tympanometry on all patients between the ages of 18 to 24.

C

Which diet choice by the patient with an acute exacerbation of inflammatory bowel disease (IBD) indicates a need for more teaching? a. Scrambled eggs b. White toast and jam c. Oatmeal with cream d. Pancakes with syrup

C

31. The nurse is caring for a patient with a descending aortic dissection. Which assessment finding is most important to report to the health care provider? a. Weak pedal pulses b. Absent bowel sounds c. Blood pressure of 138/88 mm Hg d. 25 mL of urine output over the past hour

C The blood pressure is typically kept at less than 120 mm Hg systolic to minimize extension of the dissection. The nurse will need to notify the health care provider so that β-blockers or other antihypertensive drugs can be prescribed. The other findings are typical with aortic dissection and should also be reported but do not require immediate action. DIF: Cognitive Level: Analyze (analysis) REF: 815 OBJ: Special Questions: Prioritization TOP: Nursing Process: Assessment MSC: NCLEX: Physiological

The patient with chronic gastritis is being put on a combination of medications to eradicate H. pylori. Which drugs does the nurse know will probably be used for this patient? A) Antibiotic(s), antacid, and corticosteroid B) Antibiotic(s), aspirin, and antiulcer/protectant C) Antibiotic(s), proton pump inhibitor, and bismuth D) Antibiotic(s) and nonsteroidal antiinflammatory drugs (NSAIDs)

C) To eradicate H. pylori, a combination of antibiotics, a proton pump inhibitor, and possibly bismuth (for quadruple therapy) will be used. Corticosteroids, aspirin, and NSAIDs are drugs that can cause gastritis and do not affect H. pylori.

Which medication information will the nurse identify as a concern for a patient's musculoskeletal status? a. The patient takes a daily multivitamin and calcium supplement. b. The patient takes hormone therapy (HT) to prevent "hot flashes." c. The patient has severe asthma and requires frequent therapy with oral corticosteroids. d. The patient has migraine headaches treated with nonsteroidal antiinflammatory drugs (NSAIDs).

C. Frequent or chronic corticosteroid use may lead to skeletal problems such as avascular necrosis and osteoporosis. The use of HT and calcium supplements will help prevent osteoporosis. NSAID use does not increase the risk for musculoskeletal problems.

21. Which action will the nurse include in the plan of care for a patient who has had a ureterolithotomy and has a left ureteral catheter and a urethral catheter in place?

Call the health care provider if the ureteral catheter output drops suddenly.

The health care provider diagnoses impetigo in a patient who has crusty vesicopustular lesions on the lower face. Which instructions should the nurse include in the teaching plan?

Clean the infected areas with soap and water. The treatment for impetigo includes softening of the crusts with warm saline soaks and then soap-and-water removal. Alcohol-based cleansers and use of petroleum jelly are not recommended for impetigo. Antibiotic ointments, such as mupirocin (Bactroban), may be applied to the lesions.

When assessing a new patient at the outpatient clinic, the nurse notes dry, scaly skin; thin hair; and thick, brittle nails. What is the nurses best action?

Consult with the health care provider about the need for further diagnostic testing. The patient has clinical manifestations that could be caused by systemic problems such as malnutrition or hypothyroidism, so further diagnostic evaluation is indicated. Patient teaching about nutrition, addressing the patients dry skin, and referral to a podiatrist may also be needed, but the priority is to rule out underlying disease that may be causing these manifestations.

A 42-year-old male patient has had a herniorrhaphy to repair an incarcerated inguinal hernia. Which patient teaching will the nurse provide before discharge? a. Soak in sitz baths several times each day. b. Cough 5 times each hour for the next 48 hours. c. Avoid use of acetaminophen (Tylenol) for pain. d. Apply a scrotal support and ice to reduce swelling.

D

A 45-year-old patient is admitted to the emergency department with severe abdominal pain and rebound tenderness. Vital signs include temperature 102° F (38.3° C), pulse 120, respirations 32, and blood pressure (BP) 82/54. Which prescribed intervention should the nurse implement first? a. Administer IV ketorolac (Toradol) 15 mg. b. Draw blood for a complete blood count (CBC). c. Obtain a computed tomography (CT) scan of the abdomen. d. Infuse 1 liter of lactated Ringer's solution over 30 minutes.

D

A 47-year-old female patient is transferred from the recovery room to a surgical unit after a transverse colostomy. The nurse observes the stoma to be deep pink with edema and a small amount of sanguineous drainage. The nurse should a. place ice packs around the stoma. b. notify the surgeon about the stoma. c. monitor the stoma every 30 minutes. d. document stoma assessment findings.

D

A 64-year-old woman who has chronic constipation asks the nurse about the use of psyllium (Metamucil). Which information will the nurse include in the response? a. Absorption of fat-soluble vitamins may be reduced by fiber-containing laxatives. b. Dietary sources of fiber should be eliminated to prevent excessive gas formation. c. Use of this type of laxative to prevent constipation does not cause adverse effects. d. Large amounts of fluid should be taken to prevent impaction or bowel obstruction.

D

A 72-year-old patient with age-related macular degeneration (AMD) has just had photodynamic therapy. Which statement by the patient indicates that the discharge teaching has been effective? a. "I will need to use bright lights to read for at least the next week." b. "I will use drops to keep my pupils dilated until my appointment." c. "I will not use facial lotions near my eyes during the recovery period." d. "I will cover up with long-sleeved shirts and pants for the next 5 days."

D

After several days of antibiotic therapy, an older hospitalized patient develops watery diarrhea. Which action should the nurse take first? a. Notify the health care provider. b. Obtain a stool specimen for analysis. c. Teach the patient about handwashing. d. Place the patient on contact precautions.

D

In reviewing a 55-year-old patient's medical record, the nurse notes that the last eye examination revealed an intraocular pressure of 28 mm Hg. The nurse will plan to assess a. visual acuity. b. pupil reaction. c. color perception. d. peripheral vision.

D

The nurse is completing the admission database for a patient admitted with abdominal pain and notes a history of hypertension and glaucoma. Which prescribed medications should the nurse question? a. Morphine sulfate 4 mg IV b. Diazepam (Valium) 5 mg IV c. Betaxolol (Betoptic) 0.25% eyedrops d. Scopolamine patch (Transderm Scop) 1.5 mg

D

The nurse is working in an urgent care clinic that has standardized treatment protocols for implementation by nursing staff. After reviewing the history, physical assessment, and vital signs for a 60-year-old patient as shown in the accompanying figure, which action should the nurse take first? History: Type 2 diabetes x 5 years, mild hearing loss, sudden loss of left eye peripheral vision today Physical assessment: PERRLA, EOMs intact, cerumen obstructs view of tympanic membranes Vitals: Pulse 102, BP 146/90 (rt arm), Resp 24, Temp 97.9 a. Check the patient's blood glucose level. b. Take the blood pressure on the left arm. c. Use an irrigating syringe to clean the ear canals. d. Report the vision change to the health care provider.

D

The nurse will plan to teach a patient with Crohn's disease who has megaloblastic anemia about the need for a. oral ferrous sulfate tablets. b. regular blood transfusions. c. iron dextran (Imferon) infusions. d. cobalamin (B12) spray or injections.

D

When assisting a blind patient in ambulating to the bathroom, the nurse should a. take the patient by the arm and lead the patient slowly to the bathroom. b. have the patient place a hand on the nurse's shoulder and guide the patient. c. stay beside the patient and describe any obstacles on the path to the bathroom. d. walk slightly ahead of the patient and allow the patient to hold the nurse's elbow.

D

Which breakfast choice indicates a patient's good understanding of information about a diet for celiac disease? a. Oatmeal with nonfat milk b. Whole wheat toast with butter c. Bagel with low-fat cream cheese d. Corn tortilla with scrambled eggs

D

Which finding in an emergency department patient who reports being struck in the right eye with a fist is a priority for the nurse to communicate to the health care provider? a. The patient complains of a right-sided headache. b. The sclera on the right eye has broken blood vessels. c. The area around the right eye is bruised and tender to the touch. d. The patient complains of "a curtain" over part of the visual field.

D

Which information will the nurse include when teaching a patient with keratitis caused by herpes simplex type 1? a. Correct use of the antifungal eyedrops natamycin (Natacyn) b. How to apply corticosteroid ophthalmic ointment to the eyes c. Avoidance of nonsteroidal antiinflammatory drugs (NSAIDs) d. Importance of taking all of the ordered oral acyclovir (Zovirax)

D

32. A patient is being evaluated for postthrombotic syndrome. Which assessment will the nurse perform? a. Ask about leg pain with exercise. b. Determine the ankle-brachial index. c. Assess capillary refill in the patient's toes. d. Inspect for presence of lipodermatosclerosis.

D Clinical signs of postthrombotic syndrome include lipodermatosclerosis. In this situation, the skin on the lower leg becomes scarred, and the leg becomes tapered like an "inverted bottle." The other assessments would be done for patients with peripheral arterial disease. DIF: Cognitive Level: Apply (application) REF: 818 TOP: Nursing Process: Evaluation MSC: NCLEX: Physiological

Which finding is of highest priority when the nurse is planning care for a 77-year-old patient seen in the outpatient clinic? a. Symmetric joint swelling of fingers b. Decreased right knee range of motion c. Report of left hip aching when jogging d. History of recent loss of balance and fall

D. A history of falls requires further assessment and development of fall prevention strategies. The other changes are more typical of bone and joint changes associated with normal aging.

The nurse finds that a patient can flex the arms when no resistance is applied but is unable to flex when the nurse applies light resistance. The nurse should document the patient's muscle strength as level a. 0. b. 1. c. 2. d. 3.

D. A level 3 indicates that the patient is unable to move against resistance but can move against gravity. Level 1 indicates minimal muscle contraction, level 2 indicates that the arm can move when gravity is eliminated, and level 4 indicates active movement with some resistance.

29. A patient who is diagnosed with nephrotic syndrome has 3+ ankle and leg edema and ascites. Which nursing diagnosis is a priority for the patient?

Excess fluid volume related to low serum protein levels

38. A 26-year-old patient with a history of polycystic kidney disease is admitted to the surgical unit after having knee surgery. Which of the routine postoperative orders is most important for the nurse to discuss with the health care provider?

Give ketorolac (Toradol) 10 mg PO PRN for pain.

A patient is undergoing psoralen plus ultraviolet A light (PUVA) therapy for treatment of psoriasis. What action should the nurse take to prevent adverse effects from this procedure?

Have the patient use protective eyewear while receiving PUVA. The eyes should be shielded from UV light (UVL) during and after PUVA therapy to prevent the development of cataracts. The patient should be taught about the effects of UVL on unaffected skin, but lead-lined drapes, use of antiseptic soap, and petroleum jelly are not used to prevent skin damage.

The nurse recording health histories in the outpatient clinic would plan a focused hearing assessment for adult patients taking which medication?

Ibuprofen (Advil) taken for 20 years to treat osteoarthritis

A patient with atopic dermatitis has a new prescription for pimecrolimus (Elidel). After teaching the patient about the medication, which statement by the patient indicates that further teaching is needed?

If the medication burns when I apply it, I will wipe it off and call the doctor. The patient should be taught that transient burning at the application site is an expected effect of pimecrolimus and that the medication should be left in place. The other statements by the patient are accurate and indicate that patient teaching has been effective.

21. A 71-year-old patient who has benign prostatic hyperplasia (BPH) with urinary retention is admitted to the hospital with elevated blood urea nitrogen (BUN) and creatinine. Which prescribed therapy should the nurse implement first?

Insert a urinary retention catheter.

30. An 88-year-old with benign prostatic hyperplasia (BPH) has a markedly distended bladder and is agitated and confused. Which of the following interventions prescribed by the health care provider should the nurse implement first?

Insert a urinary retention catheter.

1. A patient has been diagnosed with urinary tract calculi that are high in uric acid. Which foods will the nurse teach the patient to avoid (select all that apply)?

LIVER AND CHICKEN

39. Which information noted by the nurse when caring for a patient with a bladder infection is most important to report to the health care provider?

Left-sided flank pain

23. After a transurethral resection of the prostate (TURP), a 64-year-old patient with continuous bladder irrigation complains of painful bladder spasms. The nurse observes clots in the urine. Which action should the nurse takefirst?

Manually instill and then withdraw 50 mL of saline into the catheter.

A nurse develops a teaching plan for a patient diagnosed with basal cell carcinoma (BCC). Which information should the nurse include in the teaching plan?

Minimizing sun exposure will reduce risk for future BCC. BCC is frequently associated with sun exposure and preventive measures should be taken for future sun exposure. BCC spreads locally, and does not metastasize to distant tissues. Since BCC can cause local tissue destruction, treatment is indicated. Local (not systemic) chemotherapy may be used to treat BCC.

A teenaged male patient who wrestles in high school is examined by the nurse in the clinic. Which assessment finding would prompt the nurse to teach the patient about the importance of not sharing headgear to prevent the spread of pediculosis?

Papular, wheal-like lesions with white deposits on the hair shaft Pediculosis is characterized by wheal-like lesions with parasites that attach eggs to the base of the hair shaft. The other descriptions are more characteristic of other types of skin disorders.

Which action can the nurse working in the emergency department delegate to experienced unlicensed assistive personnel (UAP)?

Perform Snellen testing of visual acuity for a patient with a history of cataracts.

19. A patient in the hospital has a history of functional urinary incontinence. Which nursing action will be included in the plan of care?

Place a bedside commode near the patient's bed.

When the patient turns his head quickly during the admission assessment, the nurse observes nystagmus. What is the indicated nursing action?

Place a fall-risk bracelet on the patient.

1.

On admission of a patient to the postanesthesia care unit (PACU), the blood pressure (BP) is 122/72. Thirty minutes after admission, the BP falls to 114/62, with a pulse of 74 and warm, dry skin. Which action by the nurse is most appropriate? a. Increase the IV fluid rate. b. Continue to take vital signs every 15 minutes. c. Administer oxygen therapy at 100% per mask. d. Notify the anesthesia care provider (ACP) immediately. ANS: B A slight drop in postoperative BP with a normal pulse and warm, dry skin indicates normal response to the residual effects of anesthesia and requires only ongoing monitoring. Hypotension with tachycardia and/or cool, clammy skin would suggest hypovolemic or hemorrhagic shock and the need for notification of the ACP, increased fluids, and high-concentration oxygen administration. DIF: Cognitive Level: Analyze (analysis) REF: 356 TOP: Nursing Process: Implementation MSC: NCLEX: Physiological Integrity 2. In the postanesthesia care unit (PACU), a patient's vital signs are blood pressure 116/72, pulse 74, respirations 12, and SpO2 91%. The patient is sleepy but awakens easily. Which action should the nurse take first? a. Place the patient in a side-lying position. b. Encourage the patient to take deep breaths. c. Prepare to transfer the patient to a clinical unit. d. Increase the rate of the postoperative IV fluids. ANS: B The patient's borderline SpO2 and sleepiness indicate hypoventilation. The nurse should stimulate the patient and remind the patient to take deep breaths. Placing the patient in a lateral position is needed when the patient first arrives in the PACU and is unconscious. The stable blood pressure and pulse indicate that no changes in fluid intake are required. The patient is not fully awake and has a low SpO2, indicating that transfer from the PACU to a clinical unit is not appropriate. DIF: Cognitive Level: Analyze (analysis) REF: 353-354 OBJ: Special Questions: Prioritization TOP: Nursing Process: Implementation MSC: NCLEX: Physiological Integrity 3. An experienced nurse orients a new nurse to the postanesthesia care unit (PACU). Which action by the new nurse, if observed by the experienced nurse, indicates that the orientation was successful? a. The new nurse assists a nauseated patient to a supine position. b. The new nurse positions an unconscious patient supine with the head elevated. c. The new nurse turns an unconscious patient to the side upon arrival in the PACU. d. The new nurse places a patient in the Trendelenburg position when the blood pressure drops. ANS: C The patient should initially be positioned in the lateral "recovery" position to keep the airway open and avoid aspiration. The Trendelenburg position is avoided because it increases the work of breathing. The patient is placed supine with the head elevated after regaining consciousness. DIF: Cognitive Level: Apply (application) REF: 354 TOP: Nursing Process: Evaluation MSC: NCLEX: Safe and Effective Care Environment 4. An older patient is being discharged from the ambulatory surgical unit following left eye surgery. The patient tells the nurse, "I do not know if I can take care of myself with this patch over my eye." Which action by the nurse is most appropriate? a. Refer the patient for home health care services. b. Discuss the specific concerns regarding self-care. c. Give the patient written instructions regarding care. d. Assess the patient's support system for care at home. ANS: B The nurse's initial action should be to assess exactly the patient's concerns about self-care. Referral to home health care and assessment of the patient's support system may be appropriate actions but will be based on further assessment of the patient's concerns. Written instructions should be given to the patient, but these are unlikely to address the patient's stated concern about self-care. DIF: Cognitive Level: Apply (application) REF: 362-363 TOP: Nursing Process: Implementation MSC: NCLEX: Physiological Integrity 5. The nasogastric (NG) tube is removed on the second postoperative day, and the patient is placed on a clear liquid diet. Four hours later, the patient complains of sharp, cramping gas pains. What action by the nurse is the most appropriate? a. Reinsert the NG tube. b. Give the PRN IV opioid. c. Assist the patient to ambulate. d. Place the patient on NPO status. ANS: C Ambulation encourages peristalsis and the passing of flatus, which will relieve the patient's discomfort. If distention persists, the patient may need to be placed on NPO status, but usually this is not necessary. Morphine administration will further decrease intestinal motility. Gas pains are usually caused by trapping of flatus in the colon, and reinsertion of the NG tube will not relieve the pains. DIF: Cognitive Level: Analyze (analysis) REF: 360 TOP: Nursing Process: Implementation MSC: NCLEX: Physiological Integrity 6. A patient's T-tube is draining dark green fluid after gallbladder surgery. What action by the nurse is the most appropriate? a. Notify the patient's surgeon. b. Place the patient on bed rest. c. Document the color and amount of drainage. d. Irrigate the T-tube with sterile normal saline. ANS: C A T-tube normally drains dark green to bright yellow drainage, so no action other than to document the amount and color of the drainage is needed. The other actions are not necessary. DIF: Cognitive Level: Apply (application) REF: 361 TOP: Nursing Process: Implementation MSC: NCLEX: Physiological Integrity 7. A nurse assists a patient on the first postoperative day to ambulate, cough, deep breathe, and turn. Which action by the nurse is most helpful? a. Teach the patient to fully exhale into the incentive spirometer. b. Administer ordered analgesic medications before these activities. c. Ask the patient to state two possible complications of immobility. d. Encourage the patient to state the purpose of splinting the incision. ANS: B An important nursing action to encourage these postoperative activities is administration of adequate analgesia to allow the patient to accomplish the activities with minimal pain. Even with motivation provided by proper teaching, positive reinforcement, and concern about complications, patients will have difficulty if there is a great deal of pain involved with these activities. When using an incentive spirometer, the patient should be taught to inhale deeply, rather than exhale into the spirometer to promote lung expansion and prevent atelectasis. DIF: Cognitive Level: Apply (application) REF: 358 TOP: Nursing Process: Implementation MSC: NCLEX: Physiological Integrity 8. A postoperative patient has a nursing diagnosis of ineffective airway clearance. The nurse determines that interventions for this nursing diagnosis have been successful if which is observed? a. Patient drinks 2 to 3 L of fluid in 24 hours. b. Patient uses the spirometer 10 times every hour. c. Patient's breath sounds are clear to auscultation. d. Patient's temperature is less than 100.4° F orally. ANS: C One characteristic of ineffective airway clearance is the presence of adventitious breath sounds such as rhonchi or crackles, so clear breath sounds are an indication of resolution of the problem. Spirometer use and increased fluid intake are interventions for ineffective airway clearance but may not improve breath sounds in all patients. Elevated temperature may occur with atelectasis, but a normal or near-normal temperature does not always indicate resolution of respiratory problems. DIF: Cognitive Level: Apply (application) REF: 353 TOP: Nursing Process: Evaluation MSC: NCLEX: Physiological Integrity 9. A patient who has begun to awaken after 30 minutes in the postanesthesia care unit (PACU) is restless and shouting at the nurse. The patient's oxygen saturation is 96%, and recent laboratory results are all normal. Which action by the nurse is most appropriate? a. Increase the IV fluid rate. b. Assess for bladder distention. c. Notify the anesthesia care provider (ACP). d. Demonstrate the use of the nurse call bell button. ANS: B Because the patient's assessment indicates physiologic stability, the most likely cause of the patient's agitation is emergence delirium, which will resolve as the patient wakes up more fully. The nurse should look for a cause such as bladder distention. Although hypoxemia is the most common cause, the patient's oxygen saturation is 96%. Emergence delirium is common in patients recovering from anesthesia, so there is no need to notify the ACP. Orientation of the patient to bed controls is needed, but is not likely to be effective until the effects of anesthesia have resolved more completely. DIF: Cognitive Level: Analyze (analysis) REF: 357 TOP: Nursing Process: Implementation MSC: NCLEX: Physiological Integrity 10. Which action could the postanesthesia care unit (PACU) nurse delegate to unlicensed assistive personnel (UAP) who help with the transfer of a patient to the clinical unit? a. Clarify the postoperative orders with the surgeon. b. Help with the transfer of the patient onto a stretcher. c. Document the appearance of the patient's incision in the chart. d. Provide hand off communication to the surgical unit charge nurse. ANS: B The scope of practice of UAP includes repositioning and moving patients under the supervision of a nurse. Providing report to another nurse, assessing and documenting the wound appearance, and clarifying physician orders with another nurse require registered-nurse (RN) level education and scope of practice. DIF: Cognitive Level: Apply (application) REF: 354 OBJ: Special Questions: Delegation TOP: Nursing Process: Planning MSC: NCLEX: Safe and Effective Care Environment 11. A patient is transferred from the postanesthesia care unit (PACU) to the clinical unit. Which action by the nurse on the clinical unit should be performed first? a. Assess the patient's pain. b. Orient the patient to the unit. c. Take the patient's vital signs. d. Read the postoperative orders. ANS: C Because the priority concerns after surgery are airway, breathing, and circulation, the vital signs are assessed first. The other actions should take place after the vital signs are obtained and compared with the vital signs before transfer. DIF: Cognitive Level: Apply (application) REF: 350 OBJ: Special Questions: Prioritization TOP: Nursing Process: Implementation MSC: NCLEX: Physiological Integrity 12. An older patient who had knee replacement surgery 2 days ago can only tolerate being out of bed with physical therapy twice a day. Which collaborative problem should the nurse identify as a priority for this patient? a. Potential complication: hypovolemic shock b. Potential complication: venous thromboembolism c. Potential complication: fluid and electrolyte imbalance d. Potential complication: impaired surgical wound healing ANS: B The patient is older and relatively immobile, which are two risk factors for development of deep vein thrombosis. The other potential complications are possible postoperative problems, but they are not supported by the data about this patient. DIF: Cognitive Level: Apply (application) REF: 356 OBJ: Special Questions: Prioritization TOP: Nursing Process: Diagnosis MSC: NCLEX: Physiological Integrity 13. A patient who is just waking up after having hip replacement surgery is agitated and confused. Which action should the nurse take first? a. Administer the ordered opioid. b. Check the oxygen (O2) saturation. c. Take the blood pressure and pulse. d. Apply wrist restraints to secure IV lines. ANS: B Emergence delirium may be caused by a variety of factors. However, the nurse should first assess for hypoxemia. The other actions also may be appropriate, but are not the best initial action. DIF: Cognitive Level: Apply (application) REF: 357 OBJ: Special Questions: Prioritization TOP: Nursing Process: Implementation MSC: NCLEX: Physiological Integrity 14. A postoperative patient has not voided for 8 hours after return to the clinical unit. Which action should the nurse take first? a. Perform a bladder scan. b. Encourage increased oral fluid intake. c. Assist the patient to ambulate to the bathroom. d. Insert a straight catheter as indicated on the PRN order. ANS: A The initial action should be to assess the bladder for distention. If the bladder is distended, providing the patient with privacy (by walking with them to the bathroom) will be helpful. Because of the risk for urinary tract infection, catheterization should only be done after other measures have been tried without success. There is no indication to notify the surgeon about this common postoperative problem unless all measures to empty the bladder are unsuccessful. DIF: Cognitive Level: Apply (application) REF: 360-361 OBJ: Special Questions: Prioritization TOP: Nursing Process: Implementation MSC: NCLEX: Physiological Integrity 15. The nurse is caring for a patient the first postoperative day following a laparotomy for a small bowel obstruction. The nurse notices new bright-red drainage about 5 cm in diameter on the dressing. Which action should the nurse take first? a. Reinforce the dressing. b. Apply an abdominal binder. c. Take the patient's vital signs. d. Recheck the dressing in 1 hour for increased drainage. ANS: C New bright-red drainage may indicate hemorrhage, and the nurse should initially assess the patient's vital signs for tachycardia and hypotension. The surgeon should then be notified of the drainage and the vital signs. The dressing may be changed or reinforced, based on the surgeon's orders or institutional policy. The nurse should not wait an hour to recheck the dressing. DIF: Cognitive Level: Apply (application) REF: 355 OBJ: Special Questions: Prioritization TOP: Nursing Process: Implementation MSC: NCLEX: Physiological Integrity 16. When caring for a patient the second postoperative day after abdominal surgery for removal of a large pancreatic cyst, the nurse obtains an oral temperature of 100.8° F. Which action should the nurse take first? a. Have the patient use the incentive spirometer. b. Assess the surgical incision for redness and swelling. c. Administer the ordered PRN acetaminophen (Tylenol). d. Ask the health care provider to prescribe a different antibiotic. ANS: A A temperature of 100.8° F in the first 48 hours is usually caused by atelectasis, and the nurse should have the patient cough and deep breathe. This problem may be resolved by nursing intervention, and therefore notifying the health care provider is not necessary. Acetaminophen will reduce the temperature, but it will not resolve the underlying respiratory congestion. Because a wound infection does not usually occur before the third postoperative day, a wound infection is not a likely source of the elevated temperature. DIF: Cognitive Level: Apply (application) REF: 359 OBJ: Special Questions: Prioritization TOP: Nursing Process: Implementation MSC: NCLEX: Physiological Integrity 17. The nurse assesses that the oxygen saturation is 89% in an unconscious patient who was transferred from surgery to the postanesthesia care unit (PACU) 15 minutes ago. Which action should the nurse take first? a. Elevate the patient's head. b. Suction the patient's mouth. c. Increase the oxygen flow rate. d. Perform the jaw-thrust maneuver. ANS: D In an unconscious postoperative patient, a likely cause of hypoxemia is airway obstruction by the tongue, and the first action is to clear the airway by maneuvers such as the jaw thrust or chin lift. Increasing the oxygen flow rate and suctioning are not helpful when the airway is obstructed by the tongue. Elevating the patient's head will not be effective in correcting the obstruction but may help with oxygenation after the patient is awake. DIF: Cognitive Level: Apply (application) REF: 351 | 352 | 323 OBJ: Special Questions: Prioritization TOP: Nursing Process: Implementation MSC: NCLEX: Physiological Integrity 18. The nurse assesses a patient who had a total abdominal hysterectomy 2 days ago. Which information about the patient is most important to communicate to the health care provider? a. The right calf is swollen, warm, and painful. b. The patient's temperature is 100.3° F (37.9° C). c. The 24-hour oral intake is 600 mL greater than the total output. d. The patient complains of abdominal pain at level 6 (0 to 10 scale) when ambulating. ANS: A The calf pain, swelling, and warmth suggest that the patient has a deep vein thrombosis, which will require the health care provider to order diagnostic tests and/or anticoagulants. Because the stress response causes fluid retention for the first 2 to 5 days postoperatively, the difference between intake and output is expected. A temperature elevation to 100.3° F on the second postoperative day suggests atelectasis, and the nurse should have the patient deep breathe and cough. Pain with ambulation is normal, and the nurse should administer the ordered analgesic before patient activities. DIF: Cognitive Level: Apply (application) REF: 363 OBJ: Special Questions: Prioritization TOP: Nursing Process: Assessment MSC: NCLEX: Physiological Integrity 19. A patient who had knee surgery received intramuscular ketorolac (Toradol) 30 minutes ago and continues to complain of pain at a level of 7 (0 to 10 scale). Which action is best for the nurse to take at this time? a. Administer the prescribed PRN IV morphine sulfate. b. Notify the health care provider about the ongoing knee pain. c. Reassure the patient that postoperative pain is expected after knee surgery. d. Teach the patient that the effects of ketorolac typically last about 6 to 8 hours. ANS: A The priority at this time is pain relief. Concomitant use of opioids and nonsteroidal antiinflammatory drugs (NSAIDs) improves pain control in postoperative patients. Patient teaching and reassurance are appropriate, but should be done after the patient's pain is relieved. If the patient continues to have pain after the morphine is administered, the health care provider should be notified. DIF: Cognitive Level: Apply (application) REF: 358 OBJ: Special Questions: Prioritization TOP: Nursing Process: Implementation MSC: NCLEX: Safe and Effective Care Environment 20. The nurse working in the postanesthesia care unit (PACU) notes that a patient who has just been transported from the operating room is shivering and has a temperature of 96.5° F (35.8° C). Which action should the nurse take? a. Cover the patient with a warm blanket and put on socks. b. Notify the anesthesia care provider about the temperature. c. Avoid the use of opioid analgesics until the patient is warmer. d. Administer acetaminophen (Tylenol) 650 mg suppository rectally. ANS: A The patient assessment indicates the need for active rewarming. There is no indication of a need for acetaminophen. Opioid analgesics may help reduce shivering. Because hypothermia is common in the immediate postoperative period, there is no need to notify the anesthesia care provider, unless the patient continues to be hypothermic after active rewarming. DIF: Cognitive Level: Apply (application) REF: 359 TOP: Nursing Process: Implementation MSC: NCLEX: Physiological Integrity 21. The nurse reviews the laboratory results for a patient on the first postoperative day after a hiatal hernia repair. Which finding would indicate to the nurse that the patient is at increased risk for poor wound healing? a. Potassium 3.5 mEq/L b. Albumin level 2.2 g/dL c. Hemoglobin 11.2 g/dL d. White blood cells 11,900/µL ANS: B Because proteins are needed for an appropriate inflammatory response and wound healing, the low serum albumin level (normal level 3.5 to 5.0 g/dL) indicates a risk for poor wound healing. The potassium level is normal. Because a small amount of blood loss is expected with surgery, the hemoglobin level is not indicative of an increased risk for wound healing. WBC count is expected to increase after surgery as a part of the normal inflammatory response. DIF: Cognitive Level: Apply (application) REF: 173 TOP: Nursing Process: Assessment MSC: NCLEX: Physiological Integrity 22. The nurse assesses a patient on the second postoperative day after abdominal surgery to repair a perforated duodenal ulcer. Which finding is most important for the nurse to report to the surgeon? a. Tympanic temperature 99.2° F (37.3° C) b. Fine crackles audible at both lung bases c. Redness and swelling along the suture line d. 200 mL sanguineous fluid in the wound drain ANS: D Wound drainage should decrease and change in color from sanguineous to serosanguineous by the second postoperative day. The color and amount of drainage for this patient are abnormal and should be reported. Redness and swelling along the suture line and a slightly elevated temperature are normal signs of postoperative inflammation. Atelectasis is common after surgery. The nurse should have the patient cough and deep breathe, but there is no urgent need to notify the surgeon. DIF: Cognitive Level: Apply (application) REF: 361 OBJ: Special Questions: Prioritization TOP: Nursing Process: Assessment MSC: NCLEX: Safe and Effective Care Environment 23. After receiving change-of-shift report about these postoperative patients, which patient should the nurse assess first? a. Obese patient who had abdominal surgery 3 days ago and whose wound edges are separating b. Patient who has 30 mL of sanguineous drainage in the wound drain 10 hours after hip replacement surgery c. Patient who has bibasilar crackles and a temperature of 100°F (37.8°C) on the first postoperative day after chest surgery d. Patient who continues to have incisional pain 15 minutes after hydrocodone and acetaminophen (Vicodin) administration ANS: A The patient's history and assessment suggests possible wound dehiscence, which should be reported immediately to the surgeon. Although the information about the other patients indicates a need for ongoing assessment and/or possible intervention, the data do not suggest any acute complications. Small amounts of red drainage are common in the first postoperative hours. Bibasilar crackles and a slightly elevated temperature are common after surgery, although the nurse will need to have the patient cough and deep breathe. Oral medications typically take more than 15 minutes for effective pain relief. DIF: Cognitive Level: Analyze (analysis) REF: 361 OBJ: Special Questions: Prioritization TOP: Nursing Process: Assessment MSC: NCLEX: Safe and Effective Care Environment OTHER 1. While ambulating in the room, a patient complains of feeling dizzy. In what order will the nurse accomplish the following activities? (Put a comma and a space between each answer choice [A, B, C, D].) a. Have the patient sit down in a chair. b. Give the patient something to drink. c. Take the patient's blood pressure (BP). d. Notify the patient's health care provider. ANS: A, C, B, D The first priority for the patient with syncope is to prevent a fall, so the patient should be assisted to a chair. Assessment of the BP will determine whether the dizziness is due to orthostatic hypotension, which occurs because of hypovolemia. Increasing the fluid intake will help prevent orthostatic dizziness. Because this is a common postoperative problem that is usually resolved through nursing measures such as increasing fluid intake and making position changes more slowly, there is no urgent need to notify the health care provider. DIF: Cognitive Level: Apply (application) REF: 357 OBJ: Special Questions: Prioritization TOP: Nursing Process: Implementation MSC: NCLEX: Physiological Integrity 2. A patient's blood pressure in the postanesthesia care unit (PACU) has dropped from an admission blood pressure of 140/86 to 102/60 with a pulse change of 70 to 96. SpO2 is 92% on 3 L of oxygen. In which order should the nurse take these actions? (Put a comma and a space between each answer choice [A, B, C, D].) a. Increase the IV infusion rate. b. Assess the patient's dressing. c. Increase the oxygen flow rate. d. Check the patient's temperature. ANS: A, C, B, D The first nursing action should be to increase the IV infusion rate. Because the most common cause of hypotension is volume loss, the IV rate should be increased. The next action should be to increase the oxygen flow rate to maximize oxygenation of hypoperfused organs. Because hemorrhage is a common cause of postoperative volume loss, the nurse should check the dressing. Finally, the patient's temperature should be assessed to determine the effects of vasodilation caused by rewarming. DIF: Cognitive Level: Analyze (analysis) REF: 355-356 OBJ: Special Questions: Prioritization TOP: Nursing Process: Implementation MSC: NCLEX: Physiological Integrity

A patient in the dermatology clinic has a thin, scaly erythematous plaque on the right cheek. Which action should the nurse take?

Prepare the patient for a biopsy. Because the appearance of the lesion suggests actinic keratosis or possible squamous cell carcinoma (SCC), the appropriate treatment would be excision and biopsy. Over-the-counter (OTC) corticosteroids, topical antibiotics, and Retin-A would not be used for this lesion.

34. When caring for a patient with continuous bladder irrigation after having transurethral resection of the prostate, which action could the nurse delegate to unlicensed assistive personnel (UAP)?

Report any complaints of pain or spasms to the nurse.

A patient who underwent eye surgery is required to wear an eye patch until the scheduled postoperative clinic visit. Which nursing diagnosis will the nurse include in the plan of care?

Risk for falls related to temporary decrease in stereoscopic vision

A patient with an enlarging, irregular mole that is 7 mm in diameter is scheduled for outpatient treatment. The nurse should plan to prepare the patient for which procedure?

Surgical excision The description of the mole is consistent with malignancy, so excision and biopsy are indicated. Curettage and cryosurgery are not used if malignancy is suspected. A punch biopsy would not be done for a lesion greater than 5 mm in diameter.

33. Which action by the unlicensed assistive personnel (UAP) who are assisting with the care of patients with male reproductive problems indicates that the nurse should provide more teaching?

The UAP leave the foreskin pulled back after cleaning the glans of a patient who has a retention catheter.

The nurse instructs a patient about application of corticosteroid cream to an area of contact dermatitis on the right leg. Which patient action indicates that further teaching is needed?

The patient applies a thick layer of the cream to the affected skin. Creams and ointments should be applied in a thin layer to avoid wasting the medication. The other actions by the patient indicate that the teaching has been successful.

24. A patient who has had a transurethral resection with fulguration for bladder cancer 3 days previously calls the nurse at the urology clinic. Which information given by the patient is most important to report to the health care provider?

The patient has noticed clots in the urine.

The nurse is interviewing a patient with contact dermatitis. Which finding indicates a need for patient teaching?

The patient uses Neosporin ointment on minor cuts or abrasions. Neosporin can cause contact dermatitis. The other medications are being used appropriately by the patient.

A patient has the following risk factors for melanoma. Which risk factor should the nurse assign as the priority focus of patient teaching?

The patient uses a tanning booth throughout the winter. Because the only risk factor that the patient can change is the use of a tanning booth, the nurse should focus teaching about melanoma prevention on this factor. The other factors also will contribute to increased risk for melanoma.

The nurse assesses a patient who has just arrived in the postanesthesia recovery area (PACU) after a blepharoplasty. Which assessment data should be reported to the surgeon immediately?

The skin around the incision is pale and cold when palpated Pale, cool skin indicates a possible decrease in circulation, so the surgeon should be notified immediately. The other assessment data indicate a need for ongoing assessment or nursing action. A heart rate of 110 beats/minute may be related to the stress associated with surgery. Assessment of other vital signs and continued monitoring are appropriate. Because local anesthesia would be used for the procedure, numbness of the incisional area is expected immediately after surgery. The nurse should monitor for return of feeling.

The nurse should report which assessment finding immediately to the health care provider?

The tympanum is blue-tinged.

A patient with atopic dermatitis has been using a high-potency topical corticosteroid ointment for several weeks. The nurse should assess for which adverse effect?

Thinning of the affected skin Thinning of the skin indicates that atrophy, a possible adverse effect of topical corticosteroids, is occurring. The health care provider should be notified so that the medication can be changed or tapered. Alopecia, red-brown discoloration, and dryness/scaling of the skin are not adverse effects of topical corticosteroid use.

Which equipment will the nurse obtain to perform a Rinne test?

Tuning fork

Which information should the nurse include when teaching a patient who has just received a prescription for ciprofloxacin (Cipro) to treat a urinary tract infection?

Use a sunscreen with a high SPF when exposed to the sun. The patient should stay out of the sun. If that is not possible, teach them to wear sunscreen when taking medications that can cause photosensitivity. The other statements are not accurate.

The nurse in the eye clinic is examining a 67-year-old patient who says "I see small spots that move around in front of my eyes." Which action will the nurse take first?

Use an ophthalmoscope to examine the posterior eye chambers.

Which information will the nurse include when teaching an older patient about skin care?

Use warm water and a moisturizing soap when bathing. Warm water and moisturizing soap will avoid overdrying the skin. Because older patients have dryer skin, daily bathing and shampooing are not necessary and may dry the skin unnecessarily. Antibacterial soaps are not necessary. Lotions should be applied while the skin is still damp to seal moisture in.

What is the best method to prevent the spread of infection when the nurse is changing the dressing over a wound infected with Staphylococcus aureus?

Wash hands and properly dispose of soiled dressings. Careful hand washing and the safe disposal of soiled dressings are the best means of preventing the spread of skin problems. Sterile glove and sterile saline use during wound care will not necessarily prevent spread of infection. Applying antibiotic ointment will treat the bacteria but not necessarily prevent the spread of infection.

The health care provider prescribes topical 5-FU for a patient with actinic keratosis on the left cheek. The nurse should include which statement in the patients instructions?

Your cheek area will be painful and develop eroded areas that will take weeks to heal. Topical 5-FU causes an initial reaction of erythema, itching, and erosion that lasts 4 weeks after application of the medication is stopped. The medication is topical, so there are no systemic effects such as increased infection risk, anorexia, or nausea.

The nurse assesses a patient who has numerous petechiae on both arms. Which question should the nurse ask the patient?

a. "Do you take salicylates?" Salicylates interfere with platelet function and can lead to petechiae and ecchymoses.

10. A 70-year-old patient who has had a transurethral resection of the prostate (TURP) for benign prostatic hyperplasia (BPH) is being discharged from the hospital today, The nurse determines that additional instruction is needed when the patient says which of the following?

a. "I should call the doctor if I have incontinence at home."

An appropriate nursing intervention for a patient with non-Hodgkin's lymphoma whose platelet count drops to 18,000/µL during chemotherapy is to

a. check all stools for occult blood. Because the patient is at risk for spontaneous bleeding, the nurse should check stools for occult blood.

9. A 56-year-old female patient is admitted to the hospital with new onset nephrotic syndrome. Which assessment data will the nurse expect?

b. Recent weight gain

Which patient requires the most rapid assessment and care by the emergency department nurse?

b. The patient with neutropenia who has a temperature of 101.8° F A neutropenic patient with a fever is assumed to have an infection and is at risk for rapidly developing sepsis.

The nurse notes scleral jaundice in a patient being admitted with hemolytic anemia. The nurse will plan to check the laboratory results for the

b. bilirubin level. Jaundice is caused by the elevation of bilirubin level associated with red blood cell (RBC) hemolysis.

It is important for the nurse providing care for a patient with sickle cell crisis to

b. evaluate the effectiveness of opioid analgesics. Pain is the most common clinical manifestation of a crisis and usually requires large doses of continuous opioids for control.

9. A 57-year-old patient is incontinent of urine following a radical retropubic prostatectomy. The nurse will plan to teach the patient

b. pelvic floor muscle exercises.

6. It is most important that the nurse ask a patient admitted with acute glomerulonephritis about

b. recent sore throat and fever.

13. The nurse performing a focused examination to determine possible causes of infertility will assess for

b. varicocele.

12. Which information will the nurse teach a patient who has chronic prostatitis?

c. Intercourse or masturbation will help relieve symptoms.

7. Which finding for a patient admitted with glomerulonephritis indicates to the nurse that treatment has been effective?

d. The periorbital and peripheral edema is resolved.

A 62-year old man with chronic anemia is experiencing increased fatigue and occasional palpitations at rest. The nurse would expect the patient's laboratory findings to include

d. a hemoglobin (Hgb) of 8.6 g/dL (86 g/L). The patient's clinical manifestations indicate moderate anemia, which is consistent with a Hgb of 6 to 10 g/dL.

1.To determine the severity of the symptoms for a 68-year-old patient with benign prostatic hyperplasia (BPH) the nurse will ask the patient about

d. force of the urinary stream.

A critical action by the nurse caring for a patient with an acute exacerbation of polycythemia vera is to

d. monitor fluid intake and output. Monitoring hydration status is important during an acute exacerbation because the patient is at risk for fluid overload or underhydration.

4. The nurse will anticipate that a 61-year-old patient who has an enlarged prostate detected by digital rectal examination (DRE) and an elevated prostate specific antigen (PSA) level will need teaching about

d. transrectal ultrasonography (TRUS).

The charge nurse must intervene immediately if observing a nurse who is caring for a patient with vestibular disease

encouraging the patient to ambulate independently.

11. The nurse will inform a patient with cancer of the prostate that side effects of leuprolide (Lupron) may include

flushing

14. When assessing a 30-year-old man who complains of a feeling of incomplete bladder emptying and a split, spraying urine stream, the nurse asks about a history of

gonococcal urethritis.

28. A 76-year-old patient who has been diagnosed with stage 2 prostate cancer chooses the option of active surveillance. The nurse will plan to

schedule the patient for annual prostate-specific antigen testing.

When assessing a patient's consensual pupil response, the nurse should

shine a light into one pupil and observe the response of both pupils.

The nurse is testing the visual acuity of a patient in the outpatient clinic. The nurse's instructions for this test include asking the patient to

stand 20 feet from the wall chart.

25. A patient with bladder cancer is scheduled for intravesical chemotherapy. In preparation for the treatment the nurse will teach the patient about

the need to empty the bladder before treatment.

The nurse performing an eye examination will document normal findings for accommodation when

the pupils constrict while fixating on an object being moved closer to the patient's eyes.

19. The nurse taking a focused health history for a patient with possible testicular cancer will ask the patient about a history of

undescended testicles.

15. After obtaining the health history for a 25-year-old who smokes two packs of cigarettes daily, the nurse will plan to do teaching about the increased risk for

bladder cancer.

When a patient with splenomegaly is scheduled for splenectomy, which action will the nurse include in the preoperative plan of care?

c. Schedule immunization with the pneumococcal vaccine (Pneumovax). Asplenic patients are at high risk for infection with Pneumococcus and immunization reduces this risk.

8. The nurse will anticipate teaching a patient with nephrotic syndrome who develops flank pain about treatment with

c. anticoagulants.

A 54-year-old woman with acute myelogenous leukemia (AML) is considering treatment with a hematopoietic stem cell transplant (HSCT). The best approach for the nurse to assist the patient with a treatment decision is to

c. ask the patient whether there are any questions or concerns about HSCT. Offering the patient an opportunity to ask questions or discuss concerns about HSCT will encourage the patient to voice concerns about this treatment

A 52-year-old patient has a new diagnosis of pernicious anemia. The nurse determines that the patient understands the teaching about the disorder when the patient states, "I

c. could choose nasal spray rather than injections of vitamin B12." Because pernicious anemia prevents the absorption of vitamin B12, this patient requires injections or intranasal administration of cobalamin.

8. A patient returning from surgery for a perineal radical prostatectomy will have a nursing diagnosis of risk for infection related to

c. possible fecal wound contamination.

7. A 53-year-old man is scheduled for an annual physical exam. The nurse will plan to teach the patient about the purpose of

c. prostate specific antigen (PSA) testing.

The nurse is observing a student who is preparing to perform an ear examination for a 30-year-old patient. The nurse will need to intervene if the student

chooses a speculum larger than the ear canal.

20. The nurse will plan to provide teaching for a 67-year-old patient who has been diagnosed with orchitis about

pain management.

31. When obtaining the pertinent health history for a man who is being evaluated for infertility, which question is most important for the nurse to ask?

"Do you use medications to improve muscle mass?"

18. A patient with urinary obstruction from benign prostatic hyperplasia (BPH) tells the nurse, "My symptoms are much worse this week." Which response by the nurse is most appropriate?

"Have you been taking any over-the-counter (OTC) medications recently?"

When the nurse is taking a health history of a new patient at the ear clinic, the patient states, "I have to sleep with the television on." Which follow-up question is most appropriate to obtain more information about possible hearing problems?

"Have you noticed ringing in your ears?"

The nurse is assessing a 65-year-old patient for presbyopia. Which instruction will the nurse give the patient before the test?

"Hold this card and read the print out loud."

When obtaining a health history from a 49-year-old patient, which patient statement is most important to communicate to the primary health care provider?

"I can't see as far over to the side."

A 65-year-old patient is being evaluated for glaucoma. Which information given by the patient has implications for the patient's treatment?

"I take metoprolol (Lopressor) daily for angina."

20. After the home health nurse teaches a patient with a neurogenic bladder how to use intermittent catheterization for bladder emptying, which patient statement indicates that the teaching has been effective?

"I will clean the catheter carefully before and after each catheterization."

Which information will the nurse provide to the patient scheduled for refractometry?

"You will need to wear sunglasses for a few hours after the exam."

A patient is to receive an infusion of 250 mL of platelets over 2 hours through tubing that is labeled: 1 mL equals 10 drops. How many drops per minute will the nurse infuse?

21 To infuse 250 mL over 2 hours, the calculated drip rate is 20.8 drops/minute or 21 drops/minute.

30. Several patients call the urology clinic requesting appointments with the health care provider as soon as possible. Which patient will the nurse schedule to be seen first?

22-year-old who has noticed a firm, nontender lump on his scrotum

A 42-year-old woman with Ménière's disease is admitted with vertigo, nausea, and vomiting. Which nursing intervention will be included in the care plan? a. Dim the lights in the patient's room. b. Encourage increased oral fluid intake. c. Change the patient's position every 2 hours. d. Keep the head of the bed elevated 30 degrees.

A

A 50-year-old female patient calls the clinic to report a new onset of severe diarrhea. The nurse anticipates that the patient will need to a. collect a stool specimen. b. prepare for colonoscopy. c. schedule a barium enema. d. have blood cultures drawn.

A

A 51-year-old male patient has a new diagnosis of Crohn's disease after having frequent diarrhea and a weight loss of 10 pounds (4.5 kg) over 2 months. The nurse will plan to teach about a. medication use. b. fluid restriction. c. enteral nutrition. d. activity restrictions.

A

A 51-year-old woman with Crohn's disease who is taking infliximab (Remicade) calls the nurse in the outpatient clinic about new symptoms. Which symptom is most important to communicate to the health care provider? a. Fever b. Nausea c. Joint pain d. Headache

A

A 75-year-old patient with presbycusis is fitted with binaural hearing aids. Which information will the nurse include when teaching the patient how to use the hearing aids? a. Experiment with volume and hearing ability in a quiet environment initially. b. Keep the volume low on the hearing aids for the first week while adjusting to them. c. Add a second hearing aid after making the initial adjustment to the first hearing aid. d. Wear the hearing aids for about an hour a day at first, gradually increasing the time of use.

A

A female patient is awaiting surgery for acute peritonitis. Which action will the nurse include in the plan of care? a. Position patient with the knees flexed. b. Avoid use of opioids or sedative drugs. c. Offer frequent small sips of clear liquids. d. Assist patient to breathe deeply and cough.

A

The nurse is developing a plan of care for an adult patient diagnosed with adult inclusion conjunctivitis (AIC) caused by Chlamydia trachomatis. Which action should be included in the plan of care? a. Discussing the need for sexually transmitted infection testing b. Applying topical corticosteroids to prevent further inflammation c. Assisting with applying for community visual rehabilitation services d. Educating about the use of antiviral eyedrops to treat the infection

A

Which menu choice by the patient with diverticulosis is best for preventing diverticulitis? a. Navy bean soup and vegetable salad b. Whole grain pasta with tomato sauce c. Baked potato with low-fat sour cream d. Roast beef sandwich on whole wheat bread

A

Which nursing action will be included in the plan of care for a 27-year-old male patient with bowel irregularity and a new diagnosis of irritable bowel syndrome (IBS)? a. Encourage the patient to express concerns and ask questions about IBS. b. Suggest that the patient increase the intake of milk and other dairy products. c. Educate the patient about the use of alosetron (Lotronex) to reduce symptoms. d. Teach the patient to avoid using nonsteroidal antiinflammatory drugs (NSAIDs).

A

Which patient arriving at the urgent care center will the nurse assess first? a. Patient with acute right eye pain that occurred while using home power tools b. Patient with purulent left eye discharge, pruritus, and conjunctival inflammation c. Patient who is complaining of intense discomfort after an insect crawled into the right ear d. Patient who has Ménière's disease and is complaining of nausea, vomiting, and dizziness

A

28. The nurse is developing a discharge teaching plan for a patient diagnosed with thromboangiitis obliterans (Buerger's disease). Which expected outcome has the highest priority for this patient? a. Cessation of all tobacco use b. Control of serum lipid levels c. Maintenance of appropriate weight d. Demonstration of meticulous foot care

A Absolute cessation of nicotine use is needed to reduce the risk for amputation in patients with Buerger's disease. Other therapies have limited success in treatment of this disease. DIF: Cognitive Level: Analyze (analysis) REF: 809 OBJ: Special Questions: Prioritization TOP: Nursing Process: Planning MSC: NCLEX: Physiological

16. Which patient statement to the nurse is most consistent with the diagnosis of venous insufficiency? a. "I can't get my shoes on at the end of the day." b. "I can't ever seem to get my feet warm enough." c. "I have burning leg pains after I walk two blocks." d. "I wake up during the night because my legs hurt."

A Because the edema associated with venous insufficiency increases when the patient has been standing, shoes will feel tighter at the end of the day. The other patient statements are characteristic of peripheral artery disease. DIF: Cognitive Level: Apply (application) REF: 826 TOP: Nursing Process: Assessment MSC: NCLEX: Physiological

22. An older patient with a history of an abdominal aortic aneurysm arrives at the emergency department (ED) with severe back pain and absent pedal pulses. Which action should the nurse take first? a. Check the blood pressure. b. Draw blood for laboratory testing. c. Assess for the presence of an abdominal bruit. d. Determine any family history of heart disease.

A Because the patient appears to be experiencing aortic dissection, the nurse's first action should be to determine the hemodynamic status by assessing blood pressure. The other actions may also be done, but they will not provide information to determine what interventions are needed immediately. DIF: Cognitive Level: Analyze (analysis) REF: 814 OBJ: Special Questions: Prioritization TOP: Nursing Process: Implementation MSC: NCLEX: Physiological

8. When evaluating the discharge teaching for a patient with chronic peripheral artery disease (PAD), the nurse determines a need for further instruction when the patient says, "I will a. use a heating pad on my feet at night to increase the circulation." b. buy some loose clothes that do not bind across my legs or waist." c. walk to the point of pain, rest, and walk again for at least 30 minutes 3 times a week." d. change my position every hour and avoid long periods of sitting with my legs crossed."

A Because the patient has impaired circulation and sensation to the feet, the use of a heating pad could lead to burns. The other patient statements are correct and indicate that teaching has been successful. DIF: Cognitive Level: Apply (application) REF: 804 TOP: Nursing Process: Evaluation MSC: NCLEX: Physiological

26. The nurse is caring for a patient with critical limb ischemia who has just arrived on the nursing unit after having percutaneous transluminal balloon angioplasty. Which action should the nurse perform first? a. Obtain vital signs. c. Assess pedal pulses. b. Teach wound care. d. Check the wound site.

A Bleeding is a possible complication after catheterization of the femoral artery, so the nurse's first action should be to assess for changes in vital signs that might indicate hemorrhage. The other actions are also appropriate but can be done after determining that bleeding is not occurring. DIF: Cognitive Level: Analyze (analysis) REF: 804 OBJ: Special Questions: Prioritization TOP: Nursing Process: Implementation MSC: NCLEX: Physiological

9. After teaching a patient with newly diagnosed Raynaud's phenomenon about how to manage the condition, which action by the patient best demonstrates that the teaching has been effective? a. The patient exercises indoors during the winter months. b. The patient immerses hands in hot water when they turn pale. c. The patient takes pseudoephedrine (Sudafed) for cold symptoms. d. The patient avoids taking nonsteroidal antiinflammatory drugs (NSAIDs).

A Patients should avoid temperature extremes by exercising indoors when it is cold. To avoid burn injuries, the patient should use warm rather than hot water to warm the hands. Pseudoephedrine is a vasoconstrictor and should be avoided. There is no reason to avoid taking NSAIDs with Raynaud's phenomenon. DIF: Cognitive Level: Apply (application) REF: 809 TOP: Nursing Process: Evaluation MSC: NCLEX: Physiological

27. A patient who is 2 days post femoral popliteal bypass graft to the right leg is being cared for on the vascular unit. Which action by a licensed practical/vocational nurse (LPN/LVN) caring for the patient requires the registered nurse (RN) to intervene? a. The LPN/LVN has the patient to sit in a chair for 2 hours. b. The LPN/LVN gives the prescribed aspirin after breakfast. c. The LPN/LVN assists the patient to walk 40 feet in the hallway. d. The LPN/LVN places the patient in Fowler's position for meals.

A The patient should avoid sitting for long periods because of the increased stress on the suture line caused by leg edema and because of the risk for venous thromboembolism (VTE). The other actions by the LPN/LVN are appropriate. DIF: Cognitive Level: Apply (application) REF: 806 OBJ: Special Questions: Delegation TOP: Nursing Process: Implementation MSC: NCLEX: Safe and Effective Care Environment

22. The nurse in the clinic notes elevated prostate specific antigen (PSA) levels in the laboratory results of these patients. Which patient's PSA result is most important to report to the health care provider?

A 48-year-old whose father died of metastatic prostate cancer

26. The nurse working in a health clinic receives calls from all these patients. Which patient should be seen by the health care provider first?

A 66-year-old man who has a painful erection that has lasted over 7 hours

Which information will the nurse include when teaching a patient how to avoid chronic constipation (select all that apply)? a. Many over-the-counter (OTC) medications can cause constipation. b. Stimulant and saline laxatives can be used regularly. c. Bulk-forming laxatives are an excellent source of fiber. d. Walking or cycling frequently will help bowel motility. e. A good time for a bowel movement may be after breakfast.

A C D E

The nurse is providing health promotion teaching to a group of older adults. Which information will the nurse include when teaching about routine glaucoma testing?

A Tono-pen will be applied to the surface of the eye.

Which teaching point should the nurse plan to include when caring for a patient whose vision is corrected to 20/200? a. How to access audio books b. How to use a white cane safely c. Where Braille instruction is available d. Where to obtain specialized magnifiers

D

A pt with a history of peptic ulcer disease has presented to the emergency department with complaints of severe abdominal pain and a rigid, boardlike abdomen, prompting the health care team to suspect a perforated ulcer. Which of the following actions should the nurse anticipate? A) Providing IV fluids and inserting a nasogastric tube B) Administering oral bicarbonate and testing the patient's gastric pH level C) Performing a fecal occult blood test and administering IV calcium gluconate D) Starting parenteral nutrition and placing the patient in a high-Fowler's position

A) Providing IV fluids and inserting a nasogastric tube A perforated peptic ulcer requires IV replacement of fluid losses and continued gastric aspiration by NG tube. Nothing is given by mouth and gastric pH testing is not a priority. Calcium gluconate is not a medication directly relevant to the patient's suspected diagnosis and parenteral nutrition is not a priority in the short term.

When administered a dose of metoclopramide (Reglan), a patient complains of nausea. The nurse would teach the patient to report which of the following potential adverse effects? A) Tremors B) Constipation C) Double vision D) Numbness in the fingers and dose

A) Tremors (Extrapyramidal side effects, including tremors and dyskinesias, may occur as a result of metoclopramide (Reglan) administration.)

A 72-year-old patient with kyphosis is scheduled for dual-energy x-ray absorptiometry (DXA) testing. The nurse will plan to a. explain the procedure. b. start an IV line for contrast medium injection. c. give an oral sedative 60 to 90 minutes before the procedure. d. screen the patient for allergies to shellfish or iodine products.

A. DXA testing is painless and noninvasive. No IV access is necessary. Contrast medium is not used. Because the procedure is painless, no antianxiety medications are required.

A patient has a new order for magnetic resonance imaging (MRI) to evaluate for left femur osteomyelitis after a hip replacement surgery. Which information indicates that the nurse should consult with the health care provider before scheduling the MRI? a. The patient has a pacemaker. b. The patient is claustrophobic. c. The patient wears a hearing aid. d. The patient is allergic to shellfish.

A. Patients with permanent pacemakers cannot have MRI because of the force exerted by the magnetic field on metal objects. An open MRI will not cause claustrophobia. The patient will need to be instructed to remove the hearing aid before the MRI, but this does not require consultation with the health care provider. Because contrast medium will not be used, shellfish allergy is not a contraindication to MRI.

Which finding from a patient's right knee arthrocentesis will be of concern to the nurse? a. Cloudy fluid b. Scant thin fluid c. Pale yellow fluid d. Straw-colored fluid

A. The presence of purulent fluid suggests a possible joint infection. Normal synovial fluid is scant in amount and pale yellow/straw-colored.

Which nursing action is correct when performing the straight-leg raising test for an ambulatory patient with back pain? a. Raise the patient's legs to a 60-degree angle from the bed. b. Place the patient initially in the prone position on the exam table. c. Have the patient dangle both legs over the edge of the exam table. d. Instruct the patient to elevate the legs and tense the abdominal muscles.

A. When performing the straight leg-raising test, the patient is in the supine position and the nurse passively lifts the patient's legs to a 60-degree angle. The other actions would not be correct for this test.

1. A 63-year-old patient who began experiencing right arm and leg weakness is admitted to the emergency department. In which order will the nurse implement these actions included in the stroke protocol? (Put a comma and a space between each answer choice [A, B, C, D].) a. Obtain computed tomography (CT) scan without contrast. b. Infuse tissue plasminogen activator (tPA). c. Administer oxygen to keep O2 saturation >95%. d. Use National Institute of Health Stroke Scale to assess patient.

ANS: C, D, A, B The initial actions should be those that help with airway, breathing, and circulation. Baseline neurologic assessments should be done next. A CT scan will be needed to rule out hemorrhagic stroke before tPA can be administered. DIF: Cognitive Level: Apply (application) REF: 1401 | 1404 | 1398 OBJ: Special Questions: Prioritization TOP: Nursing Process: Implementation MSC: NCLEX: Physiological Integrity

A patient who is having an acute exacerbation of multiple sclerosis has a prescription for methylprednisolone (Solu-Medrol) 160 mg IV. The label on the vial reads: methylprednisolone 125 mg in 2 mL. How many mL will the nurse administer?

ANS: 2.56 With a concentration of 125 mg/2 mL, the nurse will need to administer 2.56 mL to obtain 160 mg of methylprednisolone. DIF: Cognitive Level: Understand (comprehension) REF: 1430-1431 TOP: Nursing Process: Implementation MSC: NCLEX: Physiological Integrity

Vasopressin (Pitressin) 0.2 units/min infusion is prescribed for a patient with acute arterial gastrointestinal (GI) bleeding. The vasopressin label states vasopressin 100 units/250 mL normal saline. How many mL/hr will the nurse infuse?

ANS: 30 There are 0.4 units/1 mL. An infusion of 30 mL/hr will result in the patient receiving 0.2 units/min as prescribed.

33. When the nurse is caring for a patient who has had left-sided extracorporeal shock wave lithotripsy, which assessment finding is most important to report to the health care provider?

Drop in urine output

A patient with asthma has a personal best peak expiratory flow rate (PEFR) of 400 L/minute. When explaining the asthma action plan, the nurse will teach the patient that a change in therapy is needed when the PEFR is less than ___ L/minute

ANS: 320 A PEFR less than 80% of the personal best indicates that the patient is in the yellow zone where changes in therapy are needed to prevent progression of the airway narrowing. DIF: Cognitive Level: Apply (application) REF: 579 TOP: Nursing Process: Evaluation MSC: NCLEX: Physiological Integrity

A patient in the oliguric phase after an acute kidney injury has had a 250 mL urine output and an emesis of 100 mL in the past 24 hours. What is the patient's fluid restriction for the next 24 hours?

ANS: 950 mL The general rule for calculating fluid restrictions is to add all fluid losses for the previous 24 hours, plus 600 mL for insensible losses: (250 + 100 + 600 = 950 mL).

1. The nurse is caring for a 47-year-old female patient who is comatose and is receiving continuous enteral nutrition through a soft nasogastric tube. The nurse notes the presence of new crackles in the patient's lungs. In which order will the nurse take action? (Put a comma and a space between each answer choice [A, B, C, D].) a. Check the patient's oxygen saturation. b. Notify the patient's health care provider. c. Measure the tube feeding residual volume. d. Stop administering the continuous feeding.

ANS: D, A, C, B The assessment data indicate that aspiration may have occurred, and the nurse's first action should be to turn off the tube feeding to avoid further aspiration. The next action should be to check the oxygen saturation because this may indicate the need for immediate respiratory suctioning or oxygen administration. The residual volume should be obtained because it provides data about possible causes of aspiration. Finally, the health care provider should be notified and informed of all the assessment data the nurse has just obtained.

A female patient who had a stroke 24 hours ago has expressive aphasia. The nurse identifies the nursing diagnosis of impaired verbal communication. An appropriate nursing intervention to help the patient communicate is to a. ask questions that the patient can answer with "yes" or "no." b. develop a list of words that the patient can read and practice reciting. c. have the patient practice her facial and tongue exercises with a mirror. d. prevent embarrassing the patient by answering for her if she does not respond.

ANS: A Communication will be facilitated and less frustrating to the patient when questions that require a "yes" or "no" response are used. When the language areas of the brain are injured, the patient might not be able to read or recite words, which will frustrate the patient without improving communication. Expressive aphasia is caused by damage to the language areas of the brain, not by the areas that control the motor aspects of speech. The nurse should allow time for the patient to respond. DIF: Cognitive Level: Apply (application) REF: 1407 TOP: Nursing Process: Implementation MSC: NCLEX: Physiological Integrity

A patient with carotid atherosclerosis asks the nurse to describe a carotid endarterectomy. Which response by the nurse is accurate? a. "The obstructing plaque is surgically removed from an artery in the neck." b. "The diseased portion of the artery in the brain is replaced with a synthetic graft." c. "A wire is threaded through an artery in the leg to the clots in the carotid artery and the clots are removed." d. "A catheter with a deflated balloon is positioned at the narrow area, and the balloon is inflated to flatten the plaque."

ANS: A In a carotid endarterectomy, the carotid artery is incised and the plaque is removed. The response beginning, "The diseased portion of the artery in the brain is replaced" describes an arterial graft procedure. The answer beginning, "A catheter with a deflated balloon is positioned at the narrow area" describes an angioplasty. The final response beginning, "A wire is threaded through the artery" describes the mechanical embolus removal in cerebral ischemia (MERCI) procedure. DIF: Cognitive Level: Understand (comprehension) REF: 1397 TOP: Nursing Process: Implementation MSC: NCLEX: Physiological Integrity

A 40-year-old patient has a ruptured cerebral aneurysm and subarachnoid hemorrhage. Which intervention will be included in the care plan? a. Apply intermittent pneumatic compression stockings. b. Assist to dangle on edge of bed and assess for dizziness. c. Encourage patient to cough and deep breathe every 4 hours. d. Insert an oropharyngeal airway to prevent airway obstruction.

ANS: A The patient with a subarachnoid hemorrhage usually has minimal activity to prevent cerebral vasospasm or further bleeding and is at risk for venous thromboembolism (VTE). Activities such as coughing and sitting up that might increase intracranial pressure (ICP) or decrease cerebral blood flow are avoided. Because there is no indication that the patient is unconscious, an oropharyngeal airway is inappropriate. DIF: Cognitive Level: Apply (application) REF: 1405 TOP: Nursing Process: Planning MSC: NCLEX: Physiological Integrity

For a patient who had a right hemisphere stroke the nurse establishes a nursing diagnosis of a. risk for injury related to denial of deficits and impulsiveness. b. impaired physical mobility related to right-sided hemiplegia. c. impaired verbal communication related to speech-language deficits. d. ineffective coping related to depression and distress about disability.

ANS: A The patient with right-sided brain damage typically denies any deficits and has poor impulse control, leading to risk for injury when the patient attempts activities such as transferring from a bed to a chair. Right-sided brain damage causes left hemiplegia. Left-sided brain damage typically causes language deficits. Left-sided brain damage is associated with depression and distress about the disability. DIF: Cognitive Level: Apply (application) REF: 1407 TOP: Nursing Process: Diagnosis MSC: NCLEX: Physiological Integrity

After receiving change-of-shift report on the following four patients, which patient should the nurse see first? a. A 60-year-old patient with right-sided weakness who has an infusion of tPA prescribed b. A 50-year-old patient who has atrial fibrillation and a new order for warfarin (Coumadin) c. A 40-year-old patient who experienced a transient ischemic attack yesterday who has a dose of aspirin due d. A 30-year-old patient with a subarachnoid hemorrhage 2 days ago who has nimodipine (Nimotop) scheduled

ANS: A tPA needs to be infused within the first few hours after stroke symptoms start in order to be effective in minimizing brain injury. The other medications should also be given as quickly as possible, but timing of the medications is not as critical. DIF: Cognitive Level: Apply (application) REF: 1398 OBJ: Special Questions: Prioritization; Multiple Patients TOP: Nursing Process: Implementation MSC: NCLEX: Safe and Effective Care Environment

7. When performing a skin assessment, the nurse notes several angiomas on the chest of an older patient. Which action should the nurse take next? a. Assess the patient for evidence of liver disease. b. Discuss the adverse effects of sun exposure on the skin. c. Teach the patient about possible skin changes with aging. d. Suggest that the patient make an appointment with a dermatologist.

ANS: A Angiomas are a common occurrence as patients get older, but they may occur with systemic problems such as liver disease. The patient may want to see a dermatologist to have the angiomas removed, but this is not the initial action by the nurse. The nurse may need to teach the patient about the effects of aging on the skin and about the effects of sun exposure, but the initial action should be further assessment. DIF: Cognitive Level: Apply (application) REF: 402 TOP: Nursing Process: Implementation

A 42-year-old woman with Ménière's disease is admitted with vertigo, nausea, and vomiting. Which nursing intervention will be included in the care plan? a. Dim the lights in the patient's room. b. Encourage increased oral fluid intake. c. Change the patient's position every 2 hours. d. Keep the head of the bed elevated 30 degrees.

ANS: A A darkened, quiet room will decrease the symptoms of the acute attack of Ménière's disease. Because the patient will be nauseated during an acute attack, fluids are administered IV. Position changes will cause vertigo and nausea. The head of the bed can be positioned for patient comfort

51. Which menu choice by the patient with diverticulosis is best for preventing diverticulitis? a. Navy bean soup and vegetable salad b. Whole grain pasta with tomato sauce c. Baked potato with low-fat sour cream d. Roast beef sandwich on whole wheat bread

ANS: A A diet high in fiber and low in fats and red meat is recommended to prevent diverticulitis. Although all of the choices have some fiber, the bean soup and salad will be the highest in fiber and the lowest in fat.

When teaching a patient about testing to diagnose metabolic syndrome, which topic would the nurse include? a. Blood glucose test b. Cardiac enzyme tests c. Postural blood pressures d. Resting electrocardiogram

ANS: A A fasting blood glucose test >100 mg/dL is one of the diagnostic criteria for metabolic syndrome. The other tests are not used to diagnose metabolic syndrome although they may be used to check for cardiovascular complications of the disorder.

Which statement by a patient scheduled for surgery is most important to report to the health care provider? a. "I had a heart valve replacement last year." b. "I had bacterial pneumonia 3 months ago." c. "I have knee pain whenever I walk or jog." d. "I have a strong family history of breast cancer."

ANS: A A patient with a history of valve replacement is at risk for endocarditis associated with invasive procedures and may need antibiotic prophylaxis. A current respiratory infection may affect whether the patient should have surgery, but a history of pneumonia is not a reason to postpone surgery. The patient's knee pain is the likely reason for the surgery. A family history of breast cancer does not have any implications for the current surgery.

26. The nurse admitting a patient with acute diverticulitis explains that the initial plan of care is to a. administer IV fluids. b. give stool softeners and enemas. c. order a diet high in fiber and fluids. d. prepare the patient for colonoscopy.

ANS: A A patient with acute diverticulitis will be NPO and given parenteral fluids. A diet high in fiber and fluids will be implemented before discharge. Bulk-forming laxatives, rather than stool softeners, are usually given, and these will be implemented later in the hospitalization. The patient with acute diverticulitis will not have enemas or a colonoscopy because of the risk for perforation and peritonitis.

A 64-year-old patient who has amyotrophic lateral sclerosis (ALS) is hospitalized with pneumonia. Which nursing action will be included in the plan of care? a. Assist with active range of motion (ROM). b. Observe for agitation and paranoia. c. Give muscle relaxants as needed to reduce spasms. d. Use simple words and phrases to explain procedures.

ANS: A ALS causes progressive muscle weakness, but assisting the patient to perform active ROM will help maintain strength as long as possible. Psychotic manifestations such as agitation and paranoia are not associated with ALS. Cognitive function is not affected by ALS, and the patient's ability to understand procedures will not be impaired. Muscle relaxants will further increase muscle weakness and depress respirations. DIF: Cognitive Level: Apply (application) REF: 1439 TOP: Nursing Process: Planning MSC: NCLEX: Physiological Integrity

30. A 50-year-old female patient calls the clinic to report a new onset of severe diarrhea. The nurse anticipates that the patient will need to a. collect a stool specimen. b. prepare for colonoscopy. c. schedule a barium enema. d. have blood cultures drawn.

ANS: A Acute diarrhea is usually caused by an infectious process, and stool specimens are obtained for culture and examined for parasites or white blood cells. There is no indication that the patient needs a colonoscopy, blood cultures, or a barium enema.

A hospitalized 31-year-old patient with a history of cluster headache awakens during the night with a severe stabbing headache. Which action should the nurse take first? a. Start the ordered PRN oxygen at 6 L/min. b. Put a moist hot pack on the patient's neck. c. Give the ordered PRN acetaminophen (Tylenol). d. Notify the patient's health care provider immediately.

ANS: A Acute treatment for cluster headache is administration of 100% oxygen at 6 to 8 L/min. If the patient obtains relief with the oxygen, there is no immediate need to notify the health care provider. Cluster headaches last only 60 to 90 minutes, so oral pain medications have minimal effect. Hot packs are helpful for tension headaches but are not as likely to reduce pain associated with a cluster headache. DIF: Cognitive Level: Apply (application) REF: 1417 OBJ: Special Questions: Prioritization TOP: Nursing Process: Implementation MSC: NCLEX: Physiological Integrity

A patient arrives at the ambulatory surgery center for a scheduled laparoscopy procedure in outpatient surgery. Which information is of most concern to the nurse? a. The patient is planning to drive home after surgery. b. The patient had a sip of water 4 hours before arriving. c. The patient's insurance does not cover outpatient surgery. d. The patient has not had surgery using general anesthesia before.

ANS: A After outpatient surgery, the patient should not drive home and will need assistance with transportation and home care. The patient's experience with surgery is assessed, but it does not have as much application to the patient's physiologic safety. The patient's insurance coverage is important to establish, but this is not usually the nurse's role or a priority in nursing care. Having clear liquids a few hours before surgery does not usually increase risk for aspiration.

41. The nurse reviews the medication administration record (MAR) for a patient having an acute asthma attack. Which medication should the nurse administer first? a. Albuterol (Ventolin) 2.5 mg per nebulizer b. Methylprednisolone (Solu-Medrol) 60 mg IV c. Salmeterol (Serevent) 50 mcg per dry-powder inhaler (DPI) d. Triamcinolone (Azmacort) 2 puffs per metered-dose inhaler (MDI)

ANS: A Albuterol is a rapidly acting bronchodilator and is the first-line medication to reverse airway narrowing in acute asthma attacks. The other medications work more slowly. DIF: Cognitive Level: Apply (application) REF: 570 | 576 OBJ: Special Questions: Prioritization TOP: Nursing Process: Implementation MSC: NCLEX: Physiological Integrity

5. A patient complains of gas pains and abdominal distention two days after a small bowel resection. Which nursing action is best to take? a. Encourage the patient to ambulate. b. Instill a mineral oil retention enema. c. Administer the ordered IV morphine sulfate. d. Offer the ordered promethazine (Phenergan) suppository.

ANS: A Ambulation will improve peristalsis and help the patient eliminate flatus and reduce gas pain. A mineral oil retention enema is helpful for constipation with hard stool. A return-flow enema might be used to relieve persistent gas pains. Morphine will further reduce peristalsis. Promethazine (Phenergan) is used as an antiemetic rather than to decrease gas pains or distention.

A nurse should instruct a patient with recurrent staphylococcal and seborrheic blepharitis to a. irrigate the eyes with saline solution. b. apply cool compresses to the eyes three times daily. c. use a gentle baby shampoo to clean the lids as needed. d. schedule an appointment for surgical removal of the lesion.

C

A patient will need vascular access for hemodialysis. Which statement by the nurse accurately describes an advantage of a fistula over a graft? a. A fistula is much less likely to clot. b. A fistula increases patient mobility. c. A fistula can accommodate larger needles. d. A fistula can be used sooner after surgery.

ANS: A Arteriovenous (AV) fistulas are much less likely to clot than grafts, although it takes longer for them to mature to the point where they can be used for dialysis. The choice of an AV fistula or a graft does not have an impact on needle size or patient mobility

36. A patient who is experiencing an acute asthma attack is admitted to the emergency department. Which assessment should the nurse complete first? a. Listen to the patient's breath sounds. b. Ask about inhaled corticosteroid use. c. Determine when the dyspnea started. d. Obtain the forced expiratory volume (FEV) flow rate.

ANS: A Assessment of the patient's breath sounds will help determine how effectively the patient is ventilating and whether rapid intubation may be necessary. The length of time the attack has persisted is not as important as determining the patient's status at present. Most patients having an acute attack will be unable to cooperate with an FEV measurement. It is important to know about the medications the patient is using but not as important as assessing the breath sounds. DIF: Cognitive Level: Apply (application) REF: 564-565 OBJ: Special Questions: Prioritization TOP: Nursing Process: Assessment MSC: NCLEX: Physiological Integrity

After change-of-shift report, which patient should the nurse assess first? a. Patient with myasthenia gravis who is reporting increased muscle weakness b. Patient with a bilateral headache described as "like a band around my head" c. Patient with seizures who is scheduled to receive a dose of phenytoin (Dilantin) d. Patient with Parkinson's disease who has developed cogwheel rigidity of the arms

ANS: A Because increased muscle weakness may indicate the onset of a myasthenic crisis, the nurse should assess this patient first. The other patients should also be assessed, but do not appear to need immediate nursing assessments or actions to prevent life-threatening complications. DIF: Cognitive Level: Analyze (analysis) REF: 1438-1439 OBJ: Special Questions: Prioritization; Multiple Patients TOP: Nursing Process: Planning MSC: NCLEX: Safe and Effective Care Environment

During the preoperative assessment of the patient scheduled for a right cataract extraction and intraocular lens implantation, it is most important for the nurse to assess a. the visual acuity of the patient's left eye. b. how long the patient has had the cataract. c. for a white pupil in the patient's right eye. d. for a history of reactions to general anesthetics.

ANS: A Because it can take several weeks before the maximum improvement in vision occurs in the right eye, patient safety and independence are determined by the vision in the left eye. A white pupil in the operative eye would not be unusual for a patient scheduled for cataract removal and lens implantation. The length of time that the patient has had the cataract will not affect the perioperative care. Cataract surgery is done using local anesthetics rather than general anesthetics

8. Which nursing action will be included in the plan of care for a 27-year-old male patient with bowel irregularity and a new diagnosis of irritable bowel syndrome (IBS)? a. Encourage the patient to express concerns and ask questions about IBS. b. Suggest that the patient increase the intake of milk and other dairy products. c. Educate the patient about the use of alosetron (Lotronex) to reduce symptoms. d. Teach the patient to avoid using nonsteroidal antiinflammatory drugs (NSAIDs).

ANS: A Because psychologic and emotional factors can affect the symptoms for IBS, encouraging the patient to discuss emotions and ask questions is an important intervention. Alosetron has serious side effects, and is used only for female patients who have not responded to other therapies. Although yogurt may be beneficial, milk is avoided because lactose intolerance can contribute to symptoms in some patients. NSAIDs can be used by patients with IBS.

The clinic nurse teaches a patient with a 42 pack-year history of cigarette smoking about lung disease. Which information will be most important for the nurse to include? a. Options for smoking cessation b. Reasons for annual sputum cytology testing c. Erlotinib (Tarceva) therapy to prevent tumor risk d. Computed tomography (CT) screening for lung cancer

ANS: A Because smoking is the major cause of lung cancer, the most important role for the nurse is teaching patients about the benefits of and means of smoking cessation. CT scanning is currently being investigated as a screening test for high-risk patients. However, if there is a positive finding, the person already has lung cancer. Erlotinib may be used in patients who have lung cancer, but it is not used to reduce the risk of developing cancer

Which action should the perioperative nurse take to best protect the patient from burn injury during surgery? a. Ensure correct placement of the grounding pad. b. Check all emergency sprinklers in the operating room. c. Verify that a fire extinguisher is available during surgery. d. Confirm that all electrosurgical equipment has been properly serviced.

ANS: A Care must be taken to correctly place the grounding pad and all electrosurgical equipment to prevent injury from burns or fire. It is important to ensure that fire extinguishers are available and that sprinklers protect everyone in the operating room in the event of a fire, but placing the grounding pad will best prevent injury to the patient. Verifying that electrosurgical equipment works properly does not protect the patient unless all equipment and the grounding pad is placed correctly

Which patient statement indicates that the nurse's teaching following a gastroduodenostomy has been effective? a. "Vitamin supplements may prevent anemia." b. "Persistent heartburn is common after surgery." c. "I will try to drink more liquids with my meals." d. "I will need to choose high carbohydrate foods."

ANS: A Cobalamin deficiency may occur after partial gastrectomy, and the patient may need to receive cobalamin via injections or nasal spray. Although peptic ulcer disease may recur, persistent heartburn is not expected after surgery and the patient should call the health care provider if this occurs. Ingestion of liquids with meals is avoided to prevent dumping syndrome. Foods that have moderate fat and low carbohydrate should be chosen to prevent dumping syndrome

15. A 51-year-old male patient has a new diagnosis of Crohn's disease after having frequent diarrhea and a weight loss of 10 pounds (4.5 kg) over 2 months. The nurse will plan to teach about a. medication use. b. fluid restriction. c. enteral nutrition. d. activity restrictions.

ANS: A Medications are used to induce and maintain remission in patients with inflammatory bowel disease (IBD). Decreased activity level is indicated only if the patient has severe fatigue and weakness. Fluids are needed to prevent dehydration. There is no advantage to enteral feedings.

The nurse administers prescribed therapies for a patient with cor pulmonale and right-sided heart failure. Which assessment would best evaluate the effectiveness of the therapies? a. Observe for distended neck veins. b. Auscultate for crackles in the lungs. c. Palpate for heaves or thrills over the heart. d. Review hemoglobin and hematocrit values.

ANS: A Cor pulmonale is right ventricular failure caused by pulmonary hypertension, so clinical manifestations of right ventricular failure such as peripheral edema, jugular venous distention, and right upper-quadrant abdominal tenderness would be expected. Crackles in the lungs are likely to be heard with left-sided heart failure. Findings in cor pulmonale include evidence of right ventricular hypertrophy on electrocardiogram ECG and an increase in intensity of the second heart sound. Heaves or thrills are not common with cor pulmonale. Chronic hypoxemia leads to polycythemia and increased total blood volume and viscosity of the blood. The hemoglobin and hematocrit values are more likely to be elevated with cor pulmonale than decreased

A patient with bacterial pneumonia has rhonchi and thick sputum. What is the nurse's most appropriate action to promote airway clearance? a. Assist the patient to splint the chest when coughing. b. Teach the patient about the need for fluid restrictions. c. Encourage the patient to wear the nasal oxygen cannula. d. Instruct the patient on the pursed lip breathing technique.

ANS: A Coughing is less painful and more likely to be effective when the patient splints the chest during coughing. Fluids should be encouraged to help liquefy secretions. Nasal oxygen will improve gas exchange, but will not improve airway clearance. Pursed lip breathing is used to improve gas exchange in patients with COPD, but will not improve airway clearance

The nurse teaches a patient about drug therapy after a kidney transplant. Which statement by the patient would indicate a need for further instructions? a. "After a couple of years, it is likely that I will be able to stop taking the cyclosporine." b. "If I develop an acute rejection episode, I will need to have other types of drugs given IV." c. "I need to be monitored closely because I have a greater chance of developing malignant tumors." d. "The drugs are given in combination because they inhibit different ways the kidney can be rejected."

ANS: A Cyclosporine, a calcineurin inhibitor, will need to be continued for life. The other patient statements are accurate and indicate that no further teaching is necessary about those topics.

8. A 31-year-old woman who has multiple sclerosis (MS) asks the nurse about risks associated with pregnancy. Which response by the nurse is accurate? a. "MS symptoms may be worse after the pregnancy." b. "Women with MS frequently have premature labor." c. "MS is associated with an increased risk for congenital defects." d. "Symptoms of MS are likely to become worse during pregnancy."

ANS: A During the postpartum period, women with MS are at greater risk for exacerbation of symptoms. There is no increased risk for congenital defects in infants born of mothers with MS. Symptoms of MS may improve during pregnancy. Onset of labor is not affected by MS. DIF: Cognitive Level: Understand (comprehension) REF: 1429 TOP: Nursing Process: Implementation MSC: NCLEX: Health Promotion and Maintenance

The nurse is coaching a community group for individuals who are overweight. Which participant behavior is an example of the best exercise plan for weight loss? a. Walking for 40 minutes 6 or 7 days/week b. Lifting weights with friends 3 times/week c. Playing soccer for an hour on the weekend d. Running for 10 to 15 minutes 3 times/week

ANS: A Exercise should be done daily for 30 minutes to an hour. Exercising in highly aerobic activities for short bursts or only once a week is not helpful and may be dangerous in an individual who has not been exercising. Running may be appropriate, but a patient should start with an exercise that is less stressful and can be done for a longer period. Weight lifting is not as helpful as aerobic exercise in weight loss.

Which information will the nurse include for a patient contemplating a cochlear implant? a. Cochlear implants require training in order to receive the full benefit. b. Cochlear implants are not useful for patients with congenital deafness. c. Cochlear implants are most helpful as an early intervention for presbycusis. d. Cochlear implants improve hearing in patients with conductive hearing loss.

ANS: A Extensive rehabilitation is required after cochlear implants in order for patients to receive the maximum benefit. Hearing aids, rather than cochlear implants, are used initially for presbycusis. Cochlear implants are used for sensorineural hearing loss and would not be helpful for conductive loss. They are appropriate for some patients with congenital deafness

A 50-year-old patient who underwent a gastroduodenostomy (Billroth I) earlier today complains of increasing abdominal pain. The patient has no bowel sounds and 200 mL of bright red nasogastric (NG) drainage in the last hour. The highest priority action by the nurse is to a. contact the surgeon. b. irrigate the NG tube. c. monitor the NG drainage. d. administer the prescribed morphine.

ANS: A Increased pain and 200 mL of bright red NG drainage 12 hours after surgery indicate possible postoperative hemorrhage, and immediate actions such as blood transfusion and/or return to surgery are needed. Because the NG is draining, there is no indication that irrigation is needed. Continuing to monitor the NG drainage is not an adequate response. The patient may need morphine, but this is not the highest priority action

A 75-year-old patient with presbycusis is fitted with binaural hearing aids. Which information will the nurse include when teaching the patient how to use the hearing aids? a. Experiment with volume and hearing ability in a quiet environment initially. b. Keep the volume low on the hearing aids for the first week while adjusting to them. c. Add a second hearing aid after making the initial adjustment to the first hearing aid. d. Wear the hearing aids for about an hour a day at first, gradually increasing the time of use.

ANS: A Initially the patient should use the hearing aids in a quiet environment like the home, experimenting with increasing and decreasing the volume as needed. There is no need to gradually increase the time of wear. The patient should experiment with the level of volume to find what works well in various situations. Both hearing aids should be used

5. A 48-year-old man who has just been started on tube feedings of full-strength formula at 100 mL/hr has 6 diarrhea stools the first day. Which action should the nurse plan to take? a. Slow the infusion rate of the tube feeding. b. Check gastric residual volumes more frequently. c. Change the enteral feeding system and formula every 8 hours. d. Discontinue administration of water through the feeding tube.

ANS: A Loose stools indicate poor absorption of nutrients and indicate a need to slow the feeding rate or decrease the concentration of the feeding. Water should be given when patients receive enteral feedings to prevent dehydration. When a closed enteral feeding system is used, the tubing and formula are changed every 24 hours. High residual volumes do not contribute to diarrhea.

he nurse advises a patient with myasthenia gravis (MG) to a. perform physically demanding activities early in the day. b. anticipate the need for weekly plasmapheresis treatments. c. do frequent weight-bearing exercise to prevent muscle atrophy. d. protect the extremities from injury due to poor sensory perception.

ANS: A Muscles are generally strongest in the morning, and activities involving muscle activity should be scheduled then. Plasmapheresis is not routinely scheduled, but is used for myasthenia crisis or for situations in which corticosteroid therapy must be avoided. There is no decrease in sensation with MG, and muscle atrophy does not occur because although there is muscle weakness, they are still used. DIF: Cognitive Level: Apply (application) REF: 1437 TOP: Nursing Process: Implementation MSC: NCLEX: Physiological Integrity

An older patient is receiving standard multidrug therapy for tuberculosis (TB). The nurse should notify the health care provider if the patient exhibits which finding? a. Yellow-tinged skin b. Orange-colored sputum c. Thickening of the fingernails d. Difficulty hearing high-pitched voices

ANS: A Noninfectious hepatitis is a toxic effect of isoniazid (INH), rifampin, and pyrazinamide, and patients who develop hepatotoxicity will need to use other medications. Changes in hearing and nail thickening are not expected with the four medications used for initial TB drug therapy. Presbycusis is an expected finding in the older adult patient. Orange discoloration of body fluids is an expected side effect of rifampin and not an indication to call the health care provider

The nurse cares for a patient who has just had a thoracentesis. Which assessment information obtained by the nurse is a priority to communicate to the health care provider? a. Oxygen saturation is 88%. b. Blood pressure is 145/90 mm Hg. c. Respiratory rate is 22 breaths/minute when lying flat. d. Pain level is 5 (on 0 to 10 scale) with a deep breath.

ANS: A Oxygen saturation would be expected to improve after a thoracentesis. A saturation of 88% indicates that a complication such as pneumothorax may be occurring. The other assessment data also indicate a need for ongoing assessment or intervention, but the low oxygen saturation is the priority

A patient experiences a chest wall contusion as a result of being struck in the chest with a baseball bat. The emergency department nurse would be most concerned if which finding is observed during the initial assessment? a. Paradoxic chest movement b. Complaint of chest wall pain c. Heart rate of 110 beats/minute d. Large bruised area on the chest

ANS: A Paradoxic chest movement indicates that the patient may have flail chest, which can severely compromise gas exchange and can rapidly lead to hypoxemia. Chest wall pain, a slightly elevated pulse rate, and chest bruising all require further assessment or intervention, but the priority concern is poor gas exchange

The nurse is developing a plan of care for an adult patient diagnosed with adult inclusion conjunctivitis (AIC) caused by Chlamydia trachomatis. Which action should be included in the plan of care? a. Discussing the need for sexually transmitted infection testing b. Applying topical corticosteroids to prevent further inflammation c. Assisting with applying for community visual rehabilitation services d. Educating about the use of antiviral eyedrops to treat the infection

ANS: A Patients with AIC have a high risk for concurrent genital Chlamydia infection and should be referred for sexually transmitted infection (STI) testing. AIC is treated with antibiotics. Antiviral and corticosteroid medications are not appropriate therapies. Although some types of Chlamydia infection do cause blindness, AIC does not lead to blindness, so referral for visual rehabilitation is not appropriate

Which action should the nurse in the emergency department anticipate for a 23-year-old patient who has had several episodes of bloody diarrhea? a. Obtain a stool specimen for culture. b. Administer antidiarrheal medication. c. Provide teaching about antibiotic therapy. d. Teach about adverse effects of acetaminophen (Tylenol).

ANS: A Patients with bloody diarrhea should have a stool culture for E. coli O157:H7. Antidiarrheal medications are usually avoided for possible infectious diarrhea to avoid prolonging the infection. Antibiotic therapy in the treatment of infectious diarrhea is controversial because it may precipitate kidney complications. Acetaminophen does not cause bloody diarrhea

A patient has been taking phenytoin (Dilantin) for 2 years. Which action will the nurse take when evaluating for adverse effects of the medication? a. Inspect the oral mucosa. b. Listen to the lung sounds. c. Auscultate the bowel tones. d. Check pupil reaction to light.

ANS: A Phenytoin can cause gingival hyperplasia, but does not affect bowel tones, lung sounds, or pupil reaction to light. DIF: Cognitive Level: Apply (application) REF: 1424 TOP: Nursing Process: Evaluation MSC: NCLEX: Physiological Integrity

Which statement, if made by a new circulating nurse, is appropriate? a. "I will assist in preparing the operating room for the patient." b. "I will remain gloved while performing activities in the sterile field." c. "I will assist with suturing of incisions and maintaining hemostasis as needed." d. "I must don full surgical attire and sterile gloves while obtaining items from the unsterile field."

ANS: A Preparing the operating room for the patient describes the role of a circulating nurse. All other answer options describe specific types of scrub nurses. The circulating nurse performs activities in the unsterile field and is not scrubbed, gowned, or gloved. The scrub nurse follows the designated scrub procedure, is gowned and gloved in sterile attire, and performs activities in the sterile field

Which patient exposure by the nurse is most likely to require postexposure prophylaxis when the patient's human immunodeficiency virus (HIV) status is unknown? a. Needle stick with a needle and syringe used to draw blood b. Splash into the eyes when emptying a bedpan containing stool c. Contamination of open skin lesions with patient vaginal secretions d. Needle stick injury with a suture needle during a surgical procedure

ANS: A Puncture wounds are the most common means for workplace transmission of blood-borne diseases, and a needle with a hollow bore that had been contaminated with the patient's blood would be a high-risk situation. The other situations described would be much less likely to result in transmission of the virus.

A patient scheduled to undergo total knee replacement surgery under general anesthesia asks the nurse, "Will the doctor put me to sleep with a mask over my face?" Which response by the nurse is most appropriate? a. "A drug may be given to you through your IV line first. I will check with the anesthesia care provider." b. "Only your surgeon can tell you for sure what method of anesthesia will be used. Should I ask your surgeon?" c. "General anesthesia is now given by injecting medication into your veins, so you will not need a mask over your face." d. "Masks are not used anymore for anesthesia. A tube will be inserted into your throat to deliver a gas that will put you to sleep."

ANS: A Routine general anesthesia is usually induced by the IV route with a hypnotic, anxiolytic, or dissociative agent. However, general anesthesia may be induced by IV or by inhalation. The nurse should consult with the anesthesia care provider to determine the method selected for this patient. The anesthesia care provider will select the method of anesthesia, not the surgeon. Inhalation agents may be given through an endotracheal tube or a laryngeal mask airway

A patient is being evaluated for possible atopic dermatitis. The nurse expects elevation of which laboratory value? a. IgE b. IgA c. Basophils d. Neutrophils

ANS: A Serum IgE is elevated in an allergic response (type 1 hypersensitivity disorders). The eosinophil level will be elevated rather than neutrophil or basophil counts. IgA is located in body secretions and would not be tested when evaluating a patient who has symptoms of atopic dermatitis.

Sodium polystyrene sulfonate (Kayexalate) is ordered for a patient with hyperkalemia. Before administering the medication, the nurse should assess the a. bowel sounds. b. blood glucose. c. blood urea nitrogen (BUN). d. level of consciousness (LOC).

ANS: A Sodium polystyrene sulfonate (Kayexalate) should not be given to a patient with a paralytic ileus (as indicated by absent bowel sounds) because bowel necrosis can occur. The BUN and creatinine, blood glucose, and LOC would not affect the nurse's decision to give the medication

Which information would be most important to help the nurse determine if the patient needs human immunodeficiency virus (HIV) testing? a. Patient age b. Patient lifestyle c. Patient symptoms d. Patient sexual orientation

ANS: A The current Center for Disease Control (CDC) policy is to offer routine testing for HIV to all individuals age 13 to 64. Although lifestyle, symptoms, and sexual orientation may suggest increased risk for HIV infection, the goal is to test all individuals in this age range.

Which action included in the perioperative patient plan of care can the charge nurse delegate to a surgical technologist? a. Pass sterile instruments and supplies to the surgeon. b. Teach the patient about what to expect in the operating room (OR). c. Continuously monitor and interpret the patient's echocardiogram (ECG) during surgery. d. Give the postoperative report to the postanesthesia care unit (PACU) nurse.

ANS: A The education and certification for a surgical technologist includes the scrub and circulating functions in the OR. Patient teaching, communication with other departments about a patient's condition, and the admission assessment require registered-nurse (RN) level education and scope of practice. A surgical technologist is not usually trained to interpret ECG rhythms

The nurse is assessing a patient who has recently been treated with amoxicillin for acute otitis media of the right ear. Which finding is a priority to report to the health care provider? a. The patient has a temperature of 100.6° F. b. The patient complains of "popping" in the ear. c. The patient frequently asks the nurse to repeat information. d. The patient states that the right ear has a feeling of fullness.

ANS: A The fever indicates that the infection may not be resolved and the patient might need further antibiotic therapy. A feeling of fullness, "popping" of the ear, and decreased hearing are symptoms of otitis media with effusion. These symptoms are normal for weeks to months after an episode of acute otitis media and usually resolve without treatment

A 46-year-old patient tells the nurse about using acetaminophen (Tylenol) several times every day for recurrent bilateral headaches. Which action will the nurse plan to take first? a. Discuss the need to stop taking the acetaminophen. b. Suggest the use of biofeedback for headache control. c. Describe the use of botulism toxin (Botox) for headaches. d. Teach the patient about magnetic resonance imaging (MRI).

ANS: A The headache description suggests that the patient is experiencing medication overuse headache. The initial action will be withdrawal of the medication. The other actions may be needed if the headaches persist. DIF: Cognitive Level: Apply (application) REF: 1418 OBJ: Special Questions: Prioritization TOP: Nursing Process: Planning MSC: NCLEX: Physiological Integrity

Which patient arriving at the urgent care center will the nurse assess first? a. Patient with acute right eye pain that occurred while using home power tools b. Patient with purulent left eye discharge, pruritus, and conjunctival inflammation c. Patient who is complaining of intense discomfort after an insect crawled into the right ear d. Patient who has Ménière's disease and is complaining of nausea, vomiting, and dizziness

ANS: A The history and symptoms suggest eye trauma with a possible penetrating injury. Blindness may occur unless the patient is assessed and treated rapidly. The other patients should be treated as soon as possible, but do not have clinical manifestations that indicate any acute risk for vision or hearing loss

A patient reports feeling numbness and tingling of the left arm before experiencing a tonic-clonic seizure. The nurse determines that this history is consistent with what type of seizure? a. Focal b. Atonic c. Absence d. Myoclonic

ANS: A The initial symptoms of a focal seizure involve clinical manifestations that are localized to a particular part of the body or brain. Symptoms of an absence seizure are staring and a brief loss of consciousness. In an atonic seizure, the patient loses muscle tone and (typically) falls to the ground. Myoclonic seizures are characterized by a sudden jerk of the body or extremities. DIF: Cognitive Level: Understand (comprehension) REF: 1421 TOP: Nursing Process: Assessment MSC: NCLEX: Physiological Integrity

A patient who has received allergen testing using the cutaneous scratch method has developed itching and swelling at the skin site. Which action should the nurse take first? a. Administer epinephrine. b. Apply topical hydrocortisone. c. Monitor the patient for lower extremity edema. d. Ask the patient about exposure to any new lotions or soaps.

ANS: A The initial symptoms of anaphylaxis are itching and edema at the site of the exposure. Hypotension, tachycardia, dilated pupils, and wheezes occur later. Rapid administration of epinephrine when excessive itching or swelling at the skin site is observed can prevent the progression to anaphylaxis. Topical hydrocortisone would not deter an anaphylactic reaction. Exposure to lotions and soaps does not address the immediate concern of a possible anaphylactic reaction. The nurse should not wait and observe for edema. The nurse should act immediately in order to prevent progression to anaphylaxis.

The surgical unit nurse has just received a patient with a history of smoking from the postanesthesia care unit. Which action is most important at this time? a. Auscultate for adventitious breath sounds. b. Obtain the patient's blood pressure and temperature. c. Remind the patient about harmful effects of smoking. d. Ask the health care provider about prescribing a nicotine patch.

ANS: A The nurse should first ensure a patent airway and check for breathing and circulation (airway, breathing, and circulation [ABCs]). Circulation and temperature can be assessed after a patent airway and breathing have been established. The immediate postoperative period is not the optimal time for patient teaching about the harmful effects of surgery. Requesting a nicotine patch may be appropriate, but is not a priority at this time.

An older adult who takes medications for coronary artery disease has just been diagnosed with asymptomatic chronic human immunodeficiency virus (HIV) infection. Which information will the nurse include in patient teaching? a. Many medications have interactions with antiretroviral drugs. b. Less frequent CD4+ level monitoring is needed in older adults. c. Hospice care is available for patients with terminal HIV infection. d. Progression of HIV infection occurs more rapidly in older patients.

ANS: A The nurse will teach the patient about potential interactions between antiretrovirals and the medications that the patient is using for chronic health problems. Treatment and monitoring of HIV infection is not affected by age. A patient with asymptomatic HIV infection is not a candidate for hospice. Progression of HIV is not affected by age, although it may be affected by chronic disease

38. The nurse in the emergency department receives arterial blood gas results for four recently admitted patients with obstructive pulmonary disease. Which patient will require the most rapid action by the nurse? a. 22-year-old with ABG results: pH 7.28, PaCO2 60 mm Hg, and PaO2 58 mm Hg b. 34-year-old with ABG results: pH 7.48, PaCO2 30 mm Hg, and PaO2 65 mm Hg c. 45-year-old with ABG results: pH 7.34, PaCO2 33 mm Hg, and PaO2 80 mm Hg d. 65-year-old with ABG results: pH 7.31, PaCO2 58 mm Hg, and PaO2 64 mm Hg

ANS: A The pH, PaCO2, and PaO2 indicate that the patient has severe uncompensated respiratory acidosis and hypoxemia. Rapid action will be required to prevent increasing hypoxemia and correct the acidosis. The other patients also should be assessed as quickly as possible but do not require interventions as quickly as the 22-year-old. DIF: Cognitive Level: Analyze (analysis) REF: 566 OBJ: Special Questions: Prioritization; Multiple Patients TOP: Nursing Process: Assessment MSC: NCLEX: Safe and Effective Care Environment

The nurse observes a patient ambulating in the hospital hall when the patient's arms and legs suddenly jerk and the patient falls to the floor. The nurse will first a. assess the patient for a possible head injury. b. give the scheduled dose of divalproex (Depakote). c. document the timing and description of the seizure. d. notify the patient's health care provider about the seizure.

ANS: A The patient who has had a myoclonic seizure and fall is at risk for head injury and should first be evaluated and treated for this possible complication. Documentation of the seizure, notification of the seizure, and administration of antiseizure medications are also appropriate actions, but the initial action should be assessment for injury. DIF: Cognitive Level: Apply (application) REF: 1423 OBJ: Special Questions: Prioritization TOP: Nursing Process: Implementation MSC: NCLEX: Physiological Integrity

35. A patient who is experiencing an asthma attack develops bradycardia and a decrease in wheezing. Which action should the nurse take first? a. Notify the health care provider. b. Document changes in respiratory status. c. Encourage the patient to cough and deep breathe. d. Administer IV methylprednisolone (Solu-Medrol).

ANS: A The patient's assessment indicates impending respiratory failure, and the nurse should prepare to assist with intubation and mechanical ventilation after notifying the health care provider. IV corticosteroids require several hours before having any effect on respiratory status. The patient will not be able to cough or deep breathe effectively. Documentation is not a priority at this time. DIF: Cognitive Level: Apply (application) REF: 565 OBJ: Special Questions: Prioritization TOP: Nursing Process: Implementation MSC: NCLEX: Physiological Integrity

A patient who was admitted the previous day with pneumonia complains of a sharp pain of 7 (based on 0 to 10 scale) "whenever I take a deep breath." Which action will the nurse take next? a. Auscultate breath sounds. b. Administer the PRN morphine. c. Have the patient cough forcefully. d. Notify the patient's health care provider.

ANS: A The patient's statement indicates that pleurisy or a pleural effusion may have developed and the nurse will need to listen for a pleural friction rub and/or decreased breath sounds. Assessment should occur before administration of pain medications. The patient is unlikely to be able to cough forcefully until pain medication has been administered. The nurse will want to obtain more assessment data before calling the health care provider

A patient scheduled for an elective hysterectomy tells the nurse, "I am afraid that I will die in surgery like my mother did!" Which response by the nurse is most appropriate? a. "Tell me more about what happened to your mother." b. "You will receive medications to reduce your anxiety." c. "You should talk to the doctor again about the surgery." d. "Surgical techniques have improved a lot in recent years."

ANS: A The patient's statement may indicate an unusually high anxiety level or a family history of problems such as malignant hyperthermia, which will require precautions during surgery. The other statements may also address the patient's concerns, but further assessment is needed first.

10. Which nursing action will the nurse include in the plan of care for a 35-year-old male patient admitted with an exacerbation of inflammatory bowel disease (IBD)? a. Restrict oral fluid intake. b. Monitor stools for blood. c. Ambulate four times daily. d. Increase dietary fiber intake.

ANS: B Because anemia or hemorrhage may occur with IBD, stools should be assessed for the presence of blood. The other actions would not be appropriate for the patient with IBD. Because dietary fiber may increase gastrointestinal (GI) motility and exacerbate the diarrhea, severe fatigue is common with IBD exacerbations, and dehydration may occur.

13. A healthy 28-year-old woman patient who weighs 145 pounds (66 kg) asks the nurse about the minimum daily requirement for protein. How many grams of protein will the nurse recommend? a. 53 b. 66 c. 75 d. 98

ANS: A The recommended daily protein intake is 0.8 to 1 g/kg of body weight, which for this patient is 66 kg × 0.8 g = 52.8 or 53 g/day.

12. A patient is receiving continuous enteral nutrition through a small-bore silicone feeding tube. What should the nurse plan for when this patient has a computed tomography (CT) scan ordered? a. Shut the feeding off 30 to 60 minutes before the scan. b. Ask the health care provider to reschedule the CT scan. c. Connect the feeding tube to continuous suction during the scan. d. Send the patient to CT scan with oral suction in case of aspiration

ANS: A The tube feeding should be shut off 30 to 60 minutes before any procedure requiring the patient to lie flat. Because the CT scan is ordered for diagnosis of patient problems, rescheduling is not usually an option. Prevention, rather than treatment, of aspiration is needed. Small-bore feeding tubes are soft and collapse easily with aspiration or suction, making nasogastric suction of gastric contents unreliable.

A 33-year-old male patient with a gunshot wound to the abdomen undergoes surgery, and a colostomy is formed as shown in the accompanying figure. Which information will be included in patient teaching? a. Stool will be expelled from both stomas. b. This type of colostomy is usually temporary. c. Soft, formed stool can be expected as drainage. d. Irrigations can regulate drainage from the stomas.

B

48. A female patient is awaiting surgery for acute peritonitis. Which action will the nurse include in the plan of care? a. Position patient with the knees flexed. b. Avoid use of opioids or sedative drugs. c. Offer frequent small sips of clear liquids. d. Assist patient to breathe deeply and cough.

ANS: A There is less peritoneal irritation with the knees flexed, which will help decrease pain. Opioids and sedatives are typically given to control pain and anxiety. Preoperative patients with peritonitis are given IV fluids for hydration. Deep breathing and coughing will increase the patient's discomfort.

Which information will the nurse include when teaching adults to decrease the risk for cancers of the tongue and buccal mucosa? a. Avoid use of cigarettes and smokeless tobacco. b. Use sunscreen when outside even on cloudy days. c. Complete antibiotic courses used to treat throat infections. d. Use antivirals to treat herpes simplex virus (HSV) infections.

ANS: A Tobacco use greatly increases the risk for oral cancer. Acute throat infections do not increase the risk for oral cancer, although chronic irritation of the oral mucosa does increase risk. Sun exposure does not increase the risk for cancers of the buccal mucosa. Human papillomavirus (HPV) infection is associated with an increased risk, but HSV infection is not a risk factor for oral cancer

39. Which nursing action for a patient with chronic obstructive pulmonary disease (COPD) could the nurse delegate to experienced unlicensed assistive personnel (UAP)? a. Obtain oxygen saturation using pulse oximetry. b. Monitor for increased oxygen need with exercise. c. Teach the patient about safe use of oxygen at home. d. Adjust oxygen to keep saturation in prescribed parameters.

ANS: A UAP can obtain oxygen saturation (after being trained and evaluated in the skill). The other actions require more education and a scope of practice that licensed practical/vocational nurses (LPN/LVNs) or registered nurses (RNs) would have. DIF: Cognitive Level: Apply (application) REF: 592 OBJ: Special Questions: Delegation TOP: Nursing Process: Planning MSC: NCLEX: Safe and Effective Care Environment

A 54-year-old critically ill patient with sepsis is frequently incontinent of watery stools. What action by the nurse will prevent complications associated with ongoing incontinence? a. Apply incontinence briefs. b. Use a fecal management system c. Insert a rectal tube with a drainage bag. d. Assist the patient to a commode frequently.

B

The nurse cares for a patient infected with human immunodeficiency virus (HIV) who has just been diagnosed with asymptomatic chronic HIV infection. Which prophylactic measures will the nurse include in the plan of care (select all that apply)? a. Hepatitis B vaccine b. Pneumococcal vaccine c. Influenza virus vaccine d. Trimethoprim-sulfamethoxazole e. Varicella zoster immune globulin

ANS: A, B, C Asymptomatic chronic HIV infection is a stage between acute HIV infection and a diagnosis of symptomatic chronic HIV infection. Although called asymptomatic, symptoms (e.g., fatigue, headache, low-grade fever, night sweats) often occur. Prevention of other infections is an important intervention in patients who are HIV positive, and these vaccines are recommended as soon as the HIV infection is diagnosed. Antibiotics and immune globulin are used to prevent and treat infections that occur later in the course of the disease when the CD4+ counts have dropped or when infection has occurred.

Which factors will the nurse consider when calculating the CURB-65 score for a patient with pneumonia (select all that apply)? a. Age b. Blood pressure c. Respiratory rate d. Oxygen saturation e. Presence of confusion f. Blood urea nitrogen (BUN) level

ANS: A, B, C, E, F Data collected for the CURB-65 are mental status (confusion), BUN (elevated), blood pressure (decreased), respiratory rate (increased), and age (65 and older). The other information is also essential to assess, but are not used for CURB-65 scoring

A patient with Parkinson's disease is admitted to the hospital for treatment of pneumonia. Which nursing interventions will be included in the plan of care (select all that apply)? a. Use an elevated toilet seat. b. Cut patient's food into small pieces. c. Provide high-protein foods at each meal. d. Place an armchair at the patient's bedside. e. Observe for sudden exacerbation of symptoms.

ANS: A, B, D Because the patient with Parkinson's has difficulty chewing, food should be cut into small pieces. An armchair should be used when the patient is seated so that the patient can use the arms to assist with getting up from the chair. An elevated toilet seat will facilitate getting on and off the toilet. High-protein foods will decrease the effectiveness of L-dopa. Parkinson's is a steadily progressive disease without acute exacerbations. DIF: Cognitive Level: Apply (application) REF: 1436-1437 TOP: Nursing Process: Planning MSC: NCLEX: Physiological Integrity

2. Which activities can the nurse working in the outpatient clinic delegate to a licensed practical/vocational nurse (LPN/LVN) (select all that apply)? a. Administer patch testing to a patient with allergic dermatitis. b. Interview a new patient about chronic health problems and allergies. c. Apply a sterile dressing after the health care provider excises a mole. d. Teach a patient about site care after a punch biopsy of an upper arm lesion. e. Explain potassium hydroxide testing to a patient with a superficial skin infection.

ANS: A, C Skills such as administration of patch testing and sterile dressing technique are included in LPN/LVN education and scope of practice. Obtaining a health history and patient education require more critical thinking and registered nurse (RN) level education and scope of practice. DIF: Cognitive Level: Apply (application) REF: 15 OBJ: Special Questions: Delegation TOP: Nursing Process: Planning

Which information will be included when the nurse is teaching self-management to a patient who is receiving peritoneal dialysis (select all that apply)? a. Avoid commercial salt substitutes. b. Drink 1500 to 2000 mL of fluids daily. c. Take phosphate-binders with each meal. d. Choose high-protein foods for most meals. e. Have several servings of dairy products daily.

ANS: A, C, D Patients who are receiving peritoneal dialysis should have a high-protein diet. Phosphate binders are taken with meals to help control serum phosphate and calcium levels. Commercial salt substitutes are high in potassium and should be avoided. Fluid intake is limited in patients requiring dialysis. Dairy products are high in phosphate and usually are limited

A 27-year-old patient who has been treated for status epilepticus in the emergency department will be transferred to the medical nursing unit. Which equipment should the nurse have available in the patient's assigned room (select all that apply)? a. Side-rail pads b. Tongue blade c. Oxygen mask d. Suction tubing e. Urinary catheter f. Nasogastric tube

ANS: A, C, D The patient is at risk for further seizures, and oxygen and suctioning may be needed after any seizures to clear the airway and maximize oxygenation. The bed's side rails should be padded to minimize the risk for patient injury during a seizure. Use of tongue blades during a seizure is contraindicated. Insertion of a nasogastric (NG) tube is not indicated because the airway problem is not caused by vomiting or abdominal distention. A urinary catheter is not required unless there is urinary retention .DIF: Cognitive Level: Apply (application) REF: 1426 TOP: Nursing Process: Planning MSC: NCLEX: Physiological Integrity

1. Which information will the nurse include when teaching a patient how to avoid chronic constipation (select all that apply)? a. Many over-the-counter (OTC) medications can cause constipation. b. Stimulant and saline laxatives can be used regularly. c. Bulk-forming laxatives are an excellent source of fiber. d. Walking or cycling frequently will help bowel motility. e. A good time for a bowel movement may be after breakfast.

ANS: A, C, D, E Stimulant and saline laxatives should be used infrequently. Use of bulk-forming laxatives, regular early morning timing of defecation, regular exercise, and avoiding many OTC medications will help the patient avoid constipation.

Several weeks after a stroke, a 50-year-old male patient has impaired awareness of bladder fullness, resulting in urinary incontinence. Which nursing intervention will be best to include in the initial plan for an effective bladder training program? a. Limit fluid intake to 1200 mL daily to reduce urine volume. b. Assist the patient onto the bedside commode every 2 hours. c. Perform intermittent catheterization after each voiding to check for residual urine. d. Use an external "condom" catheter to protect the skin and prevent embarrassment.

ANS: B Developing a regular voiding schedule will prevent incontinence and may increase patient awareness of a full bladder. A 1200 mL fluid restriction may lead to dehydration. Intermittent catheterization and use of a condom catheter are appropriate in the acute phase of stroke, but should not be considered solutions for long-term management because of the risks for urinary tract infection (UTI) and skin breakdown. DIF: Cognitive Level: Apply (application) REF: 1406 TOP: Nursing Process: Planning MSC: NCLEX: Physiological Integrity

The nurse is caring for a patient who has just returned after having left carotid artery angioplasty and stenting. Which assessment information is of most concern to the nurse? a. The pulse rate is 102 beats/min. b. The patient has difficulty speaking. c. The blood pressure is 144/86 mm Hg. d. There are fine crackles at the lung bases.

ANS: B Small emboli can occur during carotid artery angioplasty and stenting, and the aphasia indicates a possible stroke during the procedure. Slightly elevated pulse rate and blood pressure are not unusual because of anxiety associated with the procedure. Fine crackles at the lung bases may indicate atelectasis caused by immobility during the procedure. The nurse should have the patient take some deep breaths. DIF: Cognitive Level: Apply (application) REF: 1394 OBJ: Special Questions: Prioritization TOP: Nursing Process: Assessment MSC: NCLEX: Physiological Integrity

Which information about the patient who has had a subarachnoid hemorrhage is most important to communicate to the health care provider? a. The patient complains of having a stiff neck. b. The patient's blood pressure (BP) is 90/50 mm Hg. c. The patient reports a severe and unrelenting headache. d. The cerebrospinal fluid (CSF) report shows red blood cells (RBCs).

ANS: B To prevent cerebral vasospasm and maintain cerebral perfusion, blood pressure needs to be maintained at a level higher than 90 mm Hg systolic after a subarachnoid hemorrhage. A low BP or drop in BP indicates a need to administer fluids and/or vasopressors to increase the BP. An ongoing headache, RBCs in the CSF, and a stiff neck are all typical clinical manifestations of a subarachnoid hemorrhage and do not need to be rapidly communicated to the health care provider. DIF: Cognitive Level: Apply (application) REF: 1393 OBJ: Special Questions: Prioritization TOP: Nursing Process: Assessment MSC: NCLEX: Physiological Integrity

When caring for a patient who is hospitalized with active tuberculosis (TB), the nurse observes a student nurse who is assigned to take care of a patient. Which action, if performed by the student nurse, would require an intervention by the nurse? a. The patient is offered a tissue from the box at the bedside. b. A surgical face mask is applied before visiting the patient. c. A snack is brought to the patient from the unit refrigerator. d. Hand washing is performed before entering the patient's room.

ANS: B A high-efficiency particulate-absorbing (HEPA) mask, rather than a standard surgical mask, should be used when entering the patient's room because the HEPA mask can filter out 100% of small airborne particles. Hand washing before entering the patient's room is appropriate. Because anorexia and weight loss are frequent problems in patients with TB, bringing food to the patient is appropriate. The student nurse should perform hand washing after handling a tissue that the patient has used, but no precautions are necessary when giving the patient an unused tissue

46. A 33-year-old male patient with a gunshot wound to the abdomen undergoes surgery, and a colostomy is formed as shown in the accompanying figure. Which information will be included in patient teaching? a. Stool will be expelled from both stomas. b. This type of colostomy is usually temporary. c. Soft, formed stool can be expected as drainage. d. Irrigations can regulate drainage from the stomas.

ANS: B A loop, or double-barrel stoma, is usually temporary. Stool will be expelled from the proximal stoma only. The stool from the transverse colon will be liquid and regulation through irrigations will not be possible.

Which action best describes the role of the certified registered nurse anesthetist (CRNA) on the surgical care team? a. Performs the same responsibilities as the anesthesiologist. b. Releases or discharges patients from the postanesthesia care area. c. Administers intraoperative anesthetics ordered by the anesthesiologist. d. Manages a patient's airway under the direct supervision of the anesthesiologist.

ANS: B A nurse anesthetist is a registered nurse who has graduated from an accredited nurse anesthesia program (minimally a master's degree program) and successfully completed a national certification examination to become a CRNA. The CRNA scope of practice includes, but is not limited to, the following: 1. Performing and documenting a preanesthetic assessment and evaluation 2. Developing and implementing a plan for delivering anesthesia 3. Selecting and initiating the planned anesthetic technique 4. Selecting, obtaining, and administering the anesthesia, adjuvant drugs, and fluids 5. Selecting, applying, and inserting appropriate noninvasive and invasive monitoring devices 6. Managing a patient's airway and pulmonary status 7. Managing emergence and recovery from anesthesia 8. Releasing or discharging patients from a postanesthesia care area

The registered nurse (RN) caring for an HIV-positive patient admitted with tuberculosis can delegate which action to unlicensed assistive personnel (UAP)? a. Teach the patient about how to use tissues to dispose of respiratory secretions. b. Stock the patient's room with all the necessary personal protective equipment. c. Interview the patient to obtain the names of family members and close contacts. d. Tell the patient's family members the reason for the use of airborne precautions.

ANS: B A patient diagnosed with tuberculosis would be placed on airborne precautions. Because all health care workers are taught about the various types of infection precautions used in the hospital, the UAP can safely stock the room with personal protective equipment. Obtaining contact information and patient teaching are higher-level skills that require RN education and scope of practice.

42. The nurse receives a change-of-shift report on the following patients with chronic obstructive pulmonary disease (COPD). Which patient should the nurse assess first? a. A patient with loud expiratory wheezes b. A patient with a respiratory rate of 38/minute c. A patient who has a cough productive of thick, green mucus d. A patient with jugular venous distention and peripheral edema

ANS: B A respiratory rate of 38/minute indicates severe respiratory distress, and the patient needs immediate assessment and intervention to prevent possible respiratory arrest. The other patients also need assessment as soon as possible, but they do not need to be assessed as urgently as the tachypneic patient. DIF: Cognitive Level: Analyze (analysis) REF: 576 OBJ: Special Questions: Prioritization; Multiple Patients TOP: Nursing Process: Assessment MSC: NCLEX: Safe and Effective Care Environment

27. A patient with chronic obstructive pulmonary disease (COPD) has rhonchi throughout the lung fields and a chronic, nonproductive cough. Which nursing intervention will be most effective? a. Change the oxygen flow rate to the highest prescribed rate. b. Teach the patient to use the Flutter airway clearance device. c. Reinforce the ongoing use of pursed lip breathing techniques. d. Teach the patient about consistent use of inhaled corticosteroids.

ANS: B Airway clearance devices assist with moving mucus into larger airways where it can more easily be expectorated. The other actions may be appropriate for some patients with COPD, but they are not indicated for this patient's problem of thick mucus secretions. DIF: Cognitive Level: Apply (application) REF: 595 TOP: Nursing Process: Implementation MSC: NCLEX: Physiological Integrity

53. The nurse is admitting a 67-year-old patient with new-onset steatorrhea. Which question is most important for the nurse to ask? a. "How much milk do you usually drink?" b. "Have you noticed a recent weight loss?" c. "What time of day do your bowels move?" d. "Do you eat meat or other animal products?"

ANS: B Although all of the questions provide useful information, it is most important to determine if the patient has an imbalance in nutrition because of the steatorrhea.

9. A patient being admitted with an acute exacerbation of ulcerative colitis reports crampy abdominal pain and passing 15 or more bloody stools a day. The nurse will plan to a. administer IV metoclopramide (Reglan). b. discontinue the patient's oral food intake. c. administer cobalamin (vitamin B12) injections. d. teach the patient about total colectomy surgery.

ANS: B An initial therapy for an acute exacerbation of inflammatory bowel disease (IBD) is to rest the bowel by making the patient NPO. Metoclopramide increases peristalsis and will worsen symptoms. Cobalamin (vitamin B12) is absorbed in the ileum, which is not affected by ulcerative colitis. Although total colectomy is needed for some patients, there is no indication that this patient is a candidate.

Before administration of captopril (Capoten) to a patient with stage 2 chronic kidney disease (CKD), the nurse will check the patient's a. glucose. b. potassium. c. creatinine. d. phosphate.

ANS: B Angiotensin-converting enzyme (ACE) inhibitors are frequently used in patients with CKD because they delay the progression of the CKD, but they cause potassium retention. Therefore careful monitoring of potassium levels is needed in patients who are at risk for hyperkalemia. The other laboratory values would also be monitored in patients with CKD but would not affect whether the captopril was given or not.

Which topic will the nurse teach after a patient has had outpatient cataract surgery and lens implantation? a. Use of oral opioids for pain control b. Administration of corticosteroid eye drops c. Importance of coughing and deep breathing exercises d. Need for bed rest for the first 1 to 2 days after the surgery

ANS: B Antibiotic and corticosteroid eye drops are commonly prescribed after cataract surgery. The patient should be able to administer them using safe technique. Pain is not expected after cataract surgery and opioids will not be needed. Coughing and deep breathing exercises are not needed because a general anesthetic agent is not used. There is no bed rest restriction after cataract surgery

Which nursing activity is appropriate for the registered nurse (RN) working in the eye clinic to delegate to experienced unlicensed assistive personnel (UAP)? a. Instilling antiviral drops for a patient with a corneal ulcer b. Application of a warm compress to a patient's hordeolum c. Instruction about hand washing for a patient with herpes keratitis d. Looking for eye irritation in a patient with possible conjunctivitis

ANS: B Application of cold and warm packs is included in UAP education and the ability to accomplish this safely would be expected for UAP working in an eye clinic. Medication administration, patient teaching, and assessment are high-level skills appropriate for the education and legal practice level of the RN

18. The nurse preparing for the annual physical exam of a 50-year-old man will plan to teach the patient about a. endoscopy. b. colonoscopy. c. computerized tomography screening. d. carcinoembryonic antigen (CEA) testing.

ANS: B At age 50, individuals with an average risk for colorectal cancer (CRC) should begin screening for CRC. Colonoscopy is the gold standard for CRC screening. The other diagnostic tests are not recommended as part of a routine annual physical exam at age 50.

A patient with idiopathic pulmonary arterial hypertension (IPAH) is receiving nifedipine (Procardia). Which assessment would best indicate to the nurse that the patient's condition is improving? a. Blood pressure (BP) is less than 140/90 mm Hg. b. Patient reports decreased exertional dyspnea. c. Heart rate is between 60 and 100 beats/minute. d. Patient's chest x-ray indicates clear lung fields.

ANS: B Because a major symptom of IPAH is exertional dyspnea, an improvement in this symptom would indicate that the medication was effective. Nifedipine will affect BP and heart rate, but these parameters would not be used to monitor the effectiveness of therapy for a patient with IPAH. The chest x-ray will show clear lung fields even if the therapy is not effective

The charge nurse observes a newly hired nurse performing all the following interventions for a patient who has just undergone right cataract removal and an intraocular lens implant. Which one requires that the charge nurse intervene? a. The nurse leaves the eye shield in place. b. The nurse encourages the patient to cough. c. The nurse elevates the patient's head to 45 degrees. d. The nurse applies corticosteroid drops to the right eye.

ANS: B Because coughing will increase intraocular pressure, patients are generally taught to avoid coughing during the acute postoperative time. The other actions are appropriate for a patient after having this surgery

1. A 38-year-old female patient states that she is using topical fluorouracil to treat actinic keratoses on her face. Which additional assessment information will be most important for the nurse to obtain? a. History of sun exposure by the patient b. Method of birth control used by the patient c. Length of time the patient has used fluorouracil d. Appearance of the treated areas on the patients face

ANS: B Because fluorouracil is teratogenic, it is essential that the patient use a reliable method of birth control. The other information is also important for the nurse to obtain, but lack of reliable birth control has the most potential for serious adverse medication effects. DIF: Cognitive Level: Apply (application) REF: 399 TOP: Nursing Process: Assessment

A 42-year-old patient admitted with acute kidney injury due to dehydration has oliguria, anemia, and hyperkalemia. Which prescribed actions should the nurse take first? a. Insert a urinary retention catheter. b. Place the patient on a cardiac monitor. c. Administer epoetin alfa (Epogen, Procrit). d. Give sodium polystyrene sulfonate (Kayexalate).

ANS: B Because hyperkalemia can cause fatal cardiac dysrhythmias, the initial action should be to monitor the cardiac rhythm. Kayexalate and Epogen will take time to correct the hyperkalemia and anemia. The catheter allows monitoring of the urine output but does not correct the cause of the renal failure

35. A patient in the emergency department has just been diagnosed with peritonitis caused by a ruptured diverticulum. Which prescribed intervention will the nurse implement first? a. Insert a urinary catheter to drainage. b. Infuse metronidazole (Flagyl) 500 mg IV. c. Send the patient for a computerized tomography scan. d. Place a nasogastric (NG) tube to intermittent low suction.

ANS: B Because peritonitis can be fatal if treatment is delayed, the initial action should be to start antibiotic therapy (after any ordered cultures are obtained). The other actions can be done after antibiotic therapy is initiated.

Which nursing action is appropriate when coaching obese adults enrolled in a behavior modification program? a. Having the adults write down the caloric intake of each meal b. Asking the adults about situations that tend to increase appetite c. Suggesting that the adults plan rewards, such as sugarless candy, for achieving their goals d. Encouraging the adults to eat small amounts frequently rather than having scheduled meals

ANS: B Behavior modification programs focus on how and when the person eats and de-emphasize aspects such as calorie counting. Nonfood rewards are recommended for achievement of weight-loss goals. Patients are often taught to restrict eating to designated meals when using behavior modification.

20. A 74-year-old patient preparing to undergo a colon resection for cancer of the colon asks about the elevated carcinoembryonic antigen (CEA) test result. The nurse explains that the test is used to a. identify any metastasis of the cancer. b. monitor the tumor status after surgery. c. confirm the diagnosis of a specific type of cancer. d. determine the need for postoperative chemotherapy.

ANS: B CEA is used to monitor for cancer recurrence after surgery. CEA levels do not help to determine whether there is metastasis of the cancer. Confirmation of the diagnosis is made on the basis of biopsy. Chemotherapy use is based on factors other than CEA.

The nurse provides discharge instructions to a patient who has an immune deficiency involving the T lymphocytes. Which screening should the nurse include in the teaching plan for this patient? a. Screening for allergies b. Screening for malignancy c. Antibody deficiency screening d. Screening for autoimmune disorders

ANS: B Cell-mediated immunity is responsible for the recognition and destruction of cancer cells. Allergic reactions, autoimmune disorders, and antibody deficiencies are mediated primarily by B lymphocytes and humoral immunity.

A 58-year-old man with blunt abdominal trauma from a motor vehicle crash undergoes peritoneal lavage. If the lavage returns brown fecal drainage, which action will the nurse plan to take next? a. Auscultate the bowel sounds. b. Prepare the patient for surgery. c. Check the patient's oral temperature. d. Obtain information about the accident.

B

A patient who has never had any prior surgeries tells the nurse doing the preoperative assessment about an allergy to bananas and avocados. Which action is most important for the nurse to take? a. Notify the dietitian about the food allergies. b. Alert the surgery center about a possible latex allergy. c. Reassure the patient that all allergies are noted on the medical record. d. Ask whether the patient uses antihistamines to reduce allergic reactions.

ANS: B Certain food allergies (e.g., eggs, avocados, bananas, chestnuts, potatoes, peaches) are related to latex allergies. When a patient is allergic to latex, special nonlatex materials are used during surgical procedures, and the staff will need to know about the allergy in advance to obtain appropriate nonlatex materials and have them available during surgery. The other actions also may be appropriate, but prevention of allergic reaction during surgery is the most important action.

Which information in the preoperative patient's medication history is most important to communicate to the health care provider? a. The patient uses acetaminophen (Tylenol) occasionally for aches and pains. b. The patient takes garlic capsules daily but did not take any on the surgical day. c. The patient has a history of cocaine use but quit using the drug over 10 years ago. d. The patient took a sedative medication the previous night to assist in falling asleep.

ANS: B Chronic use of garlic may predispose to intraoperative and postoperative bleeding. The use of a sedative the previous night, occasional acetaminophen use, and a distant history of cocaine use will not usually affect the surgical outcome.

A female patient with chronic kidney disease (CKD) is receiving peritoneal dialysis with 2 L inflows. Which information should the nurse report immediately to the health care provider? a. The patient has an outflow volume of 1800 mL. b. The patient's peritoneal effluent appears cloudy. c. The patient has abdominal pain during the inflow phase. d. The patient's abdomen appears bloated after the inflow.

ANS: B Cloudy appearing peritoneal effluent is a sign of peritonitis and should be reported immediately so that treatment with antibiotics can be started. The other problems can be addressed through nursing interventions such as slowing the inflow and repositioning the patient

The nurse determines that teaching regarding cobalamin injections has been effective when the patient with chronic atrophic gastritis states which of the following? a. "The cobalamin injections will prevent gastric inflammation." b. "The cobalamin injections will prevent me from becoming anemic." c. "These injections will increase the hydrochloric acid in my stomach." d. "These injections will decrease my risk for developing stomach cancer."

ANS: B Cobalamin supplementation prevents the development of pernicious anemia. Chronic gastritis may cause achlorhydria, but cobalamin does not correct this. The loss of intrinsic factor secretion with chronic gastritis is permanent, and the patient will need lifelong supplementation with cobalamin. The incidence of stomach cancer is higher in patients with chronic gastritis, but cobalamin does not reduce the risk for stomach cancer

32. The nurse is assessing a 31-year-old female patient with abdominal pain. Th nurse,who notes that there is ecchymosis around the area of umbilicus, will document this finding as a. Cullen sign. b. Rovsing sign. c. McBurney sign. d. Grey-Turner's signt.

ANS: B Cullen sign is ecchymosis around the umbilicus. Rovsing sign occurs when palpation of the left lower quadrant causes pain in the right lower quadrant. Deep tenderness at McBurney's point (halfway between the umbilicus and the right iliac crest), known as McBurney's sign, is a sign of acute appendicitis.

A patient has arrived for a scheduled hemodialysis session. Which nursing action is most appropriate for the registered nurse (RN) to delegate to a dialysis technician? a. Teach the patient about fluid restrictions. b. Check blood pressure before starting dialysis. c. Assess for causes of an increase in predialysis weight. d. Determine the ultrafiltration rate for the hemodialysis.

ANS: B Dialysis technicians are educated in monitoring for blood pressure. Assessment, adjustment of the appropriate ultrafiltration rate, and patient teaching require the education and scope of practice of an RN.

18. A patient hospitalized with chronic heart failure eats only about 50% of each meal and reports "feeling too tired to eat." Which action should the nurse take first? a. Teach the patient about the importance of good nutrition. b. Serve multiple small feedings of high-calorie, high-protein foods. c. Obtain an order for enteral feedings of liquid nutritional supplements. d. Consult with the health care provider about providing parenteral nutrition (PN).

ANS: B Eating small amounts of food frequently throughout the day is less fatiguing and will improve the patient's ability to take in more nutrients. Teaching the patient may be appropriate, but will not address the patient's inability to eat more because of fatigue. Tube feedings or PN may be needed if the patient is unable to take in enough nutrients orally, but increasing the oral intake should be attempted first.

Which assessment action will help the nurse determine if an obese patient has metabolic syndrome? a. Take the patient's apical pulse. b. Check the patient's blood pressure. c. Ask the patient about dietary intake. d. Dipstick the patient's urine for protein.

ANS: B Elevated blood pressure is one of the characteristics of metabolic syndrome. The other information also may be obtained by the nurse, but it will not assist with the diagnosis of metabolic syndrome.

14. After a total proctocolectomy and permanent ileostomy, the patient tells the nurse, "I cannot manage all these changes. I don't want to look at the stoma." What is the best action by the nurse? a. Reassure the patient that ileostomy care will become easier. b. Ask the patient about the concerns with stoma management. c. Develop a detailed written list of ostomy care tasks for the patient. d. Postpone any teaching until the patient adjusts to the ileostomy.

ANS: B Encouraging the patient to share concerns assists in helping the patient adjust to the body changes. Acknowledgment of the patient's feelings and concerns is important rather than offering false reassurance. Because the patient indicates that the feelings about the ostomy are the reason for the difficulty with the many changes, development of a detailed ostomy care plan will not improve the patient's ability to manage the ostomy. Although detailed ostomy teaching may be postponed, the nurse should offer teaching about some aspects of living with an ostomy.

A family member of a 28-year-old patient who has suffered massive abdominal trauma in an automobile accident asks the nurse why the patient is receiving famotidine (Pepcid). The nurse will explain that the medication will a. decrease nausea and vomiting. b. inhibit development of stress ulcers. c. lower the risk for H. pylori infection. d. prevent aspiration of gastric contents.

ANS: B Famotidine is administered to prevent the development of physiologic stress ulcers, which are associated with a major physiologic insult such as massive trauma. Famotidine does not decrease nausea or vomiting, prevent aspiration, or prevent H. pylori infection

A patient in the emergency department has just been diagnosed with peritonitis caused by a ruptured diverticulum. Which prescribed intervention will the nurse implement first? a. Insert a urinary catheter to drainage. b. Infuse metronidazole (Flagyl) 500 mg IV. c. Send the patient for a computerized tomography scan. d. Place a nasogastric (NG) tube to intermittent low suction.

B

33. A 54-year-old critically ill patient with sepsis is frequently incontinent of watery stools. What action by the nurse will prevent complications associated with ongoing incontinence? a. Apply incontinence briefs. b. Use a fecal management system c. Insert a rectal tube with a drainage bag. d. Assist the patient to a commode frequently.

ANS: B Fecal management systems are designed to contain loose stools and can be in place for as long as 4 weeks without causing damage to the rectum or anal sphincters. Although incontinence briefs may be helpful, unless they are changed frequently, they are likely to increase the risk for skin breakdown. Rectal tubes are avoided because of possible damage to the anal sphincter and ulceration of the rectal mucosa. A critically ill patient will not be able to tolerate getting up frequently to use the commode or bathroom.

The nurse is titrating the IV fluid infusion rate immediately after a patient has had kidney transplantation. Which parameter will be most important for the nurse to consider? a. Heart rate b. Urine output c. Creatinine clearance d. Blood urea nitrogen (BUN) level

ANS: B Fluid volume is replaced based on urine output after transplant because the urine output can be as high as a liter an hour. The other data will be monitored but are not the most important determinants of fluid infusion rate

The occupational health nurse is caring for an employee who is complaining of bilateral eye pain after a cleaning solution splashed into the employee's eyes. Which action will the nurse take first? a. Apply ice packs to both eyes. b. Flush the eyes with sterile saline. c. Apply antiseptic ophthalmic ointment to the eyes. d. Cover the eyes with dry sterile patches and shields.

ANS: B Flushing of the eyes immediately is indicated for chemical exposure. Emergency treatment of a burn or foreign-body injury to the eyes includes protecting the eyes from further injury by covering them with dry sterile dressings and protective shields. Flushing of the eyes immediately is indicated only for chemical exposure. In the case of chemical exposure, the nurse should begin treatment by flushing the eyes until the patient has been assessed by a health care provider and orders are available

14. Which finding by the nurse for a patient with a nursing diagnosis of impaired gas exchange will be most useful in evaluating the effectiveness of treatment? a. Even, unlabored respirations b. Pulse oximetry reading of 92% c. Respiratory rate of 18 breaths/minute d. Absence of wheezes, rhonchi, or crackles

ANS: B For the nursing diagnosis of impaired gas exchange, the best data for evaluation are arterial blood gases (ABGs) or pulse oximetry. The other data may indicate either improvement or impending respiratory failure caused by fatigue. DIF: Cognitive Level: Apply (application) REF: 598 TOP: Nursing Process: Evaluation MSC: NCLEX: Physiological Integrity

Which information will the nurse teach a 23-year-old patient with lactose intolerance? a. Ice cream is relatively low in lactose. b. Live-culture yogurt is usually tolerated. c. Heating milk will break down the lactose. d. Nonfat milk is a better choice than whole milk.

B

28. The nurse provides dietary teaching for a patient with chronic obstructive pulmonary disease (COPD) who has a low body mass index (BMI). Which patient statement indicates that the teaching has been effective? a. "I will drink lots of fluids with my meals." b. "I can have ice cream as a snack every day." c. "I will exercise for 15 minutes before meals." d. "I will decrease my intake of meat and poultry."

ANS: B High-calorie foods like ice cream are an appropriate snack for patients with COPD. Fluid intake of 3 L/day is recommended, but fluids should be taken between meals rather than with meals to improve oral intake of solid foods. The patient should avoid exercise for an hour before meals to prevent fatigue while eating. Meat and dairy products are high in protein and are good choices for the patient with COPD. DIF: Cognitive Level: Apply (application) REF: 595-596 TOP: Nursing Process: Evaluation MSC: NCLEX: Physiological Integrity

23. Which information will the nurse include in teaching a patient who had a proctocolectomy and ileostomy for ulcerative colitis? a. Restrict fluid intake to prevent constant liquid drainage from the stoma. b. Use care when eating high-fiber foods to avoid obstruction of the ileum. c. Irrigate the ileostomy daily to avoid having to wear a drainage appliance. d. Change the pouch every day to prevent leakage of contents onto the skin.

ANS: B High-fiber foods are introduced gradually and should be well chewed to avoid obstruction of the ileostomy. Patients with ileostomies lose the absorption of water in the colon and need to take in increased amounts of fluid. The pouch should be drained frequently but is changed every 5 to 7 days. The drainage from an ileostomy is liquid and continuous, so control by irrigation is not possible.

A nurse should include which instructions when teaching a patient with repeated hordeolum how to prevent further infection? a. Apply cold compresses at the first sign of recurrence. b. Discard all open or used cosmetics applied near the eyes. c. Wash the scalp and eyebrows with an antiseborrheic shampoo. d. Be examined for recurrent sexually transmitted infections (STIs).

ANS: B Hordeolum (styes) are commonly caused by Staphylococcus aureus, which may be present in cosmetics that the patient is using. Warm compresses are recommended to treat hordeolum. Antiseborrheic shampoos are recommended for seborrheic blepharitis. Patients with adult inclusion conjunctivitis, which is caused by Chlamydia trachomatis, should be referred for sexually transmitted infection (STI) testing

Which nursing action will the nurse include in the plan of care for a 35-year-old male patient admitted with an exacerbation of inflammatory bowel disease (IBD)? a. Restrict oral fluid intake. b. Monitor stools for blood. c. Ambulate four times daily. d. Increase dietary fiber intake.

B

The nurse prepares to administer the following medications to a hospitalized patient with human immunodeficiency (HIV). Which medication is most important to administer at the right time? a. Oral acyclovir (Zovirax) b. Oral saquinavir (Invirase) c. Nystatin (Mycostatin) tablet d. Aerosolized pentamidine (NebuPent)

ANS: B It is important that antiretrovirals be taken at the prescribed time every day to avoid developing drug-resistant HIV. The other medications should also be given as close as possible to the correct time, but they are not as essential to receive at the same time every day

10. A patient reports chronic itching of the ankles and continuously scratches the area. Which assessment finding will the nurse expect? a. Hypertrophied scars on both ankles b. Thickening of the skin around the ankles c. Yellowish-brown skin around both ankles d. Complete absence of melanin in both ankles

ANS: B Lichenification is likely to occur in areas where the patient scratches the skin frequently. Lichenification results in thickening of the skin with accentuated normal skin markings. Vitiligo is the complete absence of melanin in the skin. Keloids are hypertrophied scars. Yellowish-brown skin indicates jaundice. Vitiligo, keloids, and jaundice do not usually occur as a result of scratching the skin. DIF: Cognitive Level: Understand (comprehension) REF: 402 TOP: Nursing Process: Assessment

Which topic will the nurse teach after a patient has had outpatient cataract surgery and lens implantation? a. Use of oral opioids for pain control b. Administration of corticosteroid eye drops c. Importance of coughing and deep breathing exercises d. Need for bed rest for the first 1 to 2 days after the surgery

B

A pregnant woman with a history of asymptomatic chronic human immunodeficiency virus (HIV) infection is seen at the clinic. The patient states, "I am very nervous about making my baby sick." Which information will the nurse include when teaching the patient? a. The antiretroviral medications used to treat HIV infection are teratogenic. b. Most infants born to HIV-positive mothers are not infected with the virus. c. Because she is at an early stage of HIV infection, the infant will not contract HIV. d. It is likely that her newborn will become infected with HIV unless she uses antiretroviral therapy (ART).

ANS: B Only 25% of infants born to HIV-positive mothers develop HIV infection, even when the mother does not use ART during pregnancy. The percentage drops to 2% when ART is used. Perinatal transmission can occur at any stage of HIV infection (although it is less likely to occur when the viral load is lower). ART can safely be used in pregnancy, although some ART drugs should be avoided.

To determine whether treatment is effective for a patient with primary open-angle glaucoma (POAG), the nurse can evaluate the patient for improvement by a. questioning the patient about blurred vision. b. noting any changes in the patient's visual field. c. asking the patient to rate the pain using a 0 to 10 scale. d. assessing the patient's depth perception when climbing stairs.

ANS: B POAG develops slowly and without symptoms except for a gradual loss of visual fields. Acute closed-angle glaucoma may present with excruciating pain, colored halos, and blurred vision. Problems with depth perception are not associated with POAG

A patient who uses injectable illegal drugs asks the nurse about preventing acquired immunodeficiency syndrome (AIDS). Which response by the nurse is best? a. "Avoid sexual intercourse when using injectable drugs." b. "It is important to participate in a needle-exchange program." c. "You should ask those who share equipment to be tested for HIV." d. "I recommend cleaning drug injection equipment before each use."

ANS: B Participation in needle-exchange programs has been shown to decrease and control the rate of HIV infection. Cleaning drug equipment before use also reduces risk, but it might not be consistently practiced. HIV antibodies do not appear for several weeks to months after exposure, so testing drug users would not be very effective in reducing risk for HIV exposure. It is difficult to make appropriate decisions about sexual activity when under the influence of drugs.

The nurse receives change-of-shift report on the following four patients. Which patient should the nurse assess first? a. A 23-year-old patient with cystic fibrosis who has pulmonary function testing scheduled b. A 46-year-old patient on bed rest who is complaining of sudden onset of shortness of breath c. A 77-year-old patient with tuberculosis (TB) who has four antitubercular medications due in 15 minutes d. A 35-year-old patient who was admitted the previous day with pneumonia and has a temperature of 100.2° F (37.8° C)

ANS: B Patients on bed rest who are immobile are at high risk for deep vein thrombosis (DVT). Sudden onset of shortness of breath in a patient with a DVT suggests a pulmonary embolism and requires immediate assessment and action such as oxygen administration. The other patients should also be assessed as soon as possible, but there is no indication that they may need immediate action to prevent clinical deterioration

A patient with glaucoma who has been using timolol (Timoptic) drops for several days tells the nurse that the eye drops cause eye burning and visual blurriness for a short time after administration. The best response to the patient's statement is a. "Those symptoms may indicate a need for an increased dosage of the eye drops." b. "The drops are uncomfortable, but it is important to use them to retain your vision." c. "These are normal side effects of the drug, which should be less noticeable with time." d. "Notify your health care provider so that different eye drops can be prescribed for you."

ANS: B Patients should be instructed that eye discomfort and visual blurring are expected side effects of the ophthalmic drops but that the drops must be used to prevent further visual-field loss. The temporary burning and visual blurriness might not lessen with ongoing use, are not relieved by avoiding systemic absorption, and are not symptoms of glaucoma

50. A new 19-year-old male patient has familial adenomatous polyposis (FAP). Which action will the nurse in the gastrointestinal clinic include in the plan of care? a. Obtain blood samples for DNA analysis. b. Schedule the patient for yearly colonoscopy. c. Provide preoperative teaching about total colectomy. d. Discuss lifestyle modifications to decrease cancer risk.

ANS: B Patients with FAP should have annual colonoscopy starting at age 16 and usually have total colectomy by age 25 to avoid developing colorectal cancer. DNA analysis is used to make the diagnosis, but is not needed now for this patient. Lifestyle modifications will not decrease cancer risk for this patient.

55. Which prescribed intervention for a 61-year-old female patient with chronic short bowel syndrome will the nurse question? a. Ferrous sulfate (Feosol) 325 mg daily b. Senna (Senokot) 1 tablet every day c. Psyllium (Metamucil) 2.1 grams 3 times daily d. Diphenoxylate with atropine (Lomotil) prn loose stools

ANS: B Patients with short bowel syndrome have diarrhea because of decreased nutrient and fluid absorption and would not need stimulant laxatives. Iron supplements are used to prevent iron-deficiency anemia, bulk-forming laxatives help make stools less watery, and opioid antidiarrheal drugs are helpful in slowing intestinal transit time.

A 62-year-old female patient has been hospitalized for 8 days with acute kidney injury (AKI) caused by dehydration. Which information will be most important for the nurse to report to the health care provider? a. The creatinine level is 3.0 mg/dL. b. Urine output over an 8-hour period is 2500 mL. c. The blood urea nitrogen (BUN) level is 67 mg/dL. d. The glomerular filtration rate is <30 mL/min/1.73m2.

ANS: B The high urine output indicates a need to increase fluid intake to prevent hypovolemia. The other information is typical of AKI and will not require a change in therapy

The nurse teaches a patient diagnosed with systemic lupus erythematosus (SLE) about plasmapheresis. What instructions about plasmapheresis should the nurse include in the teaching plan? a. Plasmapheresis will eliminate eosinophils and basophils from blood. b. Plasmapheresis will remove antibody-antigen complexes from circulation. c. Plasmapheresis will prevent foreign antibodies from damaging various body tissues. d. Plasmapheresis will decrease the damage to organs caused by attacking T lymphocytes.

ANS: B Plasmapheresis is used in SLE to remove antibodies, antibody-antigen complexes, and complement from blood. T lymphocytes, foreign antibodies, eosinophils, and basophils do not directly contribute to the tissue damage in SLE.

A patient who has diabetes and uses insulin to control blood glucose has been NPO since midnight before having a knee replacement surgery. Which action should the nurse take? a. Withhold the usual scheduled insulin dose because the patient is NPO. b. Obtain a blood glucose measurement before any insulin administration. c. Give the patient the usual insulin dose because stress will increase the blood glucose. d. Administer a lower dose of insulin because there will be no oral intake before surgery.

ANS: B Preoperative insulin administration is individualized to the patient, and the current blood glucose will provide the most reliable information about insulin needs. It is not possible to predict whether the patient will require no insulin, a lower dose, or a higher dose without blood glucose monitoring.

A patient diagnosed with external otitis is being discharged from the emergency department with an ear wick in place. Which statement by the patient indicates a need for further teaching? a. "I will apply the eardrops to the cotton wick in the ear canal." b. "I can use aspirin or acetaminophen (Tylenol) for pain relief." c. "I will clean the ear canal daily with a cotton-tipped applicator." d. "I can use warm compresses to the outside of the ear for comfort."

C

Which information will the nurse include when teaching a patient with peptic ulcer disease about the effect of ranitidine (Zantac)? a. "Ranitidine absorbs the gastric acid." b. "Ranitidine decreases gastric acid secretion." c. "Ranitidine constricts the blood vessels near the ulcer." d. "Ranitidine covers the ulcer with a protective material."

ANS: B Ranitidine is a histamine-2 (H2) receptor blocker, which decreases the secretion of gastric acid. The response beginning, "Ranitidine constricts the blood vessels" describes the effect of vasopressin. The response "Ranitidine absorbs the gastric acid" describes the effect of antacids. The response beginning "Ranitidine covers the ulcer" describes the action of sucralfate (Carafate).

6. A 58-year-old man with blunt abdominal trauma from a motor vehicle crash undergoes peritoneal lavage. If the lavage returns brown fecal drainage, which action will the nurse plan to take next? a. Auscultate the bowel sounds. b. Prepare the patient for surgery. c. Check the patient's oral temperature. d. Obtain information about the accident.

ANS: B Return of brown drainage and fecal material suggests perforation of the bowel and the need for immediate surgery. Auscultation of bowel sounds, checking the temperature, and obtaining information about the accident are appropriate actions, but the priority is to prepare to send the patient for emergency surgery.

A 73-year-old patient with Parkinson's disease has a nursing diagnosis of impaired physical mobility related to bradykinesia. Which action will the nurse include in the plan of care? a. Instruct the patient in activities that can be done while lying or sitting. b. Suggest that the patient rock from side to side to initiate leg movement. c. Have the patient take small steps in a straight line directly in front of the feet. d. Teach the patient to keep the feet in contact with the floor and slide them forward.

ANS: B Rocking the body from side to side stimulates balance and improves mobility. The patient will be encouraged to continue exercising because this will maintain functional abilities. Maintaining a wide base of support will help with balance. The patient should lift the feet and avoid a shuffling gait. DIF: Cognitive Level: Apply (application) REF: 1437 TOP: Nursing Process: Planning MSC: NCLEX: Physiological Integrity

A 26-year-old patient with a family history of stomach cancer asks the nurse about ways to decrease the risk for developing stomach cancer. The nurse will teach the patient to avoid a. emotionally stressful situations. b. smoked foods such as ham and bacon. c. foods that cause distention or bloating. d. chronic use of H2 blocking medications.

ANS: B Smoked foods such as bacon, ham, and smoked sausage increase the risk for stomach cancer. Stressful situations, abdominal distention, and use of H2 blockers are not associated with an increased incidence of stomach cancer

Which action will the nurse take when performing ear irrigation for a patient with cerumen impaction? a. Assist the patient to a supine position for the irrigation. b. Fill the irrigation syringe with body-temperature solution. c. Use a sterile applicator to clean the ear canal before irrigating. d. Occlude the ear canal completely with the syringe while irrigating.

ANS: B Solution at body temperature is used for ear irrigation. The patient should be sitting for the procedure. Use of cotton-tipped applicators to clear the ear may result in forcing the cerumen deeper into the ear canal. The ear should not be completely occluded with the syringe

A 25-year-old male patient has been admitted with a severe crushing injury after an industrial accident. Which laboratory result will be most important to report to the health care provider? a. Serum creatinine level 2.1 mg/dL b. Serum potassium level 6.5 mEq/L c. White blood cell count 11,500/µL d. Blood urea nitrogen (BUN) 56 mg/dL

ANS: B The hyperkalemia associated with crushing injuries may cause cardiac arrest and should be treated immediately. The nurse also will report the other laboratory values, but abnormalities in these are not immediately life threatening

11. Which patient statement indicates that the nurse's teaching about sulfasalazine (Azulfidine) for ulcerative colitis has been effective? a. "The medication will be tapered if I need surgery." b. "I will need to use a sunscreen when I am outdoors." c. "I will need to avoid contact with people who are sick." d. "The medication will prevent infections that cause the diarrhea."

ANS: B Sulfasalazine may cause photosensitivity in some patients. It is not used to treat infections. Sulfasalazine does not reduce immune function. Unlike corticosteroids, tapering of sulfasalazine is not needed.

The nurse teaches a patient about the transmission of pulmonary tuberculosis (TB). Which statement, if made by the patient, indicates that teaching was effective? a. "I will avoid being outdoors whenever possible." b. "My husband will be sleeping in the guest bedroom." c. "I will take the bus instead of driving to visit my friends." d. "I will keep the windows closed at home to contain the germs."

ANS: B Teach the patient how to minimize exposure to close contacts and household members. Homes should be well ventilated, especially the areas where the infected person spends a lot of time. While still infectious, the patient should sleep alone, spend as much time as possible outdoors, and minimize time in congregate settings or on public transportation

Eight years after seroconversion, a human immunodeficiency virus (HIV)-infected patient has a CD4+ cell count of 800/µL and an undetectable viral load. What is the priority nursing intervention at this time? a. Teach about the effects of antiretroviral agents. b. Encourage adequate nutrition, exercise, and sleep. c. Discuss likelihood of increased opportunistic infections. d. Monitor for symptoms of acquired immunodeficiency syndrome (AIDS).

ANS: B The CD4+ level for this patient is in the normal range, indicating that the patient is the stage of asymptomatic chronic infection, when the body is able to produce enough CD4+ cells to maintain a normal CD4+ count. AIDS and increased incidence of opportunistic infections typically develop when the CD4+ count is much lower than normal. Although the initiation of ART is highly individual, it would not be likely that a patient with a normal CD4+ level would receive ART.

While in the holding area, a patient reveals to the nurse that his father had a high fever after surgery. What action by the nurse is a priority? a. Place a medical alert sticker on the front of the patient's chart. b. Alert the anesthesia care provider of the family member's reaction to surgery. c. Reassure the patient that there will be close monitoring during and after surgery. d. Administer 650 mg of acetaminophen (Tylenol) per rectum as a preventive measure.

ANS: B The anesthesia care provider (ACP) needs to be notified and made aware of the patient's family history in regards to anesthesia reactions. Malignant hyperthermia (MH) is a valid concern because the patient's father appears to have had a reaction to surgery. The ACP needs to be notified immediately, rather than waiting for a sticker to be noticed on the chart. Administering acetaminophen may not prevent MH. General anesthesia can be administered to patients with MH as long as precautions to avoid MH are taken and preparations are made to treat MH if it does occur

Which information about dietary management should the nurse include when teaching a patient with peptic ulcer disease (PUD)? a. "You will need to remain on a bland diet." b. "Avoid foods that cause pain after you eat them." c. "High-protein foods are least likely to cause you pain." d. "You should avoid eating any raw fruits and vegetables."

ANS: B The best information is that each individual should choose foods that are not associated with postprandial discomfort. Raw fruits and vegetables may irritate the gastric mucosa, but chewing well seems to decrease this problem and some patients may tolerate these foods well. High-protein foods help neutralize acid, but they also stimulate hydrochloric (HCl) acid secretion and may increase discomfort for some patients. Bland diets may be recommended during an acute exacerbation of PUD, but there is little scientific evidence to support their use

The nurse developing a teaching plan for a patient with herpes simplex keratitis should include which instruction? a. Apply antibiotic drops to the eye several times daily. b. Wash hands frequently and avoid touching the eyes. c. Apply a new occlusive dressing to the affected eye at bedtime. d. Use corticosteroid ophthalmic ointment to decrease inflammation.

ANS: B The best way to avoid the spread of infection from one eye to another is to avoid rubbing or touching the eyes and to use careful hand washing when touching the eyes is unavoidable. Occlusive dressings are not used for herpes keratitis. Herpes simplex is a virus and antibiotic drops will not be prescribed. Topical corticosteroids are immunosuppressive and typically are not ordered because they can contribute to a longer course of infection and more complications

A patient who has undergone a left tympanoplasty should be instructed to a. remain on bed rest. b. keep the head elevated. c. avoid blowing the nose. d. irrigate the left ear canal.

C

The nurse is assessing a patient 4 hours after a kidney transplant. Which information is most important to communicate to the health care provider? a. The urine output is 900 to 1100 mL/hr. b. The patient's central venous pressure (CVP) is decreased. c. The patient has a level 7 (0 to 10 point scale) incisional pain. d. The blood urea nitrogen (BUN) and creatinine levels are elevated.

ANS: B The decrease in CVP suggests hypovolemia, which must be rapidly corrected to prevent renal hypoperfusion and acute tubular necrosis. The other information is not unusual in a patient after a transplant

When a 74-year-old patient is seen in the health clinic with new development of a stooped posture, shuffling gait, and pill rolling-type tremor, the nurse will anticipate teaching the patient about a. oral corticosteroids. b. antiparkinsonian drugs. c. magnetic resonance imaging (MRI). d. electroencephalogram (EEG) testing.

ANS: B The diagnosis of Parkinson's is made when two of the three characteristic manifestations of tremor, rigidity, and bradykinesia are present. The confirmation of the diagnosis is made on the basis of improvement when antiparkinsonian drugs are administered. This patient has symptoms of tremor and bradykinesia. The next anticipated step will be treatment with medications. MRI and EEG are not useful in diagnosing Parkinson's disease, and corticosteroid therapy is not used to treat it. DIF: Cognitive Level: Apply (application) REF: 1434 TOP: Nursing Process: Planning MSC: NCLEX: Physiological Integrity

A patient is admitted to the emergency department with an open stab wound to the left chest. What is the first action that the nurse should take? a. Position the patient so that the left chest is dependent. b. Tape a nonporous dressing on three sides over the chest wound. c. Cover the sucking chest wound firmly with an occlusive dressing. d. Keep the head of the patient's bed at no more than 30 degrees elevation.

ANS: B The dressing taped on three sides will allow air to escape when intrapleural pressure increases during expiration, but it will prevent air from moving into the pleural space during inspiration. Placing the patient on the left side or covering the chest wound with an occlusive dressing will allow trapped air in the pleural space and cause tension pneumothorax. The head of the bed should be elevated to 30 to 45 degrees to facilitate breathing

Which medications will the nurse teach the patient about whose peptic ulcer disease is associated with Helicobacter pylori? a. Sucralfate (Carafate), nystatin (Mycostatin), and bismuth (Pepto-Bismol) b. Amoxicillin (Amoxil), clarithromycin (Biaxin), and omeprazole (Prilosec) c. Famotidine (Pepcid), magnesium hydroxide (Mylanta), and pantoprazole (Protonix) d. Metoclopramide (Reglan), bethanechol (Urecholine), and promethazine (Phenergan)

ANS: B The drugs used in triple drug therapy include a proton pump inhibitor such as omeprazole and the antibiotics amoxicillin and clarithromycin. The other combinations listed are not included in the protocol for H. pylori infection

16. Which action should the nurse take first when preparing to teach a frail 79-year-old Hispanic man who lives with an adult daughter about ways to improve nutrition? a. Ask the daughter about the patient's food preferences. b. Determine who shops for groceries and prepares the meals. c. Question the patient about how many meals per day are eaten. d. Assure the patient that culturally preferred foods will be included.

ANS: B The family member who shops for groceries and cooks will be in control of the patient's diet, so the nurse will need to ensure that this family member is involved in any teaching or discussion about the patient's nutritional needs. The other information will also be assessed and used but will not be useful in meeting the patient's nutritional needs unless nutritionally appropriate foods are purchased and prepared.

After 2 months of tuberculosis (TB) treatment with isoniazid (INH), rifampin (Rifadin), pyrazinamide (PZA), and ethambutol, a patient continues to have positive sputum smears for acid-fast bacilli (AFB). Which action should the nurse take next? a. Teach about treatment for drug-resistant TB treatment. b. Ask the patient whether medications have been taken as directed. c. Schedule the patient for directly observed therapy three times weekly. d. Discuss with the health care provider the need for the patient to use an injectable antibiotic.

ANS: B The first action should be to determine whether the patient has been compliant with drug therapy because negative sputum smears would be expected if the TB bacillus is susceptible to the medications and if the medications have been taken correctly. Assessment is the first step in the nursing process. Depending on whether the patient has been compliant or not, different medications or directly observed therapy may be indicated. The other options are interventions based on assumptions until an assessment has been completed

33. A patient with cystic fibrosis (CF) has blood glucose levels that are consistently between 180 to 250 mg/dL. Which nursing action will the nurse plan to implement? a. Discuss the role of diet in blood glucose control. b. Teach the patient about administration of insulin. c. Give oral hypoglycemic medications before meals. d. Evaluate the patient's home use of pancreatic enzymes.

ANS: B The glucose levels indicate that the patient has developed CF-related diabetes, and insulin therapy is required. Because the etiology of diabetes in CF is inadequate insulin production, oral hypoglycemic agents are not effective. Patients with CF need a high-calorie diet. Inappropriate use of pancreatic enzymes would not be a cause of hyperglycemia in a patient with CF. DIF: Cognitive Level: Apply (application) REF: 603 TOP: Nursing Process: Planning MSC: NCLEX: Physiological Integrity

5. The emergency department nurse is evaluating the effectiveness of therapy for a patient who has received treatment during an asthma attack. Which assessment finding is the best indicator that the therapy has been effective? a. No wheezes are audible. b. Oxygen saturation is >90%. c. Accessory muscle use has decreased. d. Respiratory rate is 16 breaths/minute.

ANS: B The goal for treatment of an asthma attack is to keep the oxygen saturation >90%. The other patient data may occur when the patient is too fatigued to continue with the increased work of breathing required in an asthma attack. DIF: Cognitive Level: Apply (application) REF: 569 TOP: Nursing Process: Evaluation MSC: NCLEX: Physiological Integrity

A patient who is anxious and has difficulty breathing seeks treatment after being stung by a wasp. What is the nurse's priority action? a. Have the patient lie down. b. Assess the patient's airway. c. Administer high-flow oxygen. d. Remove the stinger from the site.

ANS: B The initial action with any patient with difficulty breathing is to assess and maintain the airway. The other actions also are part of the emergency management protocol for anaphylaxis, but the priority is airway maintenance.

36. A 25-year-old male patient calls the clinic complaining of diarrhea for 24 hours. Which action should the nurse take first? a. Inform the patient that laboratory testing of blood and stools will be necessary. b. Ask the patient to describe the character of the stools and any associated symptoms. c. Suggest that the patient drink clear liquid fluids with electrolytes, such as Gatorade or Pedialyte. d. Advise the patient to use over-the-counter loperamide (Imodium) to slow gastrointestinal (GI) motility.

ANS: B The initial response by the nurse should be further assessment of the patient. The other responses may be appropriate, depending on what is learned in the assessment.

24. The nurse will determine that teaching a 67-year-old man to irrigate his new colostomy has been effective if the patient a. inserts the irrigation tubing 4 to 6 inches into the stoma. b. hangs the irrigating container 18 inches above the stoma. c. stops the irrigation and removes the irrigating cone if cramping occurs. d. fills the irrigating container with 1000 to 2000 mL of lukewarm tap water.

ANS: B The irrigating container should be hung 18 to 24 inches above the stoma. If cramping occurs, the irrigation should be temporarily stopped and the cone left in place. Five hundred to 1000 mL of water should be used for irrigation. An irrigation cone, rather than tubing, should be inserted into the stoma; 4 to 6 inches would be too far for safe insertion.

A high school teacher who has just been diagnosed with epilepsy after having a generalized tonic-clonic seizure tells the nurse, "I cannot teach anymore, it will be too upsetting if I have a seizure at work." Which response by the nurse is best? a. "You might benefit from some psychologic counseling." b. "Epilepsy usually can be well controlled with medications." c. "You will want to contact the Epilepsy Foundation for assistance." d. "The Department of Vocational Rehabilitation can help with work retraining."

ANS: B The nurse should inform the patient that most patients with seizure disorders are controlled with medication. The other information may be necessary if the seizures persist after treatment with antiseizure medications is implemented. DIF: Cognitive Level: Apply (application) REF: 1422 TOP: Nursing Process: Implementation MSC: NCLEX: Psychosocial Integrity

22. A severely malnourished patient reports that he is Jewish. The nurse's initial action to meet his nutritional needs will be to a. have family members bring in food. b. ask the patient about food preferences. c. teach the patient about nutritious Kosher foods. d. order nutrition supplements that are manufactured Kosher.

ANS: B The nurse's first action should be further assessment whether or not the patient follows any specific religious guidelines that impact nutrition. The other actions may also be appropriate, based on the information obtained during the assessment.

2. A 71-year-old male patient tells the nurse that growing old causes constipation so he has been using a suppository for constipation every morning. Which action should the nurse take first? a. Encourage the patient to increase oral fluid intake. b. Assess the patient about risk factors for constipation. c. Suggest that the patient increase intake of high-fiber foods. d. Teach the patient that a daily bowel movement is unnecessary.

ANS: B The nurse's initial action should be further assessment of the patient for risk factors for constipation and for his usual bowel pattern. The other actions may be appropriate but will be based on the assessment.

26. A young adult female patient with cystic fibrosis (CF) tells the nurse that she is not sure about getting married and having children some day. Which initial response by the nurse is best? a. "Are you aware of the normal lifespan for patients with CF?" b. "Do you need any information to help you with that decision?" c. "Many women with CF do not have difficulty conceiving children." d. "You will need to have genetic counseling before making a decision."

ANS: B The nurse's initial response should be to assess the patient's knowledge level and need for information. Although the lifespan for patients with CF is likely to be shorter than normal, it would not be appropriate for the nurse to address this as the initial response to the patient's comments. The other responses have accurate information, but the nurse should first assess the patient's understanding about the issues surrounding pregnancy. DIF: Cognitive Level: Apply (application) REF: 605-606 TOP: Nursing Process: Implementation MSC: NCLEX: Health Promotion and Maintenance

A patient who received a corneal transplant 2 weeks ago calls the ophthalmology clinic to report that his vision has not improved with the transplant. Which action should the nurse take? a. Suggest the patient arrange a ride to the clinic immediately. b. Ask about the presence of "floaters" in the patient's visual field. c. Remind the patient it may take months to restore vision after transplant. d. Teach the patient to continue using prescribed pupil-dilating medications.

C

17. After change-of-shift report, which patient will the nurse assess first? a. A 40-year-old woman whose parenteral nutrition infusion bag has 30 minutes of solution left b. A 40-year-old man with continuous enteral feedings who has developed pulmonary crackles c. A 30-year-old man with 4+ generalized pitting edema and severe protein-calorie malnutrition d. A 30-year-old woman whose gastrostomy tube is plugged after crushed medications were administered.

ANS: B The patient data suggest aspiration has occurred and rapid assessment and intervention are needed. The other patients should also be assessed as quickly as possible, but the data about them do not suggest any immediately life-threatening complications.

An adolescent patient seeks care in the emergency department after sharing needles for heroin injection with a friend who has hepatitis B. To provide immediate protection from infection, what medication will the nurse administer? a. Corticosteroids b. Gamma globulin c. Hepatitis B vaccine d. Fresh frozen plasma

ANS: B The patient should first receive antibodies for hepatitis B from injection of gamma globulin. The hepatitis B vaccination series should be started to provide active immunity. Fresh frozen plasma and corticosteroids will not be effective in preventing hepatitis B in the patient

13. The nurse teaches a patient about pursed lip breathing. Which action by the patient would indicate to the nurse that further teaching is needed? a. The patient inhales slowly through the nose. b. The patient puffs up the cheeks while exhaling. c. The patient practices by blowing through a straw. d. The patient's ratio of inhalation to exhalation is 1:3.

ANS: B The patient should relax the facial muscles without puffing the cheeks while doing pursed lip breathing. The other actions by the patient indicate a good understanding of pursed lip breathing. DIF: Cognitive Level: Apply (application) REF: 579 TOP: Nursing Process: Evaluation MSC: NCLEX: Physiological Integrity

Which data identified during the perioperative assessment alert the nurse that special protection techniques should be implemented during surgery? a. Stated allergy to cats and dogs b. History of spinal and hip arthritis c. Verbalization of anxiety by the patient d. Having a sip of water 3 hours previously

ANS: B The patient with arthritis may require special positioning to avoid injury and postoperative discomfort. Preoperative anxiety (unless severe) and having a sip of water 3 hours before surgery are not contraindications to having surgery. An allergy to cats and dogs will not impact the care needed during the intraoperative phase

The nurse and a licensed practical/vocational nurse (LPN/LVN) are working together to care for a patient who had an esophagectomy 2 days ago. Which action by the LPN/LVN requires that the nurse intervene? a. The LPN/LVN uses soft swabs to provide for oral care. b. The LPN/LVN positions the head of the bed in the flat position. c. The LPN/LVN encourages the patient to use pain medications before coughing. d. The LPN/LVN includes the enteral feeding volume when calculating intake and output.

ANS: B The patient's bed should be in Fowler's position to prevent reflux and aspiration of gastric contents. The other actions by the LPN/LVN are appropriate

During routine hemodialysis, the 68-year-old patient complains of nausea and dizziness. Which action should the nurse take first? a. Slow down the rate of dialysis. b. Check patient's blood pressure (BP). c. Review the hematocrit (Hct) level. d. Give prescribed PRN antiemetic drugs.

ANS: B The patient's complaints of nausea and dizziness suggest hypotension, so the initial action should be to check the BP. The other actions may also be appropriate based on the blood pressure obtained

ollowing a thymectomy, a 62-year-old male patient with myasthenia gravis receives the usual dose of pyridostigmine (Mestinon). An hour later, the patient complains of nausea and severe abdominal cramps. Which action should the nurse take first? a. Auscultate the patient's bowel sounds. b. Notify the patient's health care provider. c. Administer the prescribed PRN antiemetic drug. d. Give the scheduled dose of prednisone (Deltasone).

ANS: B The patient's history and symptoms indicate a possible cholinergic crisis. The health care provider should be notified immediately, and it is likely that atropine will be prescribed. The other actions will be appropriate if the patient is not experiencing a cholinergic crisis. DIF: Cognitive Level: Apply (application) REF: 1438-1439 OBJ: Special Questions: Prioritization TOP: Nursing Process: Implementation MSC: NCLEX: Physiological Integrity

20. A 19-year-old female admitted with anorexia nervosa is 5 ft 6 in (163 cm) tall and weighs 88 pounds (41 kg). Laboratory tests reveal hypokalemia and iron-deficiency anemia. Which nursing diagnosis has the highest priority? a. Risk for activity intolerance related to anemia b. Risk for electrolyte imbalance related to eating patterns c. Ineffective health maintenance related to body image obsession d. Imbalanced nutrition: less than body requirements related to anorexia

ANS: B The patient's hypokalemia may lead to life-threatening cardiac dysrhythmias. The other diagnoses are also appropriate for this patient but are not associated with immediate risk for fatal complications.

The nurse is administering IV fluid boluses and nasogastric irrigation to a patient with acute gastrointestinal (GI) bleeding. Which assessment finding is most important for the nurse to communicate to the health care provider? a. The bowel sounds are hyperactive in all four quadrants. b. The patient's lungs have crackles audible to the midchest. c. The nasogastric (NG) suction is returning coffee-ground material. d. The patient's blood pressure (BP) has increased to 142/84 mm Hg.

ANS: B The patient's lung sounds indicate that pulmonary edema may be developing as a result of the rapid infusion of IV fluid and that the fluid infusion rate should be slowed. The return of coffee-ground material in an NG tube is expected for a patient with upper GI bleeding. The BP is slightly elevated but would not be an indication to contact the health care provider immediately. Hyperactive bowel sounds are common when a patient has GI bleeding

40. The clinic nurse makes a follow-up telephone call to a patient with asthma. The patient reports having a baseline peak flow reading of 600 L/minute and the current peak flow is 420 L/minute. Which action should the nurse take first? a. Tell the patient to go to the hospital emergency department. b. Instruct the patient to use the prescribed albuterol (Proventil). c. Ask about recent exposure to any new allergens or asthma triggers. d. Question the patient about use of the prescribed inhaled corticosteroids.

ANS: B The patient's peak flow is 70% of normal, indicating a need for immediate use of short-acting β2-adrenergic SABA medications. Assessing for correct use of medications or exposure to allergens also is appropriate, but would not address the current decrease in peak flow. Because the patient is currently in the yellow zone, hospitalization is not needed. DIF: Cognitive Level: Analyze (analysis) REF: 580 OBJ: Special Questions: Prioritization TOP: Nursing Process: Implementation MSC: NCLEX: Physiological Integrity

A patient who has bacterial endophthalmitis in the left eye is restless, frequently asking whether the eye is healing, and whether removal of the eye will be necessary. Based on the assessment data, which nursing diagnosis is most appropriate at this time? a. Grieving related to current loss of functional vision b. Anxiety related to the possibility of permanent vision loss c. Situational low self-esteem related to loss of visual function d. Risk for falls related to inability to see environmental hazards

ANS: B The patient's restlessness and questioning of the nurse indicate anxiety about the future possible loss of vision. Because the patient can see with the right eye, functional vision is relatively intact. There is no indication of impaired self-esteem at this time

The nurse is planning care for a patient with severe heart failure who has developed elevated blood urea nitrogen (BUN) and creatinine levels. The primary collaborative treatment goal in the plan will be a. augmenting fluid volume. b. maintaining cardiac output. c. diluting nephrotoxic substances. d. preventing systemic hypertension.

ANS: B The primary goal of treatment for acute kidney injury (AKI) is to eliminate the cause and provide supportive care while the kidneys recover. Because this patient's heart failure is causing AKI, the care will be directed toward treatment of the heart failure. For renal failure caused by hypertension, hypovolemia, or nephrotoxins, the other responses would be correct

A patient with a head injury after a motorcycle crash arrives in the emergency department (ED) complaining of shortness of breath and severe eye pain. Which action will the nurse take first? a. Administer the ordered analgesic. b. Check the patient's oxygen saturation. c. Examine the eye for evidence of trauma. d. Assess each of the cranial nerve functions.

ANS: B The priority action for a patient after a head injury is to assess and maintain airway and breathing. Because the patient is complaining of shortness of breath, it is essential that the nurse assess the oxygen saturation. The other actions are also appropriate but are not the first action the nurse will take

Which patient choice for a snack 2 hours before bedtime indicates that the nurse's teaching about gastroesophageal reflux disease (GERD) has been effective? a. Chocolate pudding b. Glass of low-fat milk c. Cherry gelatin with fruit d. Peanut butter and jelly sandwich

ANS: C Gelatin and fruit are low fat and will not decrease lower esophageal sphincter (LES) pressure. Foods such as chocolate are avoided because they lower LES pressure. Milk products increase gastric acid secretion. High-fat foods such as peanut butter decrease both gastric emptying and LES pressure

A 68-year-old patient with a bleeding duodenal ulcer has a nasogastric (NG) tube in place, and the health care provider orders 30 mL of aluminum hydroxide/magnesium hydroxide (Maalox) to be instilled through the tube every hour. To evaluate the effectiveness of this treatment, the nurse a. monitors arterial blood gas values daily. b. periodically aspirates and tests gastric pH. c. checks each stool for the presence of occult blood. d. measures the volume of residual stomach contents.

ANS: B The purpose for antacids is to increase gastric pH. Checking gastric pH is the most direct way of evaluating the effectiveness of the medication. Arterial blood gases may change slightly, but this does not directly reflect the effect of antacids on gastric pH. Because the patient has upper gastrointestinal (GI) bleeding, occult blood in the stools will appear even after the acute bleeding has stopped. The amount of residual stomach contents is not a reflection of resolution of bleeding or of gastric pH

The nurse develops a plan of care to prevent aspiration in a high-risk patient. Which nursing action will be most effective? a. Turn and reposition immobile patients at least every 2 hours. b. Place patients with altered consciousness in side-lying positions. c. Monitor for respiratory symptoms in patients who are immunosuppressed. d. Insert nasogastric tube for feedings for patients with swallowing problems.

ANS: B The risk for aspiration is decreased when patients with a decreased level of consciousness are placed in a side-lying or upright position. Frequent turning prevents pooling of secretions in immobilized patients but will not decrease the risk for aspiration in patients at risk. Monitoring of parameters such as breath sounds and oxygen saturation will help detect pneumonia in immunocompromised patients, but it will not decrease the risk for aspiration. Conditions that increase the risk of aspiration include decreased level of consciousness (e.g., seizure, anesthesia, head injury, stroke, alcohol intake), difficulty swallowing, and nasogastric intubation with or without tube feeding. With loss of consciousness, the gag and cough reflexes are depressed, and aspiration is more likely to occur. Other high-risk groups are those who are seriously ill, have poor dentition, or are receiving acid-reducing medications

Which nursing action should the operating room (OR) nurse manager delegate to the registered nurse first assistant (RNFA) when caring for a surgical patient? a. Adjust the doses of administered anesthetics. b. Make surgical incision and suture incisions as needed. c. Coordinate transfer of the patient to the operating table. d. Provide postoperative teaching about coughing to the patient.

ANS: B The role of the RNFA includes skills such as making and suturing incisions and maintaining hemostasis. The other actions should be delegated to other staff members such as the circulating nurse, scrub nurse, or surgical technician. The anesthesia care provider should adjust the doses of anesthetics for patients, not the RNFA.

11. When caring for a 63-year-old woman with a soft, silicone nasogastric tube in place for enteral feedings, the nurse will a. avoid giving medications through the feeding tube. b. flush the tubing after checking for residual volumes. c. administer continuous feedings using an infusion pump. d. replace the tube every 3 days to avoid mucosal damage.

ANS: B The soft silicone feeding tubes are small in diameter and can easily become clogged unless they are flushed after the nurse checks the residual volume. Either intermittent or continuous feedings can be given. The tubes are less likely to cause mucosal damage than the stiffer polyvinyl chloride tubes used for nasogastric suction and do not need to be replaced at certain intervals. Medications can be given through these tubes, but flushing after medication administration is important to avoid clogging.

The nurse is preparing to witness the patient signing the operative consent form when the patient says, "I do not really understand what the doctor said." Which action is best for the nurse to take? a. Provide an explanation of the planned surgical procedure. b. Notify the surgeon that the informed consent process is not complete. c. Administer the prescribed preoperative antibiotics and withhold any ordered sedative medications. d. Notify the operating room staff that the surgeon needs to give a more complete explanation of the procedure.

ANS: B The surgeon is responsible for explaining the surgery to the patient, and the nurse should wait until the surgeon has clarified the surgery before having the patient sign the consent form. The nurse should communicate directly with the surgeon about the consent form rather than asking other staff to pass on the message. It is not within the nurse's legal scope of practice to explain the surgical procedure. No preoperative medications should be administered until the patient understands the surgical procedure and signs the consent form. Integrity

The health care provider is considering the use of sumatriptan (Imitrex) for a 54-year-old male patient with migraine headaches. Which information obtained by the nurse is most important to report to the health care provider? a. The patient drinks 1 to 2 cups of coffee daily. b. The patient had a recent acute myocardial infarction. c. The patient has had migraine headaches for 30 years. d. The patient has taken topiramate (Topamax) for 2 months.

ANS: B The triptans cause coronary artery vasoconstriction and should be avoided in patients with coronary artery disease. The other information will be reported to the health care provider, but none of it indicates that sumatriptan would be an inappropriate treatment. DIF: Cognitive Level: Apply (application) REF: 1416 OBJ: Special Questions: Prioritization TOP: Nursing Process: Assessment MSC: NCLEX: Physiological Integrity

Which adult will the nurse plan to teach about risks associated with obesity? a. Man who has a BMI of 18 kg/m2 b. Man with a 42 in waist and 44 in hips c. Woman who has a body mass index (BMI) of 24 kg/m2 d. Woman with a waist circumference of 34 inches (86 cm)

ANS: B The waist-to-hip ratio for this patient is 0.95, which exceeds the recommended level of <0.80. A patient with a BMI of 18 kg/m2 is considered underweight. A BMI of 24 kg/m2 is normal. Health risks associated with obesity increase in women with a waist circumference larger than 35 in (89 cm) and men with a waist circumference larger than 40 in (102 cm).

After the nurse teaches a patient about the recommended amounts of foods from animal and plant sources, which menu selections indicate that the initial instructions about diet have been understood? a. 3 oz of lean beef, 2 oz of low-fat cheese, and a tomato slice b. 3 oz of roasted pork, a cup of corn, and a cup of carrot sticks c. Cup of tossed salad and nonfat dressing topped with a chicken breast d. Half cup of tuna mixed with nonfat mayonnaise and a half cup of celery

ANS: B This selection is most consistent with the recommendation of the American Institute for Cancer Research that one third of the diet should be from animal sources and two thirds from plant source foods. The other choices all have higher ratios of animal origin foods to plant source foods than would be recommended.

After bariatric surgery, a patient who is being discharged tells the nurse, "I prefer to be independent. I am not interested in any support groups." Which response by the nurse is best? a. "I hope you change your mind so that I can suggest a group for you." b. "Tell me what types of resources you think you might use after this surgery." c. "Support groups have been found to lead to more successful weight loss after surgery." d. "Because there are many lifestyle changes after surgery, we recommend support groups."

ANS: B This statement allows the nurse to assess the individual patient's potential needs and preferences. The other statements offer the patient more information about the benefits of support groups, but fail to acknowledge the patient's preferences.

The nurse will assess a 67-year-old patient who is experiencing a cluster headache for a. nuchal rigidity. b. unilateral ptosis. c. projectile vomiting. d. throbbing, bilateral facial pain.

ANS: B Unilateral eye edema, tearing, and ptosis are characteristic of cluster headaches. Nuchal rigidity suggests meningeal irritation, such as occurs with meningitis. Although nausea and vomiting may occur with migraine headaches, projectile vomiting is more consistent with increased intracranial pressure (ICP). Unilateral sharp, stabbing pain, rather than throbbing pain, is characteristic of cluster headaches. DIF: Cognitive Level: Understand (comprehension) REF: 1414 TOP: Nursing Process: Assessment MSC: NCLEX: Physiological Integrity

7. When obtaining a health history and physical assessment for a 36-year-old female patient with possible multiple sclerosis (MS), the nurse should a. assess for the presence of chest pain. b. inquire about urinary tract problems. c. inspect the skin for rashes or discoloration. d. ask the patient about any increase in libido.

ANS: B Urinary tract problems with incontinence or retention are common symptoms of MS. Chest pain and skin rashes are not symptoms of MS. A decrease in libido is common with MS. DIF: Cognitive Level: Apply (application) REF: 1429 TOP: Nursing Process: Assessment MSC: NCLEX: Physiological Integrity

Four hours after a bowel resection, a 74-year-old male patient with a nasogastric tube to suction complains of nausea and abdominal distention. The first action by the nurse should be to a. auscultate for hypotonic bowel sounds. b. notify the patient's health care provider. c. reposition the tube and check for placement. d. remove the tube and replace it with a new one.

C

Which action could the registered nurse (RN) who is working in the eye and ear clinic delegate to a licensed practical/vocational nurse (LPN/LVN)? a. Evaluate a patient's ability to administer eye drops. b. Use a Snellen chart to check a patient's visual acuity. c. Teach a patient with otosclerosis about use of sodium fluoride and vitamin D. d. Check the patient's external ear for signs of irritation caused by a hearing aid.

ANS: B Using standardized screening tests such as a Snellen chart to test visual acuity is included in LPN education and scope of practice. Evaluation, assessment, and patient teaching are higher level skills that require RN education and scope of practice

Unlicensed assistive personnel (UAP) perform all the following actions when caring for a patient with Ménière's disease who is experiencing an acute attack. Which action by UAP indicates that the nurse should intervene immediately? a. UAP raise the side rails on the bed. b. UAP turn on the patient's television. c. UAP turn the patient to the right side. d. UAP place an emesis basin at the bedside.

ANS: B Watching television may exacerbate the symptoms of an acute attack of Ménière's disease. The other actions are appropriate because the patient will be at high fall risk and may suffer from nausea during the acute attack

The nurse identifies the nursing diagnosis of imbalanced nutrition: less than body requirements related to impaired self-feeding ability for a left-handed patient with left-sided hemiplegia. Which intervention should be included in the plan of care? a. Provide a wide variety of food choices. b. Provide oral care before and after meals. c. Assist the patient to eat with the right hand. d. Teach the patient the "chin-tuck" technique.

ANS: C Because the nursing diagnosis indicates that the patient's imbalanced nutrition is related to the left-sided hemiplegia, the appropriate interventions will focus on teaching the patient to use the right hand for self-feeding. The other interventions are appropriate for patients with other etiologies for the imbalanced nutrition. DIF: Cognitive Level: Apply (application) REF: 1407 TOP: Nursing Process: Planning MSC: NCLEX: Physiological Integrity

5. A dark-skinned patient has been admitted to the hospital with chronic heart failure. How would the nurse best assess this patient for cyanosis? a. Assess the skin color of the earlobes. b. Apply pressure to the palms of the hands. c. Check the lips and oral mucous membranes. d. Examine capillary refill time of the nail beds.

ANS: C Cyanosis in dark-skinned individuals is more easily seen in the mucous membranes. Earlobe color may change in light-skinned individuals, but this change in skin color is difficult to detect on darker skin. Application of pressure to the palms of the hands and nail bed assessment would check for adequate circulation but not for skin color. DIF: Cognitive Level: Apply (application) REF: 401 TOP: Nursing Process: Assessment

Which stroke risk factor for a 48-year-old male patient in the clinic is most important for the nurse to address? a. The patient is 25 pounds above the ideal weight. b. The patient drinks a glass of red wine with dinner daily. c. The patient's usual blood pressure (BP) is 170/94 mm Hg. d. The patient works at a desk and relaxes by watching television.

ANS: C Hypertension is the single most important modifiable risk factor. People who drink more than 1 (for women) or 2 (for men) alcoholic beverages a day may increase risk for hypertension. Physical inactivity and obesity contribute to stroke risk but not as much as hypertension. DIF: Cognitive Level: Apply (application) REF: 1390 OBJ: Special Questions: Prioritization TOP: Nursing Process: Assessment MSC: NCLEX: Health Promotion and Maintenance

A male patient who has right-sided weakness after a stroke is making progress in learning to use the left hand for feeding and other activities. The nurse observes that when the patient's wife is visiting, she feeds and dresses him. Which nursing diagnosis is most appropriate for the patient? a. Interrupted family processes related to effects of illness of a family member b. Situational low self-esteem related to increasing dependence on spouse for care c. Disabled family coping related to inadequate understanding by patient's spouse d. Impaired nutrition: less than body requirements related to hemiplegia and aphasia

ANS: C The information supports the diagnosis of disabled family coping because the wife does not understand the rehabilitation program. There are no data supporting low self-esteem, and the patient is attempting independence. The data do not support an interruption in family processes because this may be a typical pattern for the couple. There is no indication that the patient has impaired nutrition. DIF: Cognitive Level: Apply (application) REF: 1409 TOP: Nursing Process: Diagnosis MSC: NCLEX: Psychosocial Integrity

A 70-year-old female patient with left-sided hemiparesis arrives by ambulance to the emergency department. Which action should the nurse take first? a. Monitor the blood pressure. b. Send the patient for a computed tomography (CT) scan. c. Check the respiratory rate and effort. d. Assess the Glasgow Coma Scale score.

ANS: C The initial nursing action should be to assess the airway and take any needed actions to ensure a patent airway. The other activities should take place quickly after the ABCs (airway, breathing, and circulation) are completed. DIF: Cognitive Level: Apply (application) REF: 1397-1398 OBJ: Special Questions: Prioritization TOP: Nursing Process: Implementation MSC: NCLEX: Physiological Integrity

4. When examining an older patient in the home, the home health nurse notices irregular patterns of bruising at different stages of healing on the patients body. Which action should the nurse take first? a. Discourage the use of throw rugs throughout the house. b. Ensure the patient has a pair of shoes with non-slip soles. c. Talk with the patient alone and ask about what caused the bruising. d. Notify the health care provider so that x-rays can be ordered as soon as possible.

ANS: C The nurse should note irregular patterns of bruising, especially in the shapes of hands or fingers, in different stages of resolution. These may be indications of other health problems or abuse, and should be further investigated. It is important that the nurse interview the patient alone because, if mistreatment is occurring, the patient may not disclose it in the presence of the person who may be the abuser. Throw rugs and shoes with slippery surfaces may contribute to falls. X-rays may be needed if the patient has fallen recently and also has complaints of pain or decreased mobility. However, the nurses first nursing action is to further assess the patient. DIF: Cognitive Level: Apply (application) REF: 401 TOP: Nursing Process: Implementation

A 47-year-old patient will attempt oral feedings for the first time since having a stroke. The nurse should assess the gag reflex and then a. order a varied pureed diet. b. assess the patient's appetite. c. assist the patient into a chair. d. offer the patient a sip of juice.

ANS: C The patient should be as upright as possible before attempting feeding to make swallowing easier and decrease aspiration risk. To assess swallowing ability, the nurse should initially offer water or ice to the patient. Pureed diets are not recommended because the texture is too smooth. The patient may have a poor appetite, but the oral feeding should be attempted regardless. DIF: Cognitive Level: Apply (application) REF: 1406 TOP: Nursing Process: Implementation MSC: NCLEX: Physiological Integrity

19. The nurse is providing preoperative teaching for a 61-year-old man scheduled for an abdominal-perineal resection. Which information will the nurse include? a. Another surgery in 8 to 12 weeks will be used to create an ileal-anal reservoir. b. The patient will begin sitting in a chair at the bedside on the first postoperative day. c. The patient will drink polyethylene glycol lavage solution (GoLYTELY) preoperatively. d. IV antibiotics will be started at least 24 hours before surgery to reduce the bowel bacteria.

ANS: C A bowel-cleansing agent is used to empty the bowel before surgery to reduce the risk for infection. A permanent colostomy is created with this surgery. Sitting is contraindicated after an abdominal-perineal resection. Oral antibiotics (rather than IV antibiotics) are given to reduce colonic and rectal bacteria.

A patient treated for human immunodeficiency virus (HIV) infection for 6 years has developed fat redistribution to the trunk, with wasting of the arms, legs, and face. What instructions will the nurse give to the patient? a. Review foods that are higher in protein. b. Teach about the benefits of daily exercise. c. Discuss a change in antiretroviral therapy. d. Talk about treatment with antifungal agents.

ANS: C A frequent first intervention for metabolic disorders is a change in antiretroviral therapy (ART). Treatment with antifungal agents would not be appropriate because there is no indication of fungal infection. Changes in diet or exercise have not proven helpful for this problem.

A 58-year-old patient has just been admitted to the emergency department with nausea and vomiting. Which information requires the most rapid intervention by the nurse? a. The patient has been vomiting for 4 days. b. The patient takes antacids 8 to 10 times a day. c. The patient is lethargic and difficult to arouse. d. The patient has undergone a small intestinal resection.

ANS: C A lethargic patient is at risk for aspiration, and the nurse will need to position the patient to decrease aspiration risk. The other information is also important to collect, but it does not require as quick action as the risk for aspiration.

The nurse notes that a patient has incisional pain, a poor cough effort, and scattered rhonchi after a thoracotomy. Which action should the nurse take first? a. Assist the patient to sit upright in a chair. b. Splint the patient's chest during coughing. c. Medicate the patient with prescribed morphine. d. Observe the patient use the incentive spirometer.

ANS: C A major reason for atelectasis and poor airway clearance in patients after chest surgery is incisional pain (which increases with deep breathing and coughing). The first action by the nurse should be to medicate the patient to minimize incisional pain. The other actions are all appropriate ways to improve airway clearance but should be done after the morphine is given

A patient with a positive rapid antibody test result for human immunodeficiency virus (HIV) is anxious and does not appear to hear what the nurse is saying. What action by the nurse is most important at this time? a. Teach the patient about the medications available for treatment. b. Inform the patient how to protect sexual and needle-sharing partners. c. Remind the patient about the need to return for retesting to verify the results. d. Ask the patient to notify individuals who have had risky contact with the patient.

ANS: C After an initial positive antibody test, the next step is retesting to confirm the results. A patient who is anxious is not likely to be able to take in new information or be willing to disclose information about HIV status of other individuals.

Which teaching should the nurse provide about intradermal skin testing to a patient with possible allergies? a. "Do not eat anything for about 6 hours before the testing." b. "Take an oral antihistamine about an hour before the testing." c. "Plan to wait in the clinic for 20 to 30 minutes after the testing." d. "Reaction to the testing will take about 48 to 72 hours to occur."

ANS: C Allergic reactions usually occur within minutes after injection of an allergen, and the patient will be monitored for at least 20 minutes for anaphylactic reactions after the testing. Medications that might modify the response, such as antihistamines, should be avoided before allergy testing. There is no reason to be NPO for skin testing. Results with intradermal testing occur within minutes.

The nurse is assessing a patient who had a total gastrectomy 8 hours ago. What information is most important to report to the health care provider? a. Absent bowel sounds b. Complaints of incisional pain c. Temperature 102.1° F (38.9° C) d. Scant nasogastric (NG) tube drainage

ANS: C An elevation in temperature may indicate leakage at the anastomosis, which may require return to surgery or keeping the patient NPO. The other findings are expected in the immediate postoperative period for patients who have this surgery.

Immediately after the nurse administers an intracutaneous injection of an allergen on the forearm, a patient complains of itching at the site and of weakness and dizziness. What action should the nurse take first? a. Remind the patient to remain calm. b. Administer subcutaneous epinephrine. c. Apply a tourniquet above the injection site. d. Rub a local antiinflammatory cream on the site.

ANS: C Application of a tourniquet will decrease systemic circulation of the allergen and should be the first reaction. A local antiinflammatory cream may be applied to the site of a cutaneous test if the itching persists. Epinephrine will be needed if the allergic reaction progresses to anaphylaxis. The nurse should assist the patient to remain calm, but this is not an adequate initial nursing action.

Which assessment finding may indicate that a patient is experiencing adverse effects to a corticosteroid prescribed after kidney transplantation? a. Postural hypotension b. Recurrent tachycardia c. Knee and hip joint pain d. Increased serum creatinine

ANS: C Aseptic necrosis of the weight-bearing joints can occur when patients take corticosteroids over a prolonged period. Increased creatinine level, orthostatic dizziness, and tachycardia are not caused by corticosteroid use.

A nurse should instruct a patient with recurrent staphylococcal and seborrheic blepharitis to a. irrigate the eyes with saline solution. b. apply cool compresses to the eyes three times daily. c. use a gentle baby shampoo to clean the lids as needed. d. schedule an appointment for surgical removal of the lesion.

ANS: C Baby shampoo is used to soften and remove crusts associated with blepharitis. The other interventions are not used in treating this disorder.

1. Which action will the nurse include in the plan of care for a 42-year-old patient who is being admitted with Clostridium difficile? a. Educate the patient about proper food storage. b. Order a diet with no dairy products for the patient. c. Place the patient in a private room on contact isolation. d. Teach the patient about why antibiotics will not be used.

ANS: C Because C. difficile is highly contagious, the patient should be placed in a private room and contact precautions should be used. There is no need to restrict dairy products for this type of diarrhea. Metronidazole (Flagyl) is frequently used to treat C. difficile. Improper food handling and storage do not cause C. difficile.

A 68-year-old male patient with a stroke is unconscious and unresponsive to stimuli. After learning that the patient has a history of gastroesophageal reflux disease (GERD), the nurse will plan to do frequent assessments of the patient's a. apical pulse. b. bowel sounds. c. breath sounds. d. abdominal girth.

ANS: C Because GERD may cause aspiration, the unconscious patient is at risk for developing aspiration pneumonia. Bowel sounds, abdominal girth, and apical pulse will not be affected by the patient's stroke or GERD and do not require more frequent monitoring than the routine

A 49-year-old patient with multiple sclerosis (MS) is to begin treatment with glatiramer acetate (Copaxone). Which information will the nurse include in patient teaching? a. Recommendation to drink at least 4 L of fluid daily b. Need to avoid driving or operating heavy machinery c. How to draw up and administer injections of the medication d. Use of contraceptive methods other than oral contraceptives

ANS: C Copaxone is administered by self-injection. Oral contraceptives are an appropriate choice for birth control. There is no need to avoid driving or drink large fluid volumes when taking glatiramer. DIF: Cognitive Level: Apply (application) REF: 1430 TOP: Nursing Process: Implementation MSC: NCLEX: Physiological Integrity

9. After abdominal surgery, a patient with protein calorie malnutrition is receiving parenteral nutrition (PN). Which is the best indicator that the patient is receiving adequate nutrition? a. Serum albumin level is 3.5 mg/dL. b. Fluid intake and output are balanced. c. Surgical incision is healing normally. d. Blood glucose is less than 110 mg/dL.

ANS: C Because poor wound healing is a possible complication of malnutrition for this patient, normal healing of the incision is an indicator of the effectiveness of the PN in providing adequate nutrition. Blood glucose is monitored to prevent the complications of hyperglycemia and hypoglycemia, but it does not indicate that the patient's nutrition is adequate. The intake and output will be monitored, but do not indicate that the PN is effective. The albumin level is in the low-normal range but does not reflect adequate caloric intake, which is also important for the patient.

While the nurse is transporting a patient on a stretcher to the radiology department, the patient begins having a tonic-clonic seizure. Which action should the nurse take? a. Insert an oral airway during the seizure to maintain a patent airway. b. Restrain the patient's arms and legs to prevent injury during the seizure. c. Time and observe and record the details of the seizure and postictal state. d. Avoid touching the patient to prevent further nervous system stimulation.

ANS: C Because the diagnosis and treatment of seizures frequently are based on the description of the seizure, recording the length and details of the seizure is important. Insertion of an oral airway and restraining the patient during the seizure are contraindicated. The nurse may need to move the patient to decrease the risk of injury during the seizure. DIF: Cognitive Level: Apply (application) REF: 1422 TOP: Nursing Process: Implementation MSC: NCLEX: Physiological Integrity

Which order from the health care provider will the nurse implement first for a patient who has vomited 1200 mL of blood? a. Give an IV H2 receptor antagonist. b. Draw blood for typing and crossmatching. c. Administer 1000 mL of lactated Ringer's solution. d. Insert a nasogastric (NG) tube and connect to suction.

ANS: C Because the patient has vomited a large amount of blood, correction of hypovolemia and prevention of hypovolemic shock are the priorities. The other actions also are important to implement quickly but are not the highest priorities.

A patient who collects honey to earn supplemental income has developed a hypersensitivity to bee stings. Which statement, if made by the patient, would indicate a need for additional teaching? a. "I need to find another way to earn extra money." b. "I will get a prescription for epinephrine and learn to self-inject it." c. "I will plan to take oral antihistamines daily before going to work." d. "I should wear a Medic-Alert bracelet indicating my allergy to bee stings."

ANS: C Because the patient is at risk for bee stings and the severity of allergic reactions tends to increase with added exposure to allergen, taking oral antihistamines will not adequately control the patient's hypersensitivity reaction. The other patient statements indicate a good understanding of management of the problem

A 53-year-old male patient with deep partial-thickness burns from a chemical spill in the workplace experiences severe pain followed by nausea during dressing changes. Which action will be most useful in decreasing the patient's nausea? a. Keep the patient NPO for 2 hours before and after dressing changes. b. Avoid performing dressing changes close to the patient's mealtimes. c. Administer the prescribed morphine sulfate before dressing changes. d. Give the ordered prochlorperazine (Compazine) before dressing changes.

ANS: C Because the patient's nausea is associated with severe pain, it is likely that it is precipitated by stress and pain. The best treatment will be to provide adequate pain medication before dressing changes. The nurse should avoid doing painful procedures close to mealtimes, but nausea/vomiting that occur at other times also should be addressed. Keeping the patient NPO does not address the reason for the nausea and vomiting and will have an adverse effect on the patient's nutrition. Administration of antiemetics is not the best choice for a patient with nausea caused by pain

21. A 71-year-old patient had an abdominal-perineal resection for colon cancer. Which nursing action is most important to include in the plan of care for the day after surgery? a. Teach about a low-residue diet. b. Monitor output from the stoma. c. Assess the perineal drainage and incision. d. Encourage acceptance of the colostomy stoma.

ANS: C Because the perineal wound is at high risk for infection, the initial care is focused on assessment and care of this wound. Teaching about diet is best done closer to discharge from the hospital. There will be very little drainage into the colostomy until peristalsis returns. The patient will be encouraged to assist with the colostomy, but this is not the highest priority in the immediate postoperative period.

The nurse, who is reviewing a clinic patient's medical record, notes that the patient missed the previous appointment for weekly immunotherapy. Which action by the nurse is most appropriate? a. Schedule an additional dose that week. b. Administer the usual dosage of the allergen. c. Consult with the health care provider about giving a lower allergen dose. d. Re-evaluate the patient's sensitivity to the allergen with a repeat skin test.

ANS: C Because there is an increased risk for adverse reactions after a patient misses a scheduled dose of allergen, the nurse should check with the health care provider before administration of the injection. A skin test is used to identify the allergen and would not be used at this time. An additional dose for the week may increase the risk for a reaction.

Which action will the nurse include in the plan of care for a 42-year-old patient who is being admitted with Clostridium difficile? a. Educate the patient about proper food storage. b. Order a diet with no dairy products for the patient. c. Place the patient in a private room on contact isolation. d. Teach the patient about why antibiotics will not be used.

C

9. The nurse is caring for a patient with chronic obstructive pulmonary disease (COPD). Which information obtained from the patient would prompt the nurse to consult with the health care provider before administering the prescribed theophylline? a. The patient reports a recent 15-pound weight gain. b. The patient denies any shortness of breath at present. c. The patient takes cimetidine (Tagamet) 150 mg daily. d. The patient complains about coughing up green mucus.

ANS: C Cimetidine interferes with the metabolism of theophylline, and concomitant administration may lead rapidly to theophylline toxicity. The other patient information would not affect whether the theophylline should be administered or not. DIF: Cognitive Level: Apply (application) REF: 571 | 572 TOP: Nursing Process: Assessment MSC: NCLEX: Physiological Integrity

A patient who has a right-sided chest tube following a thoracotomy has continuous bubbling in the suction-control chamber of the collection device. Which action by the nurse is most appropriate? a. Document the presence of a large air leak. b. Notify the surgeon of a possible pneumothorax. c. Take no further action with the collection device. d. Adjust the dial on the wall regulator to decrease suction.

ANS: C Continuous bubbling is expected in the suction-control chamber and indicates that the suction-control chamber is connected to suction. An air leak would be detected in the water-seal chamber. There is no evidence of pneumothorax. Increasing or decreasing the vacuum source will not adjust the suction pressure. The amount of suction applied is regulated by the amount of water in this chamber and not by the amount of suction applied to the system

A patient returned from a laparoscopic Nissen fundoplication for hiatal hernia 4 hours ago. Which assessment finding is most important for the nurse to address immediately? a. The patient is experiencing intermittent waves of nausea. b. The patient complains of 7/10 (0 to 10 scale) abdominal pain. c. The patient has absent breath sounds in the left anterior chest. d. The patient has hypoactive bowel sounds in all four quadrants.

ANS: C Decreased breath sounds on one side may indicate a pneumothorax, which requires rapid diagnosis and treatment. The nausea and abdominal pain should also be addressed but they are not as high priority as the patient's respiratory status. The patient's decreased bowel sounds are expected after surgery and require ongoing monitoring but no other action

A patient who has a positive test for human immunodeficiency virus (HIV) antibodies is admitted to the hospital with Pneumocystis jiroveci pneumonia (PCP) and a CD4+ T-cell count of less than 200 cells/L. Based on diagnostic criteria established by the Centers for Disease Control and Prevention (CDC), which statement by the nurse is correct? a. "The patient meets the criteria for a diagnosis of an acute HIV infection." b. "The patient will be diagnosed with asymptomatic chronic HIV infection." c. "The patient has developed acquired immunodeficiency syndrome (AIDS)." d. "The patient will develop symptomatic chronic HIV infection in less than a year."

ANS: C Development of PCP meets the diagnostic criterion for AIDS. The other responses indicate earlier stages of HIV infection than is indicated by the PCP infection.

40. Which activity in the care of a 48-year-old female patient with a new colostomy could the nurse delegate to unlicensed assistive personnel (UAP)? a. Document the appearance of the stoma. b. Place a pouching system over the ostomy. c. Drain and measure the output from the ostomy. d. Check the skin around the stoma for breakdown.

ANS: C Draining and measuring the output from the ostomy is included in UAP education and scope of practice. The other actions should be implemented by LPNs or RNs.

11. A patient with chronic obstructive pulmonary disease (COPD) has a nursing diagnosis of imbalanced nutrition: less than body requirements. Which intervention would be most appropriate for the nurse to include in the plan of care? a. Encourage increased intake of whole grains. b. Increase the patient's intake of fruits and fruit juices. c. Offer high-calorie snacks between meals and at bedtime. d. Assist the patient in choosing foods with high vegetable and mineral content.

ANS: C Eating small amounts more frequently (as occurs with snacking) will increase caloric intake by decreasing the fatigue and feelings of fullness associated with large meals. Patients with COPD should rest before meals. Foods that have a lot of texture like whole grains may take more energy to eat and get absorbed and lead to decreased intake. Although fruits, juices, and vegetables are not contraindicated, foods high in protein are a better choice. DIF: Cognitive Level: Apply (application) REF: 596 TOP: Nursing Process: Planning MSC: NCLEX: Physiological Integrity

2. A 76-year-old woman with a body mass index (BMI) of 17 kg/m2 and a low serum albumin level is being admitted by the nurse. Which assessment finding will the nurse expect to find? a. Restlessness b. Hypertension c. Pitting edema d. Food allergies

ANS: C Edema occurs when serum albumin levels and plasma oncotic pressure decrease. The blood pressure and level of consciousness are not directly affected by malnutrition. Food allergies are not an indicator of nutritional status.

A young adult female patient who is human immunodeficiency virus (HIV)-positive has a new prescription for efavirenz (Sustiva). Which information is most important to include in the medication teaching plan? a. Driving is allowed when starting this medication. b. Report any bizarre dreams to the health care provider. c. Continue to use contraception while on this medication. d. Take this medication in the morning on an empty stomach.

ANS: C Efavirenz can cause fetal anomalies and should not be used in patients who may be pregnant. The drug should not be used during pregnancy because large doses could cause fetal anomalies. Once-a-day doses should be taken at bedtime (at least initially) to help patients cope with the side effects that include dizziness and confusion. Patients should be cautioned about driving when starting this drug. Patients should be informed that many people who use the drug have reported vivid and sometimes bizarre dreams

21. The nurse is planning care for a patient who is chronically malnourished. Which action is appropriate for the nurse to delegate to unlicensed assistive personnel (UAP)? a. Assist the patient to choose high-nutrition items from the menu. b. Monitor the patient for skin breakdown over the bony prominences. c. Offer the patient the prescribed nutritional supplement between meals. d. Assess the patient's strength while ambulating the patient in the room.

ANS: C Feeding the patient and assisting with oral intake are included in UAP education and scope of practice. Assessing the patient and assisting the patient in choosing high-nutrition foods require licensed practical/vocational nurse (LPN/LVN)-or registered nurse (RN)-level education and scope of practice.

Which action will the nurse include in the plan of care for a patient with benign paroxysmal positional vertigo (BPPV)? a. Teach the patient about use of medications to reduce symptoms. b. Place the patient in a dark, quiet room to avoid stimulating BPPV attacks. c. Teach the patient that canalith repositioning may be used to reduce dizziness. d. Speak slowly and in a low-pitch to ensure that the patient is able to hear instructions.

C

The nurse provides preoperative instruction for a patient scheduled for a left pneumonectomy for cancer of the lung. Which information should the nurse include about the patient's postoperative care? a. Positioning on the right side b. Bed rest for the first 24 hours c. Frequent use of an incentive spirometer d. Chest tube placement with continuous drainage

ANS: C Frequent deep breathing and coughing are needed after chest surgery to prevent atelectasis. To promote gas exchange, patients after pneumonectomy are positioned on the surgical side. Early mobilization decreases the risk for postoperative complications such as pneumonia and deep vein thrombosis. In a pneumonectomy, chest tubes may or may not be placed in the space from which the lung was removed. If a chest tube is used, it is clamped and only released by the surgeon to adjust the volume of serosanguineous fluid that will fill the space vacated by the lung. If the cavity overfills, it could compress the remaining lung and compromise the cardiovascular and pulmonary function. Daily chest x-rays can be used to assess the volume and space

6. The nurse prepares to obtain a culture from a patient who has a possible fungal infection on the foot. Which items should the nurse gather for this procedure? a. Sterile gloves b. Patch test instruments c. Cotton-tipped applicators d. Local anesthetic, syringe, and intradermal needle

ANS: C Fungal cultures are obtained by swabbing the affected area of the skin with cotton-tipped applicators. Sterile gloves are not needed because it is not a sterile procedure. Local injection is not needed because the swabbing is not usually painful. The patch test is done to determine whether a patient is allergic to specific testing material, not for obtaining fungal specimens. DIF: Cognitive Level: Apply (application) REF: 405 TOP: Nursing Process: Implementation

A 37-year-old female patient is hospitalized with acute kidney injury (AKI). Which information will be most useful to the nurse in evaluating improvement in kidney function? a. Urine volume b. Creatinine level c. Glomerular filtration rate (GFR) d. Blood urea nitrogen (BUN) level

ANS: C GFR is the preferred method for evaluating kidney function. BUN levels can fluctuate based on factors such as fluid volume status and protein intake. Urine output can be normal or high in patients with AKI and does not accurately reflect kidney function. Creatinine alone is not an accurate reflection of renal function

Which activity in the care of a 48-year-old female patient with a new colostomy could the nurse delegate to unlicensed assistive personnel (UAP)? a. Document the appearance of the stoma. b. Place a pouching system over the ostomy. c. Drain and measure the output from the ostomy. d. Check the skin around the stoma for breakdown.

C

A 40-year-old patient is diagnosed with early Huntington's disease (HD). When teaching the patient, spouse, and children about this disorder, the nurse will provide information about the a. use of levodopa-carbidopa (Sinemet) to help reduce HD symptoms. b. prophylactic antibiotics to decrease the risk for aspiration pneumonia. c. option of genetic testing for the patient's children to determine their own HD risks. d. lifestyle changes of improved nutrition and exercise that delay disease progression.

ANS: C Genetic testing is available to determine whether an asymptomatic individual has the HD gene. The patient and family should be informed of the benefits and problems associated with genetic testing. Sinemet will increase symptoms of HD because HD involves an increase in dopamine. Antibiotic therapy will not reduce the risk for aspiration. There are no effective treatments or lifestyle changes that delay the progression of symptoms in HD. DIF: Cognitive Level: Apply (application) REF: 1440 TOP: Nursing Process: Implementation MSC: NCLEX: Physiological Integrity

Before administration of calcium carbonate (Caltrate) to a patient with chronic kidney disease (CKD), the nurse should check laboratory results for a. potassium level. b. total cholesterol. c. serum phosphate. d. serum creatinine.

ANS: C If serum phosphate is elevated, the calcium and phosphate can cause soft tissue calcification. The calcium carbonate should not be given until the phosphate level is lowered. Total cholesterol, creatinine, and potassium values do not affect whether calcium carbonate should be administered

32. A patient newly diagnosed with asthma is being discharged. The nurse anticipates including which topic in the discharge teaching? a. Use of long-acting β-adrenergic medications b. Side effects of sustained-release theophylline c. Self-administration of inhaled corticosteroids d. Complications associated with oxygen therapy

ANS: C Inhaled corticosteroids are more effective in improving asthma than any other drug and are indicated for all patients with persistent asthma. The other therapies would not typically be first-line treatments for newly diagnosed asthma. DIF: Cognitive Level: Apply (application) REF: 569 TOP: Nursing Process: Implementation MSC: NCLEX: Physiological Integrity

A patient diagnosed with external otitis is being discharged from the emergency department with an ear wick in place. Which statement by the patient indicates a need for further teaching? a. "I will apply the eardrops to the cotton wick in the ear canal." b. "I can use aspirin or acetaminophen (Tylenol) for pain relief." c. "I will clean the ear canal daily with a cotton-tipped applicator." d. "I can use warm compresses to the outside of the ear for comfort."

ANS: C Insertion of instruments such as cotton-tipped applicators into the ear should be avoided. The other patient statements indicate that the teaching has been successful

The nurse determines that teaching about management of migraine headaches has been effective when the patient says which of the following? a. "I can take the (Topamax) as soon as a headache starts." b. "A glass of wine might help me relax and prevent a headache." c. "I will lie down someplace dark and quiet when the headaches begin." d. "I should avoid taking aspirin and sumatriptan (Imitrex) at the same time."

ANS: C It is recommended that the patient with a migraine rest in a dark, quiet area. Topiramate (Topamax) is used to prevent migraines and must be taken for several months to determine effectiveness. Aspirin or other nonsteroidal antiinflammatory medications can be taken with the triptans. Alcohol may precipitate migraine headaches. DIF: Cognitive Level: Apply (application) REF: 1416 | 1419 TOP: Nursing Process: Evaluation MSC: NCLEX: Physiological Integrity

The nurse learns that a newly admitted patient has functional blindness and that the spouse has cared for the patient for many years. During the initial assessment of the patient, it is most important for the nurse to a. obtain more information about the cause of the patient's vision loss. b. obtain information from the spouse about the patient's special needs. c. make eye contact with the patient and ask about any need for assistance. d. perform an evaluation of the patient's visual acuity using a Snellen chart.

ANS: C Making eye contact with a partially sighted patient allows the patient to hear the nurse more easily and allows the nurse to assess the patient's facial expressions. The patient (rather than the spouse) should be asked about any need for assistance. The information about the cause of the vision loss and assessment of the patient's visual acuity are not priorities during the initial assessment

Which statement by the nurse is most likely to help a morbidly obese 22-year-old man in losing weight on a 1000-calorie diet? a. "It will be necessary to change lifestyle habits permanently to maintain weight loss." b. "You will decrease your risk for future health problems such as diabetes by losing weight now." c. "You are likely to notice changes in how you feel with just a few weeks of diet and exercise." d. "Most of the weight that you lose during the first weeks of dieting is water weight rather than fat."

ANS: C Motivation is a key factor in successful weight loss and a short-term outcome provides a higher motivation. A 22-year-old patient is unlikely to be motivated by future health problems. Telling a patient that the initial weight loss is water will be discouraging, although this may be correct. Changing lifestyle habits is necessary, but this process occurs over time and discussing this is not likely to motivate the patient.

The nurse at the outpatient surgery unit obtains the following information about a patient who is scheduled for cataract extraction and implantation of an intraocular lens. Which information is most important to report to the health care provider at this time? a. The patient has had blurred vision for 3 years. b. The patient has not eaten anything for 8 hours. c. The patient takes 2 antihypertensive medications. d. The patient gets nauseated with general anesthesia.

ANS: C Mydriatic medications used for pupil dilation are sympathetic nervous system stimulants and may increase heart rate and blood pressure. Using punctal occlusion when administering the mydriatic and monitoring of blood pressure are indicated for this patient. Blurred vision is an expected finding with cataracts. Patients are expected to be NPO for 6 to 8 hours before the surgical procedure. Cataract extraction and intraocular lens implantation are done using local anesthesia

A 64-year-old male patient who has had progressive chronic kidney disease (CKD) for several years has just begun regular hemodialysis. Which information about diet will the nurse include in patient teaching? a. Increased calories are needed because glucose is lost during hemodialysis. b. Unlimited fluids are allowed because retained fluid is removed during dialysis. c. More protein is allowed because urea and creatinine are removed by dialysis. d. Dietary potassium is not restricted because the level is normalized by dialysis.

ANS: C Once the patient is started on dialysis and nitrogenous wastes are removed, more protein in the diet is encouraged. Fluids are still restricted to avoid excessive weight gain and complications such as shortness of breath. Glucose is not lost during hemodialysis. Sodium and potassium intake continues to be restricted to avoid the complications associated with high levels of these electrolytes

A 26-year-old woman has been admitted to the emergency department with nausea and vomiting. Which action could the RN delegate to unlicensed assistive personnel (UAP)? a. Auscultate the bowel sounds. b. Assess for signs of dehydration. c. Assist the patient with oral care. d. Ask the patient about the nausea.

ANS: C Oral care is included in UAP education and scope of practice. The other actions are all assessments that require more education and a higher scope of nursing practice

A licensed practical/vocational nurse (LPN/LVN) is caring for a patient with stage 2 chronic kidney disease. Which observation by the RN requires an intervention? a. The LPN/LVN administers the erythropoietin subcutaneously. b. The LPN/LVN assists the patient to ambulate out in the hallway. c. The LPN/LVN administers the iron supplement and phosphate binder with lunch. d. The LPN/LVN carries a tray containing low-protein foods into the patient's room.

ANS: C Oral phosphate binders should not be given at the same time as iron because they prevent the iron from being absorbed. The phosphate binder should be given with a meal and the iron given at a different time. The other actions by the LPN/LVN are appropriate for a patient with renal insufficiency.

22. A patient with chronic obstructive pulmonary disease (COPD) has poor gas exchange. Which action by the nurse would be most appropriate? a. Have the patient rest in bed with the head elevated to 15 to 20 degrees. b. Ask the patient to rest in bed in a high-Fowler's position with the knees flexed. c. Encourage the patient to sit up at the bedside in a chair and lean slightly forward. d. Place the patient in the Trendelenburg position with several pillows behind the head.

ANS: C Patients with COPD improve the mechanics of breathing by sitting up in the "tripod" position. Resting in bed with the head elevated in a semi-Fowler's position would be an alternative position if the patient was confined to bed, but sitting in a chair allows better ventilation. The Trendelenburg position or sitting upright in bed with the knees flexed would decrease the patient's ability to ventilate well. DIF: Cognitive Level: Apply (application) REF: 599 TOP: Nursing Process: Implementation MSC: NCLEX: Physiological Integrity

Which statement by a patient would alert the nurse to a possible immunodeficiency disorder? a. "I take one baby aspirin every day to prevent stroke." b. "I usually eat eggs or meat for at least 2 meals a day." c. "I had my spleen removed many years ago after a car accident." d. "I had a chest x-ray 6 months ago when I had walking pneumonia."

ANS: C Splenectomy increases the risk for septicemia from bacterial infections. The patient's protein intake is good and should improve immune function. Daily aspirin use does not affect immune function. A chest x-ray does not have enough radiation to suppress immune function.

The nurse palpates enlarged cervical lymph nodes on a patient diagnosed with acute human immunodeficiency virus (HIV) infection. Which action would be most appropriate for the nurse to take? a. Instruct the patient to apply ice to the neck. b. Advise the patient that this is probably the flu. c. Explain to the patient that this is an expected finding. d. Request that an antibiotic be prescribed for the patient.

ANS: C Persistent generalized lymphadenopathy is common in the early stages of HIV infection. No antibiotic is needed because the enlarged nodes are probably not caused by bacteria. Applying ice to the neck may provide comfort, but the initial action is to reassure the patient this is an expected finding. Lymphadenopathy is common with acute HIV infection and is therefore not likely the flu.

Which menu choice by the patient who is receiving hemodialysis indicates that the nurse's teaching has been successful? a. Split-pea soup, English muffin, and nonfat milk b. Oatmeal with cream, half a banana, and herbal tea c. Poached eggs, whole-wheat toast, and apple juice d. Cheese sandwich, tomato soup, and cranberry juice

ANS: C Poached eggs would provide high-quality protein, and apple juice is low in potassium. Cheese is high in salt and phosphate, and tomato soup would be high in potassium. Split-pea soup is high in potassium, and dairy products are high in phosphate. Bananas are high in potassium, and the cream would be high in phosphate

A nurse has obtained donor tissue typing information about a patient who is waiting for a kidney transplant. Which results should be reported to the transplant surgeon? a. Patient is Rh positive and donor is Rh negative b. Six antigen matches are present in HLA typing c. Results of patient-donor cross matching are positive d. Panel of reactive antibodies (PRA) percentage is low

ANS: C Positive crossmatching is an absolute contraindication to kidney transplantation, since a hyperacute rejection will occur after the transplant. The other information indicates that the tissue match between the patient and potential donor is acceptable

An occupational health nurse works at a manufacturing plant where there is potential exposure to inhaled dust. Which action, if recommended by the nurse, will be most helpful in reducing the incidence of lung disease? a. Treat workers with pulmonary fibrosis. b. Teach about symptoms of lung disease. c. Require the use of protective equipment. d. Monitor workers for coughing and wheezing.

ANS: C Prevention of lung disease requires the use of appropriate protective equipment such as masks. The other actions will help in recognition or early treatment of lung disease but will not be effective in prevention of lung damage. Repeated exposure eventually results in diffuse pulmonary fibrosis. Fibrosis is the result of tissue repair after inflammation

A few months after bariatric surgery, a 56-year-old man tells the nurse, "My skin is hanging in folds. I think I need cosmetic surgery." Which response by the nurse is most appropriate? a. "The important thing is that you are improving your health." b. "The skinfolds will disappear once most of the weight is lost." c. "Cosmetic surgery is a possibility once your weight has stabilized." d. "Perhaps you would like to talk to a counselor about your body image."

ANS: C Reconstructive surgery may be used to eliminate excess skinfolds after at least a year has passed since the surgery. Skinfolds may not disappear over time, especially in older patients. The response, "The important thing is that your weight loss is improving your health," ignores the patient's concerns about appearance and implies that the nurse knows what is important. Whereas it may be helpful for the patient to talk to a counselor, it is more likely to be helpful to know that cosmetic surgery is available.

38. Four hours after a bowel resection, a 74-year-old male patient with a nasogastric tube to suction complains of nausea and abdominal distention. The first action by the nurse should be to a. auscultate for hypotonic bowel sounds. b. notify the patient's health care provider. c. reposition the tube and check for placement. d. remove the tube and replace it with a new one.

ANS: C Repositioning the tube will frequently facilitate drainage. Because this is a common occurrence, it is not appropriate to notify the health care provider unless other interventions do not resolve the problem. Information about the presence or absence of bowel sounds will not be helpful in improving drainage. Removing the tube and replacing it are unnecessarily traumatic to the patient, so that would only be done if the tube was completely occluded.

Which patient should the nurse assess first? a. Patient with urticaria after receiving an IV antibiotic b. Patient who has graft-versus-host disease and severe diarrhea c. Patient who is sneezing after having subcutaneous immunotherapy d. Patient with multiple chemical sensitivities who has muscle stiffness

ANS: C Sneezing after subcutaneous immunotherapy may indicate impending anaphylaxis and assessment and emergency measures should be initiated. The other patients also have findings that need assessment and intervention by the nurse, but do not have evidence of life-threatening complications.

The health care provider writes an order for bacteriologic testing for a patient who has a positive tuberculosis skin test. Which action should the nurse take? a. Teach about the reason for the blood tests. b. Schedule an appointment for a chest x-ray. c. Teach about the need to get sputum specimens for 2 to 3 consecutive days. d. Instruct the patient to expectorate three specimens as soon as possible.

ANS: C Sputum specimens are obtained on 2 to 3 consecutive days for bacteriologic testing for M. tuberculosis. The patient should not provide all the specimens at once. Blood cultures are not used for tuberculosis testing. A chest x-ray is not bacteriologic testing. Although the findings on chest x-ray examination are important, it is not possible to make a diagnosis of TB solely based on chest x-ray findings because other diseases can mimic the appearance of TB

A patient who has acute glomerulonephritis is hospitalized with hyperkalemia. Which information will the nurse monitor to evaluate the effectiveness of the prescribed calcium gluconate IV? a. Urine volume b. Calcium level c. Cardiac rhythm d. Neurologic status

ANS: C The calcium gluconate helps prevent dysrhythmias that might be caused by the hyperkalemia. The nurse will monitor the other data as well, but these will not be helpful in determining the effectiveness of the calcium gluconate

A patient is being prepared for a spinal fusion. While in the holding area, which action by a member of the surgical team requires rapid intervention by the charge nurse? a. Wearing street clothes into the nursing station b. Wearing a surgical mask into the holding room c. Walking into the hallway outside an operating room without the hair covered d. Putting on a surgical mask, cap, and scrubs before entering the operating room

ANS: C The corridors outside the operating room (OR) are part of the semirestricted area where personnel must wear surgical attire and head coverings. Surgical masks may be worn in the holding room, although they are not necessary. Street clothes may be worn at the nursing station, which is part of the unrestricted area. Wearing a mask and scrubs is essential when going into the OR

47. A 76-year-old patient with obstipation has a fecal impaction and is incontinent of liquid stool. Which action should the nurse take first? a. Administer bulk-forming laxatives. b. Assist the patient to sit on the toilet. c. Manually remove the impacted stool. d. Increase the patient's oral fluid intake.

ANS: C The initial action with a fecal impaction is manual disimpaction. The other actions will be used to prevent future constipation and impactions.

39. A 19-year-old female is brought to the emergency department with a knife handle protruding from the abdomen. During the initial assessment of the patient, the nurse should a. remove the knife and assess the wound. b. determine the presence of Rovsing sign. c. check for circulation and tissue perfusion. d. insert a urinary catheter and assess for hematuria.

ANS: C The initial assessment is focused on determining whether the patient has hypovolemic shock. The knife should not be removed until the patient is in surgery, where bleeding can be controlled. Rovsing sign is assessed in the patient with suspected appendicitis. A patient with a knife in place will be taken to surgery and assessed for bladder trauma there.

A 22-year-old patient seen at the health clinic with a severe migraine headache tells the nurse about having other similar headaches recently. Which initial action should the nurse take? a. Teach about the use of triptan drugs. b. Refer the patient for stress counseling. c. Ask the patient to keep a headache diary. d. Suggest the use of muscle-relaxation techniques.

ANS: C The initial nursing action should be further assessment of the precipitating causes of the headaches, quality, and location of pain, etc. Stress reduction, muscle relaxation, and the triptan drugs may be helpful, but more assessment is needed first. DIF: Cognitive Level: Apply (application) REF: 1419 OBJ: Special Questions: Prioritization TOP: Nursing Process: Implementation MSC: NCLEX: Physiological Integrity

14. A 20-year-old female is being admitted for electrolyte disorders of unknown etiology. Which assessment finding is most important to report to the health care provider? a. The patient uses laxatives daily. b. The patient's knuckles are macerated. c. The patient's serum potassium level is 2.9 mEq/L. d. The patient has a history of large weight fluctuations.

ANS: C The low serum potassium level may cause life-threatening cardiac dysrhythmias, and potassium supplementation is needed rapidly. The other information will also be reported because it suggests that bulimia may be the etiology of the patient's electrolyte disturbances, but it does not suggest imminent life-threatening complications.

A patient in surgery receives a neuromuscular blocking agent as an adjunct to general anesthesia. While in the postanesthesia care unit (PACU), what assessment finding is most important for the nurse to report? a. Laryngospasm b. Complaint of nausea c. Weak chest wall movement d. Patient unable to recall the correct date

ANS: C The most serious adverse effect of the neuromuscular blocking agents is weakness of the respiratory muscles, which can lead to postoperative hypoxemia. Nausea and confusion are possible adverse effects of these drugs, but they are not as great of concern as respiratory depression. Because these medications decrease muscle contraction, laryngospasm is not a concern

A patient with right lower-lobe pneumonia has been treated with IV antibiotics for 3 days. Which assessment data obtained by the nurse indicates that the treatment has been effective? a. Bronchial breath sounds are heard at the right base. b. The patient coughs up small amounts of green mucus. c. The patient's white blood cell (WBC) count is 9000/µL. d. Increased tactile fremitus is palpable over the right chest.

ANS: C The normal WBC count indicates that the antibiotics have been effective. All the other data suggest that a change in treatment is needed

The nurse plans to provide preoperative teaching to an alert older man who has hearing and vision deficits. His wife usually answers most questions that are directed to the patient. Which action should the nurse take when doing the teaching? a. Use printed materials for instruction so that the patient will have more time to review the material. b. Direct the teaching toward the wife because she is the obvious support and caregiver for the patient. c. Provide additional time for the patient to understand preoperative instructions and carry out procedures. d. Ask the patient's wife to wait in the hall in order to focus preoperative teaching with the patient himself.

ANS: C The nurse should allow more time when doing preoperative teaching and preparation for older patients with sensory deficits. Because the patient has visual deficits, he will not be able to use written material for learning. The teaching should be directed toward both the patient and the wife because both will need to understand preoperative procedures and teaching.

To decrease the risk for future hearing loss, which action should the nurse who is working with college students at the on-campus health clinic implement? a. Arrange to include otoscopic examinations for all patients. b. Administer influenza immunizations to all students at the clinic. c. Discuss the importance of limiting exposure to amplified music. d. Perform tympanometry on all patients between the ages of 18 to 24.

ANS: C The nurse should discuss the impact of amplified music on hearing with young adults and discourage listening to very amplified music, especially for prolonged periods. Tympanometry measures the ability of the eardrum to vibrate and would not help prevent future hearing loss. Although students are at risk for the influenza virus, being vaccinated does not help prevent future hearing loss. Otoscopic examinations are not necessary for all patients

At his first postoperative checkup appointment after a gastrojejunostomy (Billroth II), a patient reports that dizziness, weakness, and palpitations occur about 20 minutes after each meal. The nurse will teach the patient to a. increase the amount of fluid with meals. b. eat foods that are higher in carbohydrates. c. lie down for about 30 minutes after eating. d. drink sugared fluids or eat candy after meals.

ANS: C The patient is experiencing symptoms of dumping syndrome, which may be reduced by lying down after eating. Increasing fluid intake and choosing high carbohydrate foods will increase the risk for dumping syndrome. Having a sweet drink or hard candy will correct the hypoglycemia that is associated with dumping syndrome but will not prevent dumping syndrome

1. The nurse teaches a patient with chronic bronchitis about a new prescription for Advair Diskus (combined fluticasone and salmeterol). Which action by the patient would indicate to the nurse that teaching about medication administration has been successful? a. The patient shakes the device before use. b. The patient attaches a spacer to the Diskus. c. The patient rapidly inhales the medication. d. The patient performs huff coughing after inhalation.

ANS: C The patient should inhale the medication rapidly. Otherwise the dry particles will stick to the tongue and oral mucosa and not get inhaled into the lungs. Advair Diskus is a dry powder inhaler; shaking is not recommended. Spacers are not used with dry powder inhalers. Huff coughing is a technique to move mucus into larger airways to expectorate. The patient should not huff cough or exhale forcefully after taking Advair in order to keep the medication in the lungs. DIF: Cognitive Level: Apply (application) REF: 574 TOP: Nursing Process: Evaluation MSC: NCLEX: Physiological Integrity

Which statement by a 62-year-old patient with stage 5 chronic kidney disease (CKD) indicates that the nurse's teaching about management of CKD has been effective? a. "I need to get most of my protein from low-fat dairy products." b. "I will increase my intake of fruits and vegetables to 5 per day." c. "I will measure my urinary output each day to help calculate the amount I can drink." d. "I need to take erythropoietin to boost my immune system and help prevent infection."

ANS: C The patient with end-stage kidney disease is taught to measure urine output as a means of determining an appropriate oral fluid intake. Erythropoietin is given to increase the red blood cell count and will not offer any benefit for immune function. Dairy products are restricted because of the high phosphate level. Many fruits and vegetables are high in potassium and should be restricted in the patient with CKD.

While obtaining a health history from a patient, the nurse learns that the patient has a history of allergic rhinitis and multiple food allergies. Which action by the nurse is most appropriate? a. Encourage the patient to carry an epinephrine kit in case a type IV allergic reaction to latex develops. b. Advise the patient to use oil-based hand creams to decrease contact with natural proteins in latex gloves. c. Document the patient's allergy history and be alert for any clinical manifestations of a type I latex allergy. d. Recommend that the patient use vinyl gloves instead of latex gloves in preventing blood-borne pathogen contact.

ANS: C The patient's allergy history and occupation indicate a risk of developing a latex allergy. The nurse should be prepared to manage any symptoms that may occur. Epinephrine is not an appropriate treatment for contact dermatitis that is caused by a type IV allergic reaction to latex. Oil-based creams will increase the exposure to latex from latex gloves. Vinyl gloves are appropriate to use when exposure to body fluids is unlikely.

After the insertion of an arteriovenous graft (AVG) in the right forearm, a 54-year-old patient complains of pain and coldness of the right fingers. Which action should the nurse take? a. Teach the patient about normal AVG function. b. Remind the patient to take a daily low-dose aspirin tablet. c. Report the patient's symptoms to the health care provider. d. Elevate the patient's arm on pillows to above the heart level.

ANS: C The patient's complaints suggest the development of distal ischemia (steal syndrome) and may require revision of the AVG. Elevation of the arm above the heart will further decrease perfusion. Pain and coolness are not normal after AVG insertion. Aspirin therapy is not used to maintain grafts

Which patient should the nurse assess first after receiving change-of-shift report? a. A patient with nausea who has a dose of metoclopramide (Reglan) due b. A patient who is crying after receiving a diagnosis of esophageal cancer c. A patient with esophageal varices who has a blood pressure of 92/58 mm Hg d. A patient admitted yesterday with gastrointestinal (GI) bleeding who has melena

ANS: C The patient's history and blood pressure indicate possible hemodynamic instability caused by GI bleeding. The data about the other patients do not indicate acutely life-threatening complications

6. A patient seen in the asthma clinic has recorded daily peak flows that are 75% of the baseline. Which action will the nurse plan to take next? a. Increase the dose of the leukotriene inhibitor. b. Teach the patient about the use of oral corticosteroids. c. Administer a bronchodilator and recheck the peak flow. d. Instruct the patient to keep the next scheduled follow-up appointment.

ANS: C The patient's peak flow reading indicates that the condition is worsening (yellow zone). The patient should take the bronchodilator and recheck the peak flow. Depending on whether the patient returns to the green zone, indicating well-controlled symptoms, the patient may be prescribed oral corticosteroids or a change in dosing of other medications. Keeping the next appointment is appropriate, but the patient also needs to be taught how to control symptoms now and use the bronchodilator. DIF: Cognitive Level: Apply (application) REF: 580 TOP: Nursing Process: Planning MSC: NCLEX: Physiological Integrity

What information will the nurse include for an overweight 35-year-old woman who is starting a weight-loss plan? a. Weigh yourself at the same time every morning and evening. b. Stick to a 600- to 800-calorie diet for the most rapid weight loss. c. Low carbohydrate diets lead to rapid weight loss but are difficult to maintain. d. Weighing all foods on a scale is necessary to choose appropriate portion sizes.

ANS: C The restrictive nature of fad diets makes the weight loss achieved by the patient more difficult to maintain. Portion size can be estimated in other ways besides weighing. Severely calorie-restricted diets are not necessary for patients in the overweight category of obesity and need to be closely supervised. Patients should weigh weekly rather than daily.

The nurse educator facilitates student clinical experiences in the surgical suite. Which action, if performed by a student, would require the nurse educator to intervene? a. The student wears a mask at the sink area. b. The student wears street clothes in the unrestricted area. c. The student wears surgical scrubs in the semirestricted area. d. The student covers head and facial hair in the semirestricted area.

ANS: C The surgical suite is divided into three distinct areas: unrestricted—staff and others in street clothes can interact with those in surgical attire; semirestricted—staff must wear surgical attire and cover all head and facial hair; restricted—includes the operating room, the sink area, and clean core where masks are required in addition to surgical attire

A 58-year-old woman who recently has been diagnosed with esophageal cancer tells the nurse, "I do not feel ready to die yet." Which response by the nurse is most appropriate? a. "You may have quite a few years still left to live." b. "Thinking about dying will only make you feel worse." c. "Having this new diagnosis must be very hard for you." d. "It is important that you be realistic about your prognosis."

ANS: C This response is open-ended and will encourage the patient to further discuss feelings of anxiety or sadness about the diagnosis. Patients with esophageal cancer have only a low survival rate, so the response "You may have quite a few years still left to live" is misleading. The response beginning, "Thinking about dying" indicates that the nurse is not open to discussing the patient's fears of dying. The response beginning, "It is important that you be realistic," discourages the patient from feeling hopeful, which is important to patients with any life-threatening diagnosis

19. A patient's peripheral parenteral nutrition (PN) bag is nearly empty and a new PN bag has not arrived yet from the pharmacy. Which intervention is the priority? a. Monitor the patient's capillary blood glucose until a new PN bag is hung. b. Flush the peripheral line with saline and wait until the new PN bag is available. c. Infuse 5% dextrose in water until the new PN bag is delivered from the pharmacy. d. Decrease the rate of the current PN infusion to 10 mL/hr until the new bag arrives.

ANS: C To prevent hypoglycemia, the nurse should infuse a 5% dextrose solution until the next PN bag can be started. Decreasing the rate of the ordered PN infusion is beyond the nurse's scope of practice. Flushing the line and then waiting for the next bag may lead to hypoglycemia. Monitoring the capillary blood glucose is appropriate but is not the priority.

The charge nurse is assigning rooms for new admissions. Which patient would be the most appropriate roommate for a patient who has acute rejection of an organ transplant? a. A patient who has viral pneumonia b. A patient with second-degree burns c. A patient who is recovering from an anaphylactic reaction to a bee sting d. A patient with graft-versus-host disease after a recent bone marrow transplant

ANS: C Treatment for a patient with acute rejection includes administration of additional immunosuppressants, and the patient should not be exposed to increased risk for infection as would occur from patients with viral pneumonia, graft-versus-host disease, and burns. There is no increased exposure to infection from a patient who had an anaphylactic reaction.

34. The nurse assesses a patient with a history of asthma. Which assessment finding indicates that the nurse should take immediate action? a. Pulse oximetry reading of 91% b. Respiratory rate of 26 breaths/minute c. Use of accessory muscles in breathing d. Peak expiratory flow rate of 240 L/minute

ANS: C Use of accessory muscle indicates that the patient is experiencing respiratory distress and rapid intervention is needed. The other data indicate the need for ongoing monitoring and assessment but do not suggest that immediate treatment is required. DIF: Cognitive Level: Apply (application) REF: 564-565 OBJ: Special Questions: Prioritization TOP: Nursing Process: Assessment MSC: NCLEX: Physiological Integrity

A 57-year-old man with Escherichia coli O157:H7 food poisoning is admitted to the hospital with bloody diarrhea and dehydration. Which order will the nurse question? a. Infuse lactated Ringer's solution at 250 mL/hr. b. Monitor blood urea nitrogen and creatinine daily. c. Administer loperamide (Imodium) after each stool. d. Provide a clear liquid diet and progress diet as tolerated.

ANS: C Use of antidiarrheal agents is avoided with this type of food poisoning. The other orders are appropriate

Which care activity for a patient with a paralytic ileus is appropriate for the registered nurse (RN) to delegate to unlicensed assistive personnel (UAP)? a. Auscultation for bowel sounds b. Nasogastric (NG) tube irrigation c. Applying petroleum jelly to the lips d. Assessment of the nares for irritation

C

The nurse is caring for a 54-year-old female patient on the first postoperative day after a Roux-en-Y gastric bypass procedure. Which assessment finding should be reported immediately to the surgeon? a. Bilateral crackles audible at both lung bases b. Redness, irritation, and skin breakdown in skinfolds c. Emesis of bile-colored fluid past the nasogastric (NG) tube d. Use of patient-controlled analgesia (PCA) several times an hour for pain

ANS: C Vomiting with an NG tube in place indicates that the NG tube needs to be repositioned by the surgeon to avoid putting stress on the gastric sutures. The nurse should implement actions to decrease skin irritation and have the patient cough and deep breathe, but these do not indicate a need for rapid notification of the surgeon. Frequent PCA use after bariatric surgery is expected.

12. A 22-year-old female patient with an exacerbation of ulcerative colitis is having 15 to 20 stools daily and has excoriated perianal skin. Which patient behavior indicates that teaching regarding maintenance of skin integrity has been effective? a. The patient uses incontinence briefs to contain loose stools. b. The patient asks for antidiarrheal medication after each stool. c. The patient uses witch hazel compresses to decrease irritation. d. The patient cleans the perianal area with soap after each stool.

ANS: C Witch hazel compresses are suggested to reduce anal irritation and discomfort. Incontinence briefs may trap diarrhea and increase the incidence of skin breakdown. Antidiarrheal medications are not given 15 to 20 times a day. The perianal area should be washed with plain water after each stool.

31. The nurse takes an admission history on a patient with possible asthma who has new-onset wheezing and shortness of breath. Which information may indicate a need for a change in therapy? a. The patient has chronic inflammatory bowel disease. b. The patient has a history of pneumonia 6 months ago. c. The patient takes propranolol (Inderal) for hypertension. d. The patient uses acetaminophen (Tylenol) for headaches.

ANS: C β-Blockers such as propranolol can cause bronchospasm in some patients with asthma. The other information will be documented in the health history but does not indicate a need for a change in therapy. DIF: Cognitive Level: Apply (application) REF: 576 TOP: Nursing Process: Assessment MSC: NCLEX: Physiological Integrity

Which actions will the nurse include in the surgical time-out procedure before surgery (select all that apply)? a. Check for placement of IV lines. b. Have the surgeon identify the patient. c. Have the patient state name and date of birth. d. Verify the patient identification band number. e. Ask the patient to state the surgical procedure. f. Confirm the hospital chart identification number.

ANS: C, D, E, F These actions are included in surgical time out. IV line placement and identification of the patient by the surgeon are not included in the surgical time-out procedure

Nurses in change-of-shift report are discussing the care of a patient with a stroke who has progressively increasing weakness and decreasing level of consciousness (LOC). Which nursing diagnosis do they determine has thehighest priority for the patient? a. Impaired physical mobility related to weakness b. Disturbed sensory perception related to brain injury c. Risk for impaired skin integrity related to immobility d. Risk for aspiration related to inability to protect airway

ANS: D Protection of the airway is the priority of nursing care for a patient having an acute stroke. The other diagnoses are also appropriate, but interventions to prevent aspiration are the priority at this time. DIF: Cognitive Level: Apply (application) REF: 1397 | 1401 OBJ: Special Questions: Prioritization TOP: Nursing Process: Analysis MSC: NCLEX: Physiological Integrity

A patient with left-sided weakness that started 60 minutes earlier is admitted to the emergency department and diagnostic tests are ordered. Which test should be done first? a. Complete blood count (CBC) b. Chest radiograph (Chest x-ray) c. 12-Lead electrocardiogram (ECG) d. Noncontrast computed tomography (CT) scan

ANS: D Rapid screening with a noncontrast CT scan is needed before administration of tissue plasminogen activator (tPA), which must be given within 4.5 hours of the onset of clinical manifestations of the stroke. The sooner the tPA is given, the less brain injury. The other diagnostic tests give information about possible causes of the stroke and do not need to be completed as urgently as the CT scan. DIF: Cognitive Level: Apply (application) REF: 1395 OBJ: Special Questions: Prioritization TOP: Nursing Process: Implementation MSC: NCLEX: Physiological Integrity

A 73-year-old patient with a stroke experiences facial drooping on the right side and right-sided arm and leg paralysis. When admitting the patient, which clinical manifestation will the nurse expect to find? a. Impulsive behavior b. Right-sided neglect c. Hyperactive left-sided tendon reflexes d. Difficulty comprehending instructions

ANS: D Right-sided paralysis indicates a left-brain stroke, which will lead to difficulty with comprehension and use of language. The left-side reflexes are likely to be intact. Impulsive behavior and neglect are more likely with a right-side stroke. DIF: Cognitive Level: Apply (application) REF: 1407 TOP: Nursing Process: Assessment MSC: NCLEX: Physiological Integrity

A 56-year-old patient arrives in the emergency department with hemiparesis and dysarthria that started 2 hours previously, and health records show a history of several transient ischemic attacks (TIAs). The nurse anticipates preparing the patient for a. surgical endarterectomy. b. transluminal angioplasty. c. intravenous heparin administration. d. tissue plasminogen activator (tPA) infusion.

ANS: D The patient's history and clinical manifestations suggest an acute ischemic stroke and a patient who is seen within 4.5 hours of stroke onset is likely to receive tPA (after screening with a CT scan). Heparin administration in the emergency phase is not indicated. Emergent carotid transluminal angioplasty or endarterectomy is not indicated for the patient who is having an acute ischemic stroke. DIF: Cognitive Level: Apply (application) REF: 1391 | 1398 TOP: Nursing Process: Planning MSC: NCLEX: Physiological Integrity

A patient in the emergency department with sudden-onset right-sided weakness is diagnosed with an intracerebral hemorrhage. Which information about the patient is most important to communicate to the health care provider? a. The patient's speech is difficult to understand. b. The patient's blood pressure is 144/90 mm Hg. c. The patient takes a diuretic because of a history of hypertension. d. The patient has atrial fibrillation and takes warfarin (Coumadin).

ANS: D The use of warfarin probably contributed to the intracerebral bleeding and remains a risk factor for further bleeding. Administration of vitamin K is needed to reverse the effects of the warfarin, especially if the patient is to have surgery to correct the bleeding. The history of hypertension is a risk factor for the patient but has no immediate effect on the patient's care. The BP of 144/90 indicates the need for ongoing monitoring but not for any immediate change in therapy. Slurred speech is consistent with a left-sided stroke, and no change in therapy is indicated. DIF: Cognitive Level: Apply (application) REF: 1398 OBJ: Special Questions: Prioritization TOP: Nursing Process: Assessment MSC: NCLEX: Physiological Integrity

3. A 64-year-old woman who has chronic constipation asks the nurse about the use of psyllium (Metamucil). Which information will the nurse include in the response? a. Absorption of fat-soluble vitamins may be reduced by fiber-containing laxatives. b. Dietary sources of fiber should be eliminated to prevent excessive gas formation. c. Use of this type of laxative to prevent constipation does not cause adverse effects. d. Large amounts of fluid should be taken to prevent impaction or bowel obstruction.

ANS: D A high fluid intake is needed when patients are using bulk-forming laxatives to avoid worsening constipation. Although bulk-forming laxatives are generally safe, the nurse should emphasize the possibility of constipation or obstipation if inadequate fluid intake occurs. Although increased gas formation is likely to occur with increased dietary fiber, the patient should gradually increase dietary fiber and eventually may not need the psyllium. Fat-soluble vitamin absorption is blocked by stool softeners and lubricants, not by bulk-forming laxatives.

A patient who is receiving immunotherapy has just received an allergen injection. Which assessment finding is most important to communicate to the health care provider? a. The patient's IgG level is increased. b. The injection site is red and swollen. c. The patient's allergy symptoms have not improved. d. There is a 2-cm wheal at the site of the allergen injection.

ANS: D A local reaction larger than quarter size may indicate that a decrease in the allergen dose is needed. An increase in IgG indicates that the therapy is effective. Redness and swelling at the site are not unusual. Because immunotherapy usually takes 1 to 2 years to achieve an effect, an improvement in the patient's symptoms is not expected after a few months

2. The nurse teaches a patient how to administer formoterol (Perforomist) through a nebulizer. Which action by the patient indicates good understanding of the teaching? a. The patient attaches a spacer before using the inhaler. b. The patient coughs vigorously after using the inhaler. c. The patient activates the inhaler at the onset of expiration. d. The patient removes the facial mask when misting has ceased.

ANS: D A nebulizer is used to administer aerosolized medication. A mist is seen when the medication is aerosolized, and when all of the medication has been used, the misting stops. The other options refer to inhaler use. Coughing vigorously after inhaling and activating the inhaler at the onset of expiration are both incorrect techniques when using an inhaler. DIF: Cognitive Level: Apply (application) REF: 574 TOP: Nursing Process: Evaluation MSC: NCLEX: Physiological Integrity

27. A 42-year-old male patient has had a herniorrhaphy to repair an incarcerated inguinal hernia. Which patient teaching will the nurse provide before discharge? a. Soak in sitz baths several times each day. b. Cough 5 times each hour for the next 48 hours. c. Avoid use of acetaminophen (Tylenol) for pain. d. Apply a scrotal support and ice to reduce swelling.

ANS: D A scrotal support and ice are used to reduce edema and pain. Coughing will increase pressure on the incision. Sitz baths will not relieve pain and would not be of use after this surgery. Acetaminophen can be used for postoperative pain.

17. A 73-year-old patient with diverticulosis has a large bowel obstruction. The nurse will monitor for a. referred back pain. b. metabolic alkalosis. c. projectile vomiting. d. abdominal distention.

ANS: D Abdominal distention is seen in lower intestinal obstruction. Referred back pain is not a common clinical manifestation of intestinal obstruction. Metabolic alkalosis is common in high intestinal obstruction because of the loss of HCl acid from vomiting. Projectile vomiting is associated with higher intestinal obstruction.

Which statement by the patient to the home health nurse indicates a need for more teaching about self-administering eardrops? a. "I will leave the ear wick in place while administering the drops." b. "I should lie down before and for 5 minutes after administering the drops." c. "I will hold the tip of the dropper above the ear while administering the drops." d. "I should keep the medication refrigerated until I am ready to administer the drops."

ANS: D Administration of cold eardrops can cause dizziness because of stimulation of the semicircular canals. The other patient actions are appropriate

A patient with pneumonia has a fever of 101.4° F (38.6° C), a nonproductive cough, and an oxygen saturation of 88%. The patient complains of weakness, fatigue, and needs assistance to get out of bed. Which nursing diagnosis should the nurse assign as the highest priority? a. Hyperthermia related to infectious illness b. Impaired transfer ability related to weakness c. Ineffective airway clearance related to thick secretions d. Impaired gas exchange related to respiratory congestion

ANS: D All these nursing diagnoses are appropriate for the patient, but the patient's oxygen saturation indicates that all body tissues are at risk for hypoxia unless the gas exchange is improved

A patient has received atropine before surgery and complains of dry mouth. Which action by the nurse is best? a. Check for skin tenting. b. Notify the health care provider. c. Ask the patient about any dizziness. d. Tell the patient dry mouth is an expected side effect.

ANS: D Anticholinergic medications decrease oral secretions, so the patient is taught that a dry mouth is an expected side effect. The dry mouth is not a symptom of dehydration in this case. Therefore there is no immediate need to check for skin tenting. The health care provider does not need to be notified about an expected side effect. Weakness, forgetfulness, and dizziness are side effects associated with other preoperative medications such as opioids and benzodiazepines

Which medication taken by a patient with restless legs syndrome should the nurse discuss with the patient? a. Multivitamin (Stresstabs) b. Acetaminophen (Tylenol) c. Ibuprofen (Motrin, Advil) d. Diphenhydramine (Benadryl)

ANS: D Antihistamines can aggravate restless legs syndrome. The other medications will not contribute to restless legs syndrome. DIF: Cognitive Level: Apply (application) REF: 1427 TOP: Nursing Process: Implementation MSC: NCLEX: Physiological Integrity

28. Which breakfast choice indicates a patient's good understanding of information about a diet for celiac disease? a. Oatmeal with nonfat milk b. Whole wheat toast with butter c. Bagel with low-fat cream cheese d. Corn tortilla with scrambled eggs

ANS: D Avoidance of gluten-containing foods is the only treatment for celiac disease. Corn does not contain gluten, while oatmeal and wheat do.

The nurse will anticipate teaching a patient experiencing frequent heartburn about a. a barium swallow. b. radionuclide tests. c. endoscopy procedures. d. proton pump inhibitors.

ANS: D Because diagnostic testing for heartburn that is probably caused by gastroesophageal reflux disease (GERD) is expensive and uncomfortable, proton pump inhibitors are frequently used for a short period as the first step in the diagnosis of GERD. The other tests may be used but are not usually the first step in diagnosis

The nurse cares for a patient who is human immunodeficiency virus (HIV) positive and taking antiretroviral therapy (ART). Which information is most important for the nurse to address when planning care? a. The patient's blood glucose level is 142 mg/dL. b. The patient complains of feeling "constantly tired." c. The patient is unable to state the side effects of the medications. d. The patient states, "Sometimes I miss a dose of zidovudine (AZT)."

ANS: D Because missing doses of ART can lead to drug resistance, this patient statement indicates the need for interventions such as teaching or changes in the drug scheduling. Elevated blood glucose and fatigue are common side effects of ART. The nurse should discuss medication side effects with the patient, but this is not as important as addressing the skipped doses of AZT.

The nurse obtains a health history from a patient who is scheduled for elective hip surgery in 1 week. The patient reports use of garlic and ginkgo biloba. Which action by the nurse is most appropriate? a. Ascertain that there will be no interactions with anesthetic agents. b. Teach the patient that these products may be continued preoperatively. c. Advise the patient to stop the use of all herbs and supplements at this time. d. Discuss the herb and supplement use with the patient's health care provider.

ANS: D Both garlic and ginkgo biloba increase a patient's risk for bleeding. The nurse should discuss the herb and supplement use with the patient's health care provider. The nurse should not advise the patient to stop the supplements or to continue them without consulting with the health care provider. Determining the interactions between the supplements and anesthetics is not within the nurse's scope of practice.

19. Postural drainage with percussion and vibration is ordered twice daily for a patient with chronic bronchitis. Which intervention should the nurse include in the plan of care? a. Schedule the procedure 1 hour after the patient eats. b. Maintain the patient in the lateral position for 20 minutes. c. Perform percussion before assisting the patient to the drainage position. d. Give the ordered albuterol (Proventil) before the patient receives the therapy.

ANS: D Bronchodilators are administered before chest physiotherapy. Postural drainage, percussion, and vibration should be done 1 hour before or 3 hours after meals. Patients remain in each postural drainage position for 5 minutes. Percussion is done while the patient is in the postural drainage position. DIF: Cognitive Level: Apply (application) REF: 594 TOP: Nursing Process: Planning MSC: NCLEX: Physiological Integrity

The nurse will most likely prepare a medication teaching plan about antiretroviral therapy (ART) for which patient? a. Patient who is currently HIV negative but has unprotected sex with multiple partners b. Patient who was infected with HIV 15 years ago and now has a CD4+ count of 840/µL c. HIV-positive patient with a CD4+ count of 160/µL who drinks a fifth of whiskey daily d. Patient who tested positive for HIV 2 years ago and now has cytomegalovirus (CMV) retinitis

ANS: D CMV retinitis is an acquired immunodeficiency syndrome (AIDS)-defining illness and indicates that the patient is appropriate for ART even though the HIV infection period is relatively short. An HIV-negative patient would not be offered ART. A patient with a CD4+ count in the normal range would not typically be started on ART. A patient who drinks alcohol heavily would be unlikely to be able to manage the complex drug regimen and would not be appropriate for ART despite the low CD4+ count.

31. The nurse will plan to teach a patient with Crohn's disease who has megaloblastic anemia about the need for a. oral ferrous sulfate tablets. b. regular blood transfusions. c. iron dextran (Imferon) infusions. d. cobalamin (B12) spray or injections.

ANS: D Crohn's disease frequently affects the ileum, where absorption of cobalamin occurs. Cobalamin must be administered regularly by nasal spray or IM to correct the anemia. Iron deficiency does not cause megaloblastic anemia. The patient may need occasional transfusions but not regularly scheduled transfusions.

Which information about a 60-year-old patient with MS indicates that the nurse should consult with the health care provider before giving the prescribed dose of dalfampridine (Ampyra)? a. The patient has relapsing-remitting MS. b. The patient walks a mile a day for exercise. c. The patient complains of pain with neck flexion. d. The patient has an increased serum creatinine level.

ANS: D Dalfampridine should not be given to patients with impaired renal function. The other information will not impact whether the dalfampridine should be administered. DIF: Cognitive Level: Apply (application) REF: 1431 TOP: Nursing Process: Assessment MSC: NCLEX: Physiological Integrity

A patient who is human immunodeficiency virus (HIV)-infected has a CD4+ cell count of 400/µL. Which factor is most important for the nurse to determine before the initiation of antiretroviral therapy (ART) for this patient? a. HIV genotype and phenotype b. Patient's social support system c. Potential medication side effects d. Patient's ability to comply with ART schedule

ANS: D Drug resistance develops quickly unless the patient takes ART medications on a strict, regular schedule. In addition, drug resistance endangers both the patient and the community. The other information is also important to consider, but patients who are unable to manage and follow a complex drug treatment regimen should not be considered for ART

A 76-year-old patient is being treated with carbidopa/levodopa (Sinemet) for Parkinson's disease. Which information is most important for the nurse to report to the health care provider? a. Shuffling gait b. Tremor at rest c. Cogwheel rigidity of limbs d. Uncontrolled head movement

ANS: D Dyskinesia is an adverse effect of the Sinemet, indicating a need for a change in medication or decrease in dose. The other findings are typical with Parkinson's disease. DIF: Cognitive Level: Apply (application) REF: 1435 OBJ: Special Questions: Prioritization TOP: Nursing Process: Planning MSC: NCLEX: Physiological Integrity

A patient who has just been admitted with community-acquired pneumococcal pneumonia has a temperature of 101.6° F with a frequent cough and is complaining of severe pleuritic chest pain. Which prescribed medication should the nurse give first? a. Codeine b. Guaifenesin (Robitussin) c. Acetaminophen (Tylenol) d. Piperacillin/tazobactam (Zosyn)

ANS: D Early initiation of antibiotic therapy has been demonstrated to reduce mortality. The other medications are also appropriate and should be given as soon as possible, but the priority is to start antibiotic therapy

20. The nurse develops a teaching plan to help increase activity tolerance at home for an older adult with severe chronic obstructive pulmonary disease (COPD). Which instructions would be most appropriate for the nurse to include in the plan of care? a. Stop exercising when short of breath. b. Walk until pulse rate exceeds 130 beats/minute. c. Limit exercise to activities of daily living (ADLs). d. Walk 15 to 20 minutes daily at least 3 times/week.

ANS: D Encourage the patient to walk 15 to 20 minutes a day at least three times a week with gradual increases. Shortness of breath is normal with exercise and not an indication that the patient should stop. Limiting exercise to ADLs will not improve the patient's exercise tolerance. A 70-year-old patient should have a pulse rate of 120 or less with exercise (80% of the maximal heart rate of 150). DIF: Cognitive Level: Apply (application) REF: 599 TOP: Nursing Process: Planning MSC: NCLEX: Physiological Integrity

When a 72-year-old patient is diagnosed with achalasia, the nurse will teach the patient that a. lying down after meals is recommended. b. a liquid or blenderized diet will be necessary. c. drinking fluids with meals should be avoided. d. treatment may include endoscopic procedures.

ANS: D Endoscopic and laparoscopic procedures are the most effective therapy for improving symptoms caused by achalasia. Keeping the head elevated after eating will improve esophageal emptying. A semisoft diet is recommended to improve esophageal emptying. Patients are advised to drink fluid with meals

3. Which menu choice indicates that the patient is implementing plans to choose high-calorie, high-protein foods? a. Baked fish with applesauce b. Beef noodle soup and canned corn c. Fresh fruit salad with yogurt topping d. Fried chicken with potatoes and gravy

ANS: D Foods that are high in calories include fried foods and those covered with sauces. High protein foods include meat and dairy products. The other choices are lower in calories and protein.

An older adult patient has a prescription for cyclosporine following a kidney transplant. Which information in the patient's health history has the most implications for planning patient teaching about the medication at this time? a. The patient restricts salt to treat prehypertension. b. The patient drinks 3 to 4 quarts of fluids every day. c. The patient has many concerns about the effects of cyclosporine. d. The patient has a glass of grapefruit juice every day for breakfast.

ANS: D Grapefruit juice can increase the toxicity of cyclosporine. The patient should be taught to avoid grapefruit juice. High fluid intake will not affect cyclosporine levels or renal function. Cyclosporine may cause hypertension, and the patient's many concerns should be addressed, but these are not potentially life-threatening problems.

Which action will the nurse take immediately after surgery for a patient who received ketamine (Ketalar) as an anesthetic agent? a. Administer higher doses of analgesic agents. b. Ensure that atropine is available in case of bradycardia. c. Question the order for benzodiazepines to be administered. d. Provide a quiet environment in the postanesthesia care unit.

ANS: D Hallucinations are an adverse effect associated with the dissociative anesthetics such as ketamine. Therefore the postoperative environment should be kept quiet to decrease the risk of hallucinations. Because ketamine causes profound analgesia lasting into the postoperative period, higher doses of analgesics are not needed. Ketamine causes an increase in heart rate. Benzodiazepine may be used with ketamine to decrease the incidence of hallucinations and nightmares

A 62-year-old patient who has Parkinson's disease is taking bromocriptine (Parlodel). Which information obtained by the nurse may indicate a need for a decrease in the dose? a. The patient has a chronic dry cough. b. The patient has four loose stools in a day. c. The patient develops a deep vein thrombosis. d. The patient's blood pressure is 92/52 mm Hg.

ANS: D Hypotension is an adverse effect of bromocriptine, and the nurse should check with the health care provider before giving the medication. Diarrhea, cough, and deep vein thrombosis are not associated with bromocriptine use. DIF: Cognitive Level: Apply (application) REF: 1435 TOP: Nursing Process: Evaluation MSC: NCLEX: Physiological Integrity

As the nurse prepares a patient the morning of surgery, the patient refuses to remove a wedding ring, saying, "I have never taken it off since the day I was married." Which response by the nurse is best? a. Have the patient sign a release and leave the ring on. b. Tape the wedding ring securely to the patient's finger. c. Tell the patient that the hospital is not liable for loss of the ring. d. Suggest that the patient give the ring to a family member to keep.

ANS: D Jewelry is not allowed to be worn by the patient, especially if electrocautery will be used. There is no need for a release form or to discuss liability with the patient.

A clinic patient is experiencing an allergic reaction to an unknown allergen. Which action is most appropriate for the registered nurse (RN) to delegate to a licensed practical/vocational nurse (LPN/LVN)? a. Perform a focused physical assessment. b. Obtain the health history from the patient. c. Teach the patient about the various diagnostic studies. d. Administer skin testing by the cutaneous scratch method.

ANS: D LPN/LVNs are educated and licensed to administer medications under the supervision of an RN. RN-level education and the scope of practice include assessment of health history, focused physical assessment, and patient teaching.

30. The nurse completes an admission assessment on a patient with asthma. Which information given by patient is most indicative of a need for a change in therapy? a. The patient uses albuterol (Proventil) before any aerobic exercise. b. The patient says that the asthma symptoms are worse every spring. c. The patient's heart rate increases after using the albuterol (Proventil) inhaler. d. The patient's only medications are albuterol (Proventil) and salmeterol (Serevent).

ANS: D Long-acting β2-agonists should be used only in patients who also are using an inhaled corticosteroid for long-term control. Salmeterol should not be used as the first-line therapy for long-term control. Using a bronchodilator before exercise is appropriate. The other information given by the patient requires further assessment by the nurse, but is not unusual for a patient with asthma. DIF: Cognitive Level: Apply (application) REF: 572 TOP: Nursing Process: Assessment MSC: NCLEX: Physiological Integrity

Which intervention will the nurse include in the plan of care for a patient who is diagnosed with a lung abscess? a. Teach the patient to avoid the use of over-the-counter expectorants. b. Assist the patient with chest physiotherapy and postural drainage. c. Notify the health care provider immediately about any bloody or foul-smelling sputum. d. Teach about the need for prolonged antibiotic therapy after discharge from the hospital.

ANS: D Long-term antibiotic therapy is needed for effective eradication of the infecting organisms in lung abscess. Chest physiotherapy and postural drainage are not recommended for lung abscess because they may lead to spread of the infection. Foul smelling and bloody sputum are common clinical manifestations in lung abscess. Expectorants may be used because the patient is encouraged to cough

The nurse completes discharge teaching for a patient who has had a lung transplant. The nurse evaluates that the teaching has been effective if the patient makes which statement? a. "I will make an appointment to see the doctor every year." b. "I will stop taking the prednisone if I experience a dry cough." c. "I will not worry if I feel a little short of breath with exercise." d. "I will call the health care provider right away if I develop a fever."

ANS: D Low-grade fever may indicate infection or acute rejection so the patient should notify the health care provider immediately if the temperature is elevated. Patients require frequent follow-up visits with the transplant team. Annual health care provider visits would not be sufficient. Home oxygen use is not an expectation after lung transplant. Shortness of breath should be reported. Low-grade fever, fatigue, dyspnea, dry cough, and oxygen desaturation are signs of rejection. Immunosuppressive therapy, including prednisone, needs to be continued to prevent rejection

The nurse is caring for a patient undergoing plasmapheresis. The nurse should assess the patient for which clinical manifestation? a. Shortness of breath b. High blood pressure c. Transfusion reaction d. Numbness and tingling

ANS: D Numbness and tingling may occur as the result of the hypocalcemia caused by the citrate used to prevent coagulation. The other clinical manifestations are not associated with plasmapheresis.

Which information about a 72-year-old patient who has a new prescription for phenytoin (Dilantin) indicates that the nurse should consult with the health care provider before administration of the medication? a. Patient has generalized tonic-clonic seizures. b. Patient experiences an aura before seizures. c. Patient's most recent blood pressure is 156/92 mm Hg. d. Patient has minor elevations in the liver function tests.

ANS: D Many older patients (especially with compromised liver function) may not be able to metabolize phenytoin. The health care provider may need to choose another antiseizure medication. Phenytoin is an appropriate medication for patients with tonic-clonic seizures, with or without an aura. Hypertension is not a contraindication for phenytoin therapy. DIF: Cognitive Level: Apply (application) REF: 1424 TOP: Nursing Process: Implementation MSC: NCLEX: Physiological Integrity

A 62-year-old man patient who requires daily use of a nonsteroidal antiinflammatory drug (NSAID) for the management of severe rheumatoid arthritis has recently developed melena. The nurse will anticipate teaching the patient about a. substitution of acetaminophen (Tylenol) for the NSAID. b. use of enteric-coated NSAIDs to reduce gastric irritation. c. reasons for using corticosteroids to treat the rheumatoid arthritis. d. misoprostol (Cytotec) to protect the gastrointestinal (GI) mucosa.

ANS: D Misoprostol, a prostaglandin analog, reduces acid secretion and the incidence of upper GI bleeding associated with NSAID use. Enteric coating of NSAIDs does not reduce the risk for GI bleeding. Corticosteroids increase the risk for ulcer development, and will not be substituted for NSAIDs for this patient. Acetaminophen will not be effective in treating the patient's rheumatoid arthritis

A lobectomy is scheduled for a patient with stage I non-small cell lung cancer. The patient tells the nurse, "I would rather have chemotherapy than surgery." Which response by the nurse is most appropriate? a. "Are you afraid that the surgery will be very painful?" b. "Did you have bad experiences with previous surgeries?" c. "Surgery is the treatment of choice for stage I lung cancer." d. "Tell me what you know about the various treatments available."

ANS: D More assessment of the patient's concerns about surgery is indicated. An open-ended response will elicit the most information from the patient. The answer beginning, "Surgery is the treatment of choice" is accurate, but it discourages the patient from sharing concerns about surgery. The remaining two answers indicate that the nurse has jumped to conclusions about the patient's reasons for not wanting surgery. Chemotherapy is the primary treatment for small cell lung cancer. In non-small cell lung cancer, chemotherapy may be used in the treatment of nonresectable tumors or as adjuvant therapy to surgery

A patient who is diagnosed with acquired immunodeficiency syndrome (AIDS) tells the nurse, "I feel obsessed with thoughts about dying. Do you think I am just being morbid?" Which response by the nurse is best? a. "Thinking about dying will not improve the course of AIDS." b. "It is important to focus on the good things about your life now." c. "Do you think that taking an antidepressant might be helpful to you?" d. "Can you tell me more about the kind of thoughts that you are having?"

ANS: D More assessment of the patient's psychosocial status is needed before taking any other action. The statements, "Thinking about dying will not improve the course of AIDS" and "It is important to focus on the good things in life" discourage the patient from sharing any further information with the nurse and decrease the nurse's ability to develop a trusting relationship with the patient. Although antidepressants may be helpful, the initial action should be further assessment of the patient's feelings.

A patient complains of leg cramps during hemodialysis. The nurse should first a. massage the patient's legs. b. reposition the patient supine. c. give acetaminophen (Tylenol). d. infuse a bolus of normal saline.

ANS: D Muscle cramps during dialysis are caused by rapid removal of sodium and water. Treatment includes infusion of normal saline. The other actions do not address the reason for the cramps.

A patient is admitted with active tuberculosis (TB). The nurse should question a health care provider's order to discontinue airborne precautions unless which assessment finding is documented? a. Chest x-ray shows no upper lobe infiltrates. b. TB medications have been taken for 6 months. c. Mantoux testing shows an induration of 10 mm. d. Three sputum smears for acid-fast bacilli are negative.

ANS: D Negative sputum smears indicate that Mycobacterium tuberculosis is not present in the sputum, and the patient cannot transmit the bacteria by the airborne route. Chest x-rays are not used to determine whether treatment has been successful. Taking medications for 6 months is necessary, but the multidrug-resistant forms of the disease might not be eradicated after 6 months of therapy. Repeat Mantoux testing would not be done because the result will not change even with effective treatment

Which information will the nurse include when teaching a patient with keratitis caused by herpes simplex type 1? a. Correct use of the antifungal eyedrops natamycin (Natacyn) b. How to apply corticosteroid ophthalmic ointment to the eyes c. Avoidance of nonsteroidal antiinflammatory drugs (NSAIDs) d. Importance of taking all of the ordered oral acyclovir (Zovirax)

ANS: D Oral acyclovir may be ordered for herpes simplex infections. Corticosteroid ointments are usually contraindicated because they prolong the course of the infection. Herpes simplex I is viral, not parasitic, or fungal. Natamycin may be used for Acanthamoeba keratitis caused by a parasite. NSAIDs can be used to treat the pain associated with keratitis

The nurse is performing tuberculosis (TB) skin tests in a clinic that has many patients who have immigrated to the United States. Which question is most important for the nurse to ask before the skin test? a. "Is there any family history of TB?" b. "How long have you lived in the United States?" c. "Do you take any over-the-counter (OTC) medications?" d. "Have you received the bacille Calmette-Guérin (BCG) vaccine for TB?"

ANS: D Patients who have received the BCG vaccine will have a positive Mantoux test. Another method for screening (such as a chest x-ray) will need to be used in determining whether the patient has a TB infection. The other information also may be valuable but is not as pertinent to the decision about doing TB skin testing

The nurse supervises unlicensed assistive personnel (UAP) who are providing care for a patient with right lower lobe pneumonia. The nurse should intervene if which action by UAP is observed? a. UAP splint the patient's chest during coughing. b. UAP assist the patient to ambulate to the bathroom. c. UAP help the patient to a bedside chair for meals. d. UAP lower the head of the patient's bed to 15 degrees.

ANS: D Positioning the patient with the head of the bed lowered will decrease ventilation. The other actions are appropriate for a patient with pneumonia

The nurse provides discharge teaching for a patient who has two fractured ribs from an automobile accident. Which statement, if made by the patient, would indicate that teaching has been effective? a. "I am going to buy a rib binder to wear during the day." b. "I can take shallow breaths to prevent my chest from hurting." c. "I should plan on taking the pain pills only at bedtime so I can sleep." d. "I will use the incentive spirometer every hour or two during the day."

ANS: D Prevention of the complications of atelectasis and pneumonia is a priority after rib fracture. This can be ensured by deep breathing and coughing. Use of a rib binder, shallow breathing, and taking pain medications only at night are likely to result in atelectasis

23. A 55-year-old patient with increasing dyspnea is being evaluated for a possible diagnosis of chronic obstructive pulmonary disease (COPD). When teaching a patient about pulmonary function testing (PFT) for this condition, what is the most important question the nurse should ask? a. "Are you claustrophobic?" b. "Are you allergic to shellfish?" c. "Do you have any metal implants or prostheses?" d. "Have you taken any bronchodilators in the past 6 hours?"

ANS: D Pulmonary function testing will help establish the COPD diagnosis. Bronchodilators should be avoided at least 6 hours before the test. PFTs do not involve being placed in an enclosed area such as for magnetic resonance imaging (MRI). Contrast dye is not used for PFTs. The patient may still have PFTs done if metal implants or prostheses are present, as these are contraindications for an MRI. DIF: Cognitive Level: Apply (application) REF: 566 TOP: Nursing Process: Planning MSC: NCLEX: Physiological Integrity

10. A patient with chronic obstructive pulmonary disease (COPD) has a nursing diagnosis of impaired breathing pattern related to anxiety. Which nursing action is most appropriate to include in the plan of care? a. Titrate oxygen to keep saturation at least 90%. b. Discuss a high-protein, high-calorie diet with the patient. c. Suggest the use of over-the-counter sedative medications. d. Teach the patient how to effectively use pursed lip breathing.

ANS: D Pursed lip breathing techniques assist in prolonging the expiratory phase of respiration and decrease air trapping. There is no indication that the patient requires oxygen therapy or an improved diet. Sedative medications should be avoided because they decrease respiratory drive. DIF: Cognitive Level: Apply (application) REF: 578 TOP: Nursing Process: Planning MSC: NCLEX: Physiological Integrity

7. The nurse teaches a patient who has asthma about peak flow meter use. Which action by the patient indicates that teaching was successful? a. The patient inhales rapidly through the peak flow meter mouthpiece. b. The patient takes montelukast (Singulair) for peak flows in the red zone. c. The patient calls the health care provider when the peak flow is in the green zone. d. The patient uses albuterol (Proventil) metered dose inhaler (MDI) for peak flows in the yellow zone.

ANS: D Readings in the yellow zone indicate a decrease in peak flow. The patient should use short-acting β2-adrenergic (SABA) medications. Readings in the green zone indicate good asthma control. The patient should exhale quickly and forcefully through the peak flow meter mouthpiece to obtain the readings. Readings in the red zone do not indicate good peak flow, and the patient should take a fast-acting bronchodilator and call the health care provider for further instructions. Singulair is not indicated for acute attacks but rather is used for maintenance therapy. DIF: Cognitive Level: Apply (application) REF: 568 | 580 TOP: Nursing Process: Evaluation MSC: NCLEX: Physiological Integrity

24. A young adult patient with cystic fibrosis (CF) is admitted to the hospital with increased dyspnea. Which intervention should the nurse include in the plan of care? a. Schedule a sweat chloride test. b. Arrange for a hospice nurse visit. c. Place the patient on a low-sodium diet. d. Perform chest physiotherapy every 4 hours.

ANS: D Routine scheduling of airway clearance techniques is an essential intervention for patients with CF. A sweat chloride test is used to diagnose CF, but it does not provide any information about the effectiveness of therapy. There is no indication that the patient is terminally ill. Patients with CF lose excessive sodium in their sweat and require high amounts of dietary sodium. DIF: Cognitive Level: Apply (application) REF: 594 TOP: Nursing Process: Planning MSC: NCLEX: Physiological Integrity

The nurse is completing the admission database for a patient admitted with abdominal pain and notes a history of hypertension and glaucoma. Which prescribed medications should the nurse question? a. Morphine sulfate 4 mg IV b. Diazepam (Valium) 5 mg IV c. Betaxolol (Betoptic) 0.25% eyedrops d. Scopolamine patch (Transderm Scop) 1.5 mg

ANS: D Scopolamine is a parasympathetic blocker and will relax the iris, causing blockage of aqueous humor outflow and an increase in intraocular pressure. The other medications are appropriate for this patient

The health care provider prescribes antacids and sucralfate (Carafate) for treatment of a patient's peptic ulcer. The nurse will teach the patient to take a. sucralfate at bedtime and antacids before each meal. b. sucralfate and antacids together 30 minutes before meals. c. antacids 30 minutes before each dose of sucralfate is taken. d. antacids after meals and sucralfate 30 minutes before meals.

ANS: D Sucralfate is most effective when the pH is low and should not be given with or soon after antacids. Antacids are most effective when taken after eating. Administration of sucralfate 30 minutes before eating and antacids just after eating will ensure that both drugs can be most effective. The other regimens will decrease the effectiveness of the medications.

The outpatient surgery nurse reviews the complete blood cell (CBC) count results for a patient who is scheduled for surgery in a few days. The results are white blood cell (WBC) count 10.2 × 103/µL; hemoglobin 15 g/dL; hematocrit 45%; platelets 150 × 103/µL. Which action should the nurse take? a. Call the surgeon and anesthesiologist immediately. b. Ask the patient about any symptoms of a recent infection. c. Discuss the possibility of blood transfusion with the patient. d. Send the patient to the holding area when the operating room calls.

ANS: D The CBC count results are normal. With normal results, the patient can go to the holding area when the operating room is ready for the patient. There is no need to notify the surgeon or anesthesiologist, discuss blood transfusion, or ask about recent infection.

Which nursing action will be most useful in assisting a college student to adhere to a newly prescribed antiretroviral therapy (ART) regimen? a. Give the patient detailed information about possible medication side effects. b. Remind the patient of the importance of taking the medications as scheduled. c. Encourage the patient to join a support group for students who are HIV positive. d. Check the patient's class schedule to help decide when the drugs should be taken.

ANS: D The best approach to improve adherence is to learn about important activities in the patient's life and adjust the ART around those activities. The other actions also are useful, but they will not improve adherence as much as individualizing the ART to the patient's schedule.

hich nursing diagnosis is of highest priority for a patient with Parkinson's disease who is unable to move the facial muscles? a. Activity intolerance b. Self-care deficit: toileting c. Ineffective self-health management d. Imbalanced nutrition: less than body requirements

ANS: D The data about the patient indicate that poor nutrition will be a concern because of decreased swallowing. The other diagnoses may also be appropriate for a patient with Parkinson's disease, but the data do not indicate that they are current problems for this patient. DIF: Cognitive Level: Apply (application) REF: 1436 OBJ: Special Questions: Prioritization TOP: Nursing Process: Analysis MSC: NCLEX: Physiological Integrity

9. During assessment of the patients skin, the nurse observes a similar pattern of small, raised lesions on the left and right upper back areas. Which term should the nurse use to document these lesions? a. Confluent b. Zosteriform c. Generalized d. Symmetric

ANS: D The description of the lesions indicates that they are grouped. The other terms are inconsistent with the description of the lesions. DIF: Cognitive Level: Understand (comprehension) REF: 401 TOP: Nursing Process: Assessment

After vertical banded gastroplasty, a 42-year-old male patient returns to the surgical nursing unit with a nasogastric tube to low, intermittent suction and a patient-controlled analgesia (PCA) machine for pain control. Which nursing action should be included in the postoperative plan of care? a. Offer sips of fruit juices at frequent intervals. b. Irrigate the nasogastric (NG) tube frequently. c. Remind the patient that PCA use may slow the return of bowel function. d. Support the surgical incision during patient coughing and turning in bed.

ANS: D The incision should be protected from strain to decrease the risk for wound dehiscence. The patient should be encouraged to use the PCA because pain control will improve the cough effort and patient mobility. NG irrigation may damage the suture line or overfill the stomach pouch. Sugar-free clear liquids are offered during the immediate postoperative time to decrease the risk for dumping syndrome.

The nurse interviews a patient scheduled to undergo general anesthesia for a hernia repair. Which information is most important to communicate to the surgeon and anesthesiologist before surgery? a. The patient drinks 3 or 4 cups of coffee every morning before going to work. b. The patient takes a baby aspirin daily but stopped taking aspirin 10 days ago. c. The patient drank 4 ounces of apple juice 3 hours before coming to the hospital. d. The patient's father died after receiving general anesthesia for abdominal surgery.

ANS: D The information about the patient's father suggests that there may be a family history of malignant hyperthermia and that precautions may need to be taken to prevent this complication. Current research indicates that having clear liquids 3 hours before surgery does not increase the risk for aspiration in most patients. Patients are instructed to discontinue aspirin 1 to 2 weeks before surgery. The patient should be offered caffeinated beverages postoperatively to prevent a caffeine-withdrawal headache, but this does not have preoperative implications.

A patient who takes a diuretic and a β-blocker to control blood pressure is scheduled for breast reconstruction surgery. Which patient information is most important to communicate to the health care provider before surgery? a. Hematocrit 36% b. Blood pressure 144/82 c. Pulse rate 58 beats/minute d. Serum potassium 3.2 mEq/L

ANS: D The low potassium level may increase the risk for intraoperative complications such as dysrhythmias. Slightly elevated blood pressure is common before surgery because of anxiety. The lower heart rate would be expected in a patient taking a β-blocker. The hematocrit is in the low normal range but does not require any intervention before surgery.

A patient with human immunodeficiency virus (HIV) infection has developed Mycobacterium avium complex infection. Which outcome would be appropriate for the nurse to include in the plan of care? a. The patient will be free from injury. b. The patient will receive immunizations. c. The patient will have adequate oxygenation. d. The patient will maintain intact perineal skin.

ANS: D The major manifestation of M. avium infection is loose, watery stools, which would increase the risk for perineal skin breakdown. The other outcomes would be appropriate for other complications (pneumonia, dementia, influenza, etc.) associated with HIV infection.

Which assessment should the nurse perform first for a patient who just vomited bright red blood? a. Measuring the quantity of emesis b. Palpating the abdomen for distention c. Auscultating the chest for breath sounds d. Taking the blood pressure (BP) and pulse

ANS: D The nurse is concerned about blood loss and possible hypovolemic shock in a patient with acute gastrointestinal (GI) bleeding. BP and pulse are the best indicators of these complications. The other information is important to obtain, but BP and pulse rate are the best indicators for assessing intravascular volume

A 75-year-old patient who lives alone at home tells the nurse, "I am afraid of losing my independence because my eyes don't work as well they used to." Which action should the nurse take first? a. Discuss the increased risk for falls that is associated with impaired vision. b. Explain that there are many ways to compensate for decreases in visual acuity. c. Suggest ways of improving the patient's safety, such as using brighter lighting. d. Ask the patient more about what type of vision problems are being experienced.

ANS: D The nurse's initial action should be further assessment of the patient's concerns and visual problems. The other actions may be appropriate, depending on what the nurse finds with further assessment

A 73-year-old patient is diagnosed with stomach cancer after an unintended 20-pound weight loss. Which nursing action will be included in the plan of care? a. Refer the patient for hospice services. b. Infuse IV fluids through a central line. c. Teach the patient about antiemetic therapy. d. Offer supplemental feedings between meals.

ANS: D The patient data indicate a poor nutritional state and improvement in nutrition will be helpful in improving the response to therapies such as surgery, chemotherapy, or radiation. Nausea and vomiting are not common clinical manifestations of stomach cancer. There is no indication that the patient requires hospice or IV fluid infusions

A patient with a possible pulmonary embolism complains of chest pain and difficulty breathing. The nurse finds a heart rate of 142 beats/minute, blood pressure of 100/60 mmHg, and respirations of 42 breaths/minute. Which action should the nurse take first? a. Administer anticoagulant drug therapy. b. Notify the patient's health care provider. c. Prepare patient for a spiral computed tomography (CT). d. Elevate the head of the bed to a semi-Fowler's position.

ANS: D The patient has symptoms consistent with a pulmonary embolism (PE). Elevating the head of the bed will improve ventilation and gas exchange. The other actions can be accomplished after the head is elevated (and oxygen is started). A spiral CT may be ordered by the health care provider to identify PE. Anticoagulants may be ordered after confirmation of the diagnosis of PE

An hour after a thoracotomy, a patient complains of incisional pain at a level 7 (based on 0 to 10 scale) and has decreased left-sided breath sounds. The pleural drainage system has 100 mL of bloody drainage and a large air leak. Which action is best for the nurse to take next? a. Milk the chest tube gently to remove any clots. b. Clamp the chest tube momentarily to check for the origin of the air leak. c. Assist the patient to deep breathe, cough, and use the incentive spirometer. d. Set up the patient controlled analgesia (PCA) and administer the loading dose of morphine.

ANS: D The patient is unlikely to take deep breaths or cough until the pain level is lower. A chest tube output of 100 mL is not unusual in the first hour after thoracotomy and would not require milking of the chest tube. An air leak is expected in the initial postoperative period after thoracotomy

After receiving change-of-shift report, which patient should the nurse assess first? a. Patient who is scheduled for the drain phase of a peritoneal dialysis exchange b. Patient with stage 4 chronic kidney disease who has an elevated phosphate level c. Patient with stage 5 chronic kidney disease who has a potassium level of 3.4 mEq/L d. Patient who has just returned from having hemodialysis and has a heart rate of 124/min

ANS: D The patient who is tachycardic after hemodialysis may be bleeding or excessively hypovolemic and should be assessed immediately for these complications. The other patients also need assessments or interventions but are not at risk for life-threatening complications

44. Which patient should the nurse assess first after receiving change-of-shift report? a. 60-year-old patient whose new ileostomy has drained 800 mL over the previous 8 hours b. 50-year-old patient with familial adenomatous polyposis who has occult blood in the stool c. 40-year-old patient with ulcerative colitis who has had six liquid stools in the previous 4 hours d. 30-year-old patient who has abdominal distention and an apical heart rate of 136 beats/minute

ANS: D The patient's abdominal distention and tachycardia suggest hypovolemic shock caused by problems such as peritonitis or intestinal obstruction, which will require rapid intervention. The other patients should also be assessed as quickly as possible, but the data do not indicate any life-threatening complications associated with their diagnoses.

When assessing a patient who has just arrived after an automobile accident, the emergency department nurse notes tachycardia and absent breath sounds over the right lung. For which intervention will the nurse prepare the patient? a. Emergency pericardiocentesis b. Stabilization of the chest wall with tape c. Administration of an inhaled bronchodilator d. Insertion of a chest tube with a chest drainage system

ANS: D The patient's history and absent breath sounds suggest a right-sided pneumothorax or hemothorax, which will require treatment with a chest tube and drainage. The other therapies would be appropriate for an acute asthma attack, flail chest, or cardiac tamponade, but the patient's clinical manifestations are not consistent with these problems

43. After several days of antibiotic therapy, an older hospitalized patient develops watery diarrhea. Which action should the nurse take first? a. Notify the health care provider. b. Obtain a stool specimen for analysis. c. Teach the patient about handwashing. d. Place the patient on contact precautions.

ANS: D The patient's history and new onset diarrhea suggest a C. difficile infection, which requires implementation of contact precautions to prevent spread of the infection to other patients. The other actions are also appropriate but can be accomplished after contact precautions are implemented.

After change-of-shift report, which patient should the nurse assess first? a. 72-year-old with cor pulmonale who has 4+ bilateral edema in his legs and feet b. 28-year-old with a history of a lung transplant and a temperature of 101° F (38.3° C) c. 40-year-old with a pleural effusion who is complaining of severe stabbing chest pain d. 64-year-old with lung cancer and tracheal deviation after subclavian catheter insertion

ANS: D The patient's history and symptoms suggest possible tension pneumothorax, a medical emergency. The other patients also require assessment as soon as possible, but tension pneumothorax will require immediate treatment to avoid death from inadequate cardiac output or hypoxemia

In reviewing a 55-year-old patient's medical record, the nurse notes that the last eye examination revealed an intraocular pressure of 28 mm Hg. The nurse will plan to assess a. visual acuity. b. pupil reaction. c. color perception. d. peripheral vision.

ANS: D The patient's increased intraocular pressure indicates glaucoma, which decreases peripheral vision. Because central visual acuity is unchanged by glaucoma, assessment of visual acuity could be normal even if the patient has worsening glaucoma. Color perception and pupil reaction to light are not affected by glaucoma

Which finding in an emergency department patient who reports being struck in the right eye with a fist is a priority for the nurse to communicate to the health care provider? a. The patient complains of a right-sided headache. b. The sclera on the right eye has broken blood vessels. c. The area around the right eye is bruised and tender to the touch. d. The patient complains of "a curtain" over part of the visual field.

ANS: D The patient's sensation that a curtain is coming across the field of vision suggests retinal detachment and the need for rapid action to prevent blindness. The other findings would be expected with the patient's history of being hit in the eye

21. A patient with severe chronic obstructive pulmonary disease (COPD) tells the nurse, "I wish I were dead! I'm just a burden on everybody." Based on this information, which nursing diagnosis is most appropriate? a. Complicated grieving related to expectation of death b. Ineffective coping related to unknown outcome of illness c. Deficient knowledge related to lack of education about COPD d. Chronic low self-esteem related to increased physical dependence

ANS: D The patient's statement about not being able to do anything for himself or herself supports this diagnosis. Although deficient knowledge, complicated grieving, and ineffective coping may also be appropriate diagnoses for patients with COPD, the data for this patient do not support these diagnoses. DIF: Cognitive Level: Apply (application) REF: 599-600 TOP: Nursing Process: Diagnosis MSC: NCLEX: Psychosocial Integrity

A 72-year-old patient with age-related macular degeneration (AMD) has just had photodynamic therapy. Which statement by the patient indicates that the discharge teaching has been effective? a. "I will need to use bright lights to read for at least the next week." b. "I will use drops to keep my pupils dilated until my appointment." c. "I will not use facial lotions near my eyes during the recovery period." d. "I will cover up with long-sleeved shirts and pants for the next 5 days."

ANS: D The photosensitizing drug used for photodynamic therapy is activated by exposure to bright light and can cause burns in areas exposed to light for 5 days after the treatment. There are no restrictions on the use of facial lotions, medications to keep the pupils dilated would not be appropriate, and bright lights would increase the risk for damage caused by the treatment

When caring for a patient who has received a general anesthetic, the circulating nurse notes red, raised wheals on the patient's arms. Which action should the nurse take immediately? a. Apply lotion to the affected areas. b. Cover the arms with sterile drapes. c. Recheck the patient's arms in 30 minutes. d. Notify the anesthesia care practitioner (ACP) immediately.

ANS: D The presence of wheals indicates a possible allergic or anaphylactic reaction, which may have been caused by latex or by medications administered as part of general anesthesia. Because general anesthesia may mask anaphylaxis, the nurse should report this to the ACP. The other actions are not appropriate at this time.

37. A 45-year-old patient is admitted to the emergency department with severe abdominal pain and rebound tenderness. Vital signs include temperature 102° F (38.3° C), pulse 120, respirations 32, and blood pressure (BP) 82/54. Which prescribed intervention should the nurse implement first? a. Administer IV ketorolac (Toradol) 15 mg. b. Draw blood for a complete blood count (CBC). c. Obtain a computed tomography (CT) scan of the abdomen. d. Infuse 1 liter of lactated Ringer's solution over 30 minutes.

ANS: D The priority for this patient is to treat the patient's hypovolemic shock with fluid infusion. The other actions should be implemented after starting the fluid infusion.

The nurse explaining esomeprazole (Nexium) to a patient with recurring heartburn describes that the medication a. reduces gastroesophageal reflux by increasing the rate of gastric emptying. b. neutralizes stomach acid and provides relief of symptoms in a few minutes. c. coats and protects the lining of the stomach and esophagus from gastric acid. d. treats gastroesophageal reflux disease by decreasing stomach acid production.

ANS: D The proton pump inhibitors decrease the rate of gastric acid secretion. Promotility drugs such as metoclopramide (Reglan) increase the rate of gastric emptying. Cryoprotective medications such as sucralfate (Carafate) protect the stomach. Antacids neutralize stomach acid and work rapidly

Which action best describes how the scrub nurse maintains aseptic technique during surgery? a. Uses waterproof shoe covers b. Wears personal protective equipment c. Insists that all operating room (OR) staff perform a surgical scrub d. Changes gloves after touching the upper arm of the surgeon's gown

ANS: D The sleeves of a sterile surgical gown are considered sterile only to 2 inches above the elbows, so touching the surgeon's upper arm would contaminate the nurse's gloves. Shoe covers are not sterile. Personal protective equipment is designed to protect caregivers, not the patient, and is not part of aseptic technique. Staff members such as the circulating nurse do not have to perform a surgical scrub before entering the OR

22. A 47-year-old female patient is transferred from the recovery room to a surgical unit after a transverse colostomy. The nurse observes the stoma to be deep pink with edema and a small amount of sanguineous drainage. The nurse should a. place ice packs around the stoma. b. notify the surgeon about the stoma. c. monitor the stoma every 30 minutes. d. document stoma assessment findings.

ANS: D The stoma appearance indicates good circulation to the stoma. There is no indication that surgical intervention is needed or that frequent stoma monitoring is required. Swelling of the stoma is normal for 2 to 3 weeks after surgery, and an ice pack is not needed.

17. A patient hospitalized with chronic obstructive pulmonary disease (COPD) is being discharged home on oxygen therapy. Which instruction should the nurse include in the discharge teaching? a. Storage of oxygen tanks will require adequate space in the home. b. Travel opportunities will be limited because of the use of oxygen. c. Oxygen flow should be increased if the patient has more dyspnea. d. Oxygen use can improve the patient's prognosis and quality of life.

ANS: D The use of home oxygen improves quality of life and prognosis. Because increased dyspnea may be a symptom of an acute process such as pneumonia, the patient should notify the physician rather than increasing the oxygen flow rate if dyspnea becomes worse. Oxygen can be supplied using liquid, storage tanks, or concentrators, depending on individual patient circumstances. Travel is possible using portable oxygen concentrators. DIF: Cognitive Level: Apply (application) REF: 592 TOP: Nursing Process: Implementation MSC: NCLEX: Physiological Integrity

After change-of-shift report, which patient should the nurse assess first? a. 42-year-old who has acute gastritis and ongoing epigastric pain b. 70-year-old with a hiatal hernia who experiences frequent heartburn c. 53-year-old who has dumping syndrome after a recent partial gastrectomy d. 60-year-old with nausea and vomiting who has dry oral mucosa and lethargy

ANS: D This older patient is at high risk for problems such as aspiration, dehydration, and fluid and electrolyte disturbances. The other patients will also need to be assessed, but the information about them indicates symptoms that are typical for their diagnoses and are not life threatening

Which teaching point should the nurse plan to include when caring for a patient whose vision is corrected to 20/200? a. How to access audio books b. How to use a white cane safely c. Where Braille instruction is available d. Where to obtain specialized magnifiers

ANS: D Various types of magnifiers can enhance the remaining vision enough to allow the performance of many tasks and activities of daily living (ADLs). Audio books, Braille instruction, and canes usually are reserved for patients with no functional vision

When assisting a blind patient in ambulating to the bathroom, the nurse should a. take the patient by the arm and lead the patient slowly to the bathroom. b. have the patient place a hand on the nurse's shoulder and guide the patient. c. stay beside the patient and describe any obstacles on the path to the bathroom. d. walk slightly ahead of the patient and allow the patient to hold the nurse's elbow.

ANS: D When using the sighted-guide technique, the nurse walks slightly in front and to the side of the patient and has the patient hold the nurse's elbow. The other techniques are not as safe in assisting a blind patient

1. When taking the health history of an older adult, the nurse discovers that the patient has worked in the landscaping business for 40 years. The nurse will plan to teach the patient about how to self-assess for which clinical manifestations (select all that apply)? a. Vitiligo b. Alopecia c. Intertrigo d. Erythema e. Actinic keratosis

ANS: D, E A patient who has worked as a landscaper is at risk for skin lesions caused by sun exposure such as erythema and actinic keratosis. Vitiligo, alopecia, and intertrigo are not associated with excessive sun exposure. DIF: Cognitive Level: Analyze (analysis) REF: 397 TOP: Nursing Process: Planning

A patient has a serum calcium level of 7.0 mEq/L. Which assessment finding is most important for the nurse to report to the health care provider? a. The patient is experiencing laryngeal stridor. b. The patient complains of generalized fatigue. c. The patient's bowels have not moved for 4 days. d. The patient has numbness and tingling of the lips.

Answer A Hypocalcemia can cause laryngeal stridor, which may lead to respiratory arrest. Rapid action is required to correct the patient's calcium level. The other data are also consistent with hypocalcemia, but do not indicate a need for as immediate action as laryngospasm.

The patient receiving chemotherapy rings the call bell and reports an onset of nausea. The nurse should prepare a prn dose of which of the following medications? A) Morphine sulfate B) Zolpidem (Ambien) C) Ondansetron (Zofran) D) Dexamethasone (Decadron)

C) Ondansetron (Zofran) (Ondansetron is a 5-HT3 receptor antagonist antiemetic that is especially effective in reducing cancer chemotherapy-induced nausea and vomiting.)

A 24-year-old woman with Crohn's disease develops a fever and symptoms of a urinary tract infection (UTI) with tan, fecal-smelling urine. What information will the nurse add to a general teaching plan about UTIs in order to individualize the teaching for this patient? a. Bacteria in the perianal area can enter the urethra. b. Fistulas can form between the bowel and bladder. c. Drink adequate fluids to maintain normal hydration. d. Empty the bladder before and after sexual intercourse.

B

A 25-year-old male patient calls the clinic complaining of diarrhea for 24 hours. Which action should the nurse take first? a. Inform the patient that laboratory testing of blood and stools will be necessary. b. Ask the patient to describe the character of the stools and any associated symptoms. c. Suggest that the patient drink clear liquid fluids with electrolytes, such as Gatorade or Pedialyte. d. Advise the patient to use over-the-counter loperamide (Imodium) to slow gastrointestinal (GI) motility.

B

A patient receives 3% NaCl solution for correction of hyponatremia. Which assessment is most important for the nurse to monitor for while the patient is receiving this infusion? a. Lung sounds b. Urinary output c. Peripheral pulses d. Peripheral edema

Answer A Hypertonic solutions cause water retention, so the patient should be monitored for symptoms of fluid excess. Crackles in the lungs may indicate the onset of pulmonary edema and are a serious manifestation of fluid excess. Bounding peripheral pulses, peripheral edema, or changes in urine output are also important to monitor when administering hypertonic solutions, but they do not indicate acute respiratory or cardiac decompensation.

A newly admitted patient is diagnosed with hyponatremia. When making room assignments, the charge nurse should take which action? a. Assign the patient to a room near the nurse's station. b. Place the patient in a room nearest to the water fountain. c. Place the patient on telemetry to monitor for peaked T waves. d. Assign the patient to a semi-private room and place an order for a low-salt diet.

Answer A

A postoperative patient who had surgery for a perforated gastric ulcer has been receiving nasogastric suction for 3 days. The patient now has a serum sodium level of 127 mEq/L (127 mmol/L). Which prescribed therapy should the nurse question? a. Infuse 5% dextrose in water at 125 mL/hr. b. Administer IV morphine sulfate 4 mg every 2 hours PRN. c. Give IV metoclopramide (Reglan) 10 mg every 6 hours PRN for nausea. d. Administer 3% saline if serum sodium decreases to less than 128 mEq/L.

Answer A

The home health nurse cares for an alert and oriented older adult patient with a history of dehydration. Which instructions should the nurse give to this patient related to fluid intake? a. "Increase fluids if your mouth feels dry. b. "More fluids are needed if you feel thirsty." c. "Drink more fluids in the late evening hours." d. "If you feel lethargic or confused, you need more to drink."

Answer A

The nurse is caring for a patient with a massive burn injury and possible hypovolemia. Which assessment data will be of most concern to the nurse? a. Blood pressure is 90/40 mm Hg. b. Urine output is 30 mL over the last hour. c. Oral fluid intake is 100 mL for the last 8 hours. d. There is prolonged skin tenting over the sternum.

Answer A

A patient who has been receiving diuretic therapy is admitted to the emergency department with a serum potassium level of 3.0 mEq/L. The nurse should alert the health care provider immediately that the patient is on which medication? a. Oral digoxin (Lanoxin) 0.25 mg daily b. Ibuprofen (Motrin) 400 mg every 6 hours c. Metoprolol (Lopressor) 12.5 mg orally daily d. Lantus insulin 24 U subcutaneously every evening

Answer A Hypokalemia increases the risk for digoxin toxicity, which can cause serious dysrhythmias. The nurse will also need to do more assessment regarding the other medications, but they are not of as much concern with the potassium level.

A nurse in the outpatient clinic is caring for a patient who has a magnesium level of 1.3 mg/dL. Which assessment would be most important for the nurse to make? a. Daily alcohol intake b. Intake of dietary protein c. Multivitamin/mineral use d. Use of over-the-counter (OTC) laxatives

Answer A Hypomagnesemia is associated with alcoholism. Protein intake would not have a significant effect on magnesium level. OTC laxatives (such as milk of magnesia) and use of multivitamin/mineral supplements would tend to increase magnesium levels

A patient with renal failure has been taking aluminum hydroxide/magnesium hydroxide suspension (Maalox) at home for indigestion. The patient arrives for outpatient hemodialysis and is unresponsive to questions and has decreased deep tendon reflexes. Which action should the dialysis nurse take first? a. Notify the patient's health care provider. b. Obtain an order to draw a potassium level. c. Review the magnesium level on the patient's chart. d. Teach the patient about the risk of magnesium-containing antacids

Answer A The health care provider should be notified immediately. The patient has a history and manifestations consistent with hypermagnesemia. The nurse should check the chart for a recent serum magnesium level and make sure that blood is sent to the laboratory for immediate electrolyte and chemistry determinations. Dialysis should correct the high magnesium levels. The patient needs teaching about the risks of taking magnesium-containing antacids. Monitoring of potassium levels also is important for patients with renal failure, but the patient's current symptoms are not consistent with hyperkalemia

A patient who is lethargic and exhibits deep, rapid respirations has the following arterial blood gas (ABG) results: pH 7.32, PaO2 88 mm Hg, PaCO2 37 mm Hg, and HCO3 16 mEq/L. How should the nurse interpret these results? a. Metabolic acidosis b. Metabolic alkalosis c. Respiratory acidosis d. Respiratory alkalosis

Answer A The pH and HCO3 indicate that the patient has a metabolic acidosis. The ABGs are inconsistent with the other responses.

Following a thyroidectomy, a patient complains of "a tingling feeling around my mouth." Which assessment should the nurse complete immediately? a. Presence of the Chvostek's sign b. Abnormal serum potassium level c. Decreased thyroid hormone level d. Bleeding on the patient's dressing

Answer A The patient's symptoms indicate possible hypocalcemia, which can occur secondary to parathyroid injury/removal during thyroidectomy. There is no indication of a need to check the potassium level, the thyroid hormone level, or for bleeding

The nurse notes a serum calcium level of 7.9 mg/dL for a patient who has chronic malnutrition. Which action should the nurse take next? a. Monitor ionized calcium level. b. Give oral calcium citrate tablets. c. Check parathyroid hormone level. d. Administer vitamin D supplements.

Answer A This patient with chronic malnutrition is likely to have a low serum albumin level, which will affect the total serum calcium. A more accurate reflection of calcium balance is the ionized calcium level. Most of the calcium in the blood is bound to protein (primarily albumin). Alterations in serum albumin levels affect the interpretation of total calcium levels. Low albumin levels result in a drop in the total calcium level, although the level of ionized calcium is not affected. The other actions may be needed if the ionized calcium is also decreased.

An older adult patient who is malnourished presents to the emergency department with a serum protein level of 5.2 g/dL. The nurse would expect which clinical manifestation? a. Pallor b. Edema c. Confusion d. Restlessness

Answer B

IV potassium chloride (KCl) 60 mEq is prescribed for treatment of a patient with severe hypokalemia. Which action should the nurse take? a. Administer the KCl as a rapid IV bolus. b. Infuse the KCl at a rate of 10 mEq/hour. c. Only give the KCl through a central venous line. d. Discontinue cardiac monitoring during the infusion.

Answer B

Which action can the registered nurse (RN) who is caring for a critically ill patient with multiple IV lines delegate to an experienced licensed practical/vocational nurse (LPN/LVN)? a. Administer IV antibiotics through the implantable port. b. Monitor the IV sites for redness, swelling, or tenderness. c. Remove the patient's nontunneled subclavian central venous catheter. d. Adjust the flow rate of the 0.9% normal saline in the peripheral IV line.

Answer B An experienced LPN/LVN has the education, experience, and scope of practice to monitor IV sites for signs of infection. Administration of medications, adjustment of infusion rates, and removal of central catheters in critically ill patients require RN level education and scope of practice.

A patient comes to the clinic complaining of frequent, watery stools for the last 2 days. Which action should the nurse take first? a. Obtain the baseline weight. b. Check the patient's blood pressure. c. Draw blood for serum electrolyte levels. d. Ask about any extremity numbness or tingling.

Answer B Because the patient's history suggests that fluid volume deficit may be a problem, assessment for adequate circulation is the highest priority. The other actions are also appropriate, but are not as essential as determining the patient's perfusion status

When assessing a pregnant patient with eclampsia who is receiving IV magnesium sulfate, which finding should the nurse report to the health care provider immediately? a. The bibasilar breath sounds are decreased. b. The patellar and triceps reflexes are absent. c. The patient has been sleeping most of the day. d. The patient reports feeling "sick to my stomach."

Answer B The loss of the deep tendon reflexes indicates that the patient's magnesium level may be reaching toxic levels. Nausea and lethargy also are side effects associated with magnesium elevation and should be reported, but they are not as significant as the loss of deep tendon reflexes. The decreased breath sounds suggest that the patient needs to cough and deep breathe to prevent atelectasis.

A patient who had a transverse colectomy for diverticulosis 18 hours ago has nasogastric suction and is complaining of anxiety and incisional pain. The patient's respiratory rate is 32 breaths/minute and the arterial blood gases (ABGs) indicate respiratory alkalosis. Which action should the nurse take first? a. Discontinue the nasogastric suction. b. Give the patient the PRN IV morphine sulfate 4 mg. c. Notify the health care provider about the ABG results. d. Teach the patient how to take slow, deep breaths when anxious.

Answer B The patient's respiratory alkalosis is caused by the increased respiratory rate associated with pain and anxiety. The nurse's first action should be to medicate the patient for pain. Although the nasogastric suction may contribute to the alkalosis, it is not appropriate to discontinue the tube when the patient needs gastric suction. The health care provider may be notified about the ABGs but is likely to instruct the nurse to medicate for pain. The patient will not be able to take slow, deep breaths when experiencing pain

The nurse is caring for a patient who has a central venous access device (CVAD). Which action by the nurse is appropriate? a. Avoid using friction when cleaning around the CVAD insertion site. b. Use the push-pause method to flush the CVAD after giving medications. c. Obtain an order from the health care provider to change CVAD dressing. d. Position the patient's face toward the CVAD during injection cap changes.

Answer B The push-pause enhances the removal of debris from the CVAD lumen and decreases the risk for clotting. To decrease infection risk, friction should be used when cleaning the CVAD insertion site. The dressing should be changed whenever it becomes damp, loose, or visibly soiled. The patient should turn away from the CVAD during cap changes.

A patient is admitted to the emergency department with severe fatigue and confusion. Laboratory studies are done. Which laboratory value will require the most immediate action by the nurse? a. Arterial blood pH is 7.32. b. Serum calcium is 18 mg/dL. c. Serum potassium is 5.1 mEq/L. d. Arterial oxygen saturation is 91%.

Answer B The serum calcium is well above the normal level and puts the patient at risk for cardiac dysrhythmias. The nurse should initiate cardiac monitoring and notify the health care provider. The potassium, oxygen saturation, and pH are also abnormal, and the nurse should notify the health care provider about these values as well, but they are not immediately life threatening.

A patient who has a small cell carcinoma of the lung develops syndrome of inappropriate antidiuretic hormone (SIADH). The nurse should notify the health care provider about which assessment finding? a. Reported weight gain b. Serum hematocrit of 42% c. Serum sodium level of 120 mg/dL d. Total urinary output of 280 mL during past 8 hours

Answer C

The nurse notes that a patient who was admitted with diabetic ketoacidosis has rapid, deep respirations. Which action should the nurse take? a. Give the prescribed PRN lorazepam (Ativan). b. Start the prescribed PRN oxygen at 2 to 4 L/min. c. Administer the prescribed normal saline bolus and insulin. d. Encourage the patient to take deep, slow breaths with guided imagery.

Answer C

The long-term care nurse is evaluating the effectiveness of protein supplements for an older resident who has a low serum total protein level. Which assessment finding indicates that the patient's condition has improved? a. Hematocrit 28% b. Absence of skin tenting c. Decreased peripheral edema d. Blood pressure 110/72 mm Hg

Answer C Edema is caused by low oncotic pressure in individuals with low serum protein levels. The decrease in edema indicates an improvement in the patient's protein status. Good skin turgor is an indicator of fluid balance, not protein status. A low hematocrit could be caused by poor protein intake. Blood pressure does not provide a useful clinical tool for monitoring protein status.

A nurse is assessing a newly admitted patient with chronic heart failure who forgot to take prescribed medications and seems confused. The patient complains of "just blowing up" and has peripheral edema and shortness of breath. Which assessment should the nurse complete first? a. Skin turgor b. Heart sounds c. Mental status d. Capillary refill

Answer C Increases in extracellular fluid (ECF) can lead to swelling of cells in the central nervous system, initially causing confusion, which may progress to coma or seizures. Although skin turgor, capillary refill, and heart sounds also may be affected by increases in ECF, these are signs that do not have as immediate impact on patient outcomes as cerebral edema.

A patient has a parenteral nutrition infusion of 25% dextrose. A student nurse asks the nurse why a peripherally inserted central catheter was inserted. Which response by the nurse is most appropriate? a. "There is a decreased risk for infection when 25% dextrose is infused through a central line." b. "The prescribed infusion can be given much more rapidly when the patient has a central line." c. "The 25% dextrose is hypertonic and will be more rapidly diluted when given through a central line." d. "The required blood glucose monitoring is more accurate when samples are obtained from a central line."

Answer C The 25% dextrose solution is hypertonic. Shrinkage of red blood cells can occur when solutions with dextrose concentrations greater than 10% are administered IV. Blood glucose testing is not more accurate when samples are obtained from a central line. The infection risk is higher with a central catheter than with peripheral IV lines. Hypertonic or concentrated IV solutions are not given rapidly.

An older patient receiving iso-osmolar continuous tube feedings develops restlessness, agitation, and weakness. Which laboratory result should the nurse report to the health care provider immediately? a. K+ 3.4 mEq/L (3.4 mmol/L) b. Ca+2 7.8 mg/dL (1.95 mmol/L) c. Na+ 154 mEq/L (154 mmol/L) d. PO4-3 4.8 mg/dL (1.55 mmol/L)

Answer C The elevated serum sodium level is consistent with the patient's neurologic symptoms and indicates a need for immediate action to prevent further serious complications such as seizures. The potassium and calcium levels vary slightly from normal but do not require immediate action by the nurse. The phosphate level is normal

Which action should the nurse take first when a patient complains of acute chest pain and dyspnea soon after insertion of a centrally inserted IV catheter? a. Notify the health care provider. b. Offer reassurance to the patient. c. Auscultate the patient's breath sounds. d. Give the prescribed PRN morphine sulfate IV.

Answer C The initial action should be to assess the patient further because the history and symptoms are consistent with several possible complications of central line insertion, including embolism and pneumothorax. The other actions may be appropriate, but further assessment of the patient is needed before notifying the health care provider, offering reassurance, or administration of morphine.

After receiving change-of-shift report, which patient should the nurse assess first? a. Patient with serum potassium level of 5.0 mEq/L who is complaining of abdominal cramping b. Patient with serum sodium level of 145 mEq/L who has a dry mouth and is asking for a glass of water c. Patient with serum magnesium level of 1.1 mEq/L who has tremors and hyperactive deep tendon reflexes d. Patient with serum phosphorus level of 4.5 mg/dL who has multiple soft tissue calcium-phosphate precipitates

Answer C The low magnesium level and neuromuscular irritability suggest that the patient may be at risk for seizures. The other patients have mild electrolyte disturbances and/or symptoms that require action, but they are not at risk for life-threatening complications

The nurse assesses a patient who has been hospitalized for 2 days. The patient has been receiving normal saline IV at 100 mL/hr, has a nasogastric tube to low suction, and is NPO. Which assessment finding would be a priority for the nurse to report to the health care provider? a. Oral temperature of 100.1° F b. Serum sodium level of 138 mEq/L (138 mmol/L) c. Gradually decreasing level of consciousness (LOC) d. Weight gain of 2 pounds (1 kg) above the admission weight

Answer C The patient's history and change in LOC could be indicative of fluid and electrolyte disturbances: extracellular fluid (ECF) excess, ECF deficit, hyponatremia, hypernatremia, hypokalemia, or metabolic alkalosis. Further diagnostic information is needed to determine the cause of the change in LOC and the appropriate interventions. The weight gain, elevated temperature, crackles, and serum sodium level also will be reported, but do not indicate a need for rapid action to avoid complications.

A patient who is taking a potassium-wasting diuretic for treatment of hypertension complains of generalized weakness. It is most appropriate for the nurse to take which action? a. Assess for facial muscle spasms. b. Ask the patient about loose stools. c. Suggest that the patient avoid orange juice with meals. d. Ask the health care provider to order a basic metabolic panel.

Answer D

A patient who was involved in a motor vehicle crash has had a tracheostomy placed to allow for continued mechanical ventilation. How should the nurse interpret the following arterial blood gas results: pH 7.48, PaO2 85 mm Hg, PaCO2 32 mm Hg, and HCO3 25 mEq/L? a. Metabolic acidosis b. Metabolic alkalosis c. Respiratory acidosis d. Respiratory alkalosis

Answer D

Spironolactone (Aldactone), an aldosterone antagonist, is prescribed for a patient. Which statement by the patient indicates that the teaching about this medication has been effective? a. "I will try to drink at least 8 glasses of water every day." b. "I will use a salt substitute to decrease my sodium intake." c. "I will increase my intake of potassium-containing foods." d. "I will drink apple juice instead of orange juice for breakfast."

Answer D

A patient is receiving a 3% saline continuous IV infusion for hyponatremia. Which assessment data will require the most rapid response by the nurse? a. The patient's radial pulse is 105 beats/minute. b. There is sediment and blood in the patient's urine. c. The blood pressure increases from 120/80 to 142/94. d. There are crackles audible throughout both lung fields.

Answer D Crackles throughout both lungs suggest that the patient may be experiencing pulmonary edema, a life-threatening adverse effect of hypertonic solutions. The increased pulse rate and blood pressure and the appearance of the urine also should be reported, but they are not as dangerous as the presence of fluid in the alveoli

The nurse is caring for a patient who has a calcium level of 12.1 mg/dL. Which nursing action should the nurse include on the care plan? a. Maintain the patient on bed rest. b. Auscultate lung sounds every 4 hours. c. Monitor for Trousseau's and Chvostek's signs. d. Encourage fluid intake up to 4000 mL every day.

Answer D To decrease the risk for renal calculi, the patient should have a fluid intake of 3000 to 4000 mL daily. Ambulation helps decrease the loss of calcium from bone and is encouraged in patients with hypercalcemia. Trousseau's and Chvostek's signs are monitored when there is a possibility of hypocalcemia. There is no indication that the patient needs frequent assessment of lung sounds, although these would be assessed every shift

25. A 58-year-old man with erectile dysfunction (ED) tells the nurse he is interested in using sildenafil (Viagra). Which action should the nurse take first?

Ask the patient about any prescription drugs he is taking.

The nurse notes darker skin pigmentation in the skinfolds of a middle-aged patient who has a body mass index of 40 kg/m2. What is the nurses best action?

Ask the patient about type 2 diabetes or if there is a family history of it. The presence of acanthosis nigricans in skinfolds suggests either having type 2 diabetes or being at an increased risk for it. The description of the patients skin does not indicate problems with fungal infection, poor hygiene, or the need to dry the skinfolds better.

A patient who has severe refractory psoriasis on the face, neck, and extremities is socially withdrawn because of the appearance of the lesions. Which action should the nurse take first?

Ask the patient to describe the impact of psoriasis on quality of life. The nurses initial actions should be to assess the impact of the disease on the patients life and to allow the patient to verbalize feelings about the psoriasis. Depending on the assessment findings, other actions may be appropriate.

42. Which information about a patient with Goodpasture syndrome requires the most rapid action by the nurse?

Audible crackles bilaterally over the posterior chest to the midscapular level.

26. Which nursing action will be most helpful in decreasing the risk for hospital-acquired infection (HAI) of the urinary tract in patients admitted to the hospital?

Avoid unnecessary catheterizations.

A patient who has bacterial endophthalmitis in the left eye is restless, frequently asking whether the eye is healing, and whether removal of the eye will be necessary. Based on the assessment data, which nursing diagnosis is most appropriate at this time? a. Grieving related to current loss of functional vision b. Anxiety related to the possibility of permanent vision loss c. Situational low self-esteem related to loss of visual function d. Risk for falls related to inability to see environmental hazards

B

A patient with a head injury after a motorcycle crash arrives in the emergency department (ED) complaining of shortness of breath and severe eye pain. Which action will the nurse take first? a. Administer the ordered analgesic. b. Check the patient's oxygen saturation. c. Examine the eye for evidence of trauma. d. Assess each of the cranial nerve functions.

B

After a total proctocolectomy and permanent ileostomy, the patient tells the nurse, "I cannot manage all these changes. I don't want to look at the stoma." What is the best action by the nurse? a. Reassure the patient that ileostomy care will become easier. b. Ask the patient about the concerns with stoma management. c. Develop a detailed written list of ostomy care tasks for the patient. d. Postpone any teaching until the patient adjusts to the ileostomy.

B

After change-of-shift report, which patient should the nurse assess first? a. 40-year-old male with celiac disease who has frequent frothy diarrhea b. 30-year-old female with a femoral hernia who has abdominal pain and vomiting c. 30-year-old male with ulcerative colitis who has severe perianal skin breakdown d. 40-year-old female with a colostomy bag that is pulling away from the adhesive wafer

B

An 82-year-old patient who is being admitted to the hospital repeatedly asks the nurse to "speak up so that I can hear you." Which action should the nurse take? a. Overenunciate while speaking. b. Speak normally but more slowly. c. Increase the volume when speaking. d. Use more facial expressions when talking.

B

The charge nurse observes a newly hired nurse performing all the following interventions for a patient who has just undergone right cataract removal and an intraocular lens implant. Which one requires that the charge nurse intervene? a. The nurse leaves the eye shield in place. b. The nurse encourages the patient to cough. c. The nurse elevates the patient's head to 45 degrees. d. The nurse applies corticosteroid drops to the right eye.

B

The nurse at the eye clinic made a follow-up telephone call to a patient who underwent cataract extraction and intraocular lens implantation the previous day. Which information is the priority to communicate to the health care provider? a. The patient has questions about the ordered eye drops. b. The patient has eye pain rated at a 5 (on a 0 to 10 scale). c. The patient has poor depth perception when wearing an eye patch. d. The patient complains that the vision has not improved very much.

B

The nurse developing a teaching plan for a patient with herpes simplex keratitis should include which instruction? a. Apply antibiotic drops to the eye several times daily. b. Wash hands frequently and avoid touching the eyes. c. Apply a new occlusive dressing to the affected eye at bedtime. d. Use corticosteroid ophthalmic ointment to decrease inflammation.

B

The nurse evaluates that wearing bifocals improved the patient's myopia and presbyopia by assessing for a. strength of the eye muscles. b. both near and distant vision. c. cloudiness in the eye lenses. d. intraocular pressure changes.

B

The nurse is admitting a 67-year-old patient with new-onset steatorrhea. Which question is most important for the nurse to ask? a. "How much milk do you usually drink?" b. "Have you noticed a recent weight loss?" c. "What time of day do your bowels move?" d. "Do you eat meat or other animal products?"

B

The nurse is assessing a 31-year-old female patient with abdominal pain. Th nurse,who notes that there is ecchymosis around the area of umbilicus, will document this finding as a. Cullen sign. b. Rovsing sign. c. McBurney sign. d. Grey-Turner's signt.

B

Unlicensed assistive personnel (UAP) perform all the following actions when caring for a patient with Ménière's disease who is experiencing an acute attack. Which action by UAP indicates that the nurse should intervene immediately? a. UAP raise the side rails on the bed. b. UAP turn on the patient's television. c. UAP turn the patient to the right side. d. UAP place an emesis basin at the bedside.

B

When teaching a patient about the treatment of acoustic neuroma, the nurse will include information about a. a low sodium diet. b. ways to avoid falls. c. how to apply sunscreen. d. the chemotherapy side effects.

B

Which action could the registered nurse (RN) who is working in the eye and ear clinic delegate to a licensed practical/vocational nurse (LPN/LVN)? a. Evaluate a patient's ability to administer eye drops. b. Use a Snellen chart to check a patient's visual acuity. c. Teach a patient with otosclerosis about use of sodium fluoride and vitamin D. d. Check the patient's external ear for signs of irritation caused by a hearing aid.

B

Which action will the nurse take when performing ear irrigation for a patient with cerumen impaction? a. Assist the patient to a supine position for the irrigation. b. Fill the irrigation syringe with body-temperature solution. c. Use a sterile applicator to clean the ear canal before irrigating. d. Occlude the ear canal completely with the syringe while irrigating.

B

Which information about a patient who had a stapedotomy yesterday is most important for the nurse to communicate to the health care provider? a. The patient complains of "fullness" in the ear. b. The patient's oral temperature is 100.8° F (38.1° C). c. The patient says "My hearing is worse now than it was right after surgery." d. There is a small amount of dried bloody drainage on the patient's dressing.

B

Which information will the nurse include in teaching a patient who had a proctocolectomy and ileostomy for ulcerative colitis? a. Restrict fluid intake to prevent constant liquid drainage from the stoma. b. Use care when eating high-fiber foods to avoid obstruction of the ileum. c. Irrigate the ileostomy daily to avoid having to wear a drainage appliance. d. Change the pouch every day to prevent leakage of contents onto the skin.

B

17. Which nursing action should be included in the plan of care after endovascular repair of an abdominal aortic aneurysm? a. Record hourly chest tube drainage. b. Monitor fluid intake and urine output. c. Assess the abdominal incision for redness. d. Teach the patient to plan for a long recovery period.

B Because renal artery occlusion can occur after endovascular repair, the nurse should monitor parameters of renal function such as intake and output. Chest tubes will not be needed for endovascular surgery, the recovery period will be short, and there will not be an abdominal wound. DIF: Cognitive Level: Apply (application) REF: 815 TOP: Nursing Process: Planning MSC: NCLEX: Physiological

35. The nurse is admitting a patient newly diagnosed with peripheral artery disease. Which admission order should the nurse question? a. Cilostazol drug therapy b. Omeprazole drug therapy c. Use of treadmill for exercise d. Exercise to the point of discomfort

B Because the antiplatelet effect of clopidogrel is reduced when it is used with omeprazole, the nurse should clarify this order with the health care provider. The other interventions are appropriate for a patient with peripheral artery disease. DIF: Cognitive Level: Apply (application) REF: 805 TOP: Nursing Process: Assessment MSC: NCLEX: Physiological

20. While working in the outpatient clinic, the nurse notes that a patient has a history of intermittent claudication. Which statement by the patient would support this information? a. "When I stand too long, my feet start to swell." b. "My legs cramp when I walk more than a block." c. "I get short of breath when I climb a lot of stairs." d. "My fingers hurt when I go outside in cold weather."

B Cramping that is precipitated by a consistent level of exercise is descriptive of intermittent claudication. Finger pain associated with cold weather is typical of Raynaud's phenomenon. Shortness of breath that occurs with exercise is not typical of intermittent claudication, which is reproducible. Swelling associated with prolonged standing is typical of venous disease. DIF: Cognitive Level: Apply (application) REF: 803 TOP: Nursing Process: Assessment MSC: NCLEX: Physiological

2. A patient has a 6-cm thoracic aortic aneurysm that was discovered during routine chest x-ray. When obtaining an admission history from the patient, it will be most important for the nurse to ask about a. low back pain. c. abdominal tenderness. b. trouble swallowing. d. changes in bowel habits.

B Difficulty swallowing may occur with a thoracic aneurysm because of pressure on the esophagus. The other symptoms will be important to assess for in patients with abdominal aortic aneurysms. DIF: Cognitive Level: Analyze (analysis) REF: 810 TOP: Nursing Process: Assessment MSC: NCLEX: Physiological

11. The health care provider prescribes an infusion of heparin and daily partial thromboplastin time (PTT) testing for a patient with venous thromboembolism (VTE). The nurse will plan to a. decrease the infusion when the PTT value is 65 seconds. b. avoid giving IM medications to prevent localized bleeding. c. have vitamin K available in case reversal of the heparin is needed. d. monitor posterior tibial and dorsalis pedis pulses with the Doppler.

B Intramuscular injections are avoided in patients receiving anticoagulation to prevent hematoma formation and bleeding from the site. A PTT of 65 seconds is within the therapeutic range. Vitamin K is used to reverse warfarin. Pulse quality is not affected by VTE. DIF: Cognitive Level: Apply (application) REF: 823 TOP: Nursing Process: Planning MSC: NCLEX: Physiological

34. The nurse who works in the vascular clinic has several patients with venous insufficiency scheduled today. Which patient should the nurse assign to an experienced licensed practical/vocational nurse (LPN/LVN)? a. Patient who has been complaining of increased edema and skin changes in the legs b. Patient who needs wound care for a chronic venous stasis ulcer on the right lower leg c. Patient who has a history of venous thromboembolism and is complaining of dyspnea d. Patient who needs teaching about elastic compression stockings for venous insufficiency

B LPN education and scope of practice includes wound care. The other patients, which require more complex assessments or education, should be managed by the RN. DIF: Cognitive Level: Apply (application) REF: 827 OBJ: Special Questions: Delegation | Special Questions: Multiple Patients TOP: Nursing Process: Planning MSC: NCLEX: Safe and Effective Care Environment

12. A patient with a venous thromboembolism (VTE) is started on enoxaparin (Lovenox) and warfarin (Coumadin). The patient asks the nurse why two medications are necessary. Which response by the nurse is most accurate? a. "Taking two blood thinners greatly reduces the risk for another clot to form." b. "Enoxaparin will work right away, but warfarin takes several days to begin preventing clots." c. "Enoxaparin will start to dissolve the clot, and warfarin will prevent any more clots from forming." d. "Because of the risk for a blood clot in the lungs, it is important for you to take more than one blood thinner."

B Low molecular weight heparin (LMWH) is used because of the immediate effect on coagulation and discontinued once the international normalized ratio (INR) value indicates that the warfarin has reached a therapeutic level. LMWH has no thrombolytic properties. The use of two anticoagulants is not related to the risk for pulmonary embolism, and two are not necessary to reduce the risk for another VTE. Anticoagulants do not thin the blood. DIF: Cognitive Level: Apply (application) REF: 820 TOP: Nursing Process: Implementation MSC: NCLEX: Physiological

13. The nurse has started discharge teaching for a patient who is to continue warfarin (Coumadin) after hospitalization for venous thromboembolism (VTE). The nurse determines that additional teaching is needed when the patient says which of the following? a. "I should get a Medic Alert device stating that I take warfarin." b. "I should reduce the amount of green, leafy vegetables that I eat." c. "I will need routine blood tests to monitor the effects of the warfarin." d. "I will check with my health care provider before I begin any new drugs."

B Patients taking warfarin are taught to follow a consistent diet with regard to foods that are high in vitamin K, such as green, leafy vegetables. The other patient statements are accurate. DIF: Cognitive Level: Apply (application) REF: 820 TOP: Nursing Process: Evaluation MSC: NCLEX: Physiological

A patient with left knee pain is diagnosed with bursitis. The nurse will explain that bursitis is an inflammation of a. the synovial membrane that lines the joint. b. a small, fluid-filled sac found at some joints. c. the fibrocartilage that acts as a shock absorber in the knee joint. d. any connective tissue that is found supporting the joints of the body.

B. Bursae are fluid-filled sacs that cushion joints and bony prominences. Fibrocartilage is a solid tissue that cushions some joints. Bursae are a specific type of connective tissue. The synovial membrane lines many joints but is not a bursa.

21. Which instructions should the nurse include in a teaching plan for an older patient newly diagnosed with peripheral artery disease (PAD)? a. "Exercise only if you do not experience any pain." b. "It is very important that you stop smoking cigarettes." c. "Try to keep your legs elevated whenever you are sitting." d. "Put elastic compression stockings on early in the morning."

B Smoking cessation is essential for slowing the progression of PAD to critical limb ischemia and reducing the risk of myocardial infarction and death. Circulation to the legs will decrease if the legs are elevated. Patients with PAD are taught to exercise to the point of feeling pain, rest, and then resume walking. Support hose are not used for patients with PAD. DIF: Cognitive Level: Apply (application) REF: 817 TOP: Nursing Process: Planning MSC: NCLEX: Physiological

23. After receiving change of shift report, which patient admitted to the emergency department should the nurse assess first? a. A 67-yr-old patient who has a gangrenous left foot ulcer with a weak pedal pulse b. A 50-yr-old patient who is complaining of sudden sharp and severe upper back pain c. A 39-yr-old patient who has right calf tenderness, redness, and swelling after a plane ride d. A 58-yr-old patient who is taking anticoagulants for atrial fibrillation and has black stools

B The patient's presentation of sudden sharp and severe upper back pain is consistent with dissecting thoracic aneurysm, which will require the most rapid intervention. The other patients also require rapid intervention but not before the patient with severe pain. DIF: Cognitive Level: Analyze (analysis) REF: 810 OBJ: Special Questions: Prioritization | Special Questions: Multiple Patients TOP: Nursing Process: Assessment MSC: NCLEX: Physiological

33. Which actions could the nurse delegate to unlicensed assistive personnel (UAP) who are providing care for a patient who is at risk for venous thromboembolism? a. Monitor for any bleeding after anticoagulation therapy is started. b. Apply sequential compression device whenever the patient is in bed. c. Ask the patient about use of herbal medicines or dietary supplements. d. Instruct the patient to call immediately if any shortness of breath occurs.

B UAP training includes the use of equipment that requires minimal nursing judgment, such as sequential compression devices. Patient assessment and teaching require more education and critical thinking and should be done by the registered nurse (RN). DIF: Cognitive Level: Apply (application) REF: 824 OBJ: Special Questions: Delegation TOP: Nursing Process: Planning MSC: NCLEX: Safe and Effective Care Environment

The results of a patients recent endoscopy indicate the presence of peptic ulcer disease (PUD). Which of the following teaching points should the nurse provide to the pt in light of his new diagnosis? A) "You'll need to drink at least two to three glasses of milk daily." B) "It would likely be beneficial for you to eliminate drinking alcohol." C) "Many people find that a minced or pureed diet eases their sxs of PUD." D) "Your medications should allow you to maintain your present diet while minimizing symptoms."

B) "It would likely be beneficial for you to eliminate drinking alcohol." Although there is no specific recommended dietary modification for PUD, most patients find it necessary to make some sort of dietary modifications to minimize symptoms. Milk may exacerbate PUD and alcohol is best avoided because it can delay healing.

A patient is seeking emergency care after choking on a piece of steak. The nursing assessment reveals a history of alcoholism, cigarette smoking, and hemoptysis. Which diagnostic study is most likely to be performed on this patient? A) Barium swallow B) Endoscopic biopsy C) Capsule endoscopy D) Endoscopic ultrasonography

B) Because of this patient's history of excessive alcohol intake, smoking, hemoptysis, and the current choking episode, cancer may be present. A biopsy is necessary to make a definitive diagnosis of carcinoma, so an endoscope will be used to obtain a biopsy and observe other abnormalities as well. A barium swallow may show narrowing of the esophagus, but it is more diagnostic for achalasia. An endoscopic ultrasonography may be used to stage esophageal cancer. Capsule endoscopy can show alterations in the esophagus but is more often used for small intestine problems. A barium swallow, capsule endoscopy, and endoscopic ultrasonography cannot provide a definitive diagnosis for cancer when it is suspected.

The nurse is caring for a patient treated with intravenous fluid therapy for severe vomiting. As the pt recovers and begins to tolerate oral intake, the N understands that which of the following food choices would be most appropriate? A) Ice tea B) Dry toast C) Warm broth D) Plain hamburger

B) Dry toast (Dry toast or crackers may alleviate the feeling of nausea and prevent further vomiting. Extremely hot or cold liquids and fatty foods are generally not well tolerated.)

The pt who is admitted with a diagnosis of diverticulitis and a history of irritable bowel disease and gastroesophageal reflux disease (GERD) has received a dose of Mylanta 30 ml PO. The nurse would evaluate its effectiveness by questioning the patient as to whether which of the following sxs has been resolved? A) Diarrhea B) Heartburn C) Constipation D) Lower abdominal pain

B) Heartburn (Mylanta is an antacid that contains both aluminum and magnesium. It is indicated for the relief of GI discomfort, such as with heartburn associated with GERD.)

Which information in a 67-year-old woman's health history will alert the nurse to the need for a more focused assessment of the musculoskeletal system? a. The patient sprained her ankle at age 13. b. The patient's mother became shorter with aging. c. The patient takes ibuprofen (Advil) for occasional headaches. d. The patient's father died of complications of miliary tuberculosis.

B. A family history of height loss with aging may indicate osteoporosis, and the nurse should perform a more thorough assessment of the patient's current height and other risk factors for osteoporosis. A sprained ankle during adolescence does not place the patient at increased current risk for musculoskeletal problems. A family history of tuberculosis is not a risk factor. Occasional nonsteroidal antiinflammatory drug (NSAID) use does not indicate any increased musculoskeletal risk.

The nurse notes crackling sounds and a grating sensation with palpation of an older patient's elbow. How will this finding be documented? a. Torticollis b. Crepitation c. Subluxation d. Epicondylitis

B. Crackling sounds and a grating sensation that accompany movement are described as crepitus or crepitation. Torticollis is a twisting of the neck to one side, subluxation is a partial dislocation of the joint, and epicondylitis is an inflammation of the elbow that causes a dull ache that increases with movement.

Which action can the nurse delegate to unlicensed assistive personnel (UAP) who are working in the orthopedic clinic? a. Grade leg muscle strength for a patient with back pain. b. Obtain blood sample for uric acid from a patient with gout. c. Perform straight-leg-raise testing for a patient with sciatica. d. Check for knee joint crepitation before arthroscopic surgery.

B. Drawing blood specimens is a common skill performed by UAP in clinic settings. The other actions are assessments and require registered nurse (RN)-level judgment and critical thinking.

A 42-year-old male patient complains of shoulder pain when the nurse moves his arm behind the back. Which question should the nurse ask? a. "Are you able to feed yourself without difficulty?" b. "Do you have difficulty when you are putting on a shirt?" c. "Are you able to sleep through the night without waking?" d. "Do you ever have trouble lowering yourself to the toilet?"

B. The patient's pain will make it more difficult to accomplish tasks like putting on a shirt or jacket. This pain should not affect the patient's ability to feed himself or use the toilet because these tasks do not involve moving the arm behind the patient. The arm will not usually be positioned behind the patient during sleeping.

35. After reviewing the electronic medical record shown in the accompanying figure for a patient who had transurethral resection of the prostate the previous day, which information requires the most rapid action by the nurse?

Bladder spasms and decreased urine output

28. Which assessment finding for a patient who has just been admitted with acute pyelonephritis is most important for the nurse to report to the health care provider?

Blood pressure 88/45 mm Hg

A 22-year-old female patient with an exacerbation of ulcerative colitis is having 15 to 20 stools daily and has excoriated perianal skin. Which patient behavior indicates that teaching regarding maintenance of skin integrity has been effective? a. The patient uses incontinence briefs to contain loose stools. b. The patient asks for antidiarrheal medication after each stool. c. The patient uses witch hazel compresses to decrease irritation. d. The patient cleans the perianal area with soap after each stool.

C

A 62-year-old patient has had a hemorrhoidectomy at an outpatient surgical center. Which instructions will the nurse include in discharge teaching? a. Maintain a low-residue diet until the surgical area is healed. b. Use ice packs on the perianal area to relieve pain and swelling. c. Take prescribed pain medications before a bowel movement is expected. d. Delay having a bowel movement for several days until healing has occurred.

C

A 71-year-old patient had an abdominal-perineal resection for colon cancer. Which nursing action is most important to include in the plan of care for the day after surgery? a. Teach about a low-residue diet. b. Monitor output from the stoma. c. Assess the perineal drainage and incision. d. Encourage acceptance of the colostomy stoma.

C

A 72-year-old male patient with dehydration caused by an exacerbation of ulcerative colitis is receiving 5% dextrose in normal saline at 125 mL/hour. Which assessment finding by the nurse is most important to report to the health care provider? a. Patient has not voided for the last 4 hours. b. Skin is dry with poor turgor on all extremities. c. Crackles are heard halfway up the posterior chest. d. Patient has had 5 loose stools over the last 6 hours.

C

A 76-year-old patient with obstipation has a fecal impaction and is incontinent of liquid stool. Which action should the nurse take first? a. Administer bulk-forming laxatives. b. Assist the patient to sit on the toilet. c. Manually remove the impacted stool. d. Increase the patient's oral fluid intake.

C

7. The nurse performing an assessment of a patient who has chronic peripheral artery disease (PAD) of the legs and an ulcer on the right second toe would expect to find a. dilated superficial veins. b. swollen, dry, scaly ankles. c. prolonged capillary refill in all the toes. d. serosanguineous drainage from the ulcer.

C Capillary refill is prolonged in PAD because of the slower and decreased blood flow to the periphery. The other listed clinical manifestations are consistent with chronic venous disease. DIF: Cognitive Level: Apply (application) REF: 807 TOP: Nursing Process: Assessment MSC: NCLEX: Physiological

14. A 46-yr-old service-counter worker undergoes sclerotherapy for treatment of superficial varicose veins at an outpatient center. Which instructions should the nurse provide to the patient before discharge? a. Sitting at the work counter, rather than standing, is recommended. b. Exercise, such as walking or jogging, can cause recurrence of varicosities. c. Elastic compression stockings should be applied before getting out of bed. d. Taking an aspirin daily will help prevent clots from forming around venous valves.

C Elastic compression stockings are applied with the legs elevated to reduce pressure in the lower legs. Walking is recommended to prevent recurrent varicosities. Sitting and standing are both risk factors for varicose veins and venous insufficiency. An aspirin a day is not adequate to prevent venous thrombosis and would not be recommended for a patient who had just had sclerotherapy. DIF: Cognitive Level: Apply (application) REF: 825 TOP: Nursing Process: Implementation MSC: NCLEX: Physiological

25. When caring for a patient on the first postoperative day after an abdominal aortic aneurysm repair, which assessment finding is most important for the nurse to communicate to the health care provider? a. Presence of flatus c. Maroon-colored liquid stool b. Hypoactive bowel sounds d. Abdominal pain with palpation

C Loose, bloody (maroon colored) stools at this time may indicate intestinal ischemia or infarction and should be reported immediately because the patient may need an emergency bowel resection. The other findings are normal on the first postoperative day after abdominal surgery. DIF: Cognitive Level: Analyze (analysis) REF: 813 OBJ: Special Questions: Prioritization TOP: Nursing Process: Assessment MSC: NCLEX: Physiological

29. Which assessment finding for a patient who has been admitted with a right calf venous thromboembolism (VTE) requires immediate action by the nurse? a. Erythema of right lower leg c. New onset shortness of breath b. Complaint of right calf pain d. Temperature of 100.4°F (38°C)

C New onset dyspnea suggests a pulmonary embolus, which will require rapid actions such as O2 administration and notification of the health care provider. The other findings are typical of VTE. DIF: Cognitive Level: Analyze (analysis) REF: 824 OBJ: Special Questions: Prioritization TOP: Nursing Process: Planning MSC: NCLEX: Physiological

19. A young adult patient tells the health care provider about experiencing cold, numb fingers when running during the winter, and Raynaud's phenomenon is suspected. The nurse will anticipate teaching the patient about tests for a. hyperglycemia. c. autoimmune disorders. b. hyperlipidemia. d. coronary artery disease.

C Secondary Raynaud's phenomenon may occur in conjunction with autoimmune diseases such as rheumatoid arthritis. Patients should be screened for autoimmune disorders. Raynaud's phenomenon is not associated with hyperlipidemia, hyperglycemia, or coronary artery disease. DIF: Cognitive Level: Apply (application) REF: 809 TOP: Nursing Process: Planning MSC: NCLEX: Physiological

18. Which action by a new nurse who is giving fondaparinux (Arixtra) to a patient with a lower leg venous thromboembolism (VTE) indicates that more education about the drug is needed? a. The nurse avoids rubbing the injection site after giving the drug. b. The nurse injects the drug into the abdominal subcutaneous tissue. c. The nurse ejects the air bubble from the syringe before giving the drug. d. The nurse does not check partial thromboplastin time (PTT) before giving the drug.

C The air bubble is not ejected before giving fondaparinux to avoid loss of drug. The other actions by the nurse are appropriate for subcutaneous administration of a low molecular weight heparin (LMWH). LMWHs typically do not require ongoing PTT monitoring and dose adjustment. DIF: Cognitive Level: Apply (application) REF: 820 TOP: Nursing Process: Implementation MSC: NCLEX: Safe and Effective Care Environment

The patient is having a gastroduodenostomy (Billroth I operation) for stomach cancer. What long-term complication is occurring when the patient reports generalized weakness, sweating, palpitations, and dizziness 15 to 30 minutes after eating? A) Malnutrition B) Bile reflux gastritis C) Dumping syndrome D) Postprandial hypoglycemia

C) After a Billroth I operation, dumping syndrome may occur 15 to 30 minutes after eating because of the hypertonic fluid going to the intestine and additional fluid being drawn into the bowel. Malnutrition may occur but does not cause these symptoms. Bile reflux gastritis cannot happen when the stomach has been removed. Postprandial hypoglycemia occurs with similar symptoms, but 2 hours after eating.

A female patient has a sliding hiatal hernia. What nursing interventions will prevent the symptoms of heartburn and dyspepsia that she is experiencing? A) Keep the patient NPO. B) Put the bed in the Trendelenberg position. C) Have the patient eat 4 to 6 smaller meals each day. D) Give various antacids to determine which one works for the patient.

C) Eating smaller meals during the day will decrease the gastric pressure and the symptoms of hiatal hernia. Keeping the patient NPO or in a Trendelenberg position are not safe or realistic for a long period of time for any patient. Varying antacids will only be done with the care provider's prescription, so this is not a nursing intervention.

The nurse determines that a pnt has experienced the beneficial effects of medication therapy with famotidine (Pepcid) when which of the following symptoms is relieved? A) Nausea B) Belching C) Epigastric pain D) Difficulty swallowing

C) Epigastric pain Famotidine is an H2-receptor antagonist that inhibits parietal cell output of HCl acid and minimizes damage to gastric mucosa related to hyperacidity, thus relieving epigastric pain.

Following administration of a dose of metoclopramide (Reglan) to the patient, the nurse determines that the medication has been effective when what is noted? A) Decreased blood pressure B) Absence of muscle tremors C) Relief of nausea and vomiting D) No further episodes of diarrhea

C) Metoclopramide is classified as a prokinetic and antiemetic medication. If it is effective, the patient's nausea and vomiting should resolve. Metoclopramide does not affect blood pressure, muscle tremors, or diarrhea.

The patient is having an esophagoenterostomy with anastomosis of a segment of the colon to replace the resected portion. What initial postoperative care should the nurse expect when this patient returns to the nursing unit? A) Turn, deep breathe, cough, and use spirometer every 4 hours. B) Maintain an upright position for at least 2 hours after eating. C) NG will have bloody drainage, and it should not be repositioned. D) Keep in a supine position to prevent movement of the anastomosis.

C) The patient will have bloody drainage from the NG tube for 8 to 12 hours, and it should not be repositioned or reinserted without contacting the surgeon. Turning and deep breathing will be done every 2 hours, and the spirometer will be used more often than every 4 hours. Coughing would put too much pressure in the area and should not be done. Because the patient will have the NG tube, the patient will not be eating yet. The patient should be kept in a semi-Fowler's or Fowler's position, not supine, to prevent reflux and aspiration of secretions.

Which information obtained during the nurse's assessment of a 30-year-old patient's nutritional-metabolic pattern may indicate the risk for musculoskeletal problems? a. The patient takes a multivitamin daily. b. The patient dislikes fruits and vegetables. c. The patient is 5 ft 2 in and weighs 180 lb. d. The patient prefers whole milk to nonfat milk.

C. The patient's height and weight indicate obesity, which places stress on weight-bearing joints. The use of whole milk, avoiding fruits and vegetables, and use of a daily multivitamin are not risk factors for musculoskeletal problems.

After completing the health history, the nurse assessing the musculoskeletal system will begin by a. having the patient move the extremities against resistance. b. feeling for the presence of crepitus during joint movement. c. observing the patient's body build and muscle configuration. d. checking active and passive range of motion for the extremities.

C. The usual technique in the physical assessment is to begin with inspection. Abnormalities in muscle mass or configuration will allow the nurse to perform a more focused assessment of abnormal areas. The other assessments are also included in the assessment but are usually done after inspection.

22. A patient who has bladder cancer had a cystectomy with creation of an Indiana pouch. Which topic will be included in patient teaching?

Catheterization technique and schedule

29. The health care provider prescribes the following interventions for a patient with acute prostatitis caused by E. coli. Which intervention should the nurse question?

Catheterize the patient as needed if symptoms of urinary retention develop.

32. Which nursing action should the nurse who is caring for a patient who has had an ileal conduit for several years delegate to nursing assistive personnel (NAP)?

Change the ostomy appliance.

43. A patient is admitted to the emergency department with possible renal trauma after an automobile accident. Which prescribed intervention will the nurse implement first?

Check blood pressure and heart rate.

The nurse is caring for a patient diagnosed with furunculosis. Which nursing action could the nurse delegate to unlicensed assistive personnel (UAP)?

Cleaning the skin with antimicrobial soap. Cleaning the skin is within the education and scope of practice for UAP. Administration of medication, obtaining cultures, and evaluation are higher-level skills that require the education and scope of practice of licensed nursing personnel.

A 27-year-old female patient is admitted to the hospital for evaluation of right lower quadrant abdominal pain with nausea and vomiting. Which action should the nurse take? a. Encourage the patient to sip clear liquids. b. Assess the abdomen for rebound tenderness. c. Assist the patient to cough and deep breathe. d. Apply an ice pack to the right lower quadrant.

D

A 73-year-old patient with diverticulosis has a large bowel obstruction. The nurse will monitor for a. referred back pain. b. metabolic alkalosis. c. projectile vomiting. d. abdominal distention.

D

A 75-year-old patient who lives alone at home tells the nurse, "I am afraid of losing my independence because my eyes don't work as well they used to." Which action should the nurse take first? a. Discuss the increased risk for falls that is associated with impaired vision. b. Explain that there are many ways to compensate for decreases in visual acuity. c. Suggest ways of improving the patient's safety, such as using brighter lighting. d. Ask the patient more about what type of vision problems are being experienced.

D

Which patient should the nurse assess first after receiving change-of-shift report? a. 60-year-old patient whose new ileostomy has drained 800 mL over the previous 8 hours b. 50-year-old patient with familial adenomatous polyposis who has occult blood in the stool c. 40-year-old patient with ulcerative colitis who has had six liquid stools in the previous 4 hours d. 30-year-old patient who has abdominal distention and an apical heart rate of 136 beats/minute

D

Which question from the nurse would help determine if a patient's abdominal pain might indicate irritable bowel syndrome? a. "Have you been passing a lot of gas?" b. "What foods affect your bowel patterns?" c. "Do you have any abdominal distention?" d. "How long have you had abdominal pain?"

D

Which statement by the patient to the home health nurse indicates a need for more teaching about self-administering eardrops? a. "I will leave the ear wick in place while administering the drops." b. "I should lie down before and for 5 minutes after administering the drops." c. "I will hold the tip of the dropper above the ear while administering the drops." d. "I should keep the medication refrigerated until I am ready to administer the drops."

D

Chapter 37: Vascular Disorders Lewis: Medical-Surgical Nursing, 10th Edition MULTIPLE CHOICE 1. When discussing risk factor modification for a patient who has a 5-cm abdominal aortic aneurysm, the nurse will focus teaching on which patient risk factor? a. Male gender c. Abdominal trauma history b. Turner syndrome d. Uncontrolled hypertension

D All of the factors contribute to the patient's risk, but only hypertension can potentially be modified to decrease the patient's risk for further expansion of the aneurysm. DIF: Cognitive Level: Apply (application) REF: 810 TOP: Nursing Process: Implementation MSC: NCLEX: Physiological

30. Which nursing intervention for a patient who had an open repair of an abdominal aortic aneurysm 2 days previously is appropriate for the nurse to delegate to unlicensed assistive personnel (UAP)? a. Monitor the quality and presence of the pedal pulses. b. Teach the patient the signs of possible wound infection. c. Check the lower extremities for strength and movement. d. Help the patient to use a pillow to splint while coughing.

D Assisting a patient who has already been taught how to cough is part of routine postoperative care and within the education and scope of practice for UAP. Patient teaching and assessment of essential postoperative functions such as circulation and movement should be done by RNs. DIF: Cognitive Level: Apply (application) REF: 824 OBJ: Special Questions: Delegation TOP: Nursing Process: Planning MSC: NCLEX: Safe and Effective Care Environment

15. Which topic should the nurse include in patient teaching for a patient with a venous stasis ulcer on the left lower leg? a. Need to increase carbohydrate intake b. Methods of keeping the wound area dry c. Purpose of prophylactic antibiotic therapy d. Application of elastic compression stockings

D Compression of the leg is essential to healing of venous stasis ulcers. High dietary intake of protein, rather than carbohydrates, is needed. Prophylactic antibiotics are not routinely used for venous ulcers. Moist dressings are used to hasten wound healing. DIF: Cognitive Level: Apply (application) REF: 826 TOP: Nursing Process: Planning MSC: NCLEX: Physiological

3. Several hours after a patient had an open surgical repair of an abdominal aortic aneurysm, the UAP reports to the nurse that urinary output for the past 2 hours has been 45 mL. The nurse notifies the health care provider and anticipates an order for a(n) a. hemoglobin count. c. serum creatinine level. b. additional antibiotic. d. increased IV infusion rate.

D The decreased urine output suggests decreased renal perfusion and monitoring of renal function is needed. There is no indication that infection is a concern, so antibiotic therapy and a WBC count are not needed. The IV rate may be increased because hypovolemia may be contributing to the patient's decreased urinary output. DIF: Cognitive Level: Apply (application) REF: 811 TOP: Nursing Process: Planning MSC: NCLEX: Physiological

6. A patient at the clinic says, "I always walk after dinner, but lately my leg cramps and hurts after just a few minutes of starting. The pain goes away after I stop walking, though." The nurse should a. look for the presence of tortuous veins bilaterally on the legs. b. ask about any skin color changes that occur in response to cold. c. assess for unilateral swelling, redness, and tenderness of either leg. d. palpate for the presence of dorsalis pedis and posterior tibial pulses.

D The nurse should assess for other clinical manifestations of peripheral arterial disease in a patient who describes intermittent claudication. Changes in skin color that occur in response to cold are consistent with Raynaud's phenomenon. Tortuous veins on the legs suggest venous insufficiency. Unilateral leg swelling, redness, and tenderness indicate venous thromboembolism. DIF: Cognitive Level: Apply (application) REF: 814 TOP: Nursing Process: Assessment MSC: NCLEX: Physiological

5. An older patient with chronic atrial fibrillation develops sudden severe pain, pulselessness, pallor, and coolness in the right leg. The nurse should notify the health care provider and immediately a. apply a compression stocking to the leg. b. elevate the leg above the level of the heart. c. assist the patient in gently exercising the leg. d. keep the patient in bed in the supine position.

D The patient's history and clinical manifestations are consistent with acute arterial occlusion, and resting the leg will decrease the O2 demand of the tissues and minimize ischemic damage until circulation can be restored. Elevating the leg or applying an elastic wrap will further compromise blood flow to the leg. Exercise will increase oxygen demand for the tissues of the leg. DIF: Cognitive Level: Apply (application) REF: 808 TOP: Nursing Process: Implementation MSC: NCLEX: Physiological

10. The health care provider has prescribed bed rest with the feet elevated for a patient admitted to the hospital with venous thromboembolism. Which action by the nurse to elevate the patient's feet is best? a. The patient is placed in the Trendelenburg position. b. Two pillows are positioned under the affected leg. c. The bed is elevated at the knee and pillows are placed under the feet. d. One pillow is placed under the thighs and two pillows are placed under the lower legs.

D The purpose of elevating the feet is to enhance venous flow from the feet to the right atrium, which is best accomplished by placing two pillows under the feet and one under the thighs. Placing the patient in the Trendelenburg position will lower the head below heart level, which is not indicated for this patient. Placing pillows under the calf or elevating the bed at the knee may cause blood stasis at the calf level. DIF: Cognitive Level: Analyze (analysis) REF: 819 TOP: Nursing Process: Implementation MSC: NCLEX: Physiological

A 72-year-old patient was admitted with epigastric pain due to a gastric ulcer. Which patient assessment warrants an urgent change in the nursing plan of care? A) Chest pain relieved with eating or drinking water B) Back pain 3 or 4 hours after eating a meal C) Burning epigastric pain 90 minutes after breakfast D) Rigid abdomen and vomiting following indigestion

D) A rigid abdomen with vomiting in a patient who has a gastric ulcer indicates a perforation of the ulcer, especially if the manifestations of perforation appear suddenly. Midepigastric pain is relieved by eating, drinking water, or antacids with duodenal ulcers, not gastric ulcers. Back pain 3-4 hours after a meal is more likely to occur with a duodenal ulcer. Burning epigastric pain 1-2 hours after a meal is an expected manifestation of a gastric ulcer related to increased gastric secretions and does not cause an urgent change in the nursing plan of care.

A pt reports having dry mouth and asks for some liquid to drink. The nurse reasons that this symptom can most likely be attributed to a common adverse effect of which of the following medications? A) Digoxin (Lanoxin) B) Cefotetan (Cefotan) C) Famotidine (Pepcid) D) Promethazine (Phenergan)

D) Promethazine (Phenergan) A common adverse effect of promethazine, an antihistamine antiemetic agent, is dry mouth; another is blurred vision.

The nurse is teaching a group of high school students about the prevention of food poisoning. Which comment by the student shows understanding of foodborne illness protection? A) "We like to mix up the ingredients so the flavors will melt before we cook our beef stew." B) "For a snack, I like to eat raw cookie dough from the package instead of baking the cookies." C) "We only have one cutting board, so we cut up our chicken and salad vegetables at the same time." D) "When they gave me a pink hamburger I sent it back and asked for a new bun and clean plate."

D) The student who did not accept the pink hamburger and asked for a new bun and clean plate understood that the pink meat may not have reached 160° and could be contaminated with bacteria. Mixing ingredients and leaving them long enough for the flavors to melt, eating raw cookie dough from a refrigerated package, and only using one cutting board without washing it with hot soapy water between the chicken and salad vegetables could all lead to food poisoning from contamination.

The nurse who notes that a 59-year-old female patient has lost 1 inch in height over the past 2 years will plan to teach the patient about a. discography studies. b. myelographic testing. c. magnetic resonance imaging (MRI). d. dual-energy x-ray absorptiometry (DXA).

D. The decreased height and the patient's age suggest that the patient may have osteoporosis and that bone density testing is needed. Discography, MRI, and myelography are typically done for patients with current symptoms caused by musculoskeletal dysfunction and are not the initial diagnostic tests for osteoporosis.

35. The nurse observes nursing assistive personnel (NAP) taking the following actions when caring for a patient with a retention catheter. Which action requires that the nurse intervene?

Disconnecting the catheter from the drainage tube to obtain a specimen

26. The nurse plans a presentation for community members about how to decrease the risk for antibiotic-resistant infections. Which information will the nurse include in the teaching plan (select all that apply)? o Continue taking antibiotics until all the medication is gone. o Antibiotics may sometimes be prescribed to prevent infection. o Unused antibiotics that are more than a year old should be discarded. o Antibiotics are effective in treating influenza associated with high fevers. o Hand washing is effective in preventing many viral and bacterial infections.

Hand washing is effective in preventing many viral and bacterial infections. Antibiotics may sometimes be prescribed to prevent infection Continue taking antibiotics until all the medication is gone.

37. Which assessment finding for a patient who has had a cystectomy with an ileal conduit the previous day is most important for the nurse to communicate to the physician?

Heart rate 102 beats/minute

Assessment of a patient's visual acuity reveals that the left eye can see at 20 feet what a person with normal vision can see at 50 feet and the right eye can see at 20 feet what a person with normal vision can see at 40 feet. The nurse records which finding?

OS 20/50; OD 20/40

An older adult patient with a squamous cell carcinoma (SCC) on the lower arm has a Mohs procedure in the dermatology clinic. Which nursing action will be included in the postoperative plan of care?

Teach about the use of cold packs to reduce bruising and swelling. Application of cold packs to the incision after the surgery will help decrease bruising and swelling at the site. Since the Mohs procedure results in complete excision of the lesion, topical fluorouracil is not needed after surgery. After the Mohs procedure the edges of the wound can be left open to heal or the edges can be approximated and sutured together. The suture line can be cleaned with tap water. No debridement with wet-to-dry dressings is indicated

40. A patient in the urology clinic is diagnosed with monilial urethritis. Which action will the nurse include in the plan of care?

Teach the patient about the use of antifungal medications.

17. A 62-year-old asks the nurse for a perineal pad, stating that laughing or coughing causes leakage of urine. Which intervention is most appropriate to include in the care plan?

Teach the patient how to perform Kegel exercises.

The nurse working in the dermatology clinic assesses a young adult female patient who is taking isotretinoin (Accutane) to treat severe cystic acne. Which assessment finding is most indicative of a need for further questioning of the patient?

The patient recently had an intrauterine device removed. Because isotretinoin is teratogenic, contraception is required for women who are using this medication. The nurse will need to determine whether the patient is using other birth control methods. More information about the other patient data may also be needed, but the other data do not indicate contraindications to isotretinoin use.

The nurse is performing an eye examination on a 76-year-old patient. The nurse should refer the patient for a more extensive assessment based on which finding?

The patient reports persistent photophobia.

Which information should the nurse include when teaching patients about decreasing the risk for sun damage to the skin?

Try to stay out of the sun between the hours of 10 AM and 2 PM (regular time). The risk for skin damage from the sun is highest with exposure between 10 AM and 2 PM. No sunscreen is completely water resistant. Sunscreens classified as water resistant sunscreens still need to be reapplied after swimming. Sunscreen with an SPF of at least 15 is recommended for people at normal risk for skin cancer. Although gradually increasing sun exposure may decrease the risk for burning, the risk for skin cancer is not decreased.

36. A patient undergoes a nephrectomy after having massive trauma to the kidney. Which assessment finding obtained postoperatively is most important to communicate to the surgeon?

Urine output is 20 mL/hr for 2 hours.

18. Following rectal surgery, a patient voids about 50 mL of urine every 30 to 60 minutes. Which nursing action is most appropriate?

Use an ultrasound scanner to check the postvoiding residual.

Which patient statement to the nurse indicates a need for additional instruction about taking oral ferrous sulfate?

a. "I will call my health care provider if my stools turn black." It is normal for the stools to appear black when a patient is taking iron, and the patient should not call the doctor about this.

The nurse examines the lymph nodes of a patient during a physical assessment. Which assessment finding would be of most concern to the nurse?

a. A 2-cm nontender supraclavicular node Enlarged and nontender nodes are suggestive of malignancies such as lymphoma.

Which patient should the nurse assign as the roommate for a patient who has aplastic anemia?

a. A patient with chronic heart failure Patients with aplastic anemia are at risk for infection because of the low white blood cell production associated with this type of anemia, so the nurse should avoid assigning a roommate with any possible infectious process.

The complete blood count (CBC) indicates that a patient is thrombocytopenic. Which action should the nurse include in the plan of care?

a. Avoid intramuscular injections. Thrombocytopenia is a decreased number of platelets, which places the patient at high risk for bleeding.

Which action will the nurse include in the plan of care for a 72-year-old woman admitted with multiple myeloma?

a. Monitor fluid intake and output. A high fluid intake and urine output helps prevent the complications of kidney stones caused by hypercalcemia and renal failure caused by deposition of Bence-Jones protein in the renal tubules.

The nurse has obtained the health history, physical assessment data, and laboratory results shown in the accompanying figure for a patient admitted with aplastic anemia. Which information is most important to communicate to the health care provider?

a. Neutropenia The low white blood cell count indicates that the patient is at high risk for infection and needs immediate actions to diagnose and treat the cause of the leucopenia.

Which menu choice indicates that the patient understands the nurse's teaching about best dietary choices for iron-deficiency anemia?

a. Omelet and whole wheat toast Eggs and whole grain breads are high in iron.

44. After change-of-shift report, which patient should the nurse assess first?

a. Patient with a urethral stricture who has not voided for 12 hours

Which information shown in the accompanying figure about a patient who has just arrived in the emergency department is most urgent for the nurse to communicate to the health care provider?

a. Platelet count The platelet count is severely decreased and places the patient at risk for spontaneous bleeding.

Which information obtained by the nurse assessing a patient admitted with multiple myeloma is most important to report to the health care provider?

a. Serum calcium level is 15 mg/dL. Hypercalcemia may lead to complications such as dysrhythmias or seizures, and should be addressed quickly.

14. Which information will the nurse plan to include when teaching a community health group about testicular self-examination?

a. Testicular self-examination should be done in a warm room.

A 19-year-old woman with immune thrombocytopenic purpura (ITP) has an order for a platelet transfusion. Which information indicates that the nurse should consult with the health care provider before obtaining and administering platelets?

a. The platelet count is 42,000/μL. Platelet transfusions are not usually indicated until the platelet count is below 10,000 to 20,000/μL unless the patient is actively bleeding.

The nurse caring for a patient with type A hemophilia being admitted to the hospital with severe pain and swelling in the right knee will

a. immobilize the joint. The initial action should be total rest of the knee to minimize bleeding. Ice packs are used to decrease bleeding.

16. When performing discharge teaching for a patient after a vasectomy, the nurse instructs the patient that he

a. should continue to use other methods of birth control for 6 weeks.

15. A 27-year-old man who has testicular cancer is being admitted for a unilateral orchiectomy. The patient does not talk to his wife and speaks to the nurse only to answer the admission questions. Which action is best for the nurse to take?

b. Ask the patient if he has any questions or concerns about the diagnosis and treatment.

Which intervention will be included in the nursing care plan for a patient with immune thrombocytopenic purpura (ITP)?

b. Avoid intramuscular (IM) injections. IM or subcutaneous injections should be avoided because of the risk for bleeding.

A patient's complete blood count (CBC) shows a hemoglobin of 19 g/dL and a hematocrit of 54%. Which question should the nurse ask to determine possible causes of this finding?

b. "Do you have any history of lung disease?" The hemoglobin and hematocrit results indicate polycythemia, which can be associated with chronic obstructive pulmonary disease (COPD).

A 68-year-old woman with acute myelogenous leukemia (AML) asks the nurse whether the planned chemotherapy will be worth undergoing. Which response by the nurse is appropriate?

b. "The side effects of chemotherapy are difficult, but AML frequently goes into remission with chemotherapy." This response uses therapeutic communication by addressing the patient's question and giving accurate information.

Several patients call the outpatient clinic and ask to make an appointment as soon as possible. Which patient should the nurse schedule to be seen first?

b. 23-year-old with no previous health problems who has a nontender lump in the axilla The patient's age and presence of a nontender axillary lump suggest possible lymphoma, which needs rapid diagnosis and treatment.

After receiving change-of-shift report for several patients with neutropenia, which patient should the nurse assess first?

b. 33-year-old with a fever of 100.8° F (38.2° C) Any fever in a neutropenic patient indicates infection and can quickly lead to sepsis and septic shock.

A patient with pancytopenia of unknown origin is scheduled for the following diagnostic tests. The nurse will provide a consent form to sign for which test?

b. Bone marrow biopsy A bone marrow biopsy is a minor surgical procedure that requires the patient or guardian to sign a surgical consent form.

Which action will the admitting nurse include in the care plan for a 30-year old woman who is neutropenic?

b. Check temperature every 4 hours. The earliest sign of infection in a neutropenic patient is an elevation in temperature.

Patient diagnosed with benign prostatic hyperplasia (BPH) tells the nurse that he does not want a transurethral resection of the prostate (TURP) because it might affect his ability to maintain an erection during intercourse. Which action should the nurse take?

b. Discuss that TURP does not commonly affect erectile function.

The nurse is reviewing laboratory results and notes an aPTT level of 28 seconds. The nurse should notify the health care provider in anticipation of adjusting which medication?

b. Heparin Activated partial thromboplastin time (aPTT) assesses intrinsic coagulation by measuring factors I, II, V, VIII, IX, X, XI, XII. aPTT is increased (prolonged) in heparin administration.

A patient with septicemia develops prolonged bleeding from venipuncture sites and blood in the stools. Which action is most important for the nurse to take?

b. Notify the patient's physician. The patient's new onset of bleeding and diagnosis of sepsis suggest that disseminated intravascular coagulation (DIC) may have developed, which will require collaborative actions such as diagnostic testing, blood product administration, and heparin administration.

The nurse is planning to administer a transfusion of packed red blood cells (PRBCs) to a patient with blood loss from gastrointestinal hemorrhage. Which action can the nurse delegate to unlicensed assistive personnel (UAP)?

b. Obtain the temperature, blood pressure, and pulse before the transfusion. UAP education includes measurement of vital signs. UAP would report the vital signs to the registered nurse (RN).

Which collaborative problem will the nurse include in a care plan for a patient admitted to the hospital with idiopathic aplastic anemia?

b. Potential complication: infection Because the patient with aplastic anemia has pancytopenia, the patient is at risk for infection and bleeding.

41. Which action will the nurse anticipate taking for an otherwise healthy 50-year-old who has just been diagnosed with Stage 1 renal cell carcinoma?

b. Provide preoperative teaching about nephrectomy.

Which information will the nurse include when teaching the patient with a urinary tract infection (UTI) about the use of phenazopyridine (Pyridium)?

b. Pyridium may change the urine color.

Which problem reported by a patient with hemophilia is most important for the nurse to communicate to the physician?

b. Tarry stools Melena is a sign of gastrointestinal bleeding and requires collaborative actions such as checking hemoglobin and hematocrit and administration of coagulation factors.

A 30-year-old man with acute myelogenous leukemia develops an absolute neutrophil count of 850/µL while receiving outpatient chemotherapy. Which action by the outpatient clinic nurse is most appropriate?

b. Teach the patient to administer filgrastim (Neupogen) injections. The patient may be taught to self-administer filgrastim injections.

Which information obtained by the nurse caring for a patient with thrombocytopenia should be immediately communicated to the health care provider?

b. The patient is difficult to arouse. Difficulty in arousing the patient may indicate a cerebral hemorrhage, which is life threatening and requires immediate action.

A patient with a history of a transfusion-related acute lung injury (TRALI) is to receive a transfusion of packed red blood cells (PRBCs). Which action by the nurse will decrease the risk for TRALI for this patient?

b. Transfuse only leukocyte-reduced PRBCs. TRALI is caused by a reaction between the donor and the patient leukocytes that causes pulmonary inflammation and capillary leaking.

An appropriate nursing intervention for a hospitalized patient with severe hemolytic anemia is to

b. alternate periods of rest and activity. Nursing care for patients with anemia should alternate periods of rest and activity to encourage activity without causing undue fatigue.

A 28-year-old man with von Willebrand disease is admitted to the hospital for minor knee surgery. The nurse will review the coagulation survey to check the

b. bleeding time. The bleeding time is affected by von Willebrand disease. Platelet count, prothrombin time, and thrombin time are normal in von Willebrand disease.

A routine complete blood count indicates that an active 80-year-old man may have myelodysplastic syndrome. The nurse will plan to teach the patient about

b. bone marrow biopsy. Bone marrow biopsy is needed to make the diagnosis and determine the specific type of myelodysplastic syndrome.

A patient who is receiving methotrexate for severe rheumatoid arthritis develops a megaloblastic anemia. The nurse will anticipate teaching the patient about increasing oral intake of

b. folic acid. Methotrexate use can lead to folic acid deficiency. Supplementation with oral folic acid supplements is the usual treatment.

3. The health care provider prescribes finasteride (Proscar) for a 67-year-old patient who has benign prostatic hyperplasia (BPH). When teaching the patient about the drug, the nurse informs him that

b. his interest in sexual activity may decrease while he is taking the medication.

6. The nurse will plan to teach the patient scheduled for photovaporization of the prostate (PVP)

b. how to care for an indwelling urinary catheter.

17. A 52-year-old man tells the nurse that he decided to seek treatment for erectile dysfunction (ED) because his wife "is losing patience with the situation." The most appropriate nursing diagnosis for the patient is

b. ineffective role performance related to effects of ED.

5. The nurse determines that further instruction is needed for a patient with interstitial cystitis when the patient says which of the following?

c. "I will start taking high potency multiple vitamins every morning."

A postoperative patient receiving a transfusion of packed red blood cells develops chills, fever, headache, and anxiety 35 minutes after the transfusion is started. After stopping the transfusion, what action should the nurse take?

c. Administer PRN acetaminophen (Tylenol). The patient's clinical manifestations are consistent with a febrile, nonhemolytic transfusion reaction.

Which action will the nurse include in the plan of care for a patient who has thalassemia major?

c. Administer iron chelation therapy as needed. The frequent transfusions used to treat thalassemia major lead to iron toxicity in patients unless iron chelation therapy is consistently used.

Which action for a patient with neutropenia is appropriate for the registered nurse (RN) to delegate to a licensed practical/vocational nurse (LPN/LVN)?

c. Administering subcutaneous filgrastim (Neupogen) injection Administration of subcutaneous medications is included in LPN/LVN education and scope of practice.

Which instruction will the nurse plan to include in discharge teaching for the patient admitted with a sickle cell crisis?

c. Avoid exposure to crowds when possible. Exposure to crowds increases the patient's risk for infection, the most common cause of sickle cell crisis.

Which finding about a patient with polycythemia vera is most important for the nurse to report to the health care provider?

c. Calf swelling and pain The calf swelling and pain suggest that the patient may have developed a deep vein thrombosis, which will require diagnosis and treatment to avoid complications such as pulmonary embolus.

13. A 32-year-old patient is diagnosed with polycystic kidney disease. Which information is most appropriate for the nurse to include in teaching at this time?

c. Discussion of options for genetic counseling

A patient who had a total hip replacement had an intraoperative hemorrhage 14 hours ago. Which laboratory result would the nurse expect to find?

c. Elevated reticulocyte count Hemorrhage causes the release of reticulocytes (immature red blood cells) from the bone marrow into circulation.

The health care provider's progress note for a patient states that the complete blood count (CBC) shows a "shift to the left." Which assessment finding will the nurse expect?

c. Elevated temperature The term shift to the left indicates that the number of immature polymorphonuclear neutrophils (bands) is elevated and that finding is a sign of infection.

A patient with possible disseminated intravascular coagulation arrives in the emergency department with a blood pressure of 82/40, temperature 102° F (38.9° C), and severe back pain. Which physician order will the nurse implement first?

c. Infuse normal saline 500 mL over 30 minutes. The patient's blood pressure indicates hypovolemia caused by blood loss and should be addressed immediately to improve perfusion to vital organs.

The nurse assesses a patient with pernicious anemia. Which assessment finding would the nurse expect?

c. Numbness of the extremities Extremity numbness is associated with cobalamin (vitamin B12) deficiency or pernicious anemia.

A 46-year-old female patient returns to the clinic with recurrent dysuria after being treated with trimethoprim and sulfamethoxazole (Bactrim) for 3 days. Which action will the nurse plan to take?

c. Obtain a midstream urine specimen for culture and sensitivity testing.

The nurse determines that instruction regarding prevention of future urinary tract infections (UTIs) has been effective for a 22-year-old female patient with cystitis when the patient states which of the following?

d. "I will empty my bladder every 3 to 4 hours during the day."

Which statement by a patient indicates good understanding of the nurse's teaching about prevention of sickle cell crisis?

d. "Risk for a crisis is decreased by having an annual influenza vaccination." Because infection is the most common cause of a sickle cell crisis, influenza, Haemophilus influenzae, pneumococcal pneumonia, and hepatitis immunizations should be administered.

Which laboratory test will the nurse use to determine whether filgrastim (Neupogen) is effective for a patient with acute lymphocytic leukemia who is receiving chemotherapy?

d. Absolute neutrophil count Filgrastim increases the neutrophil count and function in neutropenic patients.

Which laboratory result will the nurse expect to show a decreased value if a patient develops heparin-induced thrombocytopenia (HIT)?

d. Activated partial thromboplastin time Platelet aggregation in HIT causes neutralization of heparin, so that the activated partial thromboplastin time will be shorter and more heparin will be needed to maintain therapeutic levels.

The health care provider orders a liver/spleen scan for a patient who has been in a motor vehicle accident. Which action should the nurse take before this procedure?

d. Assist the patient to a flat position. During a liver/spleen scan, a radioactive isotope is injected IV and images from the radioactive emission are used to evaluate the structure of the spleen and liver.

5. Which information about continuous bladder irrigation will the nurse teach to a patient who is being admitted for a transurethral resection of the prostate (TURP)?

d. Bladder irrigation prevents obstruction of the catheter after surgery.

4. Which finding by the nurse will be most helpful in determining whether a 67-year-old patient with benign prostatic hyperplasia has an upper urinary tract infection (UTI)?

d. Costovertebral tenderness

A patient who has been receiving a heparin infusion and warfarin (Coumadin) for a deep vein thrombosis (DVT) is diagnosed with heparin-induced thrombocytopenia (HIT) when her platelet level drops to 110,000/µL. Which action will the nurse include in the plan of care?

d. Discontinue heparin and flush intermittent IV lines using normal saline. All heparin is discontinued when the HIT is diagnosed.

A patient with pancytopenia has a bone marrow aspiration from the left posterior iliac crest. Which action would be important for the nurse to take after the procedure?

d. Have the patient lie on the left side for 1 hour. To decrease the risk for bleeding, the patient should lie on the left side for 30 to 60 minutes.

When assessing a newly admitted patient, the nurse notes pallor of the skin and nail beds. The nurse should ensure that which laboratory test has been ordered?

d. Hemoglobin (Hgb) level Pallor of the skin or nail beds is indicative of anemia, which would be indicated by a low Hgb level.

A patient who has non-Hodgkin's lymphoma is receiving combination treatment with rituximab (Rituxan) and chemotherapy. Which patient assessment finding requires the most rapid action by the nurse?

d. Lip swelling Lip swelling in angioedema may indicate a hypersensitivity reaction to the rituximab. The nurse should stop the infusion and further assess for anaphylaxis.

Following successful treatment of Hodgkin's lymphoma for a 55-year-old woman, which topic will the nurse include in patient teaching?

d. Need for follow-up appointments to screen for malignancy The chemotherapy used in treating Hodgkin's lymphoma results in a high incidence of secondary malignancies; follow-up screening is needed.

Which patient information is most important for the nurse to monitor when evaluating the effectiveness of deferoxamine (Desferal) for a patient with hemochromatosis?

d. Serum iron level Because iron chelating agents are used to lower serum iron levels, the most useful information will be the patient's iron level.

27. Which assessment information is most important for the nurse to report to the health care provider when a patient asks for a prescription for testosterone replacement therapy (TRT)?

d. The patient has had a gradual decrease in the force of his urinary stream.

The nurse is caring for a patient who is being discharged after an emergency splenectomy following an automobile accident. Which instructions should the nurse include in the discharge teaching?

d. Wash hands and avoid persons who are ill. Splenectomy increases the risk for infection, especially with gram-positive bacteria.

A nurse reviews the laboratory data for an older patient. The nurse would be most concerned about which finding?

d. White blood cell (WBC) count of 2800/µL Because the total WBC count is not usually affected by aging, the low WBC count in this patient would indicate that the patient's immune function may be compromised and the underlying cause of the problem needs to be investigated

The nurse reviews the complete blood count (CBC) and white blood cell (WBC) differential of a patient admitted with abdominal pain. Which information will be most important for the nurse to communicate to the health care provider?

d. White blood cells (WBCs) 15,500/µL The elevation in WBCs indicates that the patient has an inflammatory or infectious process ongoing, which may be the cause of the patient's pain, and that further diagnostic testing is needed.

A patient in the emergency department complains of back pain and difficulty breathing 15 minutes after a transfusion of packed red blood cells is started. The nurse's first action should be to

d. disconnect the transfusion and infuse normal saline. The patient's symptoms indicate a possible acute hemolytic reaction caused by the transfusion.

11. The nurse teaches a 64-year-old woman to prevent the recurrence of renal calculi by

d. drinking 2000 to 3000 mL of fluid a day.

23. Two days after surgery for an ileal conduit, the patient will not look at the stoma or participate in care. The patient insists that no one but the ostomy nurse specialist care for the stoma. The nurse identifies a nursing diagnosis of

disturbed body image related to change in body function.

12. When planning teaching for a patient with benign nephrosclerosis the nurse should include instructions regarding

monitoring and recording blood pressure.

A patient's renal calculus is analyzed as being very high in uric acid. To prevent recurrence of stones, the nurse teaches the patient to avoid eating

organ meats and sardines.

A patient arrives in the emergency department complaining of eye itching and pain after sleeping with contact lenses in place. To facilitate further examination of the eye, fluorescein angiography is ordered. The nurse will teach the patient to

report any burning or pain at the IV site.

A patient complains of dizziness when bending over and of nausea and dizziness associated with physical activities. The nurse will plan to teach the patient about

rotary chair testing.


Related study sets

Ch 19: Drug Therapy with Aminoglycosides and Fluoroquinolones

View Set

Table 6-5 Pink Lesions: Basal Cell Carcinoma and Its Mimics

View Set

module 5 Psychology ch 9, 10, 11, 12

View Set

N300 Exam 2: Gallbladder Disorders

View Set

World History Semester 2 Answers

View Set